Вы находитесь на странице: 1из 773

8/12/2016

MyPastest

Back to Filters (/Secure/TestMe/Filter/429893/QA)

Question 1 of 179

A 24-year-old man is admitted from a local night club after suffering from an epileptic
seizure. You understand from other people with him that he was acutely anxious and
suffering from paranoid thoughts, and the nurse finds a small amount of what looks like
cocaine in his jacket. On examination he is very anxious, and is reluctant even to let you take
blood. His temperature is 38.0C, blood pressure 155/90 mmHg, pulse 120 bpm, regular. He
develops generalised seizures that fail to respond to three successive intravenous doses of
diazepam 5 mg. His ECG shows 2 mm of ST depression in the anterolateral leads.
Which one of the following is the most appropriate next intervention?
A

Intubation and ventilation

IV esmolol

IV phenytoin

IV sodium valproate

IV verapamil

Explanation

The answer is Intubation and ventilation


This man has suffered a further series of seizures, coupled with a rise in blood pressure
and heart rate. He is at high risk of myocardial ischaemic damage and hyperthermia,
which may lead to rhabdomyolysis and worsening renal failure. Cocaine overdose may
cause haemorrhagic stroke. Airway intubation and ventilation may lower his blood
pressure and improve ischaemia; subsequent steps would involve measures to control
blood pressure and seizure activity.

IV esmolol (Option B) is incorrect. Esmolol, a short acting -blocker may assist in controlling
heart rate, but would be of limited value in controlling blood pressure or seizure risk.
IV phenytoin (Option C) is incorrect. IV phenytoin may help control seizures, but would not
address the underlying agitation, pyrexia and high blood pressure that may be provoking
seizures.
https://mypastest.pastest.com/Secure/TestMe/Browser/429893

1/2

8/12/2016

MyPastest

IV sodium valproate (Option D) is incorrect. IV sodium valproate may help control seizures,
but would not address the underlying agitation, pyrexia and high blood pressure that may be
provoking seizures.
IV verapamil (Option E) is incorrect. Verapamil, a calcium channel blocker, may assist in
controlling heart rate, but would be of limited value in controlling blood pressure or seizure
risk.
46981

Next Question

Previous Question

Tag Question

Feedback

End Review

Difficulty: Average
Peer Responses

Session Progress
Responses Correct:

Responses Incorrect:

179

Responses Total:

179

Responses - % Correct:

0%

Blog (https://www.pastest.com/blog) About Pastest (https://www.pastest.com/about-us)


Contact Us (https://www.pastest.com/contact-us) Help (https://www.pastest.com/help)
Pastest 2016

https://mypastest.pastest.com/Secure/TestMe/Browser/429893

2/2

8/12/2016

MyPastest

Back to Filters (/Secure/TestMe/Filter/429893/QA)

Question 2 of 179

A 16-year-old woman attends the Emergency Department and reports taking around 30 g of
paracetamol and 2 g of dihydrocodeine about 10 h earlier. On examination, she is drowsy with
a Glasgow Coma Scale of 14. Heart rate is 100 bpm, blood pressure is 110/66 mmHg, there are
pinpoint pupils, and oxygen saturations are 96% on air.
Which one of the following treatments would be most strongly indicated?
A

10% dextrose infusion

Activated charcoal by mouth

Gastric lavage

N-Acetylcysteine intravenously

Naloxone intravenously

Explanation

The answer is N-acetylcysteine intravenously


N-acetylcysteine is the treatment of choice for paracetamol overdose. At more than 8 h
after ingestion of a potentially toxic quantity, acetylcysteine should be commenced while
the drug concentrations and blood tests are awaited. Treatment should not be withheld
pending the blood test results.

10% dextrose infusion (Option A) is incorrect. There is no indication that the patient is
suffering from hypoglycaemia.
Activated charcoal by mouth (Option B) is incorrect. Oral activated charcoal should normally
be given within 1 h of paracetamol ingestion, although may be effective up to 2 h after in
patients that coingest opioids. At 10 h, it is too late to be effective, and in a drowsy patient,
there may be a risk of charcoal aspiration into the lungs.
Gastric lavage (Option C) is incorrect. Gastric lavage may be indicated for life-threatening
ingestions within the previous 1 h, but is rarely indicated, with the exception of certain
medications e.g. lithium.
https://mypastest.pastest.com/Secure/TestMe/Browser/429893

1/2

8/12/2016

MyPastest

Naloxone intravenously (Option E) is incorrect. There are clues in the clinical history for this
patient to suggest that she is not significantly compromised in terms of level of
consciousness or respiratory function; hence naloxone is not necessary at present.
46921

Next Question

Previous Question

Tag Question

Feedback

End Review

Difficulty: Average
Peer Responses

Session Progress
Responses Correct:

Responses Incorrect:

179

Responses Total:

179

Responses - % Correct:

0%

Blog (https://www.pastest.com/blog) About Pastest (https://www.pastest.com/about-us)


Contact Us (https://www.pastest.com/contact-us) Help (https://www.pastest.com/help)
Pastest 2016

https://mypastest.pastest.com/Secure/TestMe/Browser/429893

2/2

8/12/2016

MyPastest

Back to Filters (/Secure/TestMe/Filter/429893/QA)

Question 3 of 179

A 69-year-old man with chronic atrial fibrillation, controlled on digoxin, is found to have a
blood pressure of 178/102 mmHg. He is commenced on modified-release verapamil 240 mg
daily. One week later, he is taken to the hospital Emergency Department having been found
collapsed. He is tachycardic at 130 bpm, hypotensive with a blood pressure 90/62 mmHg, and
an electrocardiogram (ECG) shows him to be in ventricular tachycardia.
What is the most likely explanation for these findings?
A

He has developed digoxin toxicity

He has had a myocardial infarction

He has WolffParkinsonWhite syndrome

The hypotensive effect of verapamil has induced a reflex ventricular tachycardia

Verapamil has exerted a pro-arrhythmic effect

Explanation

The answer is He has developed digoxin toxicity


Digoxin toxicity can result from co-administration of verapamil, nifedipine, quinine or
quinidine, all of which inhibit tubular secretion of digoxin, so that the dose of digoxin
should normally be decreased. Digoxin toxicity may cause any one of a number of
different arrhythmias, and may be precipitated by hypokalaemia or hypomagnesaemia.

He has had a myocardial infarction (Option B) is incorrect. Although myocardial infarction


might explain the haemodynamic profile, this explanation fails to recognise the significance
of the drug interaction.
He has WolffParkinsonWhite syndrome (Option C) is incorrect. The clinical manifestion of
WolffParkinsonWhite syndrome would perhaps be less rather than more likely to occur
after initiation of verapamil.
The hypotensive effect of verapamil has induced a reflex ventricular tachycardia (Option D) is
incorrect. Verapamil would primarily act by reducing resting heart rate, rather than causing
hypotension through a direct effect on blood vessel tone.
https://mypastest.pastest.com/Secure/TestMe/Browser/429893

1/2

8/12/2016

MyPastest

Verapamil has exerted a pro-arrhythmic effect (Option E) is incorrect. Verapamil may cause
bradyarrhythmia and heart block, and would not be expected to cause tachycardia.
46915

Next Question

Previous Question

Tag Question

Feedback

End Review

Difficulty: Average
Peer Responses

Session Progress
Responses Correct:

Responses Incorrect:

179

Responses Total:

179

Responses - % Correct:

0%

Blog (https://www.pastest.com/blog) About Pastest (https://www.pastest.com/about-us)


Contact Us (https://www.pastest.com/contact-us) Help (https://www.pastest.com/help)
Pastest 2016

https://mypastest.pastest.com/Secure/TestMe/Browser/429893

2/2

8/12/2016

MyPastest

Back to Filters (/Secure/TestMe/Filter/429893/QA)

Question 4 of 179

A 78-year-old recently widowed lady is admitted by ambulance to the Emergency


Department. She was found unconscious by a neighbour, surrounded by three empty bottles
of propranolol tablets. On examination her pulse was noted to be 38 bpm and she has a
blood pressure of 78/50 mmHg. She is given activated charcoal, intravenous fluid loading and
atropine, but fails to respond.
Which one of the following is the most appropriate next step in her management?
A

Insertion of temporary pacing wire

IV adrenaline

IV calcium chloride

IV glucagon

IV phenytoin

Explanation

The answer is IV glucagon


This woman is suffering the effects of an intentional overdose of -blockers, probably
related to depression at her recent bereavement. Glucagon is a specific antidote for
excessive -blockade and is indicated in this situation.

Insertion of temporary pacing wire (Option A) is incorrect. Temporary pacing might be


considered in patients unresponsive to glucagon therapy, but it is associated with iatrogenic
complications and normally considered after medical therapies.
IV adrenaline (Option B) is incorrect. IV adrenaline in this situation may precipitate
myocardial ischaemia or actually be proarrhythmogenic.
IV calcium chloride (Option C) is incorrect. IV calcium chloride is useful in calcium channel
blocker overdose, but less so after beta-blocker overdose.
IV phenytoin (Option E) is incorrect. Phenytoin would not be expected to offer any
significant benefit in this situation.
46973

https://mypastest.pastest.com/Secure/TestMe/Browser/429893

1/2

8/12/2016

MyPastest

46973

Next Question

Previous Question

Tag Question

Feedback

End Review

Difficulty: Average
Peer Responses

Session Progress
Responses Correct:

Responses Incorrect:

179

Responses Total:

179

Responses - % Correct:

0%

Blog (https://www.pastest.com/blog) About Pastest (https://www.pastest.com/about-us)


Contact Us (https://www.pastest.com/contact-us) Help (https://www.pastest.com/help)
Pastest 2016

https://mypastest.pastest.com/Secure/TestMe/Browser/429893

2/2

8/12/2016

MyPastest

Back to Filters (/Secure/TestMe/Filter/429893/QA)

Question 5 of 179

A 29-year-old woman with a history of ulcerative colitis is reviewed on the ward 48hrs after
commencement of IV corticosteroid therapy. She was admitted after failing to respond to
high dose oral corticosteroids and maximal dose Mesalazine, and continues to pass blood
stained diarrhoeal stool up to 12 times per day. On examination she is pyrexial 37.9C, her BP
is 105/72 mmHg, pulse is 92/min and regular. Her abdomen is generally tender and mildly
distended, with more marked tenderness on the left hand side. Bowel sounds are active.
Investigations;
Hb

9.9 g/dl

WCC

13.1 x109/l

PLT

191 x109/l

Na+

137 mmol/l

K+

3.4 mmol/l

Creatinine

110 micromol/l

CRP

195 mg/l

Albumin

28 g/l

Abdominal X-ray distended colon 5.2cm in diameter


Which of the following is the most appropriate next step?
A

Add Azathioprine

Add Infliximab

Add rectal corticosteroids

Continue IV steroids and Mesalazine

Proceed to colectomy

Explanation
https://mypastest.pastest.com/Secure/TestMe/Browser/429893

1/2

8/12/2016

MyPastest

The answer is Add Infliximab In this situation the choices are proceed to colectomy or move to a trial of Infliximab or
Ciclosporin. Infliximab can be used to induce rapid remission and for maintenance therapy,
although it is unclear whether its use actually reduces the risk of colectomy over the longer
term. Ciclosporin is used to act as a bridge to other treatments, which take time to build
their effectiveness such as Azathioprine or 6-mercaptopurine. If there is no response to
Infliximab within 72hrs, colectomy is the most appropriate next intervention.
40150

Next Question

Previous Question

Tag Question

Feedback

End Review

Difficulty: Difficult
Peer Responses

Session Progress
Responses Correct:

Responses Incorrect:

179

Responses Total:

179

Responses - % Correct:

0%

Blog (https://www.pastest.com/blog) About Pastest (https://www.pastest.com/about-us)


Contact Us (https://www.pastest.com/contact-us) Help (https://www.pastest.com/help)
Pastest 2016

https://mypastest.pastest.com/Secure/TestMe/Browser/429893

2/2

8/12/2016

MyPastest

Back to Filters (/Secure/TestMe/Filter/429893/QA)

Question 6 of 179

Genetic variations may have a significant impact on the clinical response to various drugs.
One example is genetic variation of cytochrome P450 2D6 (CYP-2D6) isoenzyme activity
such that patients may be considered active or poor metabolisers.
Which of the following drugs is most likely to cause adverse effects in patients with poor
CYP-2D6 metabolism?
A

Dapsone

Diazepam

Fluoxetine

Omeprazole

Phenylzine

Explanation

The answer is Fluoxetine


There are numerous variations in the cytochrome P450 isoenzyme system leading to
differences in how patients respond to certain drugs. Patients who are slow metabolisers
will have increased concentrations of drugs and have a higher rate of occurrence of
adverse effects; rapid metabolisers may have lower drug concentrations and less
therapeutic response. Drugs that are metabolised via CYP-2D6 include tricyclic
antidepressants, -blockers (especially metoprolol), dihydrocodeine, ecstasy (MDMA)
and certain selective serotonin reuptake inhibitors. Dihydrocodeine is metabolised by
cytochrome CYP-2D6 to morphine, the active analgesic. Patients who are CYP-2D6 poor
metabolisers will have less analgesic response to dihydrocodeine. Other P450
isoenzymes are CYP-2C8 (omeprazole, diazepam, barbiturates) and CYP-2C18/19
(omeprazole, diazepam, tricyclic antidepressants, proguanil).

Dapsone (Option A) is incorrect. Dapsone is subject to metabolism by CYP-2C8 and CYP2C9.

https://mypastest.pastest.com/Secure/TestMe/Browser/429893

1/2

8/12/2016

MyPastest

Diazepam (Option B) is incorrect. Diazepam is subject to metabolism by CYP-2C8 and CYP3A4.


Omeprazole (Option D) is incorrect. Omeprazole is subject to metabolism by CYP-2C8, CYP2C18 and others, but not CYP-2D6.
Phenylzine (Option E) is incorrect. Phenylzine metabolism is not subject to genetic variation.
46385

Next Question

Previous Question

Tag Question

Feedback

End Review

Difficulty: Average
Peer Responses

Session Progress
Responses Correct:

Responses Incorrect:

179

Responses Total:

179

Responses - % Correct:

0%

Blog (https://www.pastest.com/blog) About Pastest (https://www.pastest.com/about-us)


Contact Us (https://www.pastest.com/contact-us) Help (https://www.pastest.com/help)
Pastest 2016

https://mypastest.pastest.com/Secure/TestMe/Browser/429893

2/2

8/12/2016

MyPastest

Back to Filters (/Secure/TestMe/Filter/429893/QA)

Question 7 of 179

You are asked by the ophthalmology registrar to prescribe a course of vitamin A for a young
man who has been suffering from night blindness for the past 34 months.
Which of the following substances in vitamin A is most likely to be maximally involved in
correcting the visual disturbance?
A

Beta-carotene

Retinaldehyde

Retinoic acid

Retinol

Retinyl phosphate

Explanation

The answer is Retinaldehyde


Vitamin A is often used as a collective term for several related biologically active
molecules. Retinaldehyde is derived from the oxidation of retinol, and the cis form is
found in the opsin proteins in the rods (rhodopsin) of the retina. Exposure to light causes
retinaldehyde to change to its trans isomer, and the resulting changes in membrane
potentials give rise to signals transmitted to the brain.

Beta-carotene (Option A) is incorrect. Beta-carotene is the main carotenoid found in green


vegetables, carrots and other yellow and red fruits. It is partially converted to retinol in
humans, but the process is inefficient: 6 g of -carotene give rise to 1 g of retinol.
Retinoic acid (Option C) is incorrect. Retinol and retinoic acid are involved in the control of
cell proliferation and differentiation, and not directly related to retinal function.
Retinol (Option D) is incorrect. Retinol and retinoic acid are involved in the control of cell
proliferation and differentiation, and not directly related to retinal function.
Retinyl phosphate (Option E) is incorrect. Retinyl phosphate is a cofactor in the synthesis of
glycoproteins that contain mannose.
46855

https://mypastest.pastest.com/Secure/TestMe/Browser/429893

1/2

8/12/2016

MyPastest

46855

Next Question

Previous Question

Tag Question

Feedback

End Review

Difficulty: Average
Peer Responses

Session Progress
Responses Correct:

Responses Incorrect:

179

Responses Total:

179

Responses - % Correct:

0%

Blog (https://www.pastest.com/blog) About Pastest (https://www.pastest.com/about-us)


Contact Us (https://www.pastest.com/contact-us) Help (https://www.pastest.com/help)
Pastest 2016

https://mypastest.pastest.com/Secure/TestMe/Browser/429893

2/2

8/12/2016

MyPastest

Back to Filters (/Secure/TestMe/Filter/429893/QA)

Question 8 of 179

You are reviewing a 23-year-old woman on the Acute Medical Unit who has presented to
hospital after a mixed overdose. She had been given a single dose of oral activated charcoal
in the Emergency Department.
Which of the following circumstances would most strongly indicate a need for repeated oral
activated charcoal administration?
A

If a bleeding disorder develops

If it is given within 12 h of ingestion of the poison

In cases of heavy-metal poisoning

In cases where gastric lavage is contraindicated

When the drug circulates through the enterohepatic circulation

Explanation

The answer is When the drug circulates through the enterohepatic circulation
Multiple doses of activated charcoal aid the elimination of some drugs that are prone to
enterohepatic circulation; charcoal adsorbs drug excreted in bile and prevents
reabsorption in the small bowel. Typical examples include aspirin, quinine,
carbamazepine and theophylline.

If a bleeding disorder develops (Option A) is incorrect. Charcoal administration is unrelated


to gastrointestinal bleeding.
If it is given within 12 h of ingestion of the poison (Option B) is incorrect. Multiple doses of
oral activated charcoal may be effective in enhancing drug clearance even up to several days
after ingestion.
In cases of heavy-metal poisoning (Option C) is incorrect. Charcoal does not adsorb metals
including heavy metals, iron or lithium.
In cases where gastric lavage is contraindicated (Option D) is incorrect. Gastric lavage is
irrelevant to multiple dose oral activated charcoal.
46815

https://mypastest.pastest.com/Secure/TestMe/Browser/429893

1/2

8/12/2016

MyPastest

46815

Next Question

Previous Question

Tag Question

Feedback

End Review

Difficulty: Average
Peer Responses

Session Progress
Responses Correct:

Responses Incorrect:

179

Responses Total:

179

Responses - % Correct:

0%

Blog (https://www.pastest.com/blog) About Pastest (https://www.pastest.com/about-us)


Contact Us (https://www.pastest.com/contact-us) Help (https://www.pastest.com/help)
Pastest 2016

https://mypastest.pastest.com/Secure/TestMe/Browser/429893

2/2

8/12/2016

MyPastest

Back to Filters (/Secure/TestMe/Filter/429893/QA)

Question 9 of 179

You are reviewing a 36-year-old man with hypertension and decide to commence -blocker
therapy; you are contemplating the various agents available.
Which of the following -blockers has the largest volume of distribution?
A

Atenolol

Celiprolol

Metoprolol

Nadolol

Sotalol

Explanation

The answer is Metoprolol


The volume of distribution is a theoretical volume that expresses the extent to which a
drug moves out of the circulating compartment to other tissues. The volume of
distribution has units of volume (litres). Metoprolol is the most lipid soluble of the blockers listed above and therefore has the largest volume of distribution; the clinical
significance is that the increased lipid solubility is associated with greater penetration
across the bloodbrain barrier (and also into other tissues), and therefore a greater
incidence of nightmares, headache and other central nervous system adverse effects.
Maximal gastrointestinal absorption of drugs occurs when there is intermediate lipid and
water solubility, so that drugs with greater lipid solubility, although allowing greater
tissue penetration, may be more poorly absorbed.

Atenolol (Option A) is incorrect. Atenolol has minimal lipid solubility, and comparatively
modest volume of distribution.
Celiprolol (Option B) is incorrect. Celiprolol has minimal lipid solubility, and possesses
intrinsic sympathomimetic activity (partial agonist activity) that minimises the occurrence of
resting bradycardia.

https://mypastest.pastest.com/Secure/TestMe/Browser/429893

1/2

8/12/2016

MyPastest

Nadolol (Option D) is incorrect. Nadolol has limited lipid solubility, and comparatively modest
volume of distribution.
Sotalol (Option E) is incorrect. Sotalol has a volume of distribution that approximates the
extracellular fluid compartment.
46845

Next Question

Previous Question

Tag Question

Feedback

End Review

Difficulty: Average
Peer Responses

Session Progress
Responses Correct:

Responses Incorrect:

179

Responses Total:

179

Responses - % Correct:

0%

Blog (https://www.pastest.com/blog) About Pastest (https://www.pastest.com/about-us)


Contact Us (https://www.pastest.com/contact-us) Help (https://www.pastest.com/help)
Pastest 2016

https://mypastest.pastest.com/Secure/TestMe/Browser/429893

2/2

8/12/2016

MyPastest

Back to Filters (/Secure/TestMe/Filter/429893/QA)

Question 10 of 179

You are helping to draw up some prescribing guidelines in the Elderly Medicine department
of your hospital.
Which of the following medications would require greatest caution when prescribed for older
adults?
A

Amoxicillin

Low-dose (75 mg daily) aspirin

Omeprazole

Ranitidine

Triamterene

Explanation

The answer is Triamterene


The most common drugs associated with adverse effects in older adults are diuretics,
digoxin, antihypertensives, anti-inflammatory agents and drugs acting on the central
nervous system (CNS). Age-related changes include declining renal function and
lessened capacity for hepatic drug metabolism. Triamterine is a diuretic that blocks the
epithelial sodium channels in the distal convoluted tubule. Older adults are more
susceptible to the volume depletion and hyponatraemic effects of diuretics.

Amoxicillin (Option A) is incorrect. Amoxicillin may be used safely in older adults; it is subject
to renal elimination, but has a very wide therapeutic window, such that any increased
concentrations are unlikely to cause adverse effects.
Low-dose (75 mg daily) aspirin (Option B) is incorrect. Aspirin may be slightly more likely to
provoke gastric irritation in older adults than younger adults and should be used cautiously.
However, the impact of age on drug response is much less obvious than for diuretics, which
is why triamterene is the preferred answer.
Omeprazole (Option C) is incorrect. Omeprazole may be used safely in older adults.
https://mypastest.pastest.com/Secure/TestMe/Browser/429893

1/2

8/12/2016

MyPastest

Ranitidine (Option D) is incorrect. Ranitidine may be used safely in older adults.


46911

Next Question

Previous Question

Tag Question

Feedback

End Review

Difficulty: Average
Peer Responses

Session Progress
Responses Correct:

Responses Incorrect:

179

Responses Total:

179

Responses - % Correct:

0%

Blog (https://www.pastest.com/blog) About Pastest (https://www.pastest.com/about-us)


Contact Us (https://www.pastest.com/contact-us) Help (https://www.pastest.com/help)
Pastest 2016

https://mypastest.pastest.com/Secure/TestMe/Browser/429893

2/2

8/12/2016

MyPastest

Back to Filters (/Secure/TestMe/Filter/429893/QA)

Question 11 of 179

A 50-year-man presents to the Acute Medical Unit with severe pain and inflammation
affecting his big toe. There is no history of injury or fever, and you suspect a clinical diagnosis
of gout. He has been receiving a number of different medications over many years.
Which of the following long-term treatments is most likely to have contributed to the
occurrence of acute gout?
A

Amlodipine

Aspirin

Chlorthalidone

Indometacin

Losartan

Explanation

The answer is Chlorthalidone


Long-term therapy with a thiazide diuretic and low-dose aspirin results in lessened renal
urate elimination, so that total body urate burden rises and this increases the risk of an
acute attack. The effect of thiazides (eg chlorthalidone) on renal urate elimination is
greater than that of aspirin.

Amlodipine (Option A) is incorrect. Amlodipine has no effect on urate elimination.


Aspirin (Option B) is incorrect. Aspirin interferes with renal tubular secretion of urate so that
the body urate load increases. The effect is less pronounced than for thiazides, which is why
chlorthalidone is the correct answer.
Indometacin (Option D) is incorrect. Indometacin and other non-steroidal anti-inflammatory
drugs (NSAIDs) may be highly effective in treating acute attacks, but do not have any
significant effect on renal elimination of urate.
Losartan (Option E) is incorrect. Losartan reduces serum urate through a uricosuric renal
effect; this has been demonstrated for ACE inhibitors too, and presumably occurs to some
https://mypastest.pastest.com/Secure/TestMe/Browser/429893

1/2

8/12/2016

MyPastest

extent for other angiotensin receptor blockers as well.


46904

Next Question

Previous Question

Tag Question

Feedback

End Review

Difficulty: Average
Peer Responses

Session Progress
Responses Correct:

Responses Incorrect:

179

Responses Total:

179

Responses - % Correct:

0%

Blog (https://www.pastest.com/blog) About Pastest (https://www.pastest.com/about-us)


Contact Us (https://www.pastest.com/contact-us) Help (https://www.pastest.com/help)
Pastest 2016

https://mypastest.pastest.com/Secure/TestMe/Browser/429893

2/2

8/12/2016

MyPastest

Back to Filters (/Secure/TestMe/Filter/429893/QA)

Question 12 of 179

The presence of renal failure may alter the pharmacokinetic handling of many drugs.
Which one of the following pharmacokinetic parameters is most likely to occur as a result of
chronic renal failure?
A

Altered volume of distribution

Increased drug absorption

Increased protein binding

Liver metabolism of drugs

Reduced bioavailability immediately after intravenous drug injection

Explanation

The answer is Altered volume of distribution Renal failure disturbs virtually every kinetic parameter including:
Gastric absorption
Hepatic metabolism of some drugs
Protein binding
Volume of distribution

Increased drug absorption (Option B) is incorrect. In many instances, the presence of chronic
kidney disease is associated with gut oedema and reduced drug absorption.
Increased protein binding (Option C) is incorrect. Patients with renal failure often have coexisting hypoalbuminaemia, resulting in lower protein-binding capacity for drugs.
Liver metabolism of drugs (Option D) is incorrect. Renal metabolism and clearance is often
altered significantly in renal failure, but liver metabolism is not substantially altered.

https://mypastest.pastest.com/Secure/TestMe/Browser/429893

1/2

8/12/2016

MyPastest

Reduced bioavailability immediately after intravenous drug injection (Option E) is incorrect.


The bioavailability of an intravenously administered drug is 100% and does not change in
renal failure.
46730

Next Question

Previous Question

Tag Question

Feedback

End Review

Difficulty: Average
Peer Responses

Session Progress
Responses Correct:

Responses Incorrect:

179

Responses Total:

179

Responses - % Correct:

0%

Blog (https://www.pastest.com/blog) About Pastest (https://www.pastest.com/about-us)


Contact Us (https://www.pastest.com/contact-us) Help (https://www.pastest.com/help)
Pastest 2016

https://mypastest.pastest.com/Secure/TestMe/Browser/429893

2/2

8/12/2016

MyPastest

Back to Filters (/Secure/TestMe/Filter/429893/QA)

Question 13 of 179

A 44-year-old man with a body mass index of 34 kg/m2 was found to have type-2 diabetes
mellitus on routine testing 3 months ago. He was advised a low-energy, weight-reducing diet
and exercise. He has been unable to comply with this, and recent investigations show
postprandial blood glucose concentration is 14.2 mmol/l.
What would be the best drug to initiate as first-line therapy for type-2 diabetes in this
patient?
A

Gliclazide

Insulin

Metformin

Pioglitazone

Sitagliptin

Explanation

The answer is option Metformin


Metformin is the drug of choice in overweight patients. Metformin acts by suppressing
appetite, decreasing gluconeogenesis and increasing the peripheral utilisation of glucose.
Metformin does not cause hypoglycaemia. Lactic acidosis may occur, particularly in
patients with renal impairment. Metformin is normally contraindicated in patients with
acute kidney injury, and chronic renal impairment is a relative contraindication. In
patients who fail to tolerate metformin, either a sulfonylurea (SU) or dipeptidyl peptidase
IV (DPPIV) inhibitor may be considered in addition.

Gliclazide (Option A) is incorrect. Gliclazide is effective in lowering blood glucose by


stimulating pancreatic insulin secretion. Sulfonylureas may promote weight gain, and
metformin is the preferred first-line agent.
Insulin (Option B) is incorrect. Insulin therapy may ultimately be required, but normally this is
reserved for patients who fail to achieve adequate glycaemic control through lifestyle
measures and oral antihyperglycaemic agents.
https://mypastest.pastest.com/Secure/TestMe/Browser/429893

1/2

8/12/2016

MyPastest

Pioglitazone (Option D) is incorrect. Pioglitazone would not be considered a suitable firstline therapy in type-2 diabetes, but may be considered in addition to metformin where
metformin alone is ineffective.
Sitagliptin (Option E) is incorrrect. Sitagliptin is suitable as a second-line agent, normally
given in conjunction with metformin. This may promote weight loss.
46748

Next Question

Previous Question

Tag Question

Feedback

End Review

Difficulty: Average
Peer Responses

Session Progress
Responses Correct:

Responses Incorrect:

179

Responses Total:

179

Responses - % Correct:

0%

Blog (https://www.pastest.com/blog) About Pastest (https://www.pastest.com/about-us)


Contact Us (https://www.pastest.com/contact-us) Help (https://www.pastest.com/help)
Pastest 2016

https://mypastest.pastest.com/Secure/TestMe/Browser/429893

2/2

8/12/2016

MyPastest

Back to Filters (/Secure/TestMe/Filter/429893/QA)

Question 14 of 179

A 67-year-old man who has suffered a previous stroke is admitted with collapse. His drug
history includes the use of dipyridamole. On examination he has a regular pulse of 150 bpm
despite having been given a single dose of bisoprolol 2.5 mg in the Emergency Department.
You elect to try intravenous adenosine to slow his heart down and assess his cardiac rhythm.
Which of the following is most relevant to the use of adenosine in this patient?
A

It can be used in cases of sick-sinus syndrome

It is effective in cardioverting ventricular tachycardia

It may be used after -blockade

Its half-life is decreased by dipyridamole

The half-life of adenosine is around 60 s

Explanation

The answer is It may be used after -blockade


Adenosine is an ultra-short-acting antiarrhythmic drug that is expected to cause very
temporary AV-nodal blockade for a matter of seconds to minutes. However, dipyridamole
prolongs the half-life of adenosine so that its effects may last for up to several minutes.
Adenosine may allow rapid cardioversion of some supraventricular tachycardias,
including those associated with WolffParkinsonWhite syndrome. It has no effect on
ventricular tachycardia, and therefore adenosine administration can help diagnose
tachyarrhythmia. It is contraindicated in patients with second- or third-degree heart
block, sick-sinus syndrome, or patients receiving verapamil owing to the risk of
precipitating complete heart block. Adverse effects include transient facial flushing and
bronchospasm, and a sense of profound anxiety; patients should be warned that this may
occur in advance. These effects normally last only a matter of seconds, but may be more
prolonged and severe in patients receiving dipyridamole.

It can be used in cases of sick-sinus syndrome (Option A) is incorrect. There is a risk that
adenosine may provoke heart block in some patients with sick-sinus syndrome.
https://mypastest.pastest.com/Secure/TestMe/Browser/429893#Top

1/2

8/12/2016

MyPastest

It is effective in cardioverting ventricular tachycardia (Option B) is incorrect. Adenosine has


no effect on ventricular tachycardia but is effective for many supraventricular tachycardias,
and so can be used to help distinguish these.
Its half-life is decreased by dipyridamole (Option D) is incorrect. Dipyridamole significantly
increases the half-life and duration of action of adenosine.
The half-life of adenosine is around 60 s (Option E) is incorrect. The half-life of adenosine is
around 35 s.
46880

Next Question

Previous Question

Tag Question

Feedback

End Review

Difficulty: Average
Peer Responses

Session Progress
Responses Correct:

Responses Incorrect:

179

Responses Total:

179

Responses - % Correct:

0%

Blog (https://www.pastest.com/blog) About Pastest (https://www.pastest.com/about-us)


Contact Us (https://www.pastest.com/contact-us) Help (https://www.pastest.com/help)
Pastest 2016

https://mypastest.pastest.com/Secure/TestMe/Browser/429893#Top

2/2

8/12/2016

MyPastest

Back to Filters (/Secure/TestMe/Filter/429893/QA)

Question 15 of 179

A 56-year-old woman has been receiving warfarin treatment after recurrent pulmonary
emboli.
Which of the following drugs is most likely to cause an increased INR?
A

Azathioprine

Griseofulvin

Phenobarbital

Rifampicin

Vitamin K

Explanation
The answer is Azathioprine
Warfarin interferes with the hepatic synthesis of vitamin-K-dependent clotting factors,
leading to depression of the activity of factors II, VII, IX, X, protein C and protein S in a
dose-dependent manner. Commonly used drugs that may lead to an increased INR
include enzyme inhibitors, including macrolide and quinolone antibiotics, metronidazole,
allopurinol and cimetidine. Other drugs capable of increasing INR in patients on warfarin
therapy include cephalosporins, azathioprine and testosterone derivatives.

Griseofulvin (Option B) is incorrect. Griseofulvin is an antifungal with powerful enzyme


inducing properties that lowers warfarin concentrations and INR.
Phenobarbital (Option C) is incorrect. Phenobarbital is an antiepileptic drug with powerful
enzyme inducer that lowers warfarin concentrations and INR.
Rifampicin (Option D) is incorrect. Rifampicin is an antimicrobial and powerful enzyme
inducer that lowers warfarin concentrations and INR.
Vitamin K (Option E) is incorrect. Vitamin K is a competitive inhibitor of warfarin.
46839

https://mypastest.pastest.com/Secure/TestMe/Browser/429893#Top

1/2

8/12/2016

MyPastest

Next Question

Previous Question

Tag Question

Feedback

End Review

Difficulty: Average
Peer Responses

Session Progress
Responses Correct:

Responses Incorrect:

179

Responses Total:

179

Responses - % Correct:

0%

Blog (https://www.pastest.com/blog) About Pastest (https://www.pastest.com/about-us)


Contact Us (https://www.pastest.com/contact-us) Help (https://www.pastest.com/help)
Pastest 2016

https://mypastest.pastest.com/Secure/TestMe/Browser/429893#Top

2/2

8/12/2016

MyPastest

Back to Filters (/Secure/TestMe/Filter/429893/QA)

Question 16 of 179

An 18-year-old man is brought to the Emergency Department at 0300 h by two of his friends.
He had been in a local nightclub but complained of central chest pain followed by a collapse.
On examination he is agitated, heart rate 130 bpm, blood pressure 156/92 mmHg, and pupils
are dilated. You suspect possible recreational drug use.
Which one of the following substances is most likely to explain this patients clinical features?
A

Alcohol

Amitriptyline

Cocaine

Lysergic acid diethylamide

Morphine

Explanation

The answer is Cocaine


Cocaine reduces the reuptake of dopamine into neurones by inhibiting the dopaminereuptake transporter, and provokes intense sympathetic activation and vasoconstriction
medicated via -adrenoceptors. Cocaine may provoke a hypertensive crisis, and is
associated with development of non-cardiac chest pain as well as myocardial infarction
and stroke. Cocaine overdose may be rapidly fatal due to arrhythmias, seizures or
myocardial infarction. Some similarities are seen with amphetamines and novel
recreational drugs in terms of agitation, tachycardia, psychosis and dilated pupils;
however, vasospasm and risk of hypertensive crises and myocardial infarction are less
than after cocaine.

Alcohol (Option A) is incorrect. Alcohol intoxication produces typical effects of acute


sedativehypnotic drug overdose, vasodilatation, tachycardia and gastrointestinal irritation.
Amitriptyline (Option B) is incorrect. Amitriptyline may cause tachycardia and dilated pupils
through anticholinergic mechanisms. However, significant toxicity is associated with reduced
conscious level, and it is not associated with euphoria or prone to abuse.
https://mypastest.pastest.com/Secure/TestMe/Browser/429893#Top

1/2

8/12/2016

MyPastest

Lysergic acid diethylamide (LSD) (Option D) is incorrect. LSD produces a series of somatic,
perceptual and psychological effects. Dizziness, weakness and tremors occur, along with
blurring of vision, hallucinations, impaired memory, poor judgement and altered mood. It
would not be expected to cause pupillary dilatation.
Morphine (Option E) is incorrect. Opioid overdose causes miosis, slurred speech,
disorientation and respiratory depression, and reduced conscious level.
46784

Next Question

Previous Question

Tag Question

Feedback

End Review

Difficulty: Average
Peer Responses

Session Progress
Responses Correct:

Responses Incorrect:

179

Responses Total:

179

Responses - % Correct:

0%

Blog (https://www.pastest.com/blog) About Pastest (https://www.pastest.com/about-us)


Contact Us (https://www.pastest.com/contact-us) Help (https://www.pastest.com/help)
Pastest 2016

https://mypastest.pastest.com/Secure/TestMe/Browser/429893#Top

2/2

8/12/2016

MyPastest

Back to Filters (/Secure/TestMe/Filter/429893/QA)

Question 17 of 179

A 27-year-old woman is taking the oral contraceptive pill and has a history of epilepsy. She
finds that sodium valproate is causing her to put on weight and she is keen to switch to an
alternative medication.
Which one of the following would be the most appropriate medication for her?
A

Carbamazepine

Clonazepam

Lamotrigine

Phenytoin

Topiramate

Explanation

The answer is Lamotrigine


Lamotrigine is the most appropriate choice here because it does not interfere with the
effectiveness of the oral contraceptive pill. Although limited data in pregnancy are
available, lamotrigine has a favourable profile in terms of congenital malformations
compared with sodium valproate.

Carbamazepine (Option A) is incorrect. Carbamazepine is a powerful hepatic enzyme inducer


and is expected to decrease effectiveness of the oral contraceptive pill and risk unintended
pregnancy.
Clonazepam (Option B) is incorrect. Clonazepam is less likely to be an effective antiepileptic
medication, and may cause sedation.
Phenytoin (Option D) is incorrect. Phenytoin may also cause weight gain, and is a powerful
enzyme inducer so that the oral contraceptive pill will be less effective. Phenytoin is a
recognised cause of congenital malformations in around 5%.
Topiramate (Option E) is incorrect. Topiramate may decrease oestrogen levels and thus
reduce pill effectiveness. Topiramate, when used as part of a polytherapy regimen, may be
https://mypastest.pastest.com/Secure/TestMe/Browser/429893#Top

1/2

8/12/2016

MyPastest

associated with a ten-fold risk of cleft lip compared to the background population.
46997

Next Question

Previous Question

Tag Question

Feedback

End Review

Difficulty: Average
Peer Responses

Session Progress
Responses Correct:

Responses Incorrect:

179

Responses Total:

179

Responses - % Correct:

0%

Blog (https://www.pastest.com/blog) About Pastest (https://www.pastest.com/about-us)


Contact Us (https://www.pastest.com/contact-us) Help (https://www.pastest.com/help)
Pastest 2016

https://mypastest.pastest.com/Secure/TestMe/Browser/429893#Top

2/2

8/12/2016

MyPastest

Back to Filters (/Secure/TestMe/Filter/429893/QA)

Question 18 of 179

You review a 44-year-old woman in an outpatient clinic for review of her hypertension. She
had recently been started on hydralazine on the advice of the renal specialist team. The
patient has read the information leaflet indicating that drug-induced lupus is a recognised
complication.
Which of the following factors would most strongly increase the risk of her developing
hydralazine-induced lupus?
A

Caucasian ethnicity

dsDNA antibodies are positive

Family history of systemic lupus erythematosus

HLA-B6 genotype

Rapid acetylator status is a risk factor

Explanation

The answer is Caucasian ethnicity


Systemic lupus erythematosus (SLE) is more common in Afro-Caribbeans and much
more common in women. In contrast, drug-induced lupus occurs more commonly in
Caucasians, with men and women affected equally. The reaction is drug dose- and
concentration-dependent. Other drugs characteristically associated with drug-induced
lupus are hydralazine, procainamide and isoniazid. Most cases resolve spontaneously
after removal of the causative drug.

dsDNA antibodies are positive (Option B) is incorrect. These are characteristically increased
in SLE but are normally undetectable in drug-induced lupus. Drug-induced lupus may be
associated with a positive antinuclear factor and antihistone antibody titre.
Family history of systemic lupus erythematosus (Option C) is incorrect. Positive family
history of SLE may pose a slightly increased risk of SLE and drug-induced lupus, but this is a
less preferred answer owing to the very weak association.

https://mypastest.pastest.com/Secure/TestMe/Browser/429893#Top

1/2

8/12/2016

MyPastest

HLA-B6 genotype (Option D) is incorrect. Drug-induced lupus occurs more commonly in


patients with HLA-DR4 genotype.
Rapid acetylator status is a risk factor (Option E) is incorrect. Hydralazine is subject to
acetylation; slow acetylator phenotype confers a higher risk of drug-induced lupus,
presumably because slower metabolism results in higher hydralazine concentrations.
45809

Next Question

Previous Question

Tag Question

Feedback

End Review

Difficulty: Average
Peer Responses

Session Progress
Responses Correct:

Responses Incorrect:

179

Responses Total:

179

Responses - % Correct:

0%

Blog (https://www.pastest.com/blog) About Pastest (https://www.pastest.com/about-us)


Contact Us (https://www.pastest.com/contact-us) Help (https://www.pastest.com/help)
Pastest 2016

https://mypastest.pastest.com/Secure/TestMe/Browser/429893#Top

2/2

8/12/2016

MyPastest

Back to Filters (/Secure/TestMe/Filter/429893/QA)

Question 19 of 179

You are reviewing a 53-year-old female patient with long-standing diabetes and gastric
dysmotility. She has read some information about cisapride and has asked you for advice
concerning this medication.
Which of the following statements is most accurate regarding cisapride?
A

Decreases prothrombin time in patients receiving warfarin

Delays gastric emptying time

Exacerbation of symptoms of heartburn

Relaxation of colonic musculature

QT prolongation on the ECG

Explanation

The answer is QT prolongation on the ECG


Cisapride is a prokinetic drug that reduces gastric emptying time. It interacts with
anticoagulants and increases the prothrombin time and bleeding risk. It increases the
motility of the entire gastrointestinal tract, and enhances gastric emptying. QT
abnormalities are reported with therapy, especially when given with erythromycin or
antifungal drugs like ketoconazole (enzyme inhibitor); cisapride has been associated with
fatal arrhythmias and its use has been restricted. Anticholinergics antagonise the effects
of cisapride.

Decreases prothrombin time in patients receiving warfarin (Option A) is incorrect. It may


increase prothrombin time and international normalised ratio (INR).
Delays gastric emptying time (Option B) is incorrect. It reduces gastric emptying time.
Exacerbation of symptoms of heartburn (Option C) is incorrect. It may alleviate heartburn in
patients with delayed gastric emptying.
Relaxation of colonic musculature (Option D) is incorrect. It increases motility and may
increase colonic tone.
46611

https://mypastest.pastest.com/Secure/TestMe/Browser/429893#Top

1/2

8/12/2016

MyPastest

46611

Next Question

Previous Question

Tag Question

Feedback

End Review

Difficulty: Average
Peer Responses

Session Progress
Responses Correct:

Responses Incorrect:

179

Responses Total:

179

Responses - % Correct:

0%

Blog (https://www.pastest.com/blog) About Pastest (https://www.pastest.com/about-us)


Contact Us (https://www.pastest.com/contact-us) Help (https://www.pastest.com/help)
Pastest 2016

https://mypastest.pastest.com/Secure/TestMe/Browser/429893#Top

2/2

8/12/2016

MyPastest

Back to Filters (/Secure/TestMe/Filter/429893/QA)

Question 20 of 179

You review a 78-year-old man who was admitted with hypotension and tachycardia and is
found to have atrial fibrillation. The admitting FY2 diagnosed sepsis and commenced him on
gentamicin therapy. He is on multiple drugs, including fluoxetine for depression, bumetanide
for fluid overload, amiodarone for atrial fibrillation and diazepam to help him sleep. You wish
to adjust some of his existing medications and add new treatments.
Which of the following medications if stopped or started would take longest for the patient
to respond to?
A

Amiodarone

Bumetanide

Diazepam

Fluoxetine

Gentamicin

Explanation

The answer is Amiodarone


Amiodarone has by far the longest half-life of these agents, at around 2530 days.
Initiation of therapy requires a loading regimen over the first few weeks of treatment,
and cessation may take several months before drug concentrations have fallen to an
insignificant level. The half-life is dependent upon lipid solubility and wide distribution of
the drug to extravascular tissues. With wide volume of distribution and extensive tissue
binding there is slow renal or hepatic drug clearance.

Bumetanide (Option B) is incorrect. Bumetanide has a short half-life, around 1 h, so that the
effects are comparatively short-lived.
Diazepam (Option C) is incorrect. Diazepam is highly lipid soluble, which helps it cross the
bloodbrain barrier; the plasma half-life is around 43 h, although the biological effects may
persist for longer due to persistence in the brain tissue.

https://mypastest.pastest.com/Secure/TestMe/Browser/429893#Top

1/2

8/12/2016

MyPastest

Fluoxetine (Option D) is incorrect. Fluoxetine is very lipid-soluble and has a comparatively


long half-life of around 50 h. Irrespective of the half-life, the biological response to starting
or stopping therapy may take 23 weeks (that is, a dissociation between the pharmacokinetic
profile and pharmacodynamic actions).
Gentamicin (Option E) is incorrect. Gentamicin is water soluble and clearance is highly
dependent upon renal excretion; in patients with normal renal function the half-life of
gentamicin is around 23 h.
46887

Next Question

Previous Question

Tag Question

Feedback

End Review

Difficulty: Average
Peer Responses

Session Progress
Responses Correct:

Responses Incorrect:

179

Responses Total:

179

Responses - % Correct:

0%

Blog (https://www.pastest.com/blog) About Pastest (https://www.pastest.com/about-us)


Contact Us (https://www.pastest.com/contact-us) Help (https://www.pastest.com/help)
Pastest 2016

https://mypastest.pastest.com/Secure/TestMe/Browser/429893#Top

2/2

8/12/2016

MyPastest

Back to Filters (/Secure/TestMe/Filter/429893/QA)

Question 21 of 179

A 58-year-old man attends the Nephrology Clinic for review. He admits at the end of his
consultation that he suffers from erectile dysfunction. You are considering prescribing
vardenafil for him. He is taking multiple agents for the treatment of hypertension.
Which one of the following agents would be most likely to provoke adverse effects in a
patient receiving vardenafil?
A

Atenolol

Bendrofluazide

Carbamazepine

Nicorandil

Ramipril

Explanation

The answer is Nicorandil


Nicorandil is a potassium channel activator recognised to increase nitric oxide (NO), and
is contraindicated for prescription in conjunction with vardenafil. Other drugs that are
contraindicated include nitrate donors such as isosorbide dinitrate and isosorbide
mononitrate; use of these agents with vardenafil may lead to a precipitous fall in blood
pressure and lead to profound hypotension. Similar adverse effects are likely to occur
with the use of amyl nitrate, which is used primarily for recreational purposes.

Atenolol (Option A) is incorrect. There is no significant interaction between atenolol and


vardenafil.
Bendrofluazide (Option B) is incorrect. There is no significant interaction between
bendrofluazide and vardenafil.
Carbamazepine (Option C) is incorrect. There is no significant interaction between
carbamazepine and vardenafil.

https://mypastest.pastest.com/Secure/TestMe/Browser/429893#Top

1/2

8/12/2016

MyPastest

Ramipril (Option E) is incorrect. Both vardenafil and ACE inhibitors may provoke postural
hypotension and should be used with caution; the effect is less severe than that of nicorandil
or nitric oxide donors, hence nicorandil is the preferred answer.
46975

Next Question

Previous Question

Tag Question

Feedback

End Review

Difficulty: Average
Peer Responses

Session Progress
Responses Correct:

Responses Incorrect:

179

Responses Total:

179

Responses - % Correct:

0%

Blog (https://www.pastest.com/blog) About Pastest (https://www.pastest.com/about-us)


Contact Us (https://www.pastest.com/contact-us) Help (https://www.pastest.com/help)
Pastest 2016

https://mypastest.pastest.com/Secure/TestMe/Browser/429893#Top

2/2

8/12/2016

MyPastest

Back to Filters (/Secure/TestMe/Filter/429893/QA)

Question 22 of 179

A 19-year-old woman presents to the Emergency Department. She had a row with her
boyfriend and took a handful of her mothers tablets, which she found in a bottle. Her mother
has multiple health problems, including hypertension, reflux oesophagitis, dyslipidaemia and
night cramps. The patient complains of involuntary spasms affecting the left side of her neck,
particularly the sternocleidomastoid muscle, and feels very upset. On examination her blood
pressure is 155/90 mmHg.
Which one of the following drugs is she most likely to have taken?
A

Metoclopramide

Omeprazole

Quinine

Ramipril

Simvastatin

Explanation

The answer is Metoclopramide


This woman has presented with acute dystonia, a condition known to be associated with
use of antipsychotics and antiemetics, particularly metoclopramide and
prochlorperazine. Onset is usually within 12 h of ingestion of the causative agent, and
procyclidine is the agent of choice used to treat it.

Omeprazole (Option B) is incorrect. Omeprazole may cause headache and nausea in


overdose.
Quinine (Option C) is incorrect. Quinine may cause blindness, cardiac arrhythmia and
neurotoxicity.
Ramipril (Option D) is incorrect. ACE inhibitors may cause profound hypotension, flushing
and headache in overdose (in contrast to the notional flat dose response associated with
therapeutic doses).
https://mypastest.pastest.com/Secure/TestMe/Browser/429893#Top

1/2

8/12/2016

MyPastest

Simvastatin (Option E) is incorrect. Simvastatin may cause myositis, especially with chronic
use, but overdose is not associated with dystonia.
46977

Next Question

Previous Question

Tag Question

Feedback

End Review

Difficulty: Average
Peer Responses

Session Progress
Responses Correct:

Responses Incorrect:

179

Responses Total:

179

Responses - % Correct:

0%

Blog (https://www.pastest.com/blog) About Pastest (https://www.pastest.com/about-us)


Contact Us (https://www.pastest.com/contact-us) Help (https://www.pastest.com/help)
Pastest 2016

https://mypastest.pastest.com/Secure/TestMe/Browser/429893#Top

2/2

8/12/2016

MyPastest

Back to Filters (/Secure/TestMe/Filter/429893/QA)

Question 23 of 179

A 62-year-old man with a diagnosis of subacute bacterial endocarditis is admitted to the


Cardiology Ward. He is treated with a combination of IV benzylpenicillin and gentamicin.
Other past history of note includes hypertension and chronic renal failure. On examination his
blood pressure is 152/82 mmHg and his pulse is 82 beats per minute and regular.
Investigations:
Hb

10.1 g/dl

White cell count 13.1 109/l


Platelets

202 109/l

Na+

139 mmol/l

K+

5.2 mmol/l

Creatinine

281 mol/l

The laboratory tells you that his trough gentamicin level is too high; the 12-hdosing regimen
had been calculated based on his body weight.
Why is the trough level likely to be too high?
A

Changes in the volume of distribution

Prolongation of the half-life

Decreased first-pass metabolism

Decreased protein binding

Increased bioavailability

Explanation
Altered gentamicin pharmacokinetics

https://mypastest.pastest.com/Secure/TestMe/Browser/429893#Top

1/2

8/12/2016

MyPastest

The usual half-life of gentamicin is between 2 and 3 h, although this is considerably


prolonged in patients with renal failure
At steady state around 70% of any given dose of gentamicin is recovered in the urine
24 h later, although some accumulation of gentamicin does occur, particularly in the
kidney
Particular care should be taken when administering gentamicin in conjunction with loop
diuretics because of the risk of exacerbating renal and ototoxicity
The problem of trough levels that are too high can be overcome in this case by
lengthening the dosing interval
22424

Next Question

Previous Question

Tag Question

Feedback

End Review

Difficulty: Average
Peer Responses

Session Progress
Responses Correct:

Responses Incorrect:

179

Responses Total:

179

Responses - % Correct:

0%

Blog (https://www.pastest.com/blog) About Pastest (https://www.pastest.com/about-us)


Contact Us (https://www.pastest.com/contact-us) Help (https://www.pastest.com/help)
Pastest 2016

https://mypastest.pastest.com/Secure/TestMe/Browser/429893#Top

2/2

8/12/2016

MyPastest

Back to Filters (/Secure/TestMe/Filter/429893/QA)

Question 24 of 179

A 79-year-old man is admitted to hospital with palpitations. A resting ECG shows atrial
fibrillation with a rate 107/min. You decide to prescribe digoxin therapy.
Which of the following most accurately describes the pharmacological mechanism of action
of digoxin?
A

Inhibition of sodium-potassium ATPase

Opening of calcium channels

Release of calcium from the sarcoplasmic reticulum

Stimulation of membrane phospholipase C

Stimulation of myosin ATPase

Explanation

The answer is Inhibition of sodium-potassium ATPase


The key pharmacological mechanism is inhibition of Na+/K+ ATPase, which is located in
the sarcolemmal membrane. This causes accumulation of intracellular sodium and
calcium by way of the sodiumcalcium exchange mechanism; intracellular potassium
concentrations decrease whereas extracellular potassium concentrations increase.
Digitalis glycosides increase the contractility of the heart (positively inotropic) and
decrease the resting heart rate (negatively chronotropic) and delay AV nodal
conduction.

Opening of calcium channels (Option B) is incorrect. Intracellular calcium concentrations


increase, but there is no direct action of digoxin on cardiac calcium channels.
Release of calcium from the sarcoplasmic reticulum (Option C) is incorrect. Calcium
concentrations increase due to sodiumcalcium exchange, rather than release from the
sarcoplasmic reticulum.
Stimulation of membrane phospholipase C (Option D) is incorrect. Digoxin is not known to
exert a direct effect on phospholipase C. Many other receptor systems act via phospholipase
https://mypastest.pastest.com/Secure/TestMe/Browser/429893#Top

1/2

8/12/2016

MyPastest

C, including 5-HT2 serotonergic, 1 adrenergic, calcitonin, H1 histamine, glutamate receptors,


and M1, M3 and M5 muscarinic receptors.
Stimulation of myosin ATPase (Option E) is incorrect. The effect is inhibition rather than
stimulation of the Na+/K+ ATPase.
46596

Next Question

Previous Question

Tag Question

Feedback

End Review

Difficulty: Average
Peer Responses

Session Progress
Responses Correct:

Responses Incorrect:

179

Responses Total:

179

Responses - % Correct:

0%

Blog (https://www.pastest.com/blog) About Pastest (https://www.pastest.com/about-us)


Contact Us (https://www.pastest.com/contact-us) Help (https://www.pastest.com/help)
Pastest 2016

https://mypastest.pastest.com/Secure/TestMe/Browser/429893#Top

2/2

8/12/2016

MyPastest

Back to Filters (/Secure/TestMe/Filter/429893/QA)

Question 25 of 179

A 65-year-old man has multiple medical disorders including heart failure, transient ischaemic
attacks and non-insulin-dependent diabetes mellitus. He has developed profuse and watery
diarrhoea for the past 4 days after eating a sea-food meal. Blood tests reveal urea of 15.2
mmol/l and a creatinine of 345 mmol/l.
Which one of his existing medications would be most important to stop?
A

Aspirin

Bendroflumethiazide

Digoxin

Glibenclamide

Irbesartan

Explanation

The answer is Irbesartan


The clinical scenario indicates acute gastroenteritis and renal failure. In healthy
individuals, activation of the reninangiotensinaldosterone system preserves glomerular
filtration pressure and renal function. However, in patients receiving ACE inhibitors or
angiotensin receptor antagonists this mechanism is blocked so that renal failure
develops. In cases of dehydration or sepsis, the ACE inhibitor or angiotensin receptor
blocker (eg irbesartan) should be temporarily discontinued.

Aspirin (Option A) is incorrect. Aspirin, even in small dosages, can contribute to renal
impairment in severe dehydration owing to its effects on renal prostaglandin synthesis.
However, this effect is much smaller than that of an ACE inhibitor or angiotensin receptor
blocker, hence irbesartan is the preferred answer.
Bendroflumethiazide (Option B) is incorrect. Diuretic treatment will exacerbate dehydration
and haemodynamic consequences. Although this may also need to be discontinued, it is less
hazardous than an ACE inhibitor or angiotensin receptor blocker, hence irbesartan is the
preferred answer.
https://mypastest.pastest.com/Secure/TestMe/Browser/429893#Top

1/2

8/12/2016

MyPastest

Digoxin (Option C) is incorrect. Digoxin may accumulate in renal impairment, so that a dose
reduction could be anticipated or drug concentrations measured.
Glibenclamide (Option D) is incorrect. Glibenclamide may accumulate in renal impairment so
that dose reduction may be needed. Blood glucose can be monitored and treatment dose
adjusted accordingly.
46966

Next Question

Previous Question

Tag Question

Feedback

End Review

Difficulty: Average
Peer Responses

Session Progress
Responses Correct:

Responses Incorrect:

179

Responses Total:

179

Responses - % Correct:

0%

Blog (https://www.pastest.com/blog) About Pastest (https://www.pastest.com/about-us)


Contact Us (https://www.pastest.com/contact-us) Help (https://www.pastest.com/help)
Pastest 2016

https://mypastest.pastest.com/Secure/TestMe/Browser/429893#Top

2/2

8/12/2016

MyPastest

Back to Filters (/Secure/TestMe/Filter/429893/QA)

Question 26 of 179

You review a 72-year-old man with a history of dementia. He is becoming increasingly hard to
manage at home, is agitated and difficult, and is suffering from delusions that the members
of his family who care for him are trying to poison him. You decide to add risperidone to his
existing medications.
For which one of the following receptors does risperidone have the highest affinity?
A

5HT-2 receptors

5HT-3 receptors

1-Adrenergic receptors

D1 receptors

H2 receptors

Explanation

The answer is 5HT-2 receptors


Risperidone is a novel antipsychotic belonging to the benzisoxazole derivative class; it is
an antagonist with high affinity for D2 and 5HT-2 receptors in the central nervous system.
To a lesser extent, risperidone also blocks 1-adrenergic receptors, H1-histaminergic and
2-adrenergic receptors. Common adverse effects include insomnia, agitation and
anxiety. Risperidone may also lead to impaired glucose tolerance, although the rate of
occurrence is similar to other antipsychotics.

5HT-3 receptors (Option B) is incorrect. Ondansetron is a 5HT-3 antagonist.


1-Adrenergic receptors (Option C) is incorrect. Risperidone is an antagonist at 1-adrenergic
receptors, but this is less important than its effects on 5HT-2 receptors.
D1 receptors (Option D) is incorrect. Blockade of D1 receptors causes parkinsonism.
H2 receptors (Option E) is incorrect. Risperidone is an antagonist at H1-histaminergic
receptors.
46989

https://mypastest.pastest.com/Secure/TestMe/Browser/429893#Top

1/2

8/12/2016

MyPastest

Next Question

Previous Question

Tag Question

Feedback

End Review

Difficulty: Average
Peer Responses

Session Progress
Responses Correct:

Responses Incorrect:

179

Responses Total:

179

Responses - % Correct:

0%

Blog (https://www.pastest.com/blog) About Pastest (https://www.pastest.com/about-us)


Contact Us (https://www.pastest.com/contact-us) Help (https://www.pastest.com/help)
Pastest 2016

https://mypastest.pastest.com/Secure/TestMe/Browser/429893#Top

2/2

8/12/2016

MyPastest

Back to Filters (/Secure/TestMe/Filter/429893/QA)

Question 27 of 179

A 25-year-old woman on the oral contraceptive pill is diagnosed with partial epilepsy. Over
the past 3 months, she has had a number of seizures requiring treatment. She advises you
that she wishes to continue the contraceptive pill as she has no plans to become pregnant.
Which of the following measures are appropriate?
A

Change to a progestogen-only contraceptive pill

Continue oral contraceptive, and commence on ethosuximide

Continue oral contraceptive, and commence on lamotrigine

Continue oral contraceptive, and commence on phenytoin

Stop the oral contraceptive, as this may be precipitating fits, and suggest using
barrier contraceptive methods

Explanation

The answer is Continue oral contraceptive, and commence on lamotrigine


Lamotrigine is a suitable first-line treatment for partial epilepsy, and does not
significantly alter oestrogen metabolism.

Change to a progestogen-only contraceptive pill (Option A) is incorrect. This would not


address the need for epilepsy treatment.
Continue oral contraceptive, and commence on ethosuximide (Option B) is incorrect.
Ethosuximide is a suitable treatment for absence and myoclonic attacks in children, but
would be less effective for partial seizures in adults.
Continue oral contraceptive, and commence on phenytoin (Option D) is incorrect. Phenytoin
induces liver enzymes, thereby increasing oestrogen breakdown and reducing the
effectiveness of oestrogen-containing contraceptives. Where the combined contraceptive pill
is used in conjunction with phenytoin, the contraceptive should contain high-dose oestrogen,
50 g ethinylestradiol or more (carbamazepine is also a potent enzyme inducer and should
not be used in combination with the pill).
https://mypastest.pastest.com/Secure/TestMe/Browser/429893#Top

1/2

8/12/2016

MyPastest

Stop the oral contraceptive, as this may be precipitating fits, and suggest using barrier
contraceptive methods (Option E) is incorrect. The oral contraceptive pill is not recognised
as a cause of seizures.
46897

Next Question

Previous Question

Tag Question

Feedback

End Review

Difficulty: Average
Peer Responses

Session Progress
Responses Correct:

Responses Incorrect:

179

Responses Total:

179

Responses - % Correct:

0%

Blog (https://www.pastest.com/blog) About Pastest (https://www.pastest.com/about-us)


Contact Us (https://www.pastest.com/contact-us) Help (https://www.pastest.com/help)
Pastest 2016

https://mypastest.pastest.com/Secure/TestMe/Browser/429893#Top

2/2

8/12/2016

MyPastest

Back to Filters (/Secure/TestMe/Filter/429893/QA)

Question 28 of 179

You are considering treatment options in a 36-year-old woman with thyrotoxicosis, and are
considering the relative merits of propylthiouracil versus carbimazole treatments.
Which of the following factors best characterises the distinguishing characteristics of
propylthiouracil therapy?
A

Inhibits conversion of thyroxine to tri-iodothyronine

Inhibits organification of iodine at the thyroid gland

Inhibits release of preformed thyroid hormone

It is a thiourea derivative, unlike carbimazole

It is more potent than carbimazole

Explanation

The answer is Inhibits conversion of thyroxine to tri-iodothyronine


Propylthiouracil (PTU) and carbimazole are both derivatives of thiourea with similar
chemical structure, mechanisms of action and adverse effects. Both are capable of
inhibiting organification of iodine at the thyroid gland. Neither drug affects iodine
trapping and neither drug inhibits the release of preformed thyroid hormone. A key
distinction is that PTU, but not carbimazole, inhibits iodination of thyroxine to triiodothyronine. This may give PTU a modest therapeutic advantage because it reduces
the proportion of active thyroid hormone as well as total T4. Both PTU and carbimazole
are excreted in very small quantities in breast milk, so breast feeding is not advised with
either. Thiourea derivatives have several side-effects, including a maculopapular rash,
hepatocellular damage and vasculitis. The most serious side-effect of both agents is
agranulocytosis, although it is more common to see a fall in, rather than total absence of,
white cells.

Inhibits organification of iodine at the thyroid gland (Option B) is incorrect. Both PTU and
carbimazole are capable of this.

https://mypastest.pastest.com/Secure/TestMe/Browser/429893#Top

1/2

8/12/2016

MyPastest

Inhibits release of preformed thyroid hormone (Option C) is incorrect. Neither drug inhibits
release of preformed thyroid hormone.
It is a thiourea derivative, unlike carbimazole (Option D) is incorrect. Both drugs are thiourea
derivatives.
It is more potent than carbimazole (Option E) is incorrect. Carbimazole is approximately 15
times more potent than PTU.
46631

Next Question

Previous Question

Tag Question

Feedback

End Review

Difficulty: Average
Peer Responses

Session Progress
Responses Correct:

Responses Incorrect:

179

Responses Total:

179

Responses - % Correct:

0%

Blog (https://www.pastest.com/blog) About Pastest (https://www.pastest.com/about-us)


Contact Us (https://www.pastest.com/contact-us) Help (https://www.pastest.com/help)
Pastest 2016

https://mypastest.pastest.com/Secure/TestMe/Browser/429893#Top

2/2

8/12/2016

MyPastest

Back to Filters (/Secure/TestMe/Filter/429893/QA)

Question 29 of 179

You are helping at a pulmonary hypertension follow-up clinic, and are reviewing a 57-year-old
woman treated with bosentan.
Which of the following statements best represents the pharmacological actions of bosentan?
A

Causes a dose-dependent increase in systemic vascular resistance

Haemoglobin concentrations typically rise due to haemoconcentration

It is a competitive antagonist of the ETA but not ETB receptor

May be used safely in pregnancy

Patients should be monitored for development of liver toxicity

Explanation

The answer is Patients should be monitored for development of liver toxicity


Bosentan is a competitive antagonist at both endothelin-A (ETA) and endothelin-B (ETB)
receptors, leading to falls in both pulmonary and systemic vascular resistances without
an increase in heart rate. It is one of the few treatments that show efficacy in patients
with pulmonary arterial hypertension. Common unwanted effects include flushing,
hypotension, dyspepsia and fatigue. The most serious adverse effect is dose-dependent
hepatotoxicity, and it is therefore contraindicated in patients with moderate to severe
liver disease. Hepatotoxicity normally occurs within the first 34 months of treatment,
and patients should have liver function tests (LFTs) monitored.

Causes a dose-dependent increase in systemic vascular resistance (Option A) is incorrect. It


causes a dose-dependent fall in systemic vascular resistance.
Haemoglobin concentrations typically rise due to haemoconcentration (Option B) is
incorrect. Haemoglobin concentrations can fall by up to 1 g/dl during bosentan treatment.
It is a competitive antagonist of the ETA but not ETB receptor (Option C) is incorrect.
Bosentan blocks both ETA and ETB receptor subtypes.

https://mypastest.pastest.com/Secure/TestMe/Browser/429893#Top

1/2

8/12/2016

MyPastest

May be used safely in pregnancy (Option D) is incorrect. Bosentan is teratogenic and it is


contraindicated in pregnancy.
46494

Next Question

Previous Question

Tag Question

Feedback

End Review

Difficulty: Average
Peer Responses

Session Progress
Responses Correct:

Responses Incorrect:

179

Responses Total:

179

Responses - % Correct:

0%

Blog (https://www.pastest.com/blog) About Pastest (https://www.pastest.com/about-us)


Contact Us (https://www.pastest.com/contact-us) Help (https://www.pastest.com/help)
Pastest 2016

https://mypastest.pastest.com/Secure/TestMe/Browser/429893#Top

2/2

8/12/2016

MyPastest

Back to Filters (/Secure/TestMe/Filter/429893/QA)

Question 30 of 179

You see a 27-year-old Asian man in the respiratory clinic who has been receiving
antituberculous therapy for the past 3 months. He is complaining of tingling and numbness in
his hands and feet, and on examination you find that he has impaired light touch sensation in
a glove and stocking distribution.
Which of the following medications is most likely to account for this problem?
A

Ethambutol

Isoniazid

Pyrazinamide

Rifampicin

Streptomycin

Explanation

The answer is Isoniazid


Isoniazid is a well-known cause of peripheral neuropathy. Isoniazid is subject to
acetylation; slow acetylators are at greater risk of developing peripheral neuropathy.
Other risk factors include the presence of malnutrition, alcoholism, diabetes and HIV
infection. Pyridoxine (vitamin B6) is given prophylactically to patients receiving
isoniazid, and at a higher dose to treat the occurrence of neuropathy.

Ethambutol (Option A) is incorrect. Ethambutol may cause retinal injury, resulting in


permanent loss of visual acuity, restriction of visual fields and colour blindness. Regular
ophthalmological review is needed and patients should be advised to urgently report any
visual symptoms.
Pyrazinamide (Option C) is incorrect. Pyrazinamide may cause hepatotoxicity.
Rifampicin (Option D) is incorrect. Rifampicin may cause hepatotoxicity and gives rise to
pink-coloured bodily fluids, including urine and tears.

https://mypastest.pastest.com/Secure/TestMe/Browser/429893#Top

1/2

8/12/2016

MyPastest

Streptomycin (Option E) is incorrect. Streptomycin causes ototoxicity and nephrotoxicity,


especially in elderly patients.
46619

Next Question

Previous Question

Tag Question

Feedback

End Review

Difficulty: Average
Peer Responses

Session Progress
Responses Correct:

Responses Incorrect:

179

Responses Total:

179

Responses - % Correct:

0%

Blog (https://www.pastest.com/blog) About Pastest (https://www.pastest.com/about-us)


Contact Us (https://www.pastest.com/contact-us) Help (https://www.pastest.com/help)
Pastest 2016

https://mypastest.pastest.com/Secure/TestMe/Browser/429893#Top

2/2

8/12/2016

MyPastest

Back to Filters (/Secure/TestMe/Filter/429893/QA)

Question 31 of 179

A 19-year-old woman is admitted to the Acute Medical Unit after an intentional paracetamol
overdose. You have been asked to review some blood test results that have been telephoned
to the ward.
Which one of the following metabolic abnormalities might most strongly indicate the
development of severe paracetamol poisoning?
A

Hyperkalaemia

Hypocalcaemia

Hypoglycaemia

Hypokalaemia

Hyponatraemia

Explanation

The answer is Hypoglycaemia


In paracetamol overdose, the usual pathway of elimination is overwhelmed due to
depletion of glutathione within hepatocytes by a toxic metabolite, N-acetyl
benzoquinone imine (NABQI) formed by cytochrome P450 oxidative metabolism.
Features of liver necrosis include hypoglycaemia, coagulation factor depletion, renal
impairment, cerebral oedema and coma. If the patient survives, hepatocyte regeneration
occurs without cirrhosis. Hypoglycaemia is an indicator of impaired hepatic glucose
synthesis.

Hyperkalaemia (Option A) is incorrect. Hyperkalaemia is uncommon unless patients have


developed acute tubular necrosis, which normally only becomes detectable several days
after paracetamol overdose.
Hypocalcaemia (Option B) is incorrect. Hypocalcaemia is not a recognised feature of
paracetamol toxicity; it is a characteristic finding in severe ethylene glycol toxicity.

https://mypastest.pastest.com/Secure/TestMe/Browser/429893#Top

1/2

8/12/2016

MyPastest

Hypokalaemia (Option D) is incorrect. Hypokalaemia is an important feature in paracetamol


overdose, and is caused by dose-dependent renal potassium excretion (perhaps also made
worse if the antidote is administered in dextrose rather than saline solution). However,
hypokalaemia does not correlate with the risk or extent of liver damage, hence
hypoglycaemia is the preferred answer.
Hyponatraemia (Option E) is incorrect. Hyponatraemia is not a recognised feature of
paracetamol toxicity.
46913

Next Question

Previous Question

Tag Question

Feedback

End Review

Difficulty: Average
Peer Responses

Session Progress
Responses Correct:

Responses Incorrect:

179

Responses Total:

179

Responses - % Correct:

0%

Blog (https://www.pastest.com/blog) About Pastest (https://www.pastest.com/about-us)


Contact Us (https://www.pastest.com/contact-us) Help (https://www.pastest.com/help)
Pastest 2016

https://mypastest.pastest.com/Secure/TestMe/Browser/429893#Top

2/2

8/12/2016

MyPastest

Back to Filters (/Secure/TestMe/Filter/429893/QA)

Question 32 of 179

A 50-year-old female on chemotherapy complains of tingling and numbness in her lower


limbs and weakness in her left hand. On examination, she is noted to have a wrist drop and
loss of ankle jerks. Blood tests show hyponatraemia and a plasma osmolality of 260
mosmol/kg. Her urine osmolality is 350 mosmol/kg.
Which of the following chemotherapeutic drugs is most likely to explain these features?
A

Bleomycin

Cyclophosphamide

Doxorubicin

Methotrexate

Vincristine

Explanation

The answer is Vincristine


Peripheral neuropathy is a neurotoxic feature of vincristine. Foot drop, paraesthesias, loss
of ankle jerks and wrist drop may also occur. Vincristine causes a biochemical picture
resembling syndrome of inappropriate antidiuretic hormone (SIADH), and hyponatraemia
may provoke seizures. Other adverse effects include mucositis of the entire
gastrointestinal tract and bleeding. Vincristine causes less nausea and vomiting than
other chemotherapy agents.

Bleomycin (Option A) is incorrect. Bleomycin may cause interstitial pneumonia.


Cyclophosphamide (Option B) is incorrect. Cyclophosphamide characteristically causes
haemorrhagic cystitis.
Doxorubicin (Option C) is incorrect. Doxorubicin and daunorubicin are known to cause
cardiomyopathy.
Methotrexate (Option D) is incorrect. Methotrexate characteristically causes hepatic and
pulmonary fibrosis.
46608

https://mypastest.pastest.com/Secure/TestMe/Browser/429893#Top

1/2

8/12/2016

MyPastest

46608

Next Question

Previous Question

Tag Question

Feedback

End Review

Difficulty: Average
Peer Responses

Session Progress
Responses Correct:

Responses Incorrect:

179

Responses Total:

179

Responses - % Correct:

0%

Blog (https://www.pastest.com/blog) About Pastest (https://www.pastest.com/about-us)


Contact Us (https://www.pastest.com/contact-us) Help (https://www.pastest.com/help)
Pastest 2016

https://mypastest.pastest.com/Secure/TestMe/Browser/429893#Top

2/2

8/12/2016

MyPastest

Back to Filters (/Secure/TestMe/Filter/429893/QA)

Question 33 of 179

A 64-year-old Asian man attends the Emergency Department for review. He is complaining of
aching pains in his arms and leg muscles and of lethargy; he also has minor symptoms of a
cold. He is a smoker with hypercholesterolaemia and has recently been prescribed oral
rosuvastatin 10 mg daily. He has developed aches and pains around his pelvic girdle, and his
urine is dipstick-positive for blood.
What is the most likely cause of this clinical picture?
A

Dermatomyositis

Influenza

Polymyositis

Proximal myopathy

Rhabdomyolysis

Explanation

The answer is Rhabdomyolysis


Specifically, the risk of rhabdomyolysis may be increased with the use of high-dose statin
therapy. Rhabdomyolysis is suggested by the presence of suspected myoglobinuria and
muscle pains; muscle enzymes including creatine kinase (CK) are likely to be abnormally
elevated.

Dermatomyositis (Option A) is incorrect. Dermatomyositis is associated with bronchial


carcinoma; hence, where there are skin changes and muscle aches in association with
smoking history, an underlying carcinoma must be considered.
Influenza (Option B) is incorrect. Influenza may cause generalised aches and myalgia, but
rarely causes rhabdomyolysis and therefore this is a less suitable answer.
Polymyositis (Option C) is incorrect. Polymyositis and proximal myopathy would be plausible
diagnoses in this scenario; however, the urine dipstick result makes rhabdomyolysis a more
likely option here.
https://mypastest.pastest.com/Secure/TestMe/Browser/429893#Top

1/2

8/12/2016

MyPastest

Proximal myopathy (Option D) is incorrect. Proximal myopathy might explain the patients
symptoms, but would not explain the findings on the dipstick test.
46874

Next Question

Previous Question

Tag Question

Feedback

End Review

Difficulty: Average
Peer Responses

Session Progress
Responses Correct:

Responses Incorrect:

179

Responses Total:

179

Responses - % Correct:

0%

Blog (https://www.pastest.com/blog) About Pastest (https://www.pastest.com/about-us)


Contact Us (https://www.pastest.com/contact-us) Help (https://www.pastest.com/help)
Pastest 2016

https://mypastest.pastest.com/Secure/TestMe/Browser/429893#Top

2/2

8/12/2016

MyPastest

Back to Filters (/Secure/TestMe/Filter/429893/QA)

Question 34 of 179

A 45-year-old woman is seen in the Oncology Clinic with end-stage carcinoma of the breast.
She has failed various treatments and you are considering treating her with docetaxel.
What is the mode of action of docetaxel?
A

Disrupting DNA

Binding to microtubules

Inhibiting mitochondrial energy production

Inhibiting RNA production

Inhibiting ribosome production

Explanation
Docetaxel
Docetaxel reversibly binds to microtubules with high affinity
This leads to a decrease in the availability of free tubulin, thus preventing mitotic cell
activity
Microtubules also accumulate within the cell, increasing apoptosis
Docetaxel has a further action in blocking bcl-2
Side-effects are similar to those seen with other chemotherapeutic agents
Alkylating agents are typically used to disrupt DNA
Specific anti-60S anti-ribosomal antibodies are currently under development for
ribosomal disruption in the treatment of cancer
Vitamin E analogues are currently under development for disruption of mitochondrial
function
21363

Next Question

https://mypastest.pastest.com/Secure/TestMe/Browser/429893#Top

1/2

8/12/2016

MyPastest

Previous Question

Tag Question

Feedback

End Review

Difficulty: Average
Peer Responses

Session Progress
Responses Correct:

Responses Incorrect:

179

Responses Total:

179

Responses - % Correct:

0%

Blog (https://www.pastest.com/blog) About Pastest (https://www.pastest.com/about-us)


Contact Us (https://www.pastest.com/contact-us) Help (https://www.pastest.com/help)
Pastest 2016

https://mypastest.pastest.com/Secure/TestMe/Browser/429893#Top

2/2

8/12/2016

MyPastest

Back to Filters (/Secure/TestMe/Filter/429893/QA)

Question 35 of 179

You are reviewing a 46-year-old man in the hypertension clinic who has evidence of left
ventricular hypertrophy on his resting ECG.
Which of the following medications is most likely to prevent or allow reversal of myocardial
hypertrophy?
A

Bendroflumethiazide

Hydrocortisone

Propranolol

Ramipril

Thyroxine

Explanation

The answer is Ramipril


Both -agonists and angiotensin II promote proto-oncogene expression, stimulate protein
synthesis and induce the synthesis of fetal forms of actin and myosin, thereby leading to
hypertrophy of smooth muscle. Clinical data indicate that ACE inhibitors and angiotensin
receptor antagonists prevent and allow reversal of myocardial hypertrophy; data
concerning -blockers are less compelling.

Bendroflumethiazide (Option A) is incorrect. Diuretic treatments reduce circulating volume


and give the appearance of improved left ventricular function on echocardiography, but have
no direct effect on myocardial hypertrophy.
Hydrocortisone (Option B) is incorrect. There is an association between excess cortisol and
left ventricular hypertrophy, although the mechanism underlying this is uncertain.
Propranolol (Option C) is incorrect. Beta-blockers may be capable of preventing and
reversing myocardial hypertrophy, but option D is a better answer because there is more
compelling evidence in favour of ACE inhibitors.

https://mypastest.pastest.com/Secure/TestMe/Browser/429893#Top

1/2

8/12/2016

MyPastest

Thyroxine (Option E) is incorrect. Thyroxine acts directly via nuclear receptors to regulate
myosin heavy-chain gene transcription, and may promote myocardial hypertrophy.
46598

Next Question

Previous Question

Tag Question

Feedback

End Review

Difficulty: Average
Peer Responses

Session Progress
Responses Correct:

Responses Incorrect:

179

Responses Total:

179

Responses - % Correct:

0%

Blog (https://www.pastest.com/blog) About Pastest (https://www.pastest.com/about-us)


Contact Us (https://www.pastest.com/contact-us) Help (https://www.pastest.com/help)
Pastest 2016

https://mypastest.pastest.com/Secure/TestMe/Browser/429893#Top

2/2

8/12/2016

MyPastest

Back to Filters (/Secure/TestMe/Filter/429893/QA)

Question 36 of 179

A 40-year-old obese female teacher is determined to lose weight. She exercises three times a
week at the local gym and is on a slimming diet. In the last month she managed to lose 3 kg
in weight and asked your opinion about initiating orlistat therapy.
On advising her, you would explain that orlistat therapy has which one of the following
effects?
A

Causes dramatic weight loss in the first month

Improves the bone mineral density

Increases the cholesterol level in the first year of therapy

Increases the risk of clotting

Reduces fat absorption from the intestine

Explanation

The answer is Reduces fat absorption from the intestine


Orlistat (Xenical) therapy effectively promotes weight loss and improves co-morbidities
by preventing absorption of fat molecules from the intestinal tract. Around 30% of
dietary fat that would otherwise have been absorbed passes straight through the bowel
and is excreted in the faeces. Therefore, it typically causes fatty stools, increased
frequency of defaecation, anal leakage or oily spotting. These adverse effects encourage
people taking the drug to minimise fat intake. Orlistat itself is not absorbed, except in
very small quantities, and thus its side-effects are restricted to the gastrointestinal tract.
Patients taking orlistat may require concomitant vitamin supplements because of
malabsorption of fat-soluble vitamins, such as vitamins A, D, K and E, although the
effects on vitamin absorption are substantially less than is seen with cholestyramine.

Causes dramatic weight loss in the first month (Option A) is incorrect. Orlistat is shown to be
clinically efficacious in reducing a persons weight over a period of a year, when administered
alongside other lifestyle measures intended to promote weight loss.

https://mypastest.pastest.com/Secure/TestMe/Browser/429893#Top

1/2

8/12/2016

MyPastest

Improves the bone mineral density (Option B) is incorrect. No effect on bone mineral density
is expected.
Increases the cholesterol level in the first year of therapy (Option C) is incorrect. Study
results also showed significant improvement in reducing fasting glucose, total cholesterol,
LDL-cholesterol and blood pressure.
Increases the risk of clotting (Option D) is incorrect. No significant effect on clotting is
expected; if there is interference with vitamin K absorption, this might cause coagulopathy,
although this is rarely seen in clinical practice.
46630

Next Question

Previous Question

Tag Question

Feedback

End Review

Difficulty: Average
Peer Responses

Session Progress
Responses Correct:

Responses Incorrect:

179

Responses Total:

179

Responses - % Correct:

0%

Blog (https://www.pastest.com/blog) About Pastest (https://www.pastest.com/about-us)


Contact Us (https://www.pastest.com/contact-us) Help (https://www.pastest.com/help)
Pastest 2016

https://mypastest.pastest.com/Secure/TestMe/Browser/429893#Top

2/2

8/12/2016

MyPastest

Back to Filters (/Secure/TestMe/Filter/429893/QA)

Question 37 of 179

A 72-year-old woman who was treated by a locum GP after a urinary tract infection asks to
see the emergency doctor after feeling something go in her ankle and then complaining of
right foot drop. She is concerned that it may be related to her antibiotic therapy. On
examination she appears to have a partial rupture of her right Achilles tendon.
Which one of the following antibiotics is the most likely cause of her symptoms?
A

Ciprofloxacin

Penicillin

Erythromycin

Gentamicin

Clarithromycin

Explanation
Tendon rupture as a consequence of antibiotic therapy
Extensive randomised controlled trial data to support the association between
quinolone use and tendon rupture are not easily available
However, a number of case control studies suggest an association between quinolone
use and tendon rupture, particularly in an elderly population and where concomitant
steroid use has occurred
These studies demonstrated that in patients using the recommended daily dose of
quinolone, the risk of rupture may be up to 7 times greater than that for controls
20935

Next Question

Previous Question

Tag Question

https://mypastest.pastest.com/Secure/TestMe/Browser/429893#Top

Feedback

End Review
1/2

8/12/2016

MyPastest

Difficulty: Average
Peer Responses

Session Progress
Responses Correct:

Responses Incorrect:

179

Responses Total:

179

Responses - % Correct:

0%

Blog (https://www.pastest.com/blog) About Pastest (https://www.pastest.com/about-us)


Contact Us (https://www.pastest.com/contact-us) Help (https://www.pastest.com/help)
Pastest 2016

https://mypastest.pastest.com/Secure/TestMe/Browser/429893#Top

2/2

8/12/2016

MyPastest

Back to Filters (/Secure/TestMe/Filter/429893/QA)

Question 38 of 179

A 14-year-old boy has been given cefuroxime for a respiratory tract infection.
Which of the following characteristics is most likely to increase its efficacy in this condition?
A

It fails to cross the bloodbrain barrier

It is active against Pseudomonas aeruginosa

It is available only in parenteral form

It is more effective than cefaclor against Haemophilus influenzae

It is the first-line treatment for infection with Gram-negative organisms

Explanation

The answer is It is more effective than cefaclor against Haemophilus influenzae.


Cefuroxime is a second-generation cephalosporin that is less susceptible to inactivation
by -lactamase than first-generation cephalosporins, including cefaclor, and therefore
exibits greater efficacy against Haemophillus influenzae and also Escherichia coli,
Klebsiella spp and Proteus mirabilis, but is less effective against Gram-positive
organisms.

It fails to cross the bloodbrain barrier (Option A) is incorrect. Cefuroxime readily crosses the
bloodbrain barrier, unlike first-generation cephalosporins.
It is active against Pseudomonas aeruginosa (Option B) is incorrect. It is minimally active
against P. aeruginosa.
It is available only in parenteral form (Option C) is incorrect. Cefuroxime may be
administered orally, intramuscularly, or intravenously.
It is the first-line treatment for infection with Gram-negative organisms (Option E) is
incorrect. Cefuroxime has limited activity against Gram-negative organisms; third-generation
cephalosporins, including cefotaxime and ceftriaxone, have greater activity against Gramnegative organisms.
46861

https://mypastest.pastest.com/Secure/TestMe/Browser/429893#Top

1/2

8/12/2016

MyPastest

Next Question

Previous Question

Tag Question

Feedback

End Review

Difficulty: Average
Peer Responses

Session Progress
Responses Correct:

Responses Incorrect:

179

Responses Total:

179

Responses - % Correct:

0%

Blog (https://www.pastest.com/blog) About Pastest (https://www.pastest.com/about-us)


Contact Us (https://www.pastest.com/contact-us) Help (https://www.pastest.com/help)
Pastest 2016

https://mypastest.pastest.com/Secure/TestMe/Browser/429893#Top

2/2

8/12/2016

MyPastest

Back to Filters (/Secure/TestMe/Filter/429893/QA)

Question 39 of 179

A 32-year-old glass etcher presents to the Emergency Department after being splashed with
chemicals at his work. He brought an information leafelet containing details of the liquid,
which contains hydrofluoric acid.
Which of the following statements best describes the toxicity of hydrofluoric acid?
A

Clinical features after dermal exposure are confined to the affected area only

Early application of alkaline solutions to affected areas is helpful

Hypercalcaemia is a recognised feature

May cause skin coagulative necrosis

The exposed area should be treated with heat packs

Explanation

The answer is May cause skin coagulative necrosis


Acids cause injury by coagulative necrosis. Acid effects are mainly topical, with corrosive
burns to the skin, mouth, oropharynx and stomach. Aspiration can lead to inflammation
and a chemical pneumonitis. Hydrofluoric acid causes significant hypocalcaemia as it
binds calcium, and exposure to even small amounts (topical or ingested) can produce
significant and fatal hypocalcaemia. Calcium gluconate applied both topically and
injected around the burn may be required.

Clinical features after dermal exposure are confined to the affected area only (Option A) is
incorrect. Systemic features are common even after a localised skin contact, including
systemic hypocalcaemia.
Early application of alkaline solutions to affected areas is helpful (Option B) is incorrect.
Neutralisation of acids is not appropriate, since this can generate increased heat and so
exacerbate any injury sustained.
Hypercalcaemia is a recognised feature (Option C) is incorrect. Hypocalcaemia occurs, not
hypercalcaemia.
https://mypastest.pastest.com/Secure/TestMe/Browser/429893#Top

1/2

8/12/2016

MyPastest

The exposed area should be treated with heat packs (Option E) is incorrect. Heat packs
should be avoided because these may increase the severity of local tissue damage.
46491

Next Question

Previous Question

Tag Question

Feedback

End Review

Difficulty: Average
Peer Responses

Session Progress
Responses Correct:

Responses Incorrect:

179

Responses Total:

179

Responses - % Correct:

0%

Blog (https://www.pastest.com/blog) About Pastest (https://www.pastest.com/about-us)


Contact Us (https://www.pastest.com/contact-us) Help (https://www.pastest.com/help)
Pastest 2016

https://mypastest.pastest.com/Secure/TestMe/Browser/429893#Top

2/2

8/12/2016

MyPastest

Back to Filters (/Secure/TestMe/Filter/429893/QA)

Question 40 of 179

You see a 56-year-old patient with type-2 diabetes in the hypertension clinic. He confides
that he has been suffering from erectile dysfunction, and you consider prescribing sildenafil
citrate treatment.
Which of the following best describes the mode of pharmacological action of sildenafil?
A

Inhibition of -glycosidase

Inhibition of cyclo-oxygenase II

Inhibition of H+/K+-ATPase

Inhibition of phosphodiesterase V

Inhibition of topoisomerase I

Explanation

The answer is Inhibition of phosphodiesterase V


Sildenafil citrate is an oral drug that has been approved for the treatment of erectile
dysfunction. It is a potent and selective inhibitor of type-V phosphodiesterase, the
primary form of the enzyme found in human penile erectile tissue. Enzyme inhibition
prevents breakdown of cyclic guanosine monophosphate (cGMP), the intracellular
second messenger of nitric oxide, and thereby promotes nitric oxide-mediated
vasodilation of the penile arterioles. Efficacy has been reported at 4085%, depending
upon the aetiology and severity of erectile dysfunction. It may interact with nitrates to
cause hypotension and syncope; adverse effects include headache, flushing, dyspepsia
and nasal congestion. A small percentage (23%) may experience altered colour vision
(blue halo), visual brightness, or blurred vision.

Inhibition of -glycosidase (Option A) is incorrect. -glycosidase inhibitors include miglitol,


which reduces abdominal fat accumulation and reverses some of the features of metabolic
syndrome.
Inhibition of cyclo-oxygenase II (Option B) is incorrect. Cyclo-oxygenase II may be inhibited
by selective agents including celecoxib, or non-selectively by NSAIDs and aspirin.
https://mypastest.pastest.com/Secure/TestMe/Browser/429893#Top

1/2

8/12/2016

MyPastest

Inhibition of H+/K+-ATPase (Option C) is incorrect. This is also known as the proton pump,
which may be inhibited by protein pump inhibitors, e.g. omeprazole.
Inhibition of topoisomerase I (Option E) is incorrect. This is the target mechanism of certain
anticancer agents and quinolone antibiotics.
46806

Next Question

Previous Question

Tag Question

Feedback

End Review

Difficulty: Average
Peer Responses

Session Progress
Responses Correct:

Responses Incorrect:

179

Responses Total:

179

Responses - % Correct:

0%

Blog (https://www.pastest.com/blog) About Pastest (https://www.pastest.com/about-us)


Contact Us (https://www.pastest.com/contact-us) Help (https://www.pastest.com/help)
Pastest 2016

https://mypastest.pastest.com/Secure/TestMe/Browser/429893#Top

2/2

8/12/2016

MyPastest

Back to Filters (/Secure/TestMe/Filter/429893/QA)

Question 41 of 179

A 72-year-old woman comes to the Rheumatology Clinic for review. She has rheumatoid
arthritis and you are planning to start golimumab therapy. She tells you she had tuberculosis
as a child which was treated successfully and there have been no symptoms since. She is
apyrexial and her chest is clear. There is clear evidence of active synovitis affecting her hands,
wrists and elbows in particular. A chest x-ray shows evidence of calcification, presumed to be
old, but nil else of note.
Which of the following is the optimal TB screening test in this situation?
A

Sputum culture

Mantoux test

Interferon gamma release assay

Repeat CXR in 6 weeks

Bronchoscopy with washings

Explanation
The answer is Interferon gamma release assay
Golimumab is an anti-TNF therapy used in the treatment of rheumatoid arthritis. As such
special care has to be taken before starting this treatment with respect to risk of reactivation
of tuberculosis or new infection. There is no reported contact risk from tuberculosis, but she
has had previous active TB as a child. As such tuberculin skin testing is likely to evoke a
strong reaction if she has active TB or not, and is therefore not very useful. Interferon gamma
release testing is therefore the preferred option.
36431

Next Question

Previous Question

Tag Question

Feedback

End Review

Difficulty: Average
https://mypastest.pastest.com/Secure/TestMe/Browser/429893#Top

1/2

8/12/2016

MyPastest

Peer Responses

Session Progress
Responses Correct:

Responses Incorrect:

179

Responses Total:

179

Responses - % Correct:

0%

Blog (https://www.pastest.com/blog) About Pastest (https://www.pastest.com/about-us)


Contact Us (https://www.pastest.com/contact-us) Help (https://www.pastest.com/help)
Pastest 2016

https://mypastest.pastest.com/Secure/TestMe/Browser/429893#Top

2/2

8/12/2016

MyPastest

Back to Filters (/Secure/TestMe/Filter/429893/QA)

Question 42 of 179

You are reviewing some blood results that have been telephoned to the ward from the
laboratory showing lowered serum potassium of 2.7 mmol/l. Serum sodium and creatinine are
within normal limits. You review the case notes and find out that the patient is a 76-year-old
man with a history of congestive heart failure and atrial fibrillation.
Which of the following drugs would be most likely to have caused hypokalaemia?
A

ACE inhibitors

Beta-blockers

Digoxin

Lithium

Theophylline

Explanation

The answer is Theophylline


Several drugs can alter serum potassium concentrations. Hypokalaemia may be caused
by shifts of potassium from extracellular to intracellular compartments with normal total
body potassium, e.g. theophylline, salbutamol, caffeine and insulin. Total body potassium
may become deplete with chronic diuretic use. Hyperkalaemia may be caused by
potassium shifting from the intracellular to extracellular compartment (e.g. -blockers,
metabolic acidosis, insulin insufficiency, digoxin), reduced renal excretion (e.g.
angiotensin-converting enzyme inhibitors, angiotensin receptor blockers, lithium).

ACE inhibitors (Option A) is incorrect. ACE inhibitors may impair renal excretion of
potassium and cause hyperkalaemia.
Beta-blockers (Option B) is incorrect. Beta-blockers may cause hyperkalaemia.
Digoxin (Option C) is incorrect. Digoxin inhibits the sodiumpotassium ATPase enzyme,
thereby increasing serum potassium and decreasing intracellular potassium.

https://mypastest.pastest.com/Secure/TestMe/Browser/429893#Top

1/2

8/12/2016

MyPastest

Lithium (Option D) is incorrect. Lithium may impair renal excretion of potassium and cause
hyperkalaemia.
46383

Next Question

Previous Question

Tag Question

Feedback

End Review

Difficulty: Average
Peer Responses

Session Progress
Responses Correct:

Responses Incorrect:

179

Responses Total:

179

Responses - % Correct:

0%

Blog (https://www.pastest.com/blog) About Pastest (https://www.pastest.com/about-us)


Contact Us (https://www.pastest.com/contact-us) Help (https://www.pastest.com/help)
Pastest 2016

https://mypastest.pastest.com/Secure/TestMe/Browser/429893#Top

2/2

8/12/2016

MyPastest

Back to Filters (/Secure/TestMe/Filter/429893/QA)

Question 43 of 179

A 74-year-old woman is admitted via the emergency department with palpitations and
breathlessness and found to have atrial fibrillation with ventricular rate 110 per min. You
decide to commence an IV infusion of amiodarone.
What is the predominant mode of action of amiodarone?
A

Alpha-adrenoreceptor blockade

Beta-adrenoreceptor blockade

Calcium channel blockade

Potassium channel blockade

Sodium channel blockade

Explanation

The answer is Potassium channel blockade


Amiodarone is a class III antiarrhythmic drug; drugs in this class exert their
pharmacological actions through potassium channel blockade. Others include sotalol.
They prolong the duration of the action potential with resulting prolongation of the
effective refractory period.

Alpha-adrenoreceptor blockade (Option A) is incorrect. Alpha-adrenoceptors do not confer


anti-arrhythmic properties.
Beta-adrenoreceptor blockade (Option B) is incorrect. Beta-blockers possess Class II
antiarrhythmic properties, and reduce adrenergic-mediated tachycardia.
Calcium channel blockade (Option C) is incorrect. Calcium channel blockers exert Class IV
antiarrhythmic properties and delay AV nodal conduction.
Sodium channel blockade (Option E) is incorrect. Sodium channel blockers exert Class I
antiarrhythmic effects, e.g. lidocaine.
46942

https://mypastest.pastest.com/Secure/TestMe/Browser/429893#Top

1/2

8/12/2016

MyPastest

Next Question

Previous Question

Tag Question

Feedback

End Review

Difficulty: Average
Peer Responses

Session Progress
Responses Correct:

Responses Incorrect:

179

Responses Total:

179

Responses - % Correct:

0%

Blog (https://www.pastest.com/blog) About Pastest (https://www.pastest.com/about-us)


Contact Us (https://www.pastest.com/contact-us) Help (https://www.pastest.com/help)
Pastest 2016

https://mypastest.pastest.com/Secure/TestMe/Browser/429893#Top

2/2

8/12/2016

MyPastest

Back to Filters (/Secure/TestMe/Filter/429893/QA)

Question 44 of 179

You review a 54-year-old woman who has been referred by her GP with elevated AST and
ALT to three times the upper limit of normal, accompanied by small rises in bilirubin and
alkaline phosphatase. Antinuclear antibodies are negative. She admits to drinking an
occasional glass of wine with her husband. She has a family history of mixed hyperlipidaemia
for which she takes fenofibrate.
What diagnosis fits best with this clinical history?
A

Autoimmune chronic active hepatitis

Chronic alcoholism

Drug-induced hepatitis

Haemochromatosis presenting after the menopause

Wilsons disease

Explanation

The answer is Drug-induced hepatitis


Fenofibrate is a recognised cause of drug-induced hepatitis, characterised by raised
transaminase levels and smaller rise in bilirubin and alkaline phosphatase. Other drug
causes of a chronic hepatitis biochemical picture include methyldopa, carbamazepine,
nitrofurantoin and isoniazid. Niacin promotes increased high-density lipoprotein (HDL)
cholesterol and may cause flushing and vasodilatation; an acute hepatitis biochemical
picture is a rare but recognised complication.

Autoimmune chronic active hepatitis (Option A) is incorrect. The negative antinuclear


antibody test makes autoimmune hepatitis less likely in this case.
Chronic alcoholism (Option B) is incorrect. Alcoholic liver disease is less likely in the absence
of heavier patterns of drinking and the absence of signs of chronic liver disease.
Haemochromatosis presenting after the menopause (Option D) is incorrect.
Haemochromatosis might be associated with other clinical signs such as hyperpigmentation,
arthropathy or type-2 diabetes/impaired glucose tolerance.
https://mypastest.pastest.com/Secure/TestMe/Browser/429893#Top

1/2

8/12/2016

MyPastest

Wilsons disease (Option E) is incorrect. Wilsons disease generally presents at a younger


age.
46879

Next Question

Previous Question

Tag Question

Feedback

End Review

Difficulty: Average
Peer Responses

Session Progress
Responses Correct:

Responses Incorrect:

179

Responses Total:

179

Responses - % Correct:

0%

Blog (https://www.pastest.com/blog) About Pastest (https://www.pastest.com/about-us)


Contact Us (https://www.pastest.com/contact-us) Help (https://www.pastest.com/help)
Pastest 2016

https://mypastest.pastest.com/Secure/TestMe/Browser/429893#Top

2/2

8/12/2016

MyPastest

Back to Filters (/Secure/TestMe/Filter/429893/QA)

Question 45 of 179

A 60-year-old woman with a past history of hypertension has been receiving a number of
different cardiac medications. She presents to the Emergency Department at 0200 h after a
suspected drug overdose. She smells strongly of alcohol, and is found to have heart rate 48
bpm and blood pressure 90/70 mmHg. An ECG shows sinus bradycardia with first-degree
heart block and prolongation of the PR interval.
What class of drugs is most likely to cause these effects?
A

Beta-blockers

Class III antiarrhythmic drugs (amiodarone)

Digitalis glycosides

Phenothiazines

Tricyclic antidepressants

Explanation

The answer is -blockers


The patients presentation with drowsiness, confusion, low pulse rate (48 bpm), low
systolic blood pressure (90/70 mmHg), first-degree heart block and prolongation of the
PR interval is typical of -blocker toxicity.

Class III antiarrhythmic drugs (amiodarone) (Option B) is incorrect. Amiodarone may cause
QT interval prolongation, although amiodarone overdose would rarely cause acute
bradyarrhythmia.
Digitalis glycosides (Option C) is incorrect. Digoxin toxicity may cause bradyarrhythmia and
is capable of prolonging the PR interval, but the clinical picture is usually dominated by
severe nausea and vomiting.
Phenothiazines (Option D) is incorrect. Phenothiazines may cause prolongation of the QT
interval, which may predispose to torsade de pointes arrhythmia (where the ECG looks like
ventricular tachycardia with a varying axis).
https://mypastest.pastest.com/Secure/TestMe/Browser/429893#Top

1/2

8/12/2016

MyPastest

Tricyclic antidepressants (Option E) is incorrect. Tricyclic antidepressants may cause


arrhythmia caused by sodium channel blockade, which may be detected by QRS
prolongation on the ECG.
46765

Next Question

Previous Question

Tag Question

Feedback

End Review

Difficulty: Average
Peer Responses

Session Progress
Responses Correct:

Responses Incorrect:

179

Responses Total:

179

Responses - % Correct:

0%

Blog (https://www.pastest.com/blog) About Pastest (https://www.pastest.com/about-us)


Contact Us (https://www.pastest.com/contact-us) Help (https://www.pastest.com/help)
Pastest 2016

https://mypastest.pastest.com/Secure/TestMe/Browser/429893#Top

2/2

8/12/2016

MyPastest

Back to Filters (/Secure/TestMe/Filter/429893/QA)

Question 46 of 179

You are reviewing a 46-year-old patient in the renal transplant clinic. He has been receiving
ciclosporin for the past 18 months.
Which of the following is most likely to represent an adverse effect of ciclosporin treatment?
A

Alopecia

Chronic interstitial nephritis

Myelosuppression

Stomatitis

Urolithiasis

Explanation

The answer is Chronic interstitial nephritis


The main use of ciclosporin is to reduce T-cell immune function in transplant patients and
so reduce the chances of rejection (liver, renal, pancreas, heart and bone marrow
transplants). Ciclosporin has a narrow therapeutic index and requires therapeutic drug
monitoring: renal toxicity is more common with high drug concentrations (> 200 ng/ml),
whereas rejection is common with low serum levels. Adverse effects include chronic
interstitial nephritis, hyperkalaemia and hypertrichosis. It is nephrotoxic, but virtually
non-myelotoxic.

Alopecia (Option A) is incorrect. Hypertrichosis is a common side-effect of ciclosporin,


rather than alopecia.
Myelosuppression (Option C) is incorrect. In contrast to other immunosuppressive agents,
ciclosporin does not cause significant myelosuppression.
Stomatitis (Option D) is incorrect. Stomatitis is not a recognised adverse effect of
ciclosporin.
Urolithiasis (Option E) is incorrect. Urolithiasis is not a recognised adverse effect of
ciclosporin.
46606

https://mypastest.pastest.com/Secure/TestMe/Browser/429893#Top

1/2

8/12/2016

MyPastest

46606

Next Question

Previous Question

Tag Question

Feedback

End Review

Difficulty: Average
Peer Responses

Session Progress
Responses Correct:

Responses Incorrect:

179

Responses Total:

179

Responses - % Correct:

0%

Blog (https://www.pastest.com/blog) About Pastest (https://www.pastest.com/about-us)


Contact Us (https://www.pastest.com/contact-us) Help (https://www.pastest.com/help)
Pastest 2016

https://mypastest.pastest.com/Secure/TestMe/Browser/429893#Top

2/2

8/12/2016

MyPastest

Back to Filters (/Secure/TestMe/Filter/429893/QA)

Question 47 of 179

You are asked to review a patient on the psychiatric ward, who has been receiving intensive
in-patient treatment. He has developed fever, rigidity and altered consciousness, together
with tachycardia and intermittent hypotension.
Which one of the following medications would be the most appropriate treatment?
A

Atropine

Beta-blocker

Dantrolene

Metoclopramide

Selegiline

Explanation

The answer is Dantrolene


The clinical features are indicative of neuroleptic malignant syndrome, a rare but
potentially very serious reaction to antipsychotic medications that is characterised by
fever, rigidity and myoclonus, and altered consciousness, often accompanied by
tachycardia, autonomic instability and pyrexia. Investigations normally show leukocytosis
and markedly elevated creatine kinase. The causative drug should be withdrawn and
supportive care given to manage cardiovascular, respiratory and renal complications.
Dantrolene may be helpful in treating agitation, and rigidity, particularly if patients have
a high fever. Alternatively, benzodiazepines may be used to reduce agitation and muscle
rigidity, and high doses are often required.

Atropine (Option A) is incorrect. Atropine would aggravate tachycardia and worsen delirium.
Beta-blocker (Option B) is incorrect. Beta-blockers may be considered if tachycardia
predisposes to a myocardial ischaemia in patients with known coronary artery disease.
However, it is rarely required, as heart rate normally resolves after administration of sufficient
doses of benzodiazepines.

https://mypastest.pastest.com/Secure/TestMe/Browser/429893#Top

1/2

8/12/2016

MyPastest

Metoclopramide (Option D) is incorrect. Metoclopramide should be avoided as this may itself


cause acute dystonia and worsen muscle rigidity.
Selegiline (Option E) is incorrect. Selegeline is unlikely to be of benefit and should be
avoided because it is an independent cause of acute dystonia.
46796

Next Question

Previous Question

Tag Question

Feedback

End Review

Difficulty: Average
Peer Responses

Session Progress
Responses Correct:

Responses Incorrect:

179

Responses Total:

179

Responses - % Correct:

0%

Blog (https://www.pastest.com/blog) About Pastest (https://www.pastest.com/about-us)


Contact Us (https://www.pastest.com/contact-us) Help (https://www.pastest.com/help)
Pastest 2016

https://mypastest.pastest.com/Secure/TestMe/Browser/429893#Top

2/2

8/12/2016

MyPastest

Back to Filters (/Secure/TestMe/Filter/429893/QA)

Question 48 of 179

A 62-year-old man attends the clinic complaining of lethargy. He has recently been started on
some medication for control of blood pressure but unfortunately he cannot remember its
name. On examination his blood pressure is 142/86 mmHg. His BMI is 22 kg/m2.
Investigations:
Hb

12.1 g/dl

White cell count 4.7 109/l


Platelets

193 109/l

Na+

124 mmol/l

K+

3.6 mmol/l

Creatinine

90 mol/l

Which one of the following agents is most likely to be responsible?


A

Ramipril

Spironolactone

Bendroflumethiazide

Amiloride

Furosemide

Explanation
Drug-induced electrolyte imbalance
Factors associated with thiazide-induced hyponatraemia include increased patient age,
decreased body mass and decreased serum potassium
In this case it would seem most appropriate to switch this patient to a calcium channel
antagonist
https://mypastest.pastest.com/Secure/TestMe/Browser/429893#Top

1/2

8/12/2016

MyPastest

Amiloride and spironolactone are associated with hyperkalaemia


both act at receptors in the distal convoluted tubule and promote sodium and
water excretion, with conservation of potassium
Furosemide promotes excretion of sodium and water and is less commonly associated
with hyponatraemia compared with thiazides
20940

Next Question

Previous Question

Tag Question

Feedback

End Review

Difficulty: Average
Peer Responses

Session Progress
Responses Correct:

Responses Incorrect:

179

Responses Total:

179

Responses - % Correct:

0%

Blog (https://www.pastest.com/blog) About Pastest (https://www.pastest.com/about-us)


Contact Us (https://www.pastest.com/contact-us) Help (https://www.pastest.com/help)
Pastest 2016

https://mypastest.pastest.com/Secure/TestMe/Browser/429893#Top

2/2

8/12/2016

MyPastest

Back to Filters (/Secure/TestMe/Filter/429893/QA)

Question 49 of 179

A 67-year-old man on warfarin for atrial fibrillation presents with epistaxis. On examination he
is tachycardic with a BP of 95/60 mmHg, and requires emergency treatment from the Ear,
Nose and Throat Service, but there has been ongoing nasal bleeding despite attempted
cauterisation. The INR on admission is 10.7 and the last known measurement was more than 3
months earlier.
Which of the following would be the most appropriate management?
A

Cryoprecipitate

Desmopressin

Prothrombin complex concentrate and vitamin K

Tranexamic acid

Vitamin K

Explanation

The answer is option Prothrombin complex concentrate and vitamin K


An INR around 8.7 accompanied by active bleeding indicates that urgent treatment is
required. Prothrombin complex concentrate (PCC) is an inactivated concentrate of
factors II, IX and X, and variable amounts of factor VII. Fresh frozen plasma (FFP) is a
plasma-derived blood product containing all clotting factors and fibrinogen; problems
include the time taken to thaw specimens prior to treatment (around 1 h) and risk of
transfusion reactions. PCC is lyophilised and can be reconstituted immediately, and
contains around 25-fold higher concentrations of vitamin K-dependent factors than FFP.
Treatment results in rapid and effective reversal of INR.

Cryoprecipitate (Option A) is incorrect. Cryoprecipitate is the plasma fraction that is


extracted from FFP that is rich in fibrinogen, von Willebrand factor, and factors VIII and XIII.
It may be used to restore haemostasis in patients with very low fibrinogen concentrations. It
contains insufficient vitamin K-dependent factors to effectively reverse warfarin-related
bleeding.
https://mypastest.pastest.com/Secure/TestMe/Browser/429893#Top

1/2

8/12/2016

MyPastest

Desmopressin (Option B) is incorrect. Desmopressin stimulates transient release of von


Willebrand factor by endothelial cells by blocking vasopressin V2 receptors, and is used in
the treatment of von Willebrand disease.
Tranexamic acid (Option D) is incorrect. Tranexamic acid is an antifibrinolytic agent and can
be used in the treatment of haemophilia and von Willebrands disease.
Vitamin K (Option E) is incorrect. Vitamin K (phytomenadione) is a useful adjunct but
reversal of warfarin may take many hours up to several days, and is inadequate alone in the
setting of active bleeding. Reversal by vitamin K may last for several weeks, which can make
reintroduction of warfarin more complicated.
46614

Next Question

Previous Question

Tag Question

Feedback

End Review

Difficulty: Average
Peer Responses

Session Progress
Responses Correct:

Responses Incorrect:

179

Responses Total:

179

Responses - % Correct:

0%

Blog (https://www.pastest.com/blog) About Pastest (https://www.pastest.com/about-us)


Contact Us (https://www.pastest.com/contact-us) Help (https://www.pastest.com/help)
Pastest 2016

https://mypastest.pastest.com/Secure/TestMe/Browser/429893#Top

2/2

8/12/2016

MyPastest

Back to Filters (/Secure/TestMe/Filter/429893/QA)

Question 50 of 179

A 43-year-old man with known alcoholic liver disease and cirrhosis is admitted via the
Emergency Department after a dramatic decline in his functional status, and is found to have
acute hepatic encephalopathy. His partner tells you that his GP gave him a new medication
just a few days earlier.
Which of the following drugs is most likely to have precipitated encephalopathy in this
patient?
A

Amlodipine

Diazepam

Folic acid

Thiamine

Vitamin E

Explanation

The answer is Diazepam


The presence of liver disease may influence pharmacodynamic responses to certain
drugs by several mechanisms. Patients with liver cirrhosis are more susceptible to the
effects of various sedative medications on the central nervous system including
benzodiazepines, opioids, barbiturates and chlorpromazine.

Amlodipine (Option A) is incorrect. Amlodipine and other calcium-channel blockers are


extensively metabolised by the liver, and may accumulate in liver disease to cause
hypotension and peripheral oedema.
Folic acid (Option C) is incorrect. Folate would not be expected to worsen liver disease or
provoke encephalopathy.
Thiamine (Option D) is incorrect. Thiamine is helpful for preventing Wernickes
encephalopathy in patients with a history of alcohol excess or malnutrition.

https://mypastest.pastest.com/Secure/TestMe/Browser/429893#Top

1/2

8/12/2016

MyPastest

Vitamin E (Option E) is incorrect. Vitamin E is subject to hepatic metabolism to active forms


including alpha-tocopherol; deficiency may cause ataxia.
46797

Next Question

Previous Question

Tag Question

Feedback

End Review

Difficulty: Average
Peer Responses

Session Progress
Responses Correct:

Responses Incorrect:

179

Responses Total:

179

Responses - % Correct:

0%

Blog (https://www.pastest.com/blog) About Pastest (https://www.pastest.com/about-us)


Contact Us (https://www.pastest.com/contact-us) Help (https://www.pastest.com/help)
Pastest 2016

https://mypastest.pastest.com/Secure/TestMe/Browser/429893#Top

2/2

8/12/2016

MyPastest

Back to Filters (/Secure/TestMe/Filter/429893/QA)

Question 51 of 179

You are asked to prescribe a course of antibiotics for a patient with suspected Chlamydia
pneumoniae infection.
What is the mechanism of Chlamydias resistance to cephalosporins?
A

Actively pumps cephalosporins out

Chlamydia has no peptidoglycan cell wall

Chlamydia is an extracellular organism

Forms cephalosporin-binding proteins

Hydrolysation of the -lactam ring

Explanation

The answer is Chlamydia has no peptidoglycan cell wall


The -lactams (penicillins, cephalosporins and monobactams, e.g. aztreonam) and
glycopeptides (vancomycin and teicoplanin) exert antimicrobial actions via inhibition of
cell wall synthesis. Chlamydia lacks a cell wall and, therefore, these antimicrobials are
ineffective.

Actively pumps cephalosporins out (Option A) is incorrect. This mechanism is not relevant.
Chlamydia is an extracellular organism (Option C) is incorrect. Chamydia is an intracellular
organism that lacks its own cell wall, and relies upon the host cell for protection.
Forms cephalosporin-binding proteins (Option D) is incorrect. This mechanism is not
relevant.
Hydrolysation of the -lactam ring (Option E) is incorrect. The -lactam antibiotics share a
common -lactam ring. Certain antimicrobials secrete enzymes capable of hydrolysing this
ring (penicillinase), which is responsible for treatment resistance.
46807

Next Question
https://mypastest.pastest.com/Secure/TestMe/Browser/429893#Top

1/2

8/12/2016

MyPastest

Previous Question

Tag Question

Feedback

End Review

Difficulty: Average
Peer Responses

Session Progress
Responses Correct:

Responses Incorrect:

179

Responses Total:

179

Responses - % Correct:

0%

Blog (https://www.pastest.com/blog) About Pastest (https://www.pastest.com/about-us)


Contact Us (https://www.pastest.com/contact-us) Help (https://www.pastest.com/help)
Pastest 2016

https://mypastest.pastest.com/Secure/TestMe/Browser/429893#Top

2/2

8/12/2016

MyPastest

Back to Filters (/Secure/TestMe/Filter/429893/QA)

Question 52 of 179

A 32-year-old car mechanic presents to the Emergency Department by paramedic


ambulance. He was found by his wife to be lying on the floor of his workshop, drowsy,
disorientated and hyperventilating. There have apparently been problems with money at
home and she is worried that he has taken an overdose. On examination he appears
intoxicated although there is no smell of alcohol, his BP is 90/60 mmHg and his pulse is
95/min and regular. His chest is clear although his respiratory rate is elevated at 32/min. IV
normal saline is commenced.
Investigations;
Hb

13.2 g/dl

WCC

10.4 x109/l

PLT

245 x109/l

Na+

136 mmol/l

K+

4.9 mmol/l

Bicarbonate 14 mmol/l
Cl-

104 mmol/l

Creatinine

138 micromol/l

Which of the following is the most effective intervention?


A

Forced alkaline diuresis

Gastric lavage

IV Fomepizole

IV N-acetylcysteine

IV Sodium bicarbonate

https://mypastest.pastest.com/Secure/TestMe/Browser/429893#Top

1/2

8/12/2016

MyPastest

Explanation
The answer is IV Fomepizole This patient works in a garage and is therefore likely to have ready access to ethylene glycol.
The metabolic acidosis with raised anion gap, and the intoxication without an obvious
alcoholic odour further support the diagnosis. IV Fomepizole, or 4-methylpyrazole is a
competitive inhibitor of alcohol dehydrogenase and therefore prevents metabolism of
ethylene glycol to oxalic acid, responsible for the acidosis and renal failure associated with
ethylene glycol poisoning. IV ethanol can be used as an alternative although IV ethanol will
of course lead to a worsening of intoxication. IV Sodium bicarbonate is only utilised in severe
acidosis. IV N-acetylcysteine is used in paracetamol overdose. Forced alkaline diuresis was
formerly used in the treatment of aspirin overdose, but is now not widely recommended.
38089

Next Question

Previous Question

Tag Question

Feedback

End Review

Difficulty: Difficult
Peer Responses

Session Progress
Responses Correct:

Responses Incorrect:

179

Responses Total:

179

Responses - % Correct:

0%

Blog (https://www.pastest.com/blog) About Pastest (https://www.pastest.com/about-us)


Contact Us (https://www.pastest.com/contact-us) Help (https://www.pastest.com/help)
Pastest 2016

https://mypastest.pastest.com/Secure/TestMe/Browser/429893#Top

2/2

8/12/2016

MyPastest

Back to Filters (/Secure/TestMe/Filter/429893/QA)

Question 53 of 179

You are reviewing a 75-year-old woman with mild dementia in the outpatient clinic. After
discussion with the family you decide to administer donepezil.
Which of the following side-effects should be looked out for?
A

Atrial tachycardias

Bradycardia

Constipation

Excessive somnolence

Nephritis

Explanation

The answer is Bradycardia


Donepezil is a cholinesterase inhibitor, and as such potentiates the actions of cholinergic
neurones. Adverse effects include nausea, vomiting and diarrhoea; cardiac side-effects
include bradycardia and, rarely, AV block. Less common effects include urinary
incontinence and hepatitis.

Atrial tachycardias (Option A) is incorrect. Bradycardia is a characteristic feature, not


tachycardia.
Constipation (Option C) is incorrect. Gastrointestinal disturbance typically results in
diarrhoea, not constipation.
Excessive somnolence (Option D) is incorrect. Donepezil does not confer significant
sedation.
Nephritis (Option E) is incorrect. Nephritis is not a recognised feature.
46824

Next Question

https://mypastest.pastest.com/Secure/TestMe/Browser/429893#Top

1/2

8/12/2016

MyPastest

Previous Question

Tag Question

Feedback

End Review

Difficulty: Average
Peer Responses

Session Progress
Responses Correct:

Responses Incorrect:

179

Responses Total:

179

Responses - % Correct:

0%

Blog (https://www.pastest.com/blog) About Pastest (https://www.pastest.com/about-us)


Contact Us (https://www.pastest.com/contact-us) Help (https://www.pastest.com/help)
Pastest 2016

https://mypastest.pastest.com/Secure/TestMe/Browser/429893#Top

2/2

8/12/2016

MyPastest

Back to Filters (/Secure/TestMe/Filter/429893/QA)

Question 54 of 179

A 17-year-old man has been diagnosed with tonsillitis and prescribed an antibiotic by his GP.
Three days later he attends the Emergency Department with a blotchy, non-pruritic purpuric
rash over his trunk, arms, and legs.
Which of the following antibiotics is most likely to have caused this?
A

Ampicillin

Benzylpenicillin

Cefadroxil

Cefuroxime

Erythromycin

Explanation

The answer is Ampicillin


Ampicillin, amoxicillin and other penicillins characteristically cause maculopapular skin
rashes. These are not allergic in nature, and may be more pronounced in patients with
viral illness, including when tonsillitis or pharyngitis is caused by EpsteinBarr infection.
Penicillins may also cause allergic reactions with a characteristic urticarial rash, and fixed
drug eruptions.

Benzylpenicillin (Option B) is incorrect. Benzylpenicillin is less likely to cause diffuse rash


than ampicillin or amoxicillin, and it is normally administered only by intramuscular or
intravenous routes of administration.
Cefadroxil (Option C) is incorrect. Cefuroxime and cefadroxil may produce an intensely itchy,
diffuse, maculopapular rash, but less commonly than after penicillin.
Cefuroxime (Option D) is incorrect. Cefuroxime and cefadroxil may produce an intensely
itchy, diffuse, maculopapular rash, but less commonly than after penicillin.
Erythromycin (Option E) is incorrect. Erythromycin may cause a diffuse maculopapular rash
but less commonly than after penicillins.
46777

https://mypastest.pastest.com/Secure/TestMe/Browser/429893#Top

1/2

8/12/2016

MyPastest

46777

Next Question

Previous Question

Tag Question

Feedback

End Review

Difficulty: Average
Peer Responses

Session Progress
Responses Correct:

Responses Incorrect:

179

Responses Total:

179

Responses - % Correct:

0%

Blog (https://www.pastest.com/blog) About Pastest (https://www.pastest.com/about-us)


Contact Us (https://www.pastest.com/contact-us) Help (https://www.pastest.com/help)
Pastest 2016

https://mypastest.pastest.com/Secure/TestMe/Browser/429893#Top

2/2

8/12/2016

MyPastest

Back to Filters (/Secure/TestMe/Filter/429893/QA)

Question 55 of 179

You are planning a clinical phase-II study in oncology, testing a new chemotherapy in patients
with a malignant tumour. It is a randomised, placebo-controlled, double-blind study. A total of
100 patients will receive standard therapy and 100 patients will receive the new substance
over a 5-year treatment duration.
Which statistical test is most appropriate to compare the survival times between the two
groups?
A

Analysis of variance (ANOVA)

Chi-square test

Log-rank test

Paired t-test

Unpaired t-test

Explanation

The answer is Log-rank test


The log-rank test is used to examine the null hypothesis that the survival curves in the
two groups are the same. That is to say that the test can demonstrate the likelihood that
survival in the two groups across the study period is similar; if very unlikely (eg a low
probability or P-value), then the null hypothesis can be rejected and a statistical
difference in survival claimed between the two groups.

Analysis of variance (ANOVA) (Option A) is incorrect. ANOVA tests examine whether the
means differ between multiple sets of data; it is similar to a t-test, but can consider more
than two groups. This is a less powerful test than the log-rank test.
Chi-square test (Option B) is incorrect. The chi-square test is used to evaluate the proportion
of events in one group versus another group; eg this might be used in a simplistic way to
present the proportion of survivors in group A versus group B at a particular time point.
However, it may be difficult to define the proportions clearly due to the effect of censoring,

https://mypastest.pastest.com/Secure/TestMe/Browser/429893#Top

1/2

8/12/2016

MyPastest

and the choice of time point is somewhat arbitrary. The log-rank test gives a comparison of
survival across the entire study period.
Paired t-test (Option D) is incorrect. A paired t-test is too simple a measure for comparison
between two matched groups, eg a before/after comparison in a cross-over study design.
Individual patient matching is used to ensure similar important characteristics between the
two groups; this might for instance be determined to be age, gender, cancer stage and so on.
A t-test might be used to compare a single measure in one group versus the other, but a
caveat is that the variable would need to demonstrate normal distribution; many biological
variables do not, eg creatinine.
Unpaired t-test (Option E) is incorrect. An unpaired t-test compares only a single variable
between two unlinked groups, and requires the variable to be distributed normally in both
groups. As with the paired t-test, this is too simplistic a measure to be useful in comparing
survival.
46792

Next Question

Previous Question

Tag Question

Feedback

End Review

Difficulty: Average
Peer Responses

Session Progress
Responses Correct:

Responses Incorrect:

179

Responses Total:

179

Responses - % Correct:

0%

Blog (https://www.pastest.com/blog) About Pastest (https://www.pastest.com/about-us)


Contact Us (https://www.pastest.com/contact-us) Help (https://www.pastest.com/help)
Pastest 2016

https://mypastest.pastest.com/Secure/TestMe/Browser/429893#Top

2/2

8/12/2016

MyPastest

Back to Filters (/Secure/TestMe/Filter/429893/QA)

Question 56 of 179

A 26-year-old who has developed a long-standing addiction to heroin which began 4 years
earlier whilst travelling visits you for advice. He has tried going cold turkey on a number of
occasions but develops unacceptable restlessness, anxiety, vomiting and diarrhoea. He now
has a child and is determined to stop. There is a place available on the local drug counselling
scheme.
Which one of the following is the most appropriate prescription with respect to medically
managing his withdrawal?
A

Buprenorphine

Methadone

Diazepam

Dihydrocodeine

Chlorpromazine

Explanation
Heroin substitutes
Both buprenorphine and methadone may be considered for use as heroin replacements
Buprenorphine may be associated with less risk in overdose, but NICE recommends
that unless circumstances dictate otherwise, methadone should be the first-choice
therapy
Co-abuse of alcohol and benzodiazepines may drive preferential use of buprenorphine,
as these agents increase the risk of significant CNS depression
Dihydrocodeine is not indicated for opiate withdrawal in the UK
Diazepam is not indicated for opiate replacement, it is added to other therapies in
some patients to alleviate symptoms of withdrawal, although it may actually broaden
the scope of any addictive behaviour
Atypical anti-psychotics are preferred in this situation to traditional agents such as
chlorpromazine
21361

https://mypastest.pastest.com/Secure/TestMe/Browser/429893#Top

1/2

8/12/2016

MyPastest

21361

Next Question

Previous Question

Tag Question

Feedback

End Review

Difficulty: Easy
Peer Responses

Session Progress
Responses Correct:

Responses Incorrect:

179

Responses Total:

179

Responses - % Correct:

0%

Blog (https://www.pastest.com/blog) About Pastest (https://www.pastest.com/about-us)


Contact Us (https://www.pastest.com/contact-us) Help (https://www.pastest.com/help)
Pastest 2016

https://mypastest.pastest.com/Secure/TestMe/Browser/429893#Top

2/2

8/12/2016

MyPastest

Back to Filters (/Secure/TestMe/Filter/429893/QA)

Question 57 of 179

A 57-year-old presents with a subacute history of breathlessness. Following a CT scan of his


chest, a diagnosis of pulmonary fibrosis is made and he is referred to you in the respiratory
clinic. On reviewing his past history, he has taken a number of different medications over the
past few years, and you suspect that one or more of these may have caused his lung disorder.
Which of the following medications would be most likely to cause pulmonary fibrosis?
A

Bendrofluazide

Captopril

Digoxin

Methotrexate

Prednisolone

Explanation

The answer is Methotrexate


Pulmonary fibrosis is a recognised adverse effect of treatment with a variety of drugs,
including busulfan, methotrexate, amiodarone and bleomycin.

Bendrofluazide (Option A) is incorrect. Thiazide diuretics are not a recognised cause of


pulmonary fibrosis.
Captopril (Option B) is incorrect. ACE inhibitors may cause a reversible dry cough in 1020%
of patients, but pulmonary fibrosis is not a recognised complication.
Digoxin (Option C) is incorrect. Digoxin is not a recognised cause of pulmonary fibrosis.
Prednisolone (Option E) is incorrect. Corticosteroids can be used to treat some forms of
interstitial lung disease, including intrinsic or extrinsic fibrosing alveolitis.
46901

Next Question

https://mypastest.pastest.com/Secure/TestMe/Browser/429893#Top

1/2

8/12/2016

MyPastest

Previous Question

Tag Question

Feedback

End Review

Difficulty: Average
Peer Responses

Session Progress
Responses Correct:

Responses Incorrect:

179

Responses Total:

179

Responses - % Correct:

0%

Blog (https://www.pastest.com/blog) About Pastest (https://www.pastest.com/about-us)


Contact Us (https://www.pastest.com/contact-us) Help (https://www.pastest.com/help)
Pastest 2016

https://mypastest.pastest.com/Secure/TestMe/Browser/429893#Top

2/2

8/12/2016

MyPastest

Back to Filters (/Secure/TestMe/Filter/429893/QA)

Question 58 of 179

A 54-year-old man with type-2 diabetes and dyslipidaemia has been prescribed metformin,
pioglitazone, gliclazide, simvastatin and fenofibrate.
Which of the following drugs has predominantly peroxisome proliferator-activated receptor
(PPAR-) activity?
A

Fenofibrate

Gliclazide

Metformin

Pioglitazone

Simvastatin

Explanation

The answer is Fenofibrate


The fibrate class of drugs are PPAR- agonists, their predominant action is in reducing
serum triglyceride levels and increasing HDL-cholesterol. PPAR activation results in
increased gene transcription; over 100 different genes are thought to be upregulated by
glitazone therapy. This group is distinct from PPAR- agonists (the glitazones), and
unrelated to PPAR- agonists that promote HDL-cholesterol.

Gliclazide (Option B) is incorrect. Sulfonylureas stimulate pancreatic insulin secretion.


Metformin (Option C) is incorrect. Metformin (a biguanide) inhibits lactate dehydrogenase
within the liver and bowel.
Pioglitazone (Option D) is incorrect. PPAR- agonists (the glitazones) act predominantly by
reducing free fatty acid levels, therefore improving insulin resistance and hence blood
glucose levels.
Simvastatin (Option E) is incorrect. Simvastatin inhibits activity of HMG-CoA reductase within
the liver, thereby limiting formation of cholesterol.
46851

https://mypastest.pastest.com/Secure/TestMe/Browser/429893#Top

1/2

8/12/2016

MyPastest

Next Question

Previous Question

Tag Question

Feedback

End Review

Difficulty: Average
Peer Responses

Session Progress
Responses Correct:

Responses Incorrect:

179

Responses Total:

179

Responses - % Correct:

0%

Blog (https://www.pastest.com/blog) About Pastest (https://www.pastest.com/about-us)


Contact Us (https://www.pastest.com/contact-us) Help (https://www.pastest.com/help)
Pastest 2016

https://mypastest.pastest.com/Secure/TestMe/Browser/429893#Top

2/2

8/12/2016

MyPastest

Back to Filters (/Secure/TestMe/Filter/429893/QA)

Question 59 of 179

You are asked to see a 13-year-old child in the medical clinic because the parents are
concerned that there is a medical cause for poor academic performance at school. The
mother has asked been searching the internet and is concerned about the possibility of lead
poisoning.
Which of the following features would most strongly suggest a diagnosis of chronic lead
poisoning?
A

Diarrhoea

Hypercalcaemia

Peroneal nerve palsy

Resting tremor affecting both hands

Thrombocytopenia

Explanation

The answer is Peroneal nerve palsy


Lead poisoning is due to the ingestion of contaminated water or certain traditional
remedies such as ayurvedic medicines. Typical acute effects of lead toxicity occur above
concentrations of 450 g/l, including nausea, abdominal pain and constipation.
Neurological effects are predominantly related to motor function including isolated nerve
lesions. Chronic low-grade exposure (< 450 g/l) can be associated with mild
neurodevelopmental delays. Chelation therapy with oral DMSA (dimercaptosuccinic acid)
or intravenous EDTA (ethylenediaminetetraacetic acid) should be considered for patients
with blood lead concentrations over 450 g/l or signs of severe toxicity. Lead absorption
and toxicity may be greater in patients with iron-deficiency anaemia.

Diarrhoea (Option A) is incorrect. Constipation and abdominal pain are recognised features.
Hypercalcaemia (Option B) is incorrect. Hypocalcaemia may give rise to greater lead
deposition in bones.

https://mypastest.pastest.com/Secure/TestMe/Browser/429893#Top

1/2

8/12/2016

MyPastest

Resting tremor affecting both hands (Option D) is incorrect. Parkinsonism is more closely
related to manganese toxicity, but it is not a characteristic of lead toxicity.
Thrombocytopenia (Option E) is incorrect. Characteristic haematological effects include
chronic anaemia and there may be macrocytosis and basophilic stippling on a blood film.
45798

Next Question

Previous Question

Tag Question

Feedback

End Review

Difficulty: Average
Peer Responses

Session Progress
Responses Correct:

Responses Incorrect:

179

Responses Total:

179

Responses - % Correct:

0%

Blog (https://www.pastest.com/blog) About Pastest (https://www.pastest.com/about-us)


Contact Us (https://www.pastest.com/contact-us) Help (https://www.pastest.com/help)
Pastest 2016

https://mypastest.pastest.com/Secure/TestMe/Browser/429893#Top

2/2

8/12/2016

MyPastest

Back to Filters (/Secure/TestMe/Filter/429893/QA)

Question 60 of 179

A 74-year-old man comes to the clinic some 6 weeks after a stroke for review and
management of his risk factors. He is already managed with two antihypertensives,
atorvastatin, metformin and gliclazide. He has suffered two myocardial infarctions during the
past 5 years. On examination his BP is 134/78 mmHg, pulse is 78/min and regular, his BMI is
28.
Investigations;
Hb

12.9 g/dl

WCC

7.8 x109/l

PLT

187 x109/l

Na+

138 mmol/l

K+

4.3 mmol/l

Creatinine

110 micromol/l

HbA1c

72.68 mmol/mol (8.8%)

You plan to intensify his glucose control.


Which of the following would be an appropriate target with respect to his HbA1c?
A

39.89 mmol/mol (5.8%)

47.54 mmol/mol (6.5%)

56.28 mmol/mol (7.3%)

65.03 mmol/mol (8.1%)

73.77 mmol/mol (8.9%)

Explanation
The answer is 56.28 mmol/mol (7.3%)
https://mypastest.pastest.com/Secure/TestMe/Browser/429893#Top

1/2

8/12/2016

MyPastest

The clues as to the most appropriate target for glucose control in this patient come from the
ACCORD and VADT studies, both of which targeted HbA1c of 42.07 mmol/mol (6%) in an
attempt to reduce the risk of vascular complications of diabetes. What was seen in these
studies was a small increase in sudden cardiovascular death in the group targeted for
intensive glucose control. The reason for this may have been increased sympathetic drive /
electrolyte imbalance at the time of hypoglycaemia, leading to arrhythmia. As such in
patients with Type 2 diabetes and a history of macrovascular disease, a less stringent HbA1c
target is more appropriate.
34348

Next Question

Previous Question

Tag Question

Feedback

End Review

Difficulty: Difficult
Peer Responses

Session Progress
Responses Correct:

Responses Incorrect:

179

Responses Total:

179

Responses - % Correct:

0%

Blog (https://www.pastest.com/blog) About Pastest (https://www.pastest.com/about-us)


Contact Us (https://www.pastest.com/contact-us) Help (https://www.pastest.com/help)
Pastest 2016

https://mypastest.pastest.com/Secure/TestMe/Browser/429893#Top

2/2

8/12/2016

MyPastest

Back to Filters (/Secure/TestMe/Filter/429893/QA)

Question 61 of 179

You are treating a 56-year-old woman with angina and decide to administer isosorbide
mononitrate.
Which one of the following pharmacological mechanisms makes the greatest contribution to
the beneficial effects of nitroglycerin?
A

Decreases oxygen transport to the myocardium

Dilation of systemic veins

Increase of left ventricular afterload

Increase of left ventricular preload

Reduces sodiumpotassium transport in myocardial muscle

Explanation

The answer is Dilation of systemic veins


Nitroglycerin products are both venous capacitance dilators, and coronary and systemic
artery dilators. Nitrates have complex beneficial effects on patients with coronary artery
disease; they dilate systemic veins, decrease myocardial wall tension, decrease oxygen
demand, cause vasodilation of large and medium-sized coronary arteries, increase
coronary blood flow to the subendocardium, reduce afterload, reduced preload and
increase ventricular compliance.

Decreases oxygen transport to the myocardium (Option A) is incorrect. Nitroglycerin results


in increased myocardial oxygen delivery.
Increase of left ventricular afterload (Option C) is incorrect. Nitroglycerin results in
decreased left ventricular afterload.
Increase of left ventricular preload (Option D) is incorrect. Nitroglycerin results in decreased
left ventricular preload.
Reduces sodiumpotassium transport in myocardial muscle (Option E) is incorrect.
Nitroglycerin has no direct effect on sodiumpotassium transport; this is a feature of digoxin.
46727

https://mypastest.pastest.com/Secure/TestMe/Browser/429893#Top

1/2

8/12/2016

MyPastest

46727

Next Question

Previous Question

Tag Question

Feedback

End Review

Difficulty: Average
Peer Responses

Session Progress
Responses Correct:

Responses Incorrect:

179

Responses Total:

179

Responses - % Correct:

0%

Blog (https://www.pastest.com/blog) About Pastest (https://www.pastest.com/about-us)


Contact Us (https://www.pastest.com/contact-us) Help (https://www.pastest.com/help)
Pastest 2016

https://mypastest.pastest.com/Secure/TestMe/Browser/429893#Top

2/2

8/12/2016

MyPastest

Back to Filters (/Secure/TestMe/Filter/429893/QA)

Question 62 of 179

A 28-year-old man with a known history of glucose 6-phosphate dehydrogenase (G6PD)


deficiency is prescribed an antibiotic for his urinary tract infection. He now presents to the
Emergency Department with jaundice and anaemia, and a blood film shows erythrocyte
fragments.
Which of the following drugs is most likely to have caused his condition?
A

Amoxicillin

Ampicillin

Ceftriaxone

Co-trimoxazole

Erythromycin

Explanation

The answer is Co-trimoxazole


The enzyme glucose 6-phosphate dehydrogenase (G6PD) in red blood cells is needed to
reduce nicotinic acid dinucleotide (NAD) to nicotinic acid dinucleotide phosphate
(NADPH); NADPH operates via glutathione to protect against oxidative damage. Most
sulfonamides can precipitate an acute haemolytic anaemia in patients with a G6PD
deficiency, including sulfamethoxazole (combined with trimethoprim in co-trimoxazole).

Amoxicillin (Option A) is incorrect. Penicillins are usually safe in this condition, although
autoimmune haemolytic anaemia is a recognised complication.
Ampicillin (Option B) is incorrect. Penicillins are usually safe in this condition, although
autoimmune haemolytic anaemia is a recognised complication.
Ceftriaxone (Option C) is incorrect. Cephalosporins are usually safe in this condition.
Erythromycin (Option E) is incorrect. Macrolides are usually safe in this condition.
46856

https://mypastest.pastest.com/Secure/TestMe/Browser/429893#Top

1/2

8/12/2016

MyPastest

Next Question

Previous Question

Tag Question

Feedback

End Review

Difficulty: Average
Peer Responses

Session Progress
Responses Correct:

Responses Incorrect:

179

Responses Total:

179

Responses - % Correct:

0%

Blog (https://www.pastest.com/blog) About Pastest (https://www.pastest.com/about-us)


Contact Us (https://www.pastest.com/contact-us) Help (https://www.pastest.com/help)
Pastest 2016

https://mypastest.pastest.com/Secure/TestMe/Browser/429893#Top

2/2

8/12/2016

MyPastest

Back to Filters (/Secure/TestMe/Filter/429893/QA)

Question 63 of 179

A 72-year-old woman presents to the Emergency Department with increased drowsiness and
lethargy. She is taking long-term extended-release morphine sulphate for an inoperable
gynaecological cancer for which she has received radiotherapy. Additional past medical
history of note includes type 2 diabetes, for which she is treated with BD insulin. You
understand that she has recently started some new medication for a complication of her
diabetes. She has neuropathy and has been seeing the chiropodist for an infected foot ulcer.
Her random blood glucose on admission is 5.4 mmol/l.
Which one of the following is most likely to have caused her decreased consciousness level?
A

Amitriptyline

Gliclazide

Ibuprofen

Ciprofloxacin

Clarithromycin

Explanation
Out of the drugs listed, it is amitriptyline which is most likely to be responsible for the clinical
picture seen here;
Interactions with morphine
The summary of product characteristics for morphine does not state any significant
pharmacokinetic interactions that potentiate morphines action
It does however state that morphine may reduce the effectiveness of ciprofloxacin
when co-administered and that prescribers should be aware of this interaction
A significant number of drugs exist that result in pharmacodynamic interactions with
morphine, including:
anxiolytics
phenothiazines
barbiturates
tricyclic antidepressants such as amitriptyline
The effect of morphine is markedly potentiated by monoamine oxidase inhibitors, so
co-administration should definitely be avoided
Resources
http://www.medicines.org.uk/emc/medicine/4896/SPC/Morphine+Sulphate+Injection+BP+Minijet+1mg+ml/#INTERACTIONS
(http://www.medicines.org.uk/emc/medicine/4896/SPC/Morphine+Sulphate+Injection+BP+Minijet+1mg+ml/#INTERACTIONS)
http://www.medicines.org.uk/emc/medicine/13143/SPC/Morphine+Sulphate+10mg+ml+Injection+BP/#INTERACTIONS
(http://www.medicines.org.uk/emc/medicine/13143/SPC/Morphine+Sulphate+10mg+ml+Injection+BP/#INTERACTIONS)
22522

Next Question

https://mypastest.pastest.com/Secure/TestMe/Browser/429893#Top

1/2

8/12/2016
Previous Question

MyPastest
Tag Question

Feedback

End Review

Difficulty: Average
Peer Responses

Session Progress
Responses Correct:

Responses Incorrect:

179

Responses Total:

179

Responses - % Correct:

0%

Blog (https://www.pastest.com/blog) About Pastest (https://www.pastest.com/about-us)


Contact Us (https://www.pastest.com/contact-us) Help (https://www.pastest.com/help)
Pastest 2016

https://mypastest.pastest.com/Secure/TestMe/Browser/429893#Top

2/2

8/12/2016

MyPastest

Back to Filters (/Secure/TestMe/Filter/429893/QA)

Question 64 of 179

A 22-year-old policeman attends the Emergency Department after being bitten on the hand
by his dog. You clean the wound and apply Steri-Strips and administer an anti-tetanus
booster. You are concerned about the possibility of an early infection.
Which one of the following antibiotics would be most appropriate in this case?
A

Ciprofloxacin

Co-amoxiclav

Doxycycline

Penicillin V

Trimethoprim

Explanation

The answer is Co-amoxiclav


Co-amoxiclav may be used to cover a variety of organisms that may be encountered
after a dog bite. For example is it effective against Pasteurella, Streptococcus,
Staphylococcus aureus and Escherichia coli.

Ciprofloxacin (Option A) is incorrect. Ciprofloxaxin would have activity against some of the
likely organisms and may be considered as second-line therapy.
Doxycycline (Option C) is incorrect. Doxycycline and erythromycin would have activity
against some of the possible infecting organisms, but less than co-amoxiclav.
Penicillin V (Option D) is incorrect. Penicillin would be active against Streptococcus infection,
but would have limited efficacy against other organisms.
Trimethoprim (Option E) is incorrect. Trimethoprim would be effective against Escherichia
coli but ineffective against Pasteurella, Streptococcus and Staphylococcus aureus.
46976

Next Question
https://mypastest.pastest.com/Secure/TestMe/Browser/429893#Top

1/2

8/12/2016

MyPastest

Previous Question

Tag Question

Feedback

End Review

Difficulty: Average
Peer Responses

Session Progress
Responses Correct:

Responses Incorrect:

179

Responses Total:

179

Responses - % Correct:

0%

Blog (https://www.pastest.com/blog) About Pastest (https://www.pastest.com/about-us)


Contact Us (https://www.pastest.com/contact-us) Help (https://www.pastest.com/help)
Pastest 2016

https://mypastest.pastest.com/Secure/TestMe/Browser/429893#Top

2/2

8/12/2016

MyPastest

Back to Filters (/Secure/TestMe/Filter/429893/QA)

Question 65 of 179

You see a 54-year-old man with a history of productive cough and fever, and decide to
administer a 1-week course of clarithromycin. He has been taking a number of regular
medications.
With which one of the following medications would clarithromycin be most likely to cause a
clinically important drug interaction?
A

Aspirin

Atorvastatin

Ciclosporin

Lithium

Paracetamol

Explanation

The answer is Ciclosporin


Macrolide antibiotics including clarithromycin may cause immediate inhibition of the
cytochrome P450 system, thereby increasing the circulating concentrations of drugs that
are normally metabolised by this system. This may provoke toxicity, particularly for drugs
with a narrow therapeutic index such as ciclosporin. Interaction between macrolides and
ciclosporin may provoke bone marrow failure and renal failure.

Aspirin (Option A) is incorrect. Aspirin is predominantly cleared by renal elimination.


Atorvastatin (Option B) is incorrect. Macrolides interfere with metabolism of simvastatin, and
to a lesser extent other statins, including atorvastatin. As a precaution, statin therapy is
usually withheld during treatment with clarithromycin. The interaction between
clarithromycin and atorvastatin is less hazardous than that involving ciclosporin.
Lithium (Option D) is incorrect. Lithium is a cation, and not subject to metabolism. It is
cleared by renal excretion, and lithium toxicity may be caused by drugs that interfere with
renal clearance.
https://mypastest.pastest.com/Secure/TestMe/Browser/429893#Top

1/2

8/12/2016

MyPastest

Paracetamol (Option E) is incorrect. Paracetamol is predominantly metabolised by sulfation


and glucuronidation, with only a minor contribution from P450 oxidation.
45807

Next Question

Previous Question

Tag Question

Feedback

End Review

Difficulty: Average
Peer Responses

Session Progress
Responses Correct:

Responses Incorrect:

179

Responses Total:

179

Responses - % Correct:

0%

Blog (https://www.pastest.com/blog) About Pastest (https://www.pastest.com/about-us)


Contact Us (https://www.pastest.com/contact-us) Help (https://www.pastest.com/help)
Pastest 2016

https://mypastest.pastest.com/Secure/TestMe/Browser/429893#Top

2/2

8/12/2016

MyPastest

Back to Filters (/Secure/TestMe/Filter/429893/QA)

Question 66 of 179

A 45-year-old man is admitted via the Emergency Department after intentional overdose
involving multiple medications. Your FY1 trainee has asked for advice on whether
measurements of drug concentrations would be helpful.
For which one of the following would drug concentrations be most helpful in the context of
suspected drug overdose?
A

Chlorpromazine

Diazepam

Imipramine

Morphine

Paraquat

Explanation

The answer is Paraquat


Measurement of plasma paraquat concentration can help in assessing prognosis and can
aid clinical decision-making regarding active versus palliative treatment. Plasma
concentration measurements are also useful in the management of poisoning to
determine the need for specific antidote treatment, including paracetamol, salicylates,
lithium, iron, methanol, ethylene glycol and theophylline.

Chlorpromazine (Option A) is incorrect. Chlorpromazine may cause profound central nervous


system depression; there is no correlation between circulating blood concentrations and
toxicity.
Diazepam (Option B) is incorrect. Diazepam may cause profound depression of central
nervous system and respiration, but blood concentrations would offer no clinical benefit in
assessing toxicity.
Imipramine (Option C) is incorrect. Tricyclic poisoning may cause severe or fatal toxicity
through sodium channel blockade and anticholinergic mechansism. Toxicity does not
correlate with blood drug concentrations.
https://mypastest.pastest.com/Secure/TestMe/Browser/429893#Top

1/2

8/12/2016

MyPastest

Morphine (Option D) is incorrect. Opioids may cause profound central nervous system
depression, hypoventilation and coma. Drug concentrations would add no benefit to the
clinical assessment of toxicity.
46912

Next Question

Previous Question

Tag Question

Feedback

End Review

Difficulty: Average
Peer Responses

Session Progress
Responses Correct:

Responses Incorrect:

179

Responses Total:

179

Responses - % Correct:

0%

Blog (https://www.pastest.com/blog) About Pastest (https://www.pastest.com/about-us)


Contact Us (https://www.pastest.com/contact-us) Help (https://www.pastest.com/help)
Pastest 2016

https://mypastest.pastest.com/Secure/TestMe/Browser/429893#Top

2/2

8/12/2016

MyPastest

Back to Filters (/Secure/TestMe/Filter/429893/QA)

Question 67 of 179

A 69-year-old man with long-standing cardiac failure and chronic kidney disease has been
referred urgently to the general medical clinic for review. Over the past week he has
complained of nausea, reduced oral intake and visual disturbance characterised by a
yellowish-green halo around objects. His cardiorespiratory examination is normal, pupil
responses and eye movements are normal, and fundi appear normal.
Which of the following medications is most likely to account for his present condition?
A

Digoxin

Enalapril

Furosemide

Rivaroxaban

Salbutamol

Explanation

The answer is Digoxin


Digoxin has a narrow therapeutic index, and the most common features of digoxin
toxicity are anorexia, nausea, xanthopsia as a manifestation of neurotoxicity, and
bradyarrhythmia. Toxicity is more likely to occur in patients with hypokalaemia and
hypercalcaemia.

Enalapril (Option B) is incorrect. Enalapril may cause postural hypotension, dizziness and
cough.
Furosemide (Option C) is incorrect. Furosemide may cause postural hypotension, dizziness,
renal impairment and metabolic disturbance including hypokalaemia.
Rivaroxaban (Option D) is incorrect. Rivaroxaban may cause bleeding, skin erythema and
blistering, sore throat and joint pains.
Salbutamol (Option E) is incorrect. Salbutamol may cause tachycardia, palpitations and
hypokalaemia.
46616

https://mypastest.pastest.com/Secure/TestMe/Browser/429893#Top

1/2

8/12/2016

MyPastest

46616

Next Question

Previous Question

Tag Question

Feedback

End Review

Difficulty: Average
Peer Responses

Session Progress
Responses Correct:

Responses Incorrect:

179

Responses Total:

179

Responses - % Correct:

0%

Blog (https://www.pastest.com/blog) About Pastest (https://www.pastest.com/about-us)


Contact Us (https://www.pastest.com/contact-us) Help (https://www.pastest.com/help)
Pastest 2016

https://mypastest.pastest.com/Secure/TestMe/Browser/429893#Top

2/2

8/12/2016

MyPastest

Back to Filters (/Secure/TestMe/Filter/429893/QA)

Question 68 of 179

An 18-year-old woman presents to the Emergency Department following an intentional


overdose of aspirin tablets.
Which of the following treatments would be most effective in reducing aspirin toxicity?
A

Forced alkaline diuresis

Intravenous furosemide

Intravenous insulin and glucose administration

Multi-dose oral activated charcoal

Oral sodium bicarbonate

Explanation

The answer is Multi-dose oral activated charcoal


Initial features include respiratory alkalosis due to direct stimulation of the respiratory
centre and hyperventilation, whereas a metabolic acidosis occurs due to uncoupling of
oxidative phosphorylation and the build-up of lactic acid and fatty acid metabolites.
Symptoms of salicylate ingestion include nausea, vomiting, hyperventilation and
sweating. Tinnitus typically occurs at plasma salicylate concentrations above 400500
mg/l. Other effects include hypoglycaemia, particularly in children. Severe toxic effects
are central nervous system (CNS) effects, including confusion, coma and seizures.
Intravenous bicarbonate administration causes ionisation of circulating aspirin, which
results in lower tissue concentrations and more extensive renal elimination. Fluid
administration should be directed towards maintaining hydration and correcting
electrolytes; there is no role for forced diuresis or diuretic administration.

Forced alkaline diuresis (Option A) is incorrect. This is hazardous and should be avoided.
Intravenous furosemide (Option B) is incorrect. This does not reduce aspirin toxicity. Fluids
may be administered to maintain hydration.
Intravenous insulin and glucose administration (Option C) is incorrect. This may be helpful in
-blocker or calcium channel blocker poisoning, but not aspirin poisoning.
https://mypastest.pastest.com/Secure/TestMe/Browser/429893#Top

1/2

8/12/2016

MyPastest

Oral sodium bicarbonate (Option E) is incorrect. The urgency of addressing severe aspirin
poisoning means that oral administration is less appropriate than intravenous administration.
45799

Next Question

Previous Question

Tag Question

Feedback

End Review

Difficulty: Average
Peer Responses

Session Progress
Responses Correct:

Responses Incorrect:

179

Responses Total:

179

Responses - % Correct:

0%

Blog (https://www.pastest.com/blog) About Pastest (https://www.pastest.com/about-us)


Contact Us (https://www.pastest.com/contact-us) Help (https://www.pastest.com/help)
Pastest 2016

https://mypastest.pastest.com/Secure/TestMe/Browser/429893#Top

2/2

8/12/2016

MyPastest

Back to Filters (/Secure/TestMe/Filter/429893/QA)

Question 69 of 179

A 65-year-old man has been receiving warfarin for atrial fibrillation. His INR has been
between 2.0 and 3.0 for the past 6 months. He attends the Emergency Department due to
extensive bruising and is noted to have an INR of 9.6.
Which of the following drugs is most likely to have interacted with warfarin to cause a rise in
INR?
A

Aspirin

Carbamazepine

Phenytoin

Rifampicin

Sodium valproate

Explanation

The answer is Sodium valproate


Inhibition of P450 enzymes may increase the circulating concentrations of drugs that are
metabolised by the P450 enzymes. Warfarin is an example of a drug that is subject to
liver metabolism by the P450 enzyme system. Therefore, concomitant treatment with
enzyme inhibitors may give rise to high INR; enzyme inhibitors include: acute alcohol
ingestion, cimetidine, omeprazole, disulfiram, erythromycin, ciprofloxacin, sodium
valproate and sulfonamides.

Aspirin (Option A) is incorrect. Aspirin may independently increase the risk of bleeding in
patients receiving anticoagulation, but normally has only a small effect on warfarin
concentrations due to a protein-binding interaction and would not normally cause a large
increase in INR.
Carbamazepine (Option B) is incorrect. Carbamazepine is an enzyme inducer, likely to lessen
warfarin concentrations and INR.

https://mypastest.pastest.com/Secure/TestMe/Browser/429893#Top

1/2

8/12/2016

MyPastest

Phenytoin (Option C) is incorrect. Phenytoin is an enzyme inducer, likely to lessen warfarin


concentrations and INR.
Rifampicin (Option D) is incorrect. Rifampicin is an enzyme inducer, likely to lessen warfarin
concentrations and INR.
46488

Next Question

Previous Question

Tag Question

Feedback

End Review

Difficulty: Average
Peer Responses

Session Progress
Responses Correct:

Responses Incorrect:

179

Responses Total:

179

Responses - % Correct:

0%

Blog (https://www.pastest.com/blog) About Pastest (https://www.pastest.com/about-us)


Contact Us (https://www.pastest.com/contact-us) Help (https://www.pastest.com/help)
Pastest 2016

https://mypastest.pastest.com/Secure/TestMe/Browser/429893#Top

2/2

8/12/2016

MyPastest

Back to Filters (/Secure/TestMe/Filter/429893/QA)

Question 70 of 179

A 72-year-old woman presents to the Emergency Department with palpitations and shortness
of breath. She has a history of COPD, managed with high dose Seretide, and an
echocardiogram conducted two months earlier has revealed left atrial enlargement and mild
mitral regurgitation. Examination reveals a BP of 115/75 mmHg, pulse is 145/min (atrial
fibrillation). Serum creatinine is measured at 90 micromol/l.
You elect to begin loading with digoxin.
What is the pharmacokinetic reason that drives the practice of loading with digoxin?
A

Gastrointestinal absorption

Half life

Hepatic metabolism

Renal excretion

Volume of distribution

Explanation
The answer is Volume of distribution The volume of distribution for Digoxin is very large (510 litres). This means that administered
doses are rapidly distributed to body tissues. The initial distribution lasts for some 6-8hrs,
which drives the typical loading regimen for Digoxin of two larger doses (500mcg) some 612hrs apart. Without loading Digoxin typically takes a few days to reach therapeutic effect.
GI absorption is high at approximately 60% in tablet form. The elimination half-life is some
30-40hrs in patients with normal renal function (the drug being renally excreted). In patients
with abnormal renal function it can be as long as 100hrs. This drives reduced dosing in
patients with renal impairment.
https://www.medicines.org.uk/emc/medicine/2177/SPC/Lanoxin+PG+Tablets/#PHARMACOKINETIC_PROPS
(https://www.medicines.org.uk/emc/medicine/2177/SPC/Lanoxin+PG+Tablets/#PHARMACOKINETIC_PROPS)
37786

Next Question

Previous Question

Tag Question

Feedback

End Review

Difficulty: Average
Peer Responses
https://mypastest.pastest.com/Secure/TestMe/Browser/429893#Top

1/2

8/12/2016

MyPastest

Session Progress
Responses Correct:

Responses Incorrect:

179

Responses Total:

179

Responses - % Correct:

0%

Blog (https://www.pastest.com/blog) About Pastest (https://www.pastest.com/about-us)


Contact Us (https://www.pastest.com/contact-us) Help (https://www.pastest.com/help)
Pastest 2016

https://mypastest.pastest.com/Secure/TestMe/Browser/429893#Top

2/2

8/12/2016

MyPastest

Back to Filters (/Secure/TestMe/Filter/429893/QA)

Question 71 of 179

A 34-year-old man is brought to the Emergency Department by the police. He was found in
the streets with reduced conscious level, and you suspect possible heroin intoxication.
Which of the following signs most strongly supports a diagnosis of opioid toxicity?
A

Bradycardia

Deep and rapid respiration

Excess sweating and lacrimation

Hypothermia

Mydriasis

Explanation

The answer is Bradycardia


Heroin and other opioid toxicity is classically associated with reduced conscious level,
pinpoint pupils (meiosis), reduced respiratory effort (shallow respirations), and
bradycardia and hypotension.

Deep and rapid respiration (Option B) is incorrect. Progressive opioid toxicity causes shallow
respirations and reducing tidal volume; respiratory rate may appear normal until severe
toxicity occurs and respirations slow or cease.
Excess sweating and lacrimation (Option C) is incorrect. Excess sweating and lacrimation are
recognised features of opiate withdrawal.
Hypothermia (Option D) is incorrect. Hypothermia is a non-specific finding in patients with
impaired conscious level who have been outdoors for an unknown period of time, commonly
due to alcohol toxicity or other sedative medications including benzodiazepines and
barbiturates.
Mydriasis (Option E) is incorrect. Mydriasis (dilated pupils) may suggest exposure to drugs
with anticholinergic or serotonergic properties.
46618

https://mypastest.pastest.com/Secure/TestMe/Browser/429893#Top

1/2

8/12/2016

MyPastest

Next Question

Previous Question

Tag Question

Feedback

End Review

Difficulty: Average
Peer Responses

Session Progress
Responses Correct:

Responses Incorrect:

179

Responses Total:

179

Responses - % Correct:

0%

Blog (https://www.pastest.com/blog) About Pastest (https://www.pastest.com/about-us)


Contact Us (https://www.pastest.com/contact-us) Help (https://www.pastest.com/help)
Pastest 2016

https://mypastest.pastest.com/Secure/TestMe/Browser/429893#Top

2/2

8/12/2016

MyPastest

Back to Filters (/Secure/TestMe/Filter/429893/QA)

Question 72 of 179

A 31-year-old woman with a history of hypertension has a recent blood pressure of 149/78
mmHg. She tells you that she has been trying to become pregnant and is keen to avoid any
therapy that might interfere with this.
Which of the following agents would be most suitable for managing hypertension in
pregnancy?
A

Bendrofluazide

Candesartan

Hydrochlorothiazide

Nifedipine

Ramipril

Explanation

The answer is Nifedipine


Regarding calcium antagonists such as nifedipine in pregnancy, it is important to balance
potential risks to the fetus against the risks of uncontrolled hypertension. Where a
woman is over 30 years old, has a history of hypertension and has raised blood pressure
currently, the potential risks of uncontrolled maternal hypertension are likely to be
greater than the potential adverse effects of antihypertensive therapy. It has been
reported that the non-dihydropyridine calcium antagonist diltiazem may inhibit the onset
of labour.

Bendrofluazide (Option A) is incorrect. Thiazides or any other diuretics should generally be


avoided in pregnancy due to the risk of neonatal thrombocytopaenia, albeit a rare outcome.
Candesartan (Option B) is incorrect. Angiotensin-II-receptor blockers carry broadly the same
potential hazards in pregnancy as ACE inhibitors, namely neonatal hyperkalaemia and
oligohydramnios.
Hydrochlorothiazide (Option C) is incorrect. Thiazides or any other diuretics should generally
be avoided in pregnancy due to the risk of neonatal thrombocytopaenia, albeit a rare
https://mypastest.pastest.com/Secure/TestMe/Browser/429893#Top

1/2

8/12/2016

MyPastest

outcome.
Ramipril (Option E) is incorrect. Angiotensin converting enzyme (ACE) inhibitors should be
avoided in pregnancy as they may adversely affect fetal blood pressure and renal function; in
addition, animal studies have demonstrated a potential association with fetal skull defects
and oligohydramnios.
46827

Next Question

Previous Question

Tag Question

Feedback

End Review

Difficulty: Average
Peer Responses

Session Progress
Responses Correct:

Responses Incorrect:

179

Responses Total:

179

Responses - % Correct:

0%

Blog (https://www.pastest.com/blog) About Pastest (https://www.pastest.com/about-us)


Contact Us (https://www.pastest.com/contact-us) Help (https://www.pastest.com/help)
Pastest 2016

https://mypastest.pastest.com/Secure/TestMe/Browser/429893#Top

2/2

8/12/2016

MyPastest

Back to Filters (/Secure/TestMe/Filter/429893/QA)

Question 73 of 179

A man with advanced cirrhosis attends the clinic for review of his medications. He is taking a
number of drugs, including omeprazole, propranolol and ciprofloxacin.
Which of the following statements best applies to drug metabolism in patients with
advanced liver disease?
A

Conjugation reactions are affected early in cirrhosis

Conjugation reactions are affected to a lesser extent by advanced liver disease and
only occur in very late stage disease

Drug reactions mediated by mixed function oxidases are affected late in liver disease

Intrahepatic cholestasis does not affect drug transport

Plasma proteins increase and affect drug metabolism

Explanation

The answer is Conjugation reactions are affected to a lesser extent by advanced liver
disease and only occur in very late stage disease
Drug processing via mixed function oxidases is affected early in liver disease, whereas
conjugation reactions are affected to a lesser extent and problems only occur much later
in the liver disease process. Clinical trials of drugs are rarely conducted in patients with
severe liver disease, although of course some drugs in particular (e.g. antibiotics) are
used in very sick liver patients, and limited studies may take place in these patient
groups.

Conjugation reactions are affected early in cirrhosis (Option A) is incorrect. Conjugation


reactions are affected late in cirrhosis.
Drug reactions mediated by mixed function oxidases are affected late in liver disease (Option
C) is incorrect. Mixed function oxidase activity is impaired early in liver cirrhosis.
Intrahepatic cholestasis does not affect drug transport (Option D) is incorrect. Both
intrahepatic and extrahepatic cholestasis may affect the metabolism of drugs that are
actively secreted into bile, e.g. ciprofloxacin.
https://mypastest.pastest.com/Secure/TestMe/Browser/429893#Top

1/2

8/12/2016

MyPastest

Plasma proteins increase and affect drug metabolism (Option E) is incorrect. Plasma proteins
fall in liver disease and may alter drug distribution.
46842

Next Question

Previous Question

Tag Question

Feedback

End Review

Difficulty: Average
Peer Responses

Session Progress
Responses Correct:

Responses Incorrect:

179

Responses Total:

179

Responses - % Correct:

0%

Blog (https://www.pastest.com/blog) About Pastest (https://www.pastest.com/about-us)


Contact Us (https://www.pastest.com/contact-us) Help (https://www.pastest.com/help)
Pastest 2016

https://mypastest.pastest.com/Secure/TestMe/Browser/429893#Top

2/2

8/12/2016

MyPastest

Back to Filters (/Secure/TestMe/Filter/429893/QA)

Question 74 of 179

A 60-year-old retired surgeon is brought to the Emergency Department by his wife, who has
noticed his behaviour has become bizarre with paranoid symptoms over the past 4 days. He
has evidence of coarse tremor, horizontal nystagmus and an ataxic gait. There is a history of
excessive drinking since retirement.
Which of the following immediate treatments would be most effective?
A

Acamprosate

Disulfiram

Intravenous diazepam

Intravenous thiamine

Naltrexone

Explanation

The answer is Intravenous thiamine


This patient is likely to be suffering from Wernickes encephalopathy owing to thiamine
deficiency. Although this is most often seen in alcoholics, it may also occur in severe
malnutrition, and in patients on long-term haemodialysis or with AIDS. Wernickes
encephalopathy presents as a triad of acute mental confusion, ataxia and
ophthalmoplegia. Nystagmus is usually present. Urgent thiamine should be given to
prevent the development of irreversible Korsakoff syndrome, characterised by retrograde
and anterograde amnesia with varying degrees of other cognitive defects.

Acamprosate (Option A) is incorrect. Acamprosate reduces cravings for alcohol and is useful
in the maintenance of abstinence, but is of no value in the setting of acute alcohol
withdrawal.
Disulfiram (Option B) is incorrect. Disulfiram is an aversive agent that creates adverse
symptoms when patients consume alcohol; it is helpful in maintaining abstinence but is of no
value in acute alcohol withdrawal.

https://mypastest.pastest.com/Secure/TestMe/Browser/429893#Top

1/2

8/12/2016

MyPastest

Intravenous diazepam (Option C) is incorrect. Diazepam may be effective in reducing the


risks of seizures and reduce symptoms of alcohol withdrawal, but will not correct the
neurological features attributable to thiamine deficiency.
Naltrexone (Option E) is incorrect. Naltrexone may be effective in promoting alcohol
abstinence and improves quit rates, but it is of no value in the acute setting.
46768

Next Question

Previous Question

Tag Question

Feedback

End Review

Difficulty: Average
Peer Responses

Session Progress
Responses Correct:

Responses Incorrect:

179

Responses Total:

179

Responses - % Correct:

0%

Blog (https://www.pastest.com/blog) About Pastest (https://www.pastest.com/about-us)


Contact Us (https://www.pastest.com/contact-us) Help (https://www.pastest.com/help)
Pastest 2016

https://mypastest.pastest.com/Secure/TestMe/Browser/429893#Top

2/2

8/12/2016

MyPastest

Back to Filters (/Secure/TestMe/Filter/429893/QA)

Question 75 of 179

Your hospital has been alerted to the possibility of a nearby chemical spill, and a number of
patients are on their way to hospital. There are reports of the chemical having a smell of
rotten eggs.
Which of the following agents would you most strongly suspect?
A

Arsenic

Cyanide

Hydrogen sulfide

Napthalene

Selenium

Explanation

The answer is Hydrogen sulfide


Certain toxins/poisons have characteristic smells that can assist in the identification of
various chemical substances. Not all individuals perceive smells or odours to the same
extent, such that reports may not be very reliable. The smell of rotten eggs is
characteristic for hydrogen sulfide.

Arsenic (Option A) is incorrect. The smell of garlic is characteristic for arsenic and selenium.
Cyanide (Option B) is incorrect. The smell of almonds is characteristic for cyanide.
Napthalene (Option D) is incorrect. The smell of mothballs is characteristic for naphthalene.
Selenium (Option E) is incorrect. The smell of garlic is characteristic for selenium and arsenic.
45802

Next Question

https://mypastest.pastest.com/Secure/TestMe/Browser/429893#Top

1/2

8/12/2016

MyPastest

Previous Question

Tag Question

Feedback

End Review

Difficulty: Average
Peer Responses

Session Progress
Responses Correct:

Responses Incorrect:

179

Responses Total:

179

Responses - % Correct:

0%

Blog (https://www.pastest.com/blog) About Pastest (https://www.pastest.com/about-us)


Contact Us (https://www.pastest.com/contact-us) Help (https://www.pastest.com/help)
Pastest 2016

https://mypastest.pastest.com/Secure/TestMe/Browser/429893#Top

2/2

8/12/2016

MyPastest

Back to Filters (/Secure/TestMe/Filter/429893/QA)

Question 76 of 179

A young woman has been taking the combined oral contraceptive pill regularly. Her GP treats
her for an acute skin infection. Seven weeks later she presents with irregular bleeding, and is
found to have a positive pregnancy test.
Which antibiotic medication is most likely to have been prescribed by the GP?
A

Amoxicillin

Cefalexin

Cefuroxime

Rifampicin

Tetracycline

Explanation

The answer is Rifampicin


Rifampicin is highly effective against Staphylococcus infection, although it is not
normally used as a first-line therapy in view of adverse effects including hepatitis. It is a
powerful enzyme inducer that increases the rate and extent of hepatic metabolism of the
oral contraceptive pill and other medications, including warfarin. Rifampicin treatment
will give rise to lower oestrogen concentrations and pill failure.

Amoxicillin (Option A) is incorrect. Amoxicillin is associated with pill failure in patients who
develop diarrhoea and impaired oestrogen absorption.
Cefalexin (Option B) is incorrect. There is no clear indication that cephalosporins interfere
with the oral contraceptive pill, although broad-spectrum antibiotics may provoke diarrhoea
and impaired gut absorption.
Cefuroxime (Option C) is incorrect. There is no clear indication that cephalosporins interfere
with the oral contraceptive pill, although broad-spectrum antibiotics may provoke diarrhoea
and impaired gut absorption.

https://mypastest.pastest.com/Secure/TestMe/Browser/429893#Top

1/2

8/12/2016

MyPastest

Tetracycline (Option E) is incorrect. Tetracyclines may rarely be associated with oral


contraceptive failure, although the mechanism of interaction is poorly understood.
46772

Next Question

Previous Question

Tag Question

Feedback

End Review

Difficulty: Average
Peer Responses

Session Progress
Responses Correct:

Responses Incorrect:

179

Responses Total:

179

Responses - % Correct:

0%

Blog (https://www.pastest.com/blog) About Pastest (https://www.pastest.com/about-us)


Contact Us (https://www.pastest.com/contact-us) Help (https://www.pastest.com/help)
Pastest 2016

https://mypastest.pastest.com/Secure/TestMe/Browser/429893#Top

2/2

8/12/2016

MyPastest

Back to Filters (/Secure/TestMe/Filter/429893/QA)

Question 77 of 179

You see a 59-year-old man who is about to commence a course of chemotherapy including
doxorubicin, and you wish to monitor for potential toxicity.
Which would be the most appropriate investigation to carry out before starting the
chemotherapy?
A

Creatinine clearance

CT brain

Echocardiogram

EEG

Lung function test

Explanation

The answer is Echocardiogram


The antitumour activity of doxorubicin derives partly from the triggering of
topoisomerase II-dependent DNA breakage. Doxorubicin can cause cardiotoxicity,
including cardiomyopathy, acute arrhythmias, congestive heart failure and dosedependent impairment of left ventricular ejection fraction; myocardial dysfunction is
most likely in patients that receive a cumulative dose of greater than 500 mg/m2.

Creatinine clearance (Option A) is incorrect. Nephrotoxicity is a more characteristic feature


of cisplatin therapy.
CT brain (Option B) is incorrect. Doxorubicin is not expected to significantly increase risk of
neurological toxicity.
EEG (Option D) is incorrect. Doxorubicin is not expected to increase seizure risk or cause
neurological toxicity.
Lung function test (Option E) is incorrect. Pulmonary adverse effects may be more likely to
occur after bleomycin or methotrexate therapy.
46798

https://mypastest.pastest.com/Secure/TestMe/Browser/429893#Top

1/2

8/12/2016

MyPastest

Next Question

Previous Question

Tag Question

Feedback

End Review

Difficulty: Average
Peer Responses

Session Progress
Responses Correct:

Responses Incorrect:

179

Responses Total:

179

Responses - % Correct:

0%

Blog (https://www.pastest.com/blog) About Pastest (https://www.pastest.com/about-us)


Contact Us (https://www.pastest.com/contact-us) Help (https://www.pastest.com/help)
Pastest 2016

https://mypastest.pastest.com/Secure/TestMe/Browser/429893#Top

2/2

8/12/2016

MyPastest

Back to Filters (/Secure/TestMe/Filter/429893/QA)

Question 78 of 179

Antimicrobial drugs possess a variety of different mechanisms of bactericidal and


bacteriostatic action.
Which one of the following antimicrobial agents is most likely to exert its pharmacological
actions through binding to bacterial ribosomes to disrupt protein synthesis?
A

Azithromycin

Ceftriaxone

Ciprofloxacin

Penicillin

Trimethoprim

Explanation

The answer is Azithromycin


Antimicrobial drugs Macrolides (eg erythromycin, azithromycin and clarithromycin),
aminoglycosides and chloramphenicol all effect their pharmacological actions through
binding to bacterial ribosomes and disrupting protein synthesis.

Ceftriaxone (Option B) is incorrect. Penicillins and cephalosporins interfere with bacterial


cell-wall synthesis.
Ciprofloxacin (Option C) is incorrect. Quinolones (eg ciprofloxacin, moxifloxacin) inhibit DNA
gyrase.
Penicillin (Option D) is incorrect. Penicillins and cephalosporins interfere with bacterial cellwall synthesis.
Trimethoprim (Option E) is incorrect. Trimethoprim and sulfonamides inhibit bacterial folate
synthesis.
46620

Next Question
https://mypastest.pastest.com/Secure/TestMe/Browser/429893#Top

1/2

8/12/2016

MyPastest

Previous Question

Tag Question

Feedback

End Review

Difficulty: Average
Peer Responses

Session Progress
Responses Correct:

Responses Incorrect:

179

Responses Total:

179

Responses - % Correct:

0%

Blog (https://www.pastest.com/blog) About Pastest (https://www.pastest.com/about-us)


Contact Us (https://www.pastest.com/contact-us) Help (https://www.pastest.com/help)
Pastest 2016

https://mypastest.pastest.com/Secure/TestMe/Browser/429893#Top

2/2

8/12/2016

MyPastest

Back to Filters (/Secure/TestMe/Filter/429893/QA)

Question 79 of 179

While prescribing a medication, a doctor tells his patient that its major mode of action is
through direct drug action, rather than achieving its effects through a metabolite.
Which of the following medications would best fit in this category?
A

Chloral hydrate

Diazepam

Enalapril

Oxcarbazepine

Vitamin D

Explanation

The answer is Diazepam


Diazepam is the only drug listed that does not require metabolism to an active
metabolite in order to exert its therapeutic effects.

Chloral hydrate (Option A) is incorrect. Chloral hydrate is rapidly metabolised to


trichloroethanol, which is the active molecule.
Enalapril (Option C) is incorrect. Enalapril is a prodrug that requires metabolism to
enalaprilat in order to inhibit ACE.
Oxcarbazepine (Option D) is incorrect. The major metabolite of oxcarbazepine is a 10monohydroxy derivative that exerts its therapeutic effects.
Vitamin D (Option E) is incorrect. Vitamin D (cholecalciferol) is converted by the liver to 25hydroxycholecalciferol, and then by the kidney tubule enzyme 1-hydroxylase to 1,25dihydrocholecalciferol; this metabolite is much more biologically active than the
unhydroxylated or monohydroxylated moieties.
46857

Next Question
https://mypastest.pastest.com/Secure/TestMe/Browser/429893#Top

1/2

8/12/2016

MyPastest

Previous Question

Tag Question

Feedback

End Review

Difficulty: Average
Peer Responses

Session Progress
Responses Correct:

Responses Incorrect:

179

Responses Total:

179

Responses - % Correct:

0%

Blog (https://www.pastest.com/blog) About Pastest (https://www.pastest.com/about-us)


Contact Us (https://www.pastest.com/contact-us) Help (https://www.pastest.com/help)
Pastest 2016

https://mypastest.pastest.com/Secure/TestMe/Browser/429893#Top

2/2

8/12/2016

MyPastest

Back to Filters (/Secure/TestMe/Filter/429893/QA)

Question 80 of 179

A 56-year-old man has recently been diagnosed with type-2 diabetes. His HbA1c after a
period of diet and exercise is still raised at 8.4%. You are considering initial drug treatment.
Which of the following has a primary mode of action that is most likely to lead to short-term
insulin release, e.g. when taken just before a meal?
A

Alpha-glucosidase inhibitor

Glitazones

Meglitinides

Metformin

Sulfonylureas

Explanation

The answer is Meglitinides


Meglitinides (nateglinide and repaglinide) specifically increase postprandial insulin
release. Meglitinides may allow better glycaemic control by addressing short-term fasting
or working in shift patterns.

Alpha-glucosidase inhibitor (Option A) is incorrect. Alpha-glucosidase inhibitors, eg


acarbose, act by inhibiting the intestinal breakdown of long-chain sugars, which leads to a
reduced peak of glucose after meals. Side-effects include flatulence and diarrhoea due to
bacterial metabolism of unabsorbed sugars.
Glitazones (Option B) is incorrect. Glitazones, although having some effect on hepatic
glucose output, exert their primary glucose-lowering effects by increasing glucose uptake
into skeletal muscle and fat.
Metformin (Option D) is incorrect. Metformin is a partial insulin sensitiser, exerting its primary
effects on blood glucose via a reduction in hepatic glucose output. Metformin is
contraindicated in patients with severe hepatic disease or where the serum creatinine level is
above 130 mol/l in women and 150 mol/l in men.
https://mypastest.pastest.com/Secure/TestMe/Browser/429893#Top

1/2

8/12/2016

MyPastest

Sulfonylureas (Option E) is incorrect. Sulfonylureas, eg gliclazide, increase insulin release


over a prolonged period and may be associated with an increased incidence of
hypoglycaemia compared with the meglitinides.
46832

Next Question

Previous Question

Tag Question

Feedback

End Review

Difficulty: Average
Peer Responses

Session Progress
Responses Correct:

Responses Incorrect:

179

Responses Total:

179

Responses - % Correct:

0%

Blog (https://www.pastest.com/blog) About Pastest (https://www.pastest.com/about-us)


Contact Us (https://www.pastest.com/contact-us) Help (https://www.pastest.com/help)
Pastest 2016

https://mypastest.pastest.com/Secure/TestMe/Browser/429893#Top

2/2

8/12/2016

MyPastest

Back to Filters (/Secure/TestMe/Filter/429893/QA)

Question 81 of 179

Drug metabolism may be subject to genetic variation, for example acetylation status.
Which of the following statements best describes the influence of acetylator status?
A

Acetylation status is an autosomal-dominant trait

Adverse drug effects are normally more severe in rapid acetylators

Drug acetylation is an example of drug conjugation

Explains the variation in adverse effects of penicillin

Predicts the clinical response to isoniazid

Explanation

The answer is Predicts the clinical response to isoniazid


Drug acetylation is a phase-II reaction, leading to the synthesis of water-soluble
drugs/drug metabolites. Genetic variation accounts for slow acetylator or fast acetylator
status. In most cases, slow acetylators have higher concentrations of drug due to slower
metabolism and clearance, therefore slow acetylators may be more prone to adverse
drug effects. This includes dapsone-induced haemolysis, isoniazid-induced neuropathy,
hydralazine-induced lupus and sulfasalazine-induced haemolysis.

Acetylation status is an autosomal-dominant trait (Option A) is incorrect. Acetylation status


is inherited as an autosomal-recessive trait.
Adverse drug effects are normally more severe in rapid acetylators (Option B) is incorrect.
Adverse effects are normally greater in slow acetylators. One exception is that rapid
acetylators may be more prone to isoniazid-induced hepatitis, which is caused by the
acetylation metabolite.
Drug acetylation is an example of drug conjugation (Option C) is incorrect. Acetylation is a
phase-II reaction, not drug conjugation.
Explains the variation in adverse effects of penicillin (Option D) is incorrect. Penicillin is not
extensively metabolised by acetylation.
46487

https://mypastest.pastest.com/Secure/TestMe/Browser/429893#Top

1/2

8/12/2016

MyPastest

46487

Next Question

Previous Question

Tag Question

Feedback

End Review

Difficulty: Average
Peer Responses

Session Progress
Responses Correct:

Responses Incorrect:

179

Responses Total:

179

Responses - % Correct:

0%

Blog (https://www.pastest.com/blog) About Pastest (https://www.pastest.com/about-us)


Contact Us (https://www.pastest.com/contact-us) Help (https://www.pastest.com/help)
Pastest 2016

https://mypastest.pastest.com/Secure/TestMe/Browser/429893#Top

2/2

8/12/2016

MyPastest

Back to Filters (/Secure/TestMe/Filter/429893/QA)

Question 82 of 179

A 72-year-old man is admitted unconscious. He has a history of type-2 diabetes and is taking
10 mg of glibenclamide. Blood testing reveals a serum creatinine level of 125 mol/l and a
blood glucose of 1.5 mmol/l.
Which treatment regime may be a suitable alternative therapy for his diabetes?
A

Chlorpropamide

Metformin

Metformin and insulin combination therapy

Pioglitazone added to glibenclamide

Pioglitazone and insulin combination therapy

Explanation

The answer is Metformin


Metformin avoids hypoglycaemia, although it is contraindicated in patients with renal
impairment; discontinuation is recommended when creatinine levels are above 130
mol/l in women and 150 mol/l in men.

Chlorpropamide (Option A) is incorrect. Chlorpropamide and glibenclamide are long-acting


sulfonylureas, and are contraindicated in elderly patients with renal impairment because of
the risk of hypoglycaemia.
Metformin and insulin combination therapy (Option C) is incorrect. The combination of
metformin and insulin would be associated with increased complexity and risk of
hypoglycaemia.
Pioglitazone added to glibenclamide (Option D) is incorrect. Pioglitazone would avoid
hypoglycaemia, and in patients with creatinine higher than 150 mol/l, it would be a
preferred option to metformin.
Pioglitazone and insulin combination therapy (Option E) is incorrect. The combination of
pioglitazone and insulin would be associated with increased complexity and risk of
https://mypastest.pastest.com/Secure/TestMe/Browser/429893#Top

1/2

8/12/2016

MyPastest

hypoglycaemia.
46831

Next Question

Previous Question

Tag Question

Feedback

End Review

Difficulty: Average
Peer Responses

Session Progress
Responses Correct:

Responses Incorrect:

179

Responses Total:

179

Responses - % Correct:

0%

Blog (https://www.pastest.com/blog) About Pastest (https://www.pastest.com/about-us)


Contact Us (https://www.pastest.com/contact-us) Help (https://www.pastest.com/help)
Pastest 2016

https://mypastest.pastest.com/Secure/TestMe/Browser/429893#Top

2/2

8/12/2016

MyPastest

Back to Filters (/Secure/TestMe/Filter/429893/QA)

Question 83 of 179

A 26-year-old woman, who is 9 weeks pregnant, has two episodes of convulsions. She had a
history of generalised seziures 18 months before but has not received any treatment. You
decide that it would be safest to commence antiepileptic drug treatment. She wants advice
about the antiepileptic drugs and risks to her pregnancy.
Which treatment would convey the greatest risk of congenital malformations?
A

Carbamazepine

Lamotrigine

Phenytoin

Sodium valproate

Topiramate

Explanation

The answer is Sodium valproate


It is difficult to define a safe medication in pregnancy, but the teratogenic risk of sodium
valproate is very well characterised and seems to pose at least as great a hazard
compared to other agents. There is an approximately 7% rate of congenital major
malformations, including neural tube defects and craniofacial abnormalities; risks are
even higher for patients taking valproate plus other antiepileptic drugs. The background
population rate of congenital malformations is around 1.5%. Counselling should be given
to all women of childbearing capacity who are prescribed antiepileptic drugs.

Carbamazepine (Option A) is incorrect. The rate of congenital malformations is around 3%


for carbamazepine.
Lamotrigine (Option B) is incorrect. The rate of congenital malformations is around 2% for
lamotrigine.
Phenytoin (Option C) is incorrect. The rate of congenital malformations is around 4% for
phenytoin.
https://mypastest.pastest.com/Secure/TestMe/Browser/429893#Top

1/2

8/12/2016

MyPastest

Topiramate (Option E) is incorrect. The rate of congenital malformations is around 5% for


topiramate.
46979

Next Question

Previous Question

Tag Question

Feedback

End Review

Difficulty: Average
Peer Responses

Session Progress
Responses Correct:

Responses Incorrect:

179

Responses Total:

179

Responses - % Correct:

0%

Blog (https://www.pastest.com/blog) About Pastest (https://www.pastest.com/about-us)


Contact Us (https://www.pastest.com/contact-us) Help (https://www.pastest.com/help)
Pastest 2016

https://mypastest.pastest.com/Secure/TestMe/Browser/429893#Top

2/2

8/12/2016

MyPastest

Back to Filters (/Secure/TestMe/Filter/429893/QA)

Question 84 of 179

You are asked to see a 24-year-old patient in the Emergency Department, who has recently
returned from a trip to Thailand. He is in police custody and complaining of generalised
abdominal discomfort. You are suspicious that he might be a body packer.
Which of the following statements represents the most appropriate initial management?
A

Gastric lavage should be avoided

Normal abdominal X-ray appearances exclude the diagnosis

Patients may be discharged if asymptomatic after 4 hours

Surgical intervention should be avoided in symptomatic patients

Whole-bowel irrigation is contraindicated

Explanation

The answer is Gastric lavage should be avoided


Management of body packers Gastric lavage may increase the risk of package rupture
and may provoke severe toxicity. Abdominal X-rays may show packages within the
gastrointestinal tract, but not always. A CT abdominal scan is better at identifying the
presence of drug packages. Whole-bowel irrigation with polyethylene glycol will clear
the gastrointestinal tract of all the swallowed packages. Upper endoscopy may also be
useful in removing packages that are still in the stomach, but caution is required to avoid
disrupting the packaging and drug release. Urgent surgical intervention may be needed
to remove drug packages in patients who develop signs of drug toxicity because rupture
of packages may be fatal, depending upon the quantity and type of drug in each
package.

Normal abdominal X-ray appearances exclude the diagnosis (Option B) is incorrect. Packages
may not be detectable on plain abdominal X-ray.
Patients may be discharged if asymptomatic after 4 hours (Option C) is incorrect. Patients
are at risk of developing toxicity until drug packages are cleared from the body.

https://mypastest.pastest.com/Secure/TestMe/Browser/429893#Top

1/2

8/12/2016

MyPastest

Surgical intervention should be avoided in symptomatic patients (Option D) is incorrect. In


patients that develop toxicity, surgical intervention may be needed to prevent severe or fatal
toxicity depending upon the type and quantity of drug packages.
Whole-bowel irrigation is contraindicated (Option E) is incorrect. This is often a useful
strategy for enhancing removal of packages from the body.
45805

Next Question

Previous Question

Tag Question

Feedback

End Review

Difficulty: Average
Peer Responses

Session Progress
Responses Correct:

Responses Incorrect:

179

Responses Total:

179

Responses - % Correct:

0%

Blog (https://www.pastest.com/blog) About Pastest (https://www.pastest.com/about-us)


Contact Us (https://www.pastest.com/contact-us) Help (https://www.pastest.com/help)
Pastest 2016

https://mypastest.pastest.com/Secure/TestMe/Browser/429893#Top

2/2

8/12/2016

MyPastest

Back to Filters (/Secure/TestMe/Filter/429893/QA)

Question 85 of 179

A 44-year-old woman has a fasting total cholesterol level of 6.6 mmol/l and triglycerides of
6.5 mmol/l. Dietary intervention has unfortunately shown no benefit and therefore therapy is
needed because of her risk factors.
Which one of the following is the most appropriate therapy?
A

Aspirin

Bezafibrate

Cholestyramine

Enalapril

Pravastatin

Explanation

The answer is Bezafibrate


In this patient, the predominant problem is hypertriglyceridaemia, with modest
hypercholesterolaemia. Fibrate drugs diminish serum triglyceride levels by stimulating
lipoprotein lipase and decreasing non-esterified fatty acids (NEFA) in the circulation,
probably by stimulating nuclear peroxisome proliferator-activated receptor-. Fibrate
drugs (bezafibrate, ciprofibrate, fenofibrate, gemfibrozil) are first-line therapy; they are
also often highly effective in type-III and primary type-V hyperlipoproteinaemia, and the
dyslipoproteinaemia of diabetes mellitus. Fibrates decrease very-low-density lipoprotein
cholesterol but are less effective than statins in lowering low-density lipoprotein
cholesterol. In some particularly high-risk patients with combined hyperlipidaemia, statin
therapy may be added to fibrate therapy to satisfactorily lower LDL-cholesterol, but this
poses a greater risk of myopathy and close monitoring is required.

Aspirin (Option A) is incorrect. Aspirin has no role in the treatment of hypertriglyceridaemia.


Cholestyramine (Option C) is incorrect. Cholestyramine interferes with dietary fat absorption
and fat-soluble vitamins, but has only a modest effect on circulating triglycerides compared
to fibrates.
https://mypastest.pastest.com/Secure/TestMe/Browser/429893#Top

1/2

8/12/2016

MyPastest

Enalapril (Option D) is incorrect. Enalapril has no significant effect on lipid profile.


Pravastatin (Option E) is incorrect. Pravastatin is effective in lowering LDL-cholesterol but
has only a modest effect on triglyceride concentrations. Other more potent statins including
atorvastatin and rosuvastatin are capable of lowering triglyceride concentrations, but to a
lesser extent than fibrates.
46794

Next Question

Previous Question

Tag Question

Feedback

End Review

Difficulty: Average
Peer Responses

Session Progress
Responses Correct:

Responses Incorrect:

179

Responses Total:

179

Responses - % Correct:

0%

Blog (https://www.pastest.com/blog) About Pastest (https://www.pastest.com/about-us)


Contact Us (https://www.pastest.com/contact-us) Help (https://www.pastest.com/help)
Pastest 2016

https://mypastest.pastest.com/Secure/TestMe/Browser/429893#Top

2/2

8/12/2016

MyPastest

Back to Filters (/Secure/TestMe/Filter/429893/QA)

Question 86 of 179

A 30-year-old woman was started on carbamazepine for partial complex seizures and was
also advised to discontinue her moderate alcohol consumption. Therapeutic concentrations
of carbamazepine were achieved within 4 days with a dose of 200 mg daily, but the dose
needed to be increased to 400 mg daily within 2 weeks to achieve a therapeutic plasma
concentration.
Which one of the following is the most likely reason for needing to increase the
carbamazepine dose?
A

Auto-induction of carbamazepine metabolism

Auto-inhibition of carbamazepine metabolism

Cessation of alcohol intake

Concomitant prescription of the oral contraceptive pill

Reduced bioavailability of carbamazepine

Explanation

The answer is Auto-induction of carbamazepine metabolism


Carbamazepine is a hepatic enzyme inducer, and hence the dose of carbamazepine may
need to be increased after a few weeks of therapy to achieve a steady plasma
concentration. Its properties as an enzyme inducer mean that carbamazepine often
lowers plasma concentrations of clonazepam, lamotrigine and phenytoin, so when antiepileptics are combined it is sensible to be wary of such interactions. Contraindications
for carbamazepine include atrioventricular (AV) conduction abnormalities, porphyria and
history of bone marrow depression. Common adverse effects include nausea and
vomiting, drowsiness, dizziness and headache.

Auto-inhibition of carbamazepine metabolism (Option B) is incorrect. Carbamazepine


induces liver enzyme activity.
Cessation of alcohol intake (Option C) is incorrect. Moderate to heavy alcohol consumption
may increase hepatic enzyme activity; cessation may result in a gradual fall in enzyme
https://mypastest.pastest.com/Secure/TestMe/Browser/429893#Top

1/2

8/12/2016

MyPastest

activity so that carbamazepine concentrations may rise.


Concomitant prescription of the oral contraceptive pill (Option D) is incorrect.
Carbamazepine should not be used with the contraceptive pill because enzyme induction
may lower concentrations of the pill so that it may become ineffective. The contraceptive pill
would not be expected to have any direct effect on carbamazepine concentrations.
Reduced bioavailability of carbamazepine (Option E) is incorrect. Carbamazepine absorption
is not likely to alter after continued therapy.
46922

Next Question

Previous Question

Tag Question

Feedback

End Review

Difficulty: Average
Peer Responses

Session Progress
Responses Correct:

Responses Incorrect:

179

Responses Total:

179

Responses - % Correct:

0%

Blog (https://www.pastest.com/blog) About Pastest (https://www.pastest.com/about-us)


Contact Us (https://www.pastest.com/contact-us) Help (https://www.pastest.com/help)
Pastest 2016

https://mypastest.pastest.com/Secure/TestMe/Browser/429893#Top

2/2

8/12/2016

MyPastest

Back to Filters (/Secure/TestMe/Filter/429893/QA)

Question 87 of 179

A 6-month-old child has sensorineural deafness and a ventricular septal defect. Her mother
had received medications during the pregnancy, and asks you for advice about whether these
might be relevant to her sons medical problems.
Which of the following drugs taken during pregnancy would be most likely to have caused
these clinical effects?
A

Clindamycin

Cyproterone acetate

Isotretinoin

Minocycline

Oxytetracyline

Explanation

The answer is Isotretinoin


Isotretinoin is a highly effective treatment for acne but it is highly teratogenic. Women
must have a negative pregnancy test before treatment and be taking effective
contraception for at least a month before the course begins, during the course and for a
month after it finishes. Congenital deafness and central nervous system and heart
defects may occur in children exposed to isotretinoin in utero.

Clindamycin (Option A) is incorrect. Clindamycin may be teratogenic, but limited data are
available.
Cyproterone acetate (Option B) is incorrect. Cyproterone acetate may cause feminisation of
male fetuses.
Minocycline (Option D) is incorrect. Tetracyclines may cause discoloured teeth in infants
exposed in utero.
Oxytetracyline (Option E) is incorrect. Tetracyclines may cause discoloured teeth in infants
exposed in utero.
46752

https://mypastest.pastest.com/Secure/TestMe/Browser/429893#Top

1/2

8/12/2016

MyPastest

46752

Next Question

Previous Question

Tag Question

Feedback

End Review

Difficulty: Average
Peer Responses

Session Progress
Responses Correct:

Responses Incorrect:

179

Responses Total:

179

Responses - % Correct:

0%

Blog (https://www.pastest.com/blog) About Pastest (https://www.pastest.com/about-us)


Contact Us (https://www.pastest.com/contact-us) Help (https://www.pastest.com/help)
Pastest 2016

https://mypastest.pastest.com/Secure/TestMe/Browser/429893#Top

2/2

8/12/2016

MyPastest

Back to Filters (/Secure/TestMe/Filter/429893/QA)

Question 88 of 179

A 65-year-old patient with asthma was started on theophylline due to poor control of
symptoms, despite other regular inhaler therapy. Other relevant past history includes
treatment for heart failure and hepatic steatosis, which appear to be stable, and he is
receiving long-term low-dose erythromycin for treatment of rosacea. He continues to smoke
20 cigarettes per day, but after discussion with the clinic nurse today he has decided to quit.
Which of the following factors is most likely to require reduction of theophylline dose?
A

Age above 60 years

Concomitant use of erythromycin

Heart failure

Liver steatosis

Smoking cessation

Explanation

The answer is Smoking cessation


Theophylline is a bronchodilator used to treat reversible airway obstruction and may
cause significant adverse effects, including nausea and vomiting, hypokalaemia and
metabolic acidosis. It has a narrow therapeutic window, and therapeutic drug monitoring
is needed to minimise the risk of drug toxicity, typically aiming to achieve concentrations
between 10 and 20 mg/l. Regular tobacco use up-regulates hepatic enzyme activity;
cessation will be associated with a decrease of hepatic enzyme activity, such that
theophylline concentrations may increase. Other factors that may increase theophylline
drug concentrations and provoke toxicity include heart failure, liver cirrhosis, acute viral
infection, increased age, drugs that inhibit its metabolism (eg erythromycin), or cessation
of enzyme-inducing drugs.

Age above 60 years (Option A) is incorrect. Advancing age may result in a need for lower
theophylline doses due to less extensive liver metabolism. This is unlikely to require a
dramatic change in dose provided that there is therapeutic drug monitoring.
https://mypastest.pastest.com/Secure/TestMe/Browser/429893#Top

1/2

8/12/2016

MyPastest

Concomitant use of erythromycin (Option B) is incorrect. Addition of erythromycin would


certainly risk an acute rise in theophylline concentrations and may provoke toxicity, but if the
patient was stable on long-term combined treatment this is much less likely to occur.
Heart failure (Option C) is incorrect. Heart failure may result in a need for lower theophylline
doses, but if clinical features of heart failure are stable, this is unlikely to change dramatically.
Liver steatosis (Option D) is incorrect. Impaired liver metabolism, e.g. in the presence of
drugs such as erythromycin or levofloxacin, may cause a significant acute rise in theophylline
concentrations and provoke toxicity. Hepatic steatosis is unlikely to cause dramatic changes
in the rate or extent of drug metabolism.
46801

Next Question

Previous Question

Tag Question

Feedback

End Review

Difficulty: Average
Peer Responses

Session Progress
Responses Correct:

Responses Incorrect:

179

Responses Total:

179

Responses - % Correct:

0%

Blog (https://www.pastest.com/blog) About Pastest (https://www.pastest.com/about-us)


Contact Us (https://www.pastest.com/contact-us) Help (https://www.pastest.com/help)
Pastest 2016

https://mypastest.pastest.com/Secure/TestMe/Browser/429893#Top

2/2

8/12/2016

MyPastest

Back to Filters (/Secure/TestMe/Filter/429893/QA)

Question 89 of 179

You are telephoned by the biochemistry laboratory to report a patient has potassium
concentration 5.9 mmol/l.
Which one of the following factors is most likely to account for hyperkalaemia?
A

Bartter syndrome

Ciclosporin

Liddle syndrome

Liquorice addiction

Treatment with corticosteroids

Explanation

The answer is Ciclosporin Ciclosporin is used as an immunosuppressant after renal transplant. Hyperkalaemia is a
common adverse effect, and patients treated with ciclosporin may have serum potassium
concentrations in the range 6.07.1 mmol/l. The mechanism is probably a variant of
hyporeninaemic hypoaldosteronism, and is responsive to fludrocortisone.

Bartter syndrome (Option A) is incorrect. Bartter syndrome is associated with hypokalaemia


due to a number of inherited defects of renal function.
Liddle syndrome (Option C) is incorrect. Liddle syndrome is a rare condition characterised by
hypokalaemia, hypertension and low aldosterone levels.
Liquorice addiction (Option D) is incorrect. Liquorice inhibits 11-hydroxysteroid
dehydrogenase causing potassium wasting from the distal tubule.
Treatment with corticosteroids (Option E) is incorrect. Corticosteroids are associated with
hypokalaemia, due to their mineralocorticoid effects.
46732

Next Question
https://mypastest.pastest.com/Secure/TestMe/Browser/429893#Top

1/2

8/12/2016

MyPastest

Previous Question

Tag Question

Feedback

End Review

Difficulty: Average
Peer Responses

Session Progress
Responses Correct:

Responses Incorrect:

179

Responses Total:

179

Responses - % Correct:

0%

Blog (https://www.pastest.com/blog) About Pastest (https://www.pastest.com/about-us)


Contact Us (https://www.pastest.com/contact-us) Help (https://www.pastest.com/help)
Pastest 2016

https://mypastest.pastest.com/Secure/TestMe/Browser/429893#Top

2/2

8/12/2016

MyPastest

Back to Filters (/Secure/TestMe/Filter/429893/QA)

Question 90 of 179

A 29-year-old woman has been taking antiepileptic medications for several years. She
presents to the Emergency Department with progressive imbalance and clumsiness, and has
fallen twice at home. Speech is slurred and she denies drinking any alcohol.
Which one of the following medications is most likely to be responsible for these symptoms?
A

Carbamazepine

Lamotrigine

Phenobarbital

Sodium valproate

Vigabatrin

Explanation

The answer is Carbamazepine


Adverse effects of carbamazepine include ataxia, nystagmus and diplopia. These are
normally dose-dependent and reversible after discontinuing or reducing the dose of
regular therapy. Phenytoin may also cause cerebellar ataxia, and other adverse effects
include gum hypertrophy, hirsutism, folate deficiency, coarse facial features and
osteomalacia.

Lamotrigine (Option B) is incorrect. Lamotrigine may cause significant skin effects including
blistering rashes and StevensJohnston syndrome, nightmares, blurred vision and dizziness.
Phenobarbital (Option C) is incorrect. Phenobarbital may cause folate deficiency,
megaloblastic anaemia, osteomalacia and neuropathy.
Sodium valproate (Option D) is incorrect. Sodium valproate may cause hair loss and alopecia,
liver damage, pancreatitis and weight gain.
Vigabatrin (Option E) is incorrect. Vigabatrin may cause aggression, alopecia, retinal atrophy,
reduced peripheral vision, thrombocytopaenia and other blood dyscrasias.
46757

https://mypastest.pastest.com/Secure/TestMe/Browser/429893#Top

1/2

8/12/2016

MyPastest

Next Question

Previous Question

Tag Question

Feedback

End Review

Difficulty: Average
Peer Responses

Session Progress
Responses Correct:

Responses Incorrect:

179

Responses Total:

179

Responses - % Correct:

0%

Blog (https://www.pastest.com/blog) About Pastest (https://www.pastest.com/about-us)


Contact Us (https://www.pastest.com/contact-us) Help (https://www.pastest.com/help)
Pastest 2016

https://mypastest.pastest.com/Secure/TestMe/Browser/429893#Top

2/2

8/12/2016

MyPastest

Back to Filters (/Secure/TestMe/Filter/429893/QA)

Question 91 of 179

A patient with congestive heart failure requires furosemide therapy for his oedema. On his
follow-up visit you note that serum potassium concentration has fallen to 3.1 mmol/l.
What would be the most appropriate next step in his management?
A

Add a small dose of amiloride to furosemide

Add metolazone to his furosemide

Change furosemide to bumetanide

Change furosemide to hydrochlorothiazide

Consider a potassium infusion

Explanation

The answer is Add a small dose of amiloride to furosemide


When hypokalaemia occurs after loop diuretic therapy, the best choice is to add a
potassium sparing diuretic, eg amiloride or spironolactone. Amiloride inhibits sodium
channels in the distal segment of the distal convoluted tubule.

Add metolazone to his furosemide (Option B) is incorrect. Addition of metolazone will


provoke the worsened hypokalaemia.
Change furosemide to bumetanide (Option C) is incorrect. Loop diuretics furosemide and
bumetanide inhibit Na+/K+/Cl co-transport in the ascending limb of Henles loop, and there
is little difference between them.
Change furosemide to hydrochlorothiazide (Option D) is incorrect. Thiazide diuretics inhibit
Na+/Cl co-transport in the proximal segment of the distal convoluted tubule. They are also
associated with the occurrence of hypokalaemia.
Consider a potassium infusion (Option E) is incorrect. Potassium administration fails to
address the renal potassium loss, so this will offer only temporary correction.
46819

https://mypastest.pastest.com/Secure/TestMe/Browser/429893#Top

1/2

8/12/2016

MyPastest

Next Question

Previous Question

Tag Question

Feedback

End Review

Difficulty: Average
Peer Responses

Session Progress
Responses Correct:

Responses Incorrect:

179

Responses Total:

179

Responses - % Correct:

0%

Blog (https://www.pastest.com/blog) About Pastest (https://www.pastest.com/about-us)


Contact Us (https://www.pastest.com/contact-us) Help (https://www.pastest.com/help)
Pastest 2016

https://mypastest.pastest.com/Secure/TestMe/Browser/429893#Top

2/2

8/12/2016

MyPastest

Back to Filters (/Secure/TestMe/Filter/429893/QA)

Question 92 of 179

A 35-year-old woman on carbamazepine for epilepsy was found to have Hashimotos


thyroiditis and has been prescribed thyroxine by her GP. She comes back to the clinic a week
later complaining of fatigue, depression, weight gain and constipation. Her tri-iodothyronine
and thyroxine levels are found to be low despite the proper intake of medications.
What is the most likely cause of her symptoms?
A

Adverse reaction to carbamazepine

Direct effect of carbamazepine on thyroxine formation

Increased binding of thyroxine by thyroxine-binding globulin

Increased thyroxine clearance by the action of carbamazepine on liver enzymes

Interference with intestinal absorption of thyroxine by carbamazepine

Explanation

The answer is option Increased thyroxine clearance by the action of carbamazepine on


liver enzymes
Hepatic enzyme induction, caused by carbamazepine, phenytoin, rifampicin and other
drugs, accelerates the metabolism of a number of co-administered drugs. In patients on
carbamazepine who develop Hashimotos thyroiditis the dose of thyroxine should be
increased to maintain therapeutic levels. The symptoms in this patient, who has
developed Hashimotos thyroiditis while taking carbamazepine, ie fatigue, depression,
weight gain and constipation, are due to inadequate treatment of hypothyroidism. Sideeffects of carbamazepine include drowsiness, ataxia, nystagmus, diplopia, rash, and
thrombocytopaenia and other blood dyscrasias.

Adverse reaction to carbamazepine (Option A) is incorrect. Carbamazepine may cause


severe skin reactions, and cerebellar toxicity.
Direct effect of carbamazepine on thyroxine formation (Option B) is incorrect.
Carbamazepine does not have a direct impact on thyroxine production (in contrast to
carbimazole!).
https://mypastest.pastest.com/Secure/TestMe/Browser/429893#Top

1/2

8/12/2016

MyPastest

Increased binding of thyroxine by thyroxine-binding globulin (Option C) is incorrect. Drugs


that compete for binding of thyroxine to circulating thyroxine-binding globulin increase free
thyroxine concentrations and may provoke hyperthyroidism.
Interference with intestinal absorption of thyroxine by carbamazepine (Option E) is incorrect.
Carbamazepine does not interfere with gut absorption, and thyroxine is formed by the
thyroid gland and not absorbed from the gut.
46737

Next Question

Previous Question

Tag Question

Feedback

End Review

Difficulty: Average
Peer Responses

Session Progress
Responses Correct:

Responses Incorrect:

179

Responses Total:

179

Responses - % Correct:

0%

Blog (https://www.pastest.com/blog) About Pastest (https://www.pastest.com/about-us)


Contact Us (https://www.pastest.com/contact-us) Help (https://www.pastest.com/help)
Pastest 2016

https://mypastest.pastest.com/Secure/TestMe/Browser/429893#Top

2/2

8/12/2016

MyPastest

Back to Filters (/Secure/TestMe/Filter/429893/QA)

Question 93 of 179

You are consulted by a 33-year-old woman who is due to go on a cruise. She has been on a
cruise ship previously, but was kept in her room with nausea and vomiting which she thinks
could be attributed to seasickness. She does not wish to have a similar experience on her
next cruise.
What would you prescribe for her to best avoid such symptoms?
A

Cinnarizine

Domperidone

Metoclopramide

Ondansetron

Prochlorperazine

Explanation

The answer is Cinnarizine


Cinnarizine is thought to be particularly useful for the treatment of motion sickness as it
has a dual action: first, it acts as a depressant of the vestibular system; second, it
dampens down smooth muscle contraction in the gut. Patients may experience
drowsiness and some gastrointestinal disturbance.

Domperidone (Option B) is incorrect. Domperidone may be used to treat motion sickness,


but may be less effective than cinnarizine owing to its inability to cross the bloodbrain
barrier and its actions being solely peripheral.
Metoclopramide (Option C) is incorrect. Metoclopramide and prochlorperazine may be
associated with an increased risk of acute dystonia.
Ondansetron (Option D) is incorrect. Ondansetron is predominantly centrally acting via 5HT3 receptors, and may be effective for nausea due to a range of triggers, but is less well
recognised as a treatment for motion sickness.

https://mypastest.pastest.com/Secure/TestMe/Browser/429893#Top

1/2

8/12/2016

MyPastest

Prochlorperazine (Option E) is incorrect. Metoclopramide and prochlorperazine may be


associated with an increased risk of acute dystonia.
46992

Next Question

Previous Question

Tag Question

Feedback

End Review

Difficulty: Average
Peer Responses

Session Progress
Responses Correct:

Responses Incorrect:

179

Responses Total:

179

Responses - % Correct:

0%

Blog (https://www.pastest.com/blog) About Pastest (https://www.pastest.com/about-us)


Contact Us (https://www.pastest.com/contact-us) Help (https://www.pastest.com/help)
Pastest 2016

https://mypastest.pastest.com/Secure/TestMe/Browser/429893#Top

2/2

8/12/2016

MyPastest

Back to Filters (/Secure/TestMe/Filter/429893/QA)

Question 94 of 179

A 65-year-old woman has attended the cardiology outpatient department for review of her
atrial fibrillation. She has been taking amiodarone for the past 3 years. Her only symptoms are
of lethargy, weight gain over the past 6 months and low mood.
Which of the following investigations would be most useful in this case?
A

Blood urea and electrolytes

ECG

Full blood count

Liver function tests

T3, T4 and TSH levels

Explanation

The answer is T3, T4 and TSH levels


Amiodarone is a class III antiarrhythmic drug. It may exert a number of effects on the
thyroid, including:
Commonly a rise in free thyroxine (T4) and a fall in free tri-iodothyronine (T3)
Some 2% of patients have clinically significant changes accompanied by
symptoms of hyperthyroidism or hypothyroidism
As amiodarone has a prolonged half-life, the problems may persist for up to 3
months after withdrawal of the drug.

Blood urea and electrolytes (Option A) is incorrect. These tests would perhaps identify
significant electrolyte disturbance or renal failure; neither is a characteristic adverse effect of
amiodarone.
ECG (Option B) is incorrect. An ECG would be helpful in assessing the patients cardiac
disorder but is less likely to identify a cause of her symptoms than thyroid tests.
https://mypastest.pastest.com/Secure/TestMe/Browser/429893#Top

1/2

8/12/2016

MyPastest

Full blood count (Option C) is incorrect. These tests might identify anaemia or other blood
abnormalities, but these are not a characteristic adverse effect of amiodarone.
Liver function tests (Option D) is incorrect. Amiodarone may cause hepatitis but this is much
less common than effects of thyroid function.
46760

Next Question

Previous Question

Tag Question

Feedback

End Review

Difficulty: Average
Peer Responses

Session Progress
Responses Correct:

Responses Incorrect:

179

Responses Total:

179

Responses - % Correct:

0%

Blog (https://www.pastest.com/blog) About Pastest (https://www.pastest.com/about-us)


Contact Us (https://www.pastest.com/contact-us) Help (https://www.pastest.com/help)
Pastest 2016

https://mypastest.pastest.com/Secure/TestMe/Browser/429893#Top

2/2

8/12/2016

MyPastest

Back to Filters (/Secure/TestMe/Filter/429893/QA)

Question 95 of 179

A 34-year-old woman has an acute attack of migraine, and is given a subcutaneous injection
of sumatriptan that brings about prompt relief of her symptoms.
Which of the following statements best explains the mechanism of action of sumatriptan?
A

Acting on 5-HT receptors in the CNS

Acting on opioid receptors in the central nervous system

Blocking adrenergic receptors

Blocking receptors to acetylcholine

Inhibiting cyclo-oxygenase

Explanation

The answer is Acting on 5-HT receptors in the CNS


Sumatriptan is a selective 5-HT (5-hydroxytryptamine, serotonin) receptor agonist used
to treat migraine in the acute phase. It may be effective in treating both classical and
common migraine. It may be administered by the oral, sublingual, subcutaneous route, or
nasal spray. It causes vasoconstriction of the cranial arteries and it may also act directly
on neurotransmitters involved in the pain cascade within the trigeminal nerve.

Acting on opioid receptors in the central nervous system (Option B) is incorrect. Opioids
exert their analgesic effects via opioid receptors within the central nervous system, but have
little efficacy in acute migraine.
Blocking adrenergic receptors (Option C) is incorrect. Sumatriptan has no effect on
adrenergic pathways.
Blocking receptors to acetylcholine (Option D) is incorrect. Sumatriptan has no effect on
acetylcholine pathways.
Inhibiting cyclo-oxygenase (Option E) is incorrect. NSAIDs exert their effects by inhibiting
cyclo-oxygenase, and may be effective in alleviating headache in acute migraine.
46610

https://mypastest.pastest.com/Secure/TestMe/Browser/429893#Top

1/2

8/12/2016

MyPastest

Next Question

Previous Question

Tag Question

Feedback

End Review

Difficulty: Average
Peer Responses

Session Progress
Responses Correct:

Responses Incorrect:

179

Responses Total:

179

Responses - % Correct:

0%

Blog (https://www.pastest.com/blog) About Pastest (https://www.pastest.com/about-us)


Contact Us (https://www.pastest.com/contact-us) Help (https://www.pastest.com/help)
Pastest 2016

https://mypastest.pastest.com/Secure/TestMe/Browser/429893#Top

2/2

8/12/2016

MyPastest

Back to Filters (/Secure/TestMe/Filter/429893/QA)

Question 96 of 179

A 24-year-old man is brought into the Emergency Department after he was found collapsed
in the street following a suspected drug overdose. A BM test shows hypoglycaemia, and a
subsequent formal laboratory test shows serum glucose 1.8 mmol/l; the patient is treated by
intravenous dextrose administration.
Which of the following drugs is most likely to have been responsible for hypoglycaemia?
A

Iron

Metformin

Prednisolone

Propranolol

Theophylline

Explanation

The answer is Propranolol


A number of medical disorders may give rise to hypoglycaemia. Few medicines are
capable of causing severe hypoglycaemia, notably insulin and sulfonylureas. Other
medications may cause modest hypoglycaemia, including salicylates, sodium valproate,
propranolol and severe iron or paracetamol poisoning. In contrast, a number of
medicines may cause hyperglycaemia, namely corticosteroids, thiazide diuretics,
theophylline, iron (initial period after overdose), caffeine and 2-agonists.

Iron (Option A) is incorrect. Iron causes hyperglycaemia in the initial period after overdose.
Metformin (Option B) is incorrect. Metformin is an antidiabetic drug that does not cause
hypoglycaemia, even after substantial overdose, but is a recognised cause of severe lactic
acidosis.
Prednisolone (Option C) is incorrect. Prednisolone causes hyperglycaemia.
Theophylline (Option E) is incorrect. Theophylline may cause hyperglycaemia.
45801

https://mypastest.pastest.com/Secure/TestMe/Browser/429893#Top

1/2

8/12/2016

MyPastest

Next Question

Previous Question

Tag Question

Feedback

End Review

Difficulty: Average
Peer Responses

Session Progress
Responses Correct:

Responses Incorrect:

179

Responses Total:

179

Responses - % Correct:

0%

Blog (https://www.pastest.com/blog) About Pastest (https://www.pastest.com/about-us)


Contact Us (https://www.pastest.com/contact-us) Help (https://www.pastest.com/help)
Pastest 2016

https://mypastest.pastest.com/Secure/TestMe/Browser/429893#Top

2/2

8/12/2016

MyPastest

Back to Filters (/Secure/TestMe/Filter/429893/QA)

Question 97 of 179

A 45-year-old man has a past history of hypertension and epilepsy. He has been receiving
treatment with phenytoin, clobazam, lisinopril, cimetidine, sucralfate and allopurinol. He now
presents to the Emergency Department with progressive ataxia, slurred speech and blurred
vision.
Which recently added drug is most likely to be responsible for development of his
symptoms?
A

Allopurinol

Cimetidine

Clobazam

Lisinopril

Sucralfate

Explanation

The answer is Cimetidine


The symptoms of ataxia, slurred speech and blurred vision are due to phenytoin toxicity
in this patient, which has been triggered by addition of cimetidine, an enzyme inhibitor
that reduces hepatic metabolism of certain drugs including phenytoin.

Allopurinol (Option A) is incorrect. Allopurinol is an enzyme inhibitor that may interact with
phenytoin, but the interaction is much less significant than for cimetidine, which is why
option B is the preferred answer.
Clobazam (Option C) is incorrect. There is no apparent interaction between clobazam and
phenytoin.
Lisinopril (Option D) is incorrect. There is no apparent interaction between lisinopril and
phenytoin.
Sucralfate (Option E) is incorrect. Sucralfate may decrease the pharmacological effects of
phenytoin when administered concurrently.
46766

https://mypastest.pastest.com/Secure/TestMe/Browser/429893#Top

1/2

8/12/2016

MyPastest

46766

Next Question

Previous Question

Tag Question

Feedback

End Review

Difficulty: Average
Peer Responses

Session Progress
Responses Correct:

Responses Incorrect:

179

Responses Total:

179

Responses - % Correct:

0%

Blog (https://www.pastest.com/blog) About Pastest (https://www.pastest.com/about-us)


Contact Us (https://www.pastest.com/contact-us) Help (https://www.pastest.com/help)
Pastest 2016

https://mypastest.pastest.com/Secure/TestMe/Browser/429893#Top

2/2

8/12/2016

MyPastest

Back to Filters (/Secure/TestMe/Filter/429893/QA)

Question 98 of 179

A 34-year-old woman is admitted for management of chemotherapy-induced nausea and


vomiting.
What is the most important mode of the antiemetic action of ondansetron?
A

Dopamine agonist

Dopamine antagonist

Histamine agonist

Serotonin receptor subtype 5-hydroxytryptamine agonist

Serotonin receptor subtype 5-hydroxytryptamine antagonist

Explanation

The answer is Serotonin receptor subtype 5-hydroxytryptamine (5-HT3) antagonist


Ondansetron is a selective antagonist of the serotonin receptor subtype, 5-HT3.
Cytotoxic chemotherapy and radiotherapy are associated with the release of serotonin
(5-HT) from enterochromaffin cells of the small intestine, presumably initiating a
vomiting reflex through stimulation of 5-HT3 receptors located on vagal afferents.
Ondansetron may block this reflex. Activation of vagal afferents may also cause a central
release of serotonin from the chemoreceptor trigger zone of the area postrema, located
on the floor of the fourth ventricle; the antiemetic effect of ondansetron is probably due
to the selective antagonism of 5-HT3 receptors on neurones located in either the
peripheral or central nervous system.

Dopamine agonist (Option A) is incorrect. Dopamine agonists, such as those used in


treatment of parkinsonism, often cause significant nausea.
Dopamine antagonist (Option B) is incorrect. Certain antiemetics exert their effects through
dopamine antagonism, e.g. domperidone.
Histamine agonist (Option C) is incorrect. Antihistamines possess some antiemetic
properties; this may be a relevant mechanism of cyclizine, which possesses central
antihistamine and anticholinertic properties.
https://mypastest.pastest.com/Secure/TestMe/Browser/429893#Top

1/2

8/12/2016

MyPastest

Serotonin receptor subtype 5-hydroxytryptamine (5-HT3) agonist (Option D) is incorrect.


Stimulation of 5-HT3 receptors is a central mechanism involved in development of
chemotherapy-induced nausea and vomiting.
46943

Next Question

Previous Question

Tag Question

Feedback

End Review

Difficulty: Average
Peer Responses

Session Progress
Responses Correct:

Responses Incorrect:

179

Responses Total:

179

Responses - % Correct:

0%

Blog (https://www.pastest.com/blog) About Pastest (https://www.pastest.com/about-us)


Contact Us (https://www.pastest.com/contact-us) Help (https://www.pastest.com/help)
Pastest 2016

https://mypastest.pastest.com/Secure/TestMe/Browser/429893#Top

2/2

8/12/2016

MyPastest

Back to Filters (/Secure/TestMe/Filter/429893/QA)

Question 99 of 179

A 59-year-old woman who suffers from arthritis and hypertension visits you and asks you to
prescribe increased pain relief.
Which of the following agents is most likely to exert its analgesic effects by specifically
inhibiting COX-2 activity?
A

Aspirin

Celecoxib

Co-proxamol

Diclofenac

Nefopam

Explanation

The answer is Celecoxib


Both celecoxib and rofecoxib are selective COX-2 inhibitors (coxibs). Prior to the
development of the coxibs it was postulated that the renal toxicity and increased
gastrointestinal bleeding effects of non-steroidal anti-inflammatory drugs (NSAIDs)
occur as a result of COX-1 inhibition, and that the major anti-inflammatory effects of
NSAIDs are due to inhibition of COX-2. However, recent data have shown that selective
inhibition of COX-2 may be associated with increased cardiovascular toxicity.

Aspirin (Option A) is incorrect. Aspirin inhibits cyclo-oxygenase but it is not selective for
COX-2.
Co-proxamol (Option C) is incorrect. Co-proxamol is a fixed dose combination of
paracetamol and dextropropoxyphene.
Diclofenac (Option D) is incorrect. Diclofenac is a non-selective COX inhibitor NSAID.
Nefopam (Option E) is incorrect. Nefopam is a NSAID but it is not selective for COX-2
isoenzyme.
46834

https://mypastest.pastest.com/Secure/TestMe/Browser/429893#Top

1/2

8/12/2016

MyPastest

Next Question

Previous Question

Tag Question

Feedback

End Review

Difficulty: Average
Peer Responses

Session Progress
Responses Correct:

Responses Incorrect:

179

Responses Total:

179

Responses - % Correct:

0%

Blog (https://www.pastest.com/blog) About Pastest (https://www.pastest.com/about-us)


Contact Us (https://www.pastest.com/contact-us) Help (https://www.pastest.com/help)
Pastest 2016

https://mypastest.pastest.com/Secure/TestMe/Browser/429893#Top

2/2

8/12/2016

MyPastest

Back to Filters (/Secure/TestMe/Filter/429893/QA)

Question 100 of 179

Two patients are brought in from the same house. They have been undertaking renovation
works recently, and you suspect possible carbon monoxide poisoning.
Which of the following features would most strongly indicate severe carbon monoxide
poisoning?
A

Carboxyhaemoglobin concentration of 9%

Headache

Impairment of fine motor coordination in the upper limbs

ST segment depression in leads V4, V5 and V6 on a resting ECG

Systolic blood pressure less than 100 mmHg

Explanation

The answer is ST segment depression in leads V4, V5 and V6 on a resting ECG


Carbon monoxide is a common cause of accidental poisoning fatalities in the UK. The
clinical features are non-specific and the diagnosis can be easily overlooked. Symptoms
and signs are attributable to tissue hypoxaemia, and include reduced conscious level,
cerebellar toxicity, myocardial ischaemia with corresponding ECG changes, and lactic
acidosis. Severe toxicity is often associated with high carboxyhaemoglobin
concentrations, typically > 30% if patients present early, although these may fall within a
few hours of removal from exposure. Patients with pre-existing vascular disease are at an
increased risk of morbidity and mortality from carbon monoxide poisoning. Remove the
patient from the source of the carbon monoxide, and treat with high-flow oxygen via a
tight-fitting mask without a re-breathing circuit. Hyperbaric oxygen therapy may be
considered if carboxyhaemoglobin concentrations are over 40% and there are
neurological features.

Carboxyhaemoglobin concentration of 9% (Option A) is incorrect. Carboxyhaemoglobin


concentrations are typically less than 5% in healthy adults, but as high as 10% in heavy
smokers.
https://mypastest.pastest.com/Secure/TestMe/Browser/429893#Top

1/2

8/12/2016

MyPastest

Headache (Option B) is incorrect. Headache is a recognised feature of carbon monoxide


exposure, especially chronic exposure, but is a non-specific symptom that does not reliably
correlate with severity.
Impairment of fine motor coordination in the upper limbs (Option C) is incorrect. Minor
neurological impairment may occur, including cerebellar impairment, but this is less specific
than ischaemic ECG changes and a less preferred option in this scenario.
Systolic blood pressure less than 100 mmHg (Option E) is incorrect. Inadequate delivery of
oxygen to tissues is caused by altered binding affinity to haemoglobin, rather than a fall in
blood pressure. Patients may have few haemodynamic effects, even after severe carbon
monoxide exposure.
45794

Next Question

Previous Question

Tag Question

Feedback

End Review

Difficulty: Average
Peer Responses

Session Progress
Responses Correct:

Responses Incorrect:

179

Responses Total:

179

Responses - % Correct:

0%

Blog (https://www.pastest.com/blog) About Pastest (https://www.pastest.com/about-us)


Contact Us (https://www.pastest.com/contact-us) Help (https://www.pastest.com/help)
Pastest 2016

https://mypastest.pastest.com/Secure/TestMe/Browser/429893#Top

2/2

8/12/2016

MyPastest

Back to Filters (/Secure/TestMe/Filter/429893/QA)

Question 101 of 179

A 62-year-old woman presents to the Rheumatology Clinic for review. She attended the
Emergency Department some 6 weeks earlier following a left Colles fracture and is
concerned about osteoporosis. A past history of venous thromboembolism and gastrooesophageal reflux disease for which she is treated with daily Omeprazole is noted. On
examination her BP is 132/80 mmHg; pulse is 75/min and regular. Her BMI is 23. The wrist
fracture is enclosed in a lightweight cast.
Investigations;
Hb

12.9 g/dl

WCC

7.1 x109/l

PLT

201 x109/l

Na+

137 mmol/l

K+

4.3 mmol/l

Creatinine

91 micromol/l

Glucose

5.8 mmol/l

Calcium

2.3 mmol/l

ALP

81 U/l

DEXA T score -2.7


Which of the following is the most appropriate intervention?
A

Calcium and vitamin D

Denosumab

HRT

Raloxifene

Risedronate

https://mypastest.pastest.com/Secure/TestMe/Browser/429893#Top

1/2

8/12/2016

MyPastest

Explanation
The answer is Denosumab In this situation with a history of GORD treated with PPI, and given the patients young age,
bisphosphonates are a suboptimal choice. They run the risk of significantly worsening
oesophagitis and leading to adynamic bone disease with long-term use. Whilst SERMs such
as Raloxifene are a reasonable second line choice, they are less effective than
bisphosphonates in reducing risk of fracture. Denosumab, given by 6 monthly subcutaneous
injections, is potentially as effective as bisphosphonates, with a superior adverse event
profile.
40149

Next Question

Previous Question

Tag Question

Feedback

End Review

Difficulty: Difficult
Peer Responses

Session Progress
Responses Correct:

Responses Incorrect:

179

Responses Total:

179

Responses - % Correct:

0%

Blog (https://www.pastest.com/blog) About Pastest (https://www.pastest.com/about-us)


Contact Us (https://www.pastest.com/contact-us) Help (https://www.pastest.com/help)
Pastest 2016

https://mypastest.pastest.com/Secure/TestMe/Browser/429893#Top

2/2

8/12/2016

MyPastest

Back to Filters (/Secure/TestMe/Filter/429893/QA)

Question 102 of 179

Which of the following drugs is most likely to require plasma level monitoring in routine
clinical practice?
A

Bleomycin

Cefuroxime

Ciprofloxacin

Erythromycin

Vancomycin

Explanation

The answer is Vancomycin


All patients require plasma vancomycin measurement (after three or four doses if renal
function is normal, or earlier if renal impairment is present). There is a risk of
nephrotoxicity including renal failure, interstitial nephritis and ototoxicity.

Bleomycin (Option A) is incorrect. Although bleomycin is highly toxic, there is no clear


relationship between blood concentrations and risks of toxicity.
Cefuroxime (Option B) is incorrect. There is a poor correlation between cefuroxime drug
concentrations and therapeutic efficacy or risk of adverse effects, and it has a broad
therapeutic window.
Ciprofloxacin (Option C) is incorrect. Ciprofloxacin has a wide therapeutic window and does
not require therapeutic monitoring.
Erythromycin (Option D) is incorrect. There is a poor correlation between erythromycin drug
concentrations and therapeutic efficacy or risk of adverse effects.
46946

Next Question

https://mypastest.pastest.com/Secure/TestMe/Browser/429893#Top

1/2

8/12/2016

MyPastest

Previous Question

Tag Question

Feedback

End Review

Difficulty: Average
Peer Responses

Session Progress
Responses Correct:

Responses Incorrect:

179

Responses Total:

179

Responses - % Correct:

0%

Blog (https://www.pastest.com/blog) About Pastest (https://www.pastest.com/about-us)


Contact Us (https://www.pastest.com/contact-us) Help (https://www.pastest.com/help)
Pastest 2016

https://mypastest.pastest.com/Secure/TestMe/Browser/429893#Top

2/2

8/13/2016

MyPastest

Back to Filters (/Secure/TestMe/Filter/429893/QA)

Question 103 of 179

An elderly man on digoxin treatment for atrial fibrillation presented to his GP 3 weeks ago
complaining of pitting oedema affecting both ankles and breathlessness on exertion, and was
prescribed a new medication. He now presents to the Emergency Department with severe
nausea, vomiting and visual disturbance, and a resting ECG shows complete heart block.
Which of the following new agents is most likely to account for development of his
symptoms and signs?
A

Amiodarone

Furosemide

Propranolol

Sotalol

Warfarin

Explanation

The answer is Furosemide


The signs and symptoms of complete heart block and profound nausea and vomiting are
characteristic of digoxin toxicity. Other features typically include xanthopsia (yellow
halo), bradyarrhythmia and hypotension. Administration of a thiazide or loop diuretic
may enhance digoxin concentrations and increase cardiac sensitivity to digoxin by
causing hypokalaemia.

Amiodarone (Option A) is incorrect. Amiodarone certainly interacts with digoxin by


increasing drug concentrations and predisposing to digoxin toxicity. However, it is a less
preferred answer here than option B because the patient is unlikely to have been prescribed
amiodarone for treatment of heart failure symptoms (breathlessness and peripheral
oedema).
Propranolol (Option C) is incorrect. Propranolol may enhance bradycardia and AV nodal
blockade when administered in combination with digoxin, but does not provoke digoxin
toxicity.
https://mypastest.pastest.com/Secure/TestMe/Browser/429893#Top

1/2

8/13/2016

MyPastest

Sotalol (Option D) is incorrect. Sotalol does not provoke digoxin toxicity.


Warfarin (Option E) is incorrect. Warfarin would not be expected provoke digoxin toxicity.
46770

Next Question

Previous Question

Tag Question

Feedback

End Review

Difficulty: Average
Peer Responses

Session Progress
Responses Correct:

Responses Incorrect:

179

Responses Total:

179

Responses - % Correct:

0%

Blog (https://www.pastest.com/blog) About Pastest (https://www.pastest.com/about-us)


Contact Us (https://www.pastest.com/contact-us) Help (https://www.pastest.com/help)
Pastest 2016

https://mypastest.pastest.com/Secure/TestMe/Browser/429893#Top

2/2

8/13/2016

MyPastest

Back to Filters (/Secure/TestMe/Filter/429893/QA)

Question 104 of 179

You review a 17-year-old girl who has taken 20 g of acetaminophen (paracetamol).


Acetaminophen is an important cause of acute hepatic failure.
Which of the following statements concerning paracetamol overdose is correct?
A

Alcoholics are less susceptible to liver injury even with a low dose

Haemodialysis is effective in the management of hepatotoxicity

N-acetylcysteine is most effective when administered within 8 h of ingestion

Significant liver injury rarely occurs with doses of less than 20 g

Survivors of acetaminophen-induced hepatotoxicity always experience residual liver


damage

Explanation

The answer is N-acetylcysteine is most effective when administered within 8 h of


ingestion
Acetaminophen overdose causes acute liver failure. Liver injury is caused by toxic
metabolites of acetaminophen produced by liver mixed oxidase enzymes. This
metabolite is usually conjugated with glutathione and excreted via a non-toxic pathway;
in overdose, glutathione stores are depleted. N-acetylcysteine is an SH-group donator,
providing alternative substrate for conjugation.

Alcoholics are less susceptible to liver injury even with a low dose (Option A) is incorrect.
Regular ethanol consumption is associated with liver enzyme induction, therefore severe
hepatotoxicity can be seen in alcoholics, even with lower dosages of paracetamol.
Haemodialysis is effective in the management of hepatotoxicity (Option B) is incorrect.
Haemodialysis may be useful for managing acute renal failure occurring in conjunction with
paracetamol-induced liver failure, but is not a treatment for hepatotoxicity.
Significant liver injury rarely occurs with doses of less than 20 g (Option D) is incorrect.
Significant liver injury usually occurs with doses of >10 to 15 g, particularly in patients that
https://mypastest.pastest.com/Secure/TestMe/Browser/429893#Top

1/2

8/13/2016

MyPastest

present late to hospital.


Survivors of acetaminophen-induced hepatotoxicity always experience residual liver damage
(Option E) is incorrect. In survivors of acute acetaminophen toxicity, there is usually
complete recovery without any cirrhosis or residual liver damage.
46933

Next Question

Previous Question

Tag Question

Feedback

End Review

Difficulty: Average
Peer Responses

Session Progress
Responses Correct:

Responses Incorrect:

179

Responses Total:

179

Responses - % Correct:

0%

Blog (https://www.pastest.com/blog) About Pastest (https://www.pastest.com/about-us)


Contact Us (https://www.pastest.com/contact-us) Help (https://www.pastest.com/help)
Pastest 2016

https://mypastest.pastest.com/Secure/TestMe/Browser/429893#Top

2/2

8/13/2016

MyPastest

Back to Filters (/Secure/TestMe/Filter/429893/QA)

Question 105 of 179

You are seeing a 17-year-old woman in the respiratory clinic for a review of her asthma. She
has been taking inhaled disodium cromoglicate for several years.
Which of the following mechanisms is most important in the action of disodium cromoglicate
to prevent asthma attacks?
A

Action on eosinophils to reduce the inflammatory response to inhaled allergens

Competitive blockade of histamine at receptor sites

Inhibition of acetylcholine at the synaptic junctions

Inhibition of mast-cell degranulation

Potentiation of the effects of 2-receptor agonists

Explanation

The answer is option D, inhibition of mast-cell degranulation Sodium cromoglicate principally acts by reducing the degranulation of mast cells
triggered by the interaction of antigen and IgE. The inhibitory effect on mast cells
appears to be cell-type specific, since cromoglicate has little inhibitory effect on
mediator release from human basophils.

Action on eosinophils to reduce the inflammatory response to inhaled allergens (Option A) is


incorrect. Disodium cromoglicate may also act on eosinophils to reduce their inflammatory
response to inhaled allergens, but the relevance to asthma is uncertain.
Competitive blockade of histamine at receptor sites (Option B) is incorrect. Anticholinergic
effects are not seen with disodium cromoglicate.
Inhibition of acetylcholine at the synaptic junctions (Option C) is incorrect. Disodium
cromoglicate does not exert anticholinergic effects. Ipratropium and tiotropium exert
antimuscarinic effects, and are effective in reducing airway inflammation in COPD.
Potentiation of the effects of 2-receptor agonists (Option E) is incorrect; 2-adrenoceptor
agonists also appear to be capable of reducing histamine release through a similar
https://mypastest.pastest.com/Secure/TestMe/Browser/429893#Top

1/2

8/13/2016

MyPastest

mechanism, but their actions are not potentiated by disodium cromoglicate.


46749

Next Question

Previous Question

Tag Question

Feedback

End Review

Difficulty: Average
Peer Responses

Session Progress
Responses Correct:

Responses Incorrect:

179

Responses Total:

179

Responses - % Correct:

0%

Blog (https://www.pastest.com/blog) About Pastest (https://www.pastest.com/about-us)


Contact Us (https://www.pastest.com/contact-us) Help (https://www.pastest.com/help)
Pastest 2016

https://mypastest.pastest.com/Secure/TestMe/Browser/429893#Top

2/2

8/13/2016

MyPastest

Back to Filters (/Secure/TestMe/Filter/429893/QA)

Question 106 of 179

A 44-year-old man comes to the Dermatology Clinic. He has severe psoriasis and has failed
conventional therapy. You elect to begin treatment with ciclosporin.
Which one of the following correctly reflects one of the main modes of action of ciclosporin?
A

It inhibits IL-6 activity

It inhibits IL-2 activity

It inhibits IL-1 activity

It inhibits TNF- activity

It inhibits IL-4 activity

Explanation
Ciclosporin
Ciclosporin is thought to have two main actions
arrest of activity of resting lymphocytes in the G0 or G1 phase of the growth
cycle
cytokine inhibition, the major one being inhibition of IL-2
Ciclosporin appears to act specifically on lymphocytes and does not depress red cell
production or the function of phagocytic white blood cells
Ciclosporin levels are particularly affected by drugs metabolised by the CYP3A4 p450
enzyme
For this reason particular caution is advised when prescribing a number of agents such
as statins and macrolide antibiotics
The British National Formulary needs to be consulted when considering prescribing
additional medication in patients taking ciclosporin
https://www.medicines.org.uk/EMC/medicine/1307/SPC/Neoral+Soft+Gelatin+Capsules,+Neoral+Oral+Solution/
(https://www.medicines.org.uk/EMC/medicine/1307/SPC/Neoral+Soft+Gelatin+Capsules,+Neoral+Oral+Solution/)
22487

Next Question

Previous Question

Tag Question

Feedback

End Review

Difficulty: Average
Peer Responses

https://mypastest.pastest.com/Secure/TestMe/Browser/429893#Top

1/2

8/13/2016

MyPastest

Session Progress
Responses Correct:

Responses Incorrect:

179

Responses Total:

179

Responses - % Correct:

0%

Blog (https://www.pastest.com/blog) About Pastest (https://www.pastest.com/about-us)


Contact Us (https://www.pastest.com/contact-us) Help (https://www.pastest.com/help)
Pastest 2016

https://mypastest.pastest.com/Secure/TestMe/Browser/429893#Top

2/2

8/13/2016

MyPastest

Back to Filters (/Secure/TestMe/Filter/429893/QA)

Question 107 of 179

A patient with coronary heart disease and high LDL-cholesterol was started on simvastatin 6
months ago. His GP recently started him on another medication and now, 2 weeks later, he is
complaining of muscle pain and weakness. Tests reveal an elevated creatine kinase (CK), ten
times greater than the upper limit of normal.
Which of the following additional drugs is most likely to have resulted in this adverse effect?
A

Amlodipine

Aspirin

Atenolol

Clarithromycin

Rifampicin

Explanation

The answer is Clarithromycin


Statins inhibit 3-hydroxy-3-methyl-glutaryl-CoA, the rate limiting step in cholesterol
synthesis. These can cause myopathy and rhabdomyolysis, associated with muscle pain,
tenderness or weakness. Rhabdomyolysis may cause acute renal failure secondary to
myoglobinuria. The risk of myopathy and rhabdomyolysis is greatest in patients receiving
high statin doses. The risks of simvastatin-induced myopathy are increased by
concomitant use of potent inhibitors of cytochrome P450 isoenzyme 3A4 inhibitors,
such as erythromycin, itraconazole, ketoconazole, clarithromycin, gemfibrozil and
ciclosporin (the effect of these agents on other statins is recognised but less important
than with simvastatin).

Amlodipine (Option A) is incorrect. Calcium channel blockers may cause peripheral oedema
but are unlikely to interact significantly with simvastatin.
Aspirin (Option B) is incorrect. Aspirin does not interfere with simvastatin and can be safely
co-prescribed.
https://mypastest.pastest.com/Secure/TestMe/Browser/429893#Top

1/2

8/13/2016

MyPastest

Atenolol (Option C) is incorrect. Beta-blockers may reduce peripheral blood flow through a
combination of increased peripheral resistance and reduced cardiac output. This often
causes muscle cramps but myopathy is not a recognised feature.
Rifampicin (Option E) is incorrect. Rifampicin is a powerful enzyme inducer that might with
long-term treatment enhance the metabolism of simvastatin and might lessen its
effectiveness.
46789

Next Question

Previous Question

Tag Question

Feedback

End Review

Difficulty: Average
Peer Responses

Session Progress
Responses Correct:

Responses Incorrect:

179

Responses Total:

179

Responses - % Correct:

0%

Blog (https://www.pastest.com/blog) About Pastest (https://www.pastest.com/about-us)


Contact Us (https://www.pastest.com/contact-us) Help (https://www.pastest.com/help)
Pastest 2016

https://mypastest.pastest.com/Secure/TestMe/Browser/429893#Top

2/2

8/13/2016

MyPastest

Back to Filters (/Secure/TestMe/Filter/429893/QA)

Question 108 of 179

A 22-year-old woman is admitted to the Emergency Department. Her mother suffers from
manic depression. Apparently she had a row with her boyfriend the previous evening and was
found by her mother that morning. You understand that she had two seizures during the
ambulance journey to hospital. On examination she is deeply unconscious and her blood
pressure is 148/94 mmHg. She has bilateral increased tone.
Investigations:
Hb

13.1 g/dl

White cell count 5.4 109/l


Platelets

230 109/l

Na+

139 mmol/l

K+

4.9 mmol/l

Creatinine

155 mol/l

Lithium

3.7 mmol/l (therapeutic range 0.61.5)

Which one of the following is the most appropriate way to manage her?
A

Charcoal via nasogastric tube

Forced alkaline diuresis

IV furosemide 120 mg

Haemodialysis

IV normal saline

Explanation
Lithium toxicity
https://mypastest.pastest.com/Secure/TestMe/Browser/429893#Top

1/2

8/13/2016

MyPastest

Haemodialysis is the management of choice for severe lithium toxicity


Both lithium concentration and the change in mental status drive the need for dialysis
Activated charcoal is not very effective in lithium overdose, but charcoal should still be
given if co-ingestion of another agent is suspected
Gastric lavage may be considered in patients presenting very rapidly (within an hour) of
overdose
For patients with mild to moderate lithium overdose, normal saline rehydration may
help drive lithium excretion
For most patients the outcome is favourable, although up to 10% of cases of severe
overdose may carry some form of long-term neurological deficit
20938

Next Question

Previous Question

Tag Question

Feedback

End Review

Difficulty: Average
Peer Responses

Session Progress
Responses Correct:

Responses Incorrect:

179

Responses Total:

179

Responses - % Correct:

0%

Blog (https://www.pastest.com/blog) About Pastest (https://www.pastest.com/about-us)


Contact Us (https://www.pastest.com/contact-us) Help (https://www.pastest.com/help)
Pastest 2016

https://mypastest.pastest.com/Secure/TestMe/Browser/429893#Top

2/2

8/13/2016

MyPastest

Back to Filters (/Secure/TestMe/Filter/429893/QA)

Question 109 of 179

A 67-year-old man with known long-standing asthma presents to the Emergency Department
following an intentional overdose of his regular theophylline. He reports having taken 20 of
his theophylline tablets 1 hour earlier to end his life.
Which of the following features would most strongly suggest severe theophylline toxicity?
A

Hyperkalaemia

Hypoglycaemia

Right bundle branch block on a resting ECG

Severe nausea and vomiting

Sinus bradycardia

Explanation

The correct answer is Severe nausea and vomiting


Management of a theophylline overdose includes multi-dose activated charcoal, which
increases drug clearance by interfering with enterohepatic recirculation; however, it is
often impossible due to severe vomiting. Other features of severe toxicity include
seizures, myoclonus, hyperthermia, rhabdomyolysis and acute renal failure. Nausea and
vomiting are characteristic features of theophylline poisoning.

Hyperkalaemia (Option A) is incorrect. Hypokalaemia is a characteristic feature, and may


correct rapidly, so frequent monitoring of potassium levels is important to avoid excess
potassium administration.
Hypoglycaemia (Option B) is incorrect. Hyperglycaemia may occur.
Right bundle branch block on a resting ECG (Option C) is incorrect. Conduction defects and
bundle branch block may occur but are uncommon. Cardiac arrhythmias typically include
sinus tachycardia, atrial and ventricular arrhythmias, and prolongation of the QT and QRS
intervals. Arrhythmias are more common in patients with electrolyte disturbances, including
hypokalaemia and metabolic acidosis.
https://mypastest.pastest.com/Secure/TestMe/Browser/429893#Top

1/2

8/13/2016

MyPastest

Sinus bradycardia (Option E) is incorrect. Sinus tachycardia is a common feature.


45795

Next Question

Previous Question

Tag Question

Feedback

End Review

Difficulty: Average
Peer Responses

Session Progress
Responses Correct:

Responses Incorrect:

179

Responses Total:

179

Responses - % Correct:

0%

Blog (https://www.pastest.com/blog) About Pastest (https://www.pastest.com/about-us)


Contact Us (https://www.pastest.com/contact-us) Help (https://www.pastest.com/help)
Pastest 2016

https://mypastest.pastest.com/Secure/TestMe/Browser/429893#Top

2/2

8/13/2016

MyPastest

Back to Filters (/Secure/TestMe/Filter/429893/QA)

Question 110 of 179

A 20-year-old student is admitted to the Emergency Department after an intentional


overdose involving her grandmothers analgesic medications. She is pyrexial, pale, sweating
profusely with laboured breathing. Heart rate is 102 bpm, oxygen saturations 100% on air, and
blood pressure 124/68 mmHg.
Which of the following drugs is most likely to cause these findings?
A

Aspirin

Co-proxamol

Diclofenac

Ibuprofen

Paracetamol

Explanation

The answer is Aspirin


Aspirin overdose initially causes a respiratory alkalosis, due to direct stimulation of the
central respiratory centres, which provokes rapid deep breathing air hunger. Other early
features include hyperventilation, tinnitus, vertigo, sweating and mild pyrexia. As drug
absorption ensues and toxicity progresses, the later features of aspirin toxicity include
metabolic acidosis, reduced conscious level and hypoglycaemia.

Co-proxamol (Option B) is incorrect. Co-proxamol contains paracetamol and


dextropropoxyphene, an opioid that is capable of prolonging QRS duration and a recognised
cause of fatal arrhythmia in overdose. Opioid effects in overdose include respiratory
depression, hypotension, cardiovascular collapse, coma and death.
Diclofenac (Option C) is incorrect. Acute overdose involving diclofenac may cause vomiting,
haematemesis, peptic ulceration and renal failure.
Ibuprofen (Option D) is incorrect. Acute overdose involving ibuprofen may cause vomiting,
haematemesis, peptic ulceration and renal failure.
https://mypastest.pastest.com/Secure/TestMe/Browser/429893#Top

1/2

8/13/2016

MyPastest

Paracetamol (Option E) is incorrect. Paracetamol overdose, even in severe and fatal cases,
may have few initial symptoms. Early features may include nausea and abdominal pain.
46763

Next Question

Previous Question

Tag Question

Feedback

End Review

Difficulty: Average
Peer Responses

Session Progress
Responses Correct:

Responses Incorrect:

179

Responses Total:

179

Responses - % Correct:

0%

Blog (https://www.pastest.com/blog) About Pastest (https://www.pastest.com/about-us)


Contact Us (https://www.pastest.com/contact-us) Help (https://www.pastest.com/help)
Pastest 2016

https://mypastest.pastest.com/Secure/TestMe/Browser/429893#Top

2/2

8/13/2016

MyPastest

Back to Filters (/Secure/TestMe/Filter/429893/QA)

Question 111 of 179

You are asked to review a 62-year-old man who takes nifedipine for hypertension. He asks for
advice about eating a healthy diet. His wife is following a homeopathic fruit juice regimen.
Which of the following fruit juices would he be wisest to avoid?
A

Apple juice

Blackcurrant juice

Grapefruit juice

Orange juice

Tomato juice

Explanation
The answer is Grapefruit juice
Nifedipine is metabolised via the CYP3A4 cytochrome P450 isoenzyme. Grapefruit juice
is capable of powerful inhibition of the CYP3A4 isoenzyme and may significantly
increase the toxicity of nifedipine, simvastatin and other drugs metabolised by this
isoenzyme.

Apple juice (Option A) is incorrect. Apple juice is unlikely to have any significant effect on
CYP3A4 activity.
Blackcurrant juice (Option B) is incorrect. Blackcurrant juice is unlikely to have any significant
effect on CYP3A4 activity.
Orange juice (Option D) is incorrect. Orange juice is unlikely to have any significant effect on
CYP3A4 activity.
Tomato juice (Option E) is incorrect. Tomato juice is unlikely to have any significant effect on
CYP3A4 activity.
46835

https://mypastest.pastest.com/Secure/TestMe/Browser/429893#Top

1/2

8/13/2016

MyPastest

Next Question

Previous Question

Tag Question

Feedback

End Review

Difficulty: Average
Peer Responses

Session Progress
Responses Correct:

Responses Incorrect:

179

Responses Total:

179

Responses - % Correct:

0%

Blog (https://www.pastest.com/blog) About Pastest (https://www.pastest.com/about-us)


Contact Us (https://www.pastest.com/contact-us) Help (https://www.pastest.com/help)
Pastest 2016

https://mypastest.pastest.com/Secure/TestMe/Browser/429893#Top

2/2

8/13/2016

MyPastest

Back to Filters (/Secure/TestMe/Filter/429893/QA)

Question 112 of 179

You are reviewing an 18-year-old young woman who has been brought into the Emergency
Department after a row with her boyfriend. The ambulance crew hand you a number of
empty bottles of dihydrocodeine. On examination she is unconscious with a GCS of 6.
Which of the following drugs would be the most appropriate antidote for dihydrocodeine?
A

Dantrolene

Dextropropoxyphene

Flumazenil

Naloxone

Neostigmine

Explanation

The answer is Naloxone


Naloxone is an opiate antagonist indicated for the reversal of opioid-induced respiratory
depression or opioid overdose. Naloxone is relatively short-acting compared with the
half-life of opioid analgesics; this means there is a real possibility of patients becoming
re-sedated as the effects wear off, it should therefore be given as repeated doses or as a
continuous infusion.

Dantrolene (Option A) is incorrect. Dantrolene may be useful in the treatment of malignant


hyperthermia in serotonin syndrome.
Dextropropoxyphene (Option B) is incorrect. Dextropropoxyphene is a long-acting opioid
used as a constituent of co-proxamol; it is a partial agonist and therefore the effect of
naloxone in co-proxamol overdose is limited. Dextropropoxyphene is cardiotoxic and is
responsible for sudden cardiac death in overdose.
Flumazenil (Option C) is incorrect. Flumazenil is a benzodiazepine antagonist that should be
used with caution, as rapid reversal of benzodiazepines can be associated with convulsions.

https://mypastest.pastest.com/Secure/TestMe/Browser/429893#Top

1/2

8/13/2016

MyPastest

Neostigmine (Option E) is incorrect. Neostigmine is useful for reversing the effects of


anaesthetic muscle relaxants.
46829

Next Question

Previous Question

Tag Question

Feedback

End Review

Difficulty: Average
Peer Responses

Session Progress
Responses Correct:

Responses Incorrect:

179

Responses Total:

179

Responses - % Correct:

0%

Blog (https://www.pastest.com/blog) About Pastest (https://www.pastest.com/about-us)


Contact Us (https://www.pastest.com/contact-us) Help (https://www.pastest.com/help)
Pastest 2016

https://mypastest.pastest.com/Secure/TestMe/Browser/429893#Top

2/2

8/13/2016

MyPastest

Back to Filters (/Secure/TestMe/Filter/429893/QA)

Question 113 of 179

You are asked to prescribe an antibiotic for a 72-year-old woman convalescing on the Elderly
Care Ward after a left total hip replacement. She has developed symptoms of a lower UTI.
Which of the following is likely to carry the lowest risk of C difficile infection?
A

Ceftriaxone

Cephalexin

Co-amoxiclav

Levofloxacin

Trimethoprim

Explanation
The answer is Trimethoprim.
Guidelines suggest that cephalosporins, broad spectrum penicillins such as co-amoxiclav,
clindamycin and fluoroquinolones are all associated with increased risk of C difficile infection.
As such the best option is trimethoprim. For an upper urinary tract infection the benefits of
using co-amoxyclav would outweigh the risks of C difficile infection. Guidelines also
recommend minimising the length of antibiotic therapy as much as possible and avoiding
blanket prescribing of antibiotic therapy.
Scottish Antimicrobial Prescribing Group
http://www.scottishmedicines.org.uk/files/sapg/2011_National_prescribing_indicators__Compatibility_Mode_.pdf
(http://www.scottishmedicines.org.uk/files/sapg/2011_National_prescribing_indicators__Compatibility_Mode_.pdf)
37291

Next Question

Previous Question

Tag Question

Feedback

End Review

Difficulty: Average
Peer Responses

https://mypastest.pastest.com/Secure/TestMe/Browser/429893#Top

1/2

8/13/2016

MyPastest

Session Progress
Responses Correct:

Responses Incorrect:

179

Responses Total:

179

Responses - % Correct:

0%

Blog (https://www.pastest.com/blog) About Pastest (https://www.pastest.com/about-us)


Contact Us (https://www.pastest.com/contact-us) Help (https://www.pastest.com/help)
Pastest 2016

https://mypastest.pastest.com/Secure/TestMe/Browser/429893#Top

2/2

8/13/2016

MyPastest

Back to Filters (/Secure/TestMe/Filter/429893/QA)

Question 114 of 179

A 43-year-old woman with severe rheumatoid arthritis has experienced marked symptomatic
improvement since the introduction of methotrexate therapy.
What is the most likely mechanism that accounts for the therapeutic action of methotrexate
in rheumatoid arthritis?
A

Inhibition of purine synthesis

Inhibition of thymidylate synthase

Inhibition of topoisomerase I

Inhibition of tubulin

Methylation of guanosine

Explanation

The answer is Inhibition of purine synthesis


Methotrexate is an antimetabolite that exerts its effects by inhibiting dihydrofolate
reductase, which interferes with tetrahydrofolate stores required for synthesis of purines
and thymidylate in inflammatory cells. Clinical trials have demonstrated the superior
efficacy of methotrexate over placebo in controlling the symptoms and signs of early and
established rheumatoid arthritis in patients previously treated with only non-steroidal
anti-inflammatory drugs. In clinical trials lasting 612 months, methotrexate slows the
progression of structural damage as assessed by serial radiographs of the hands and
feet.

Inhibition of thymidylate synthase (Option B) is incorrect. The anti-cancer agent 5fluorouracil is a prodrug of fluorodeoxyuridine monophosphate, which inhibits thymidylate
synthase, an enzyme required for the synthesis of deoxythymidine triphosphate and DNA.
Inhibition of topoisomerase I (Option C) is incorrect. This is a target mechanism of certain
anticancer agents and quinolone antibiotics that interfere with DNA synthesis.
Inhibition of tubulin (Option D) is incorrect. Colchicine and vinca alkaloids exert their
therapeutic effects through inhibition of tubulin.
https://mypastest.pastest.com/Secure/TestMe/Browser/429893#Top

1/2

8/13/2016

MyPastest

Methylation of guanosine (Option E) is incorrect. Some novel antiviral treatments for


hepatitis C target this mechanism.
46805

Next Question

Previous Question

Tag Question

Feedback

End Review

Difficulty: Average
Peer Responses

Session Progress
Responses Correct:

Responses Incorrect:

179

Responses Total:

179

Responses - % Correct:

0%

Blog (https://www.pastest.com/blog) About Pastest (https://www.pastest.com/about-us)


Contact Us (https://www.pastest.com/contact-us) Help (https://www.pastest.com/help)
Pastest 2016

https://mypastest.pastest.com/Secure/TestMe/Browser/429893#Top

2/2

8/13/2016

MyPastest

Back to Filters (/Secure/TestMe/Filter/429893/QA)

Question 115 of 179

A 35-year-old woman with a bipolar disorder has been prescribed lithium.


Which of the following pharmacological features should best be kept in mind when
prescribing this drug?
A

Amiloride is a specific antidote for lithium toxicity

Hyperkalaemia may occur

Lithium may cause nephrogenic diabetes insipidus

Serum lithium level should not exceed 2 mmol/l

Thyroid antibodies may be increased

Explanation

The answer is Lithium may cause nephrogenic diabetes insipidus


Long-term treatment with lithium causes insensitivity of ADH receptors causing
nephrogenic diabetes insipidus. It is also associated with progressive chronic kidney
disease.

Amiloride is a specific antidote for lithium toxicity (Option A) is incorrect. Amiloride may be
effective in treating lithium-induced nephrogenic diabetes insipidus. It is not an antidote for
lithium toxicity; haemodialysis may be required to treat lithium toxicity.
Hyperkalaemia may occur (Option B) is incorrect. Lithium nephropathy is often associated
with hypokalaemia, and may result in flattening of T-waves on the ECG.
Serum lithium level should not exceed 2 mmol/l (Option D) is incorrect. The therapeutic
range for lithium treatment is 0.51.0 mmol/l; levels between 1.0 and 2.0 mmol/l may be
associated with significant clinical toxicity.
Thyroid antibodies may be increased (Option E) is incorrect. Lithium may interfere with
thyroid function, often causing hypothyroidism due to recepton insensitivity to thyroid
stimulating hormone; the presence of thyroid antibodies may increase the risk of
hypothyroidism.
46860

https://mypastest.pastest.com/Secure/TestMe/Browser/429893#Top

1/2

8/13/2016

MyPastest

46860

Next Question

Previous Question

Tag Question

Feedback

End Review

Difficulty: Average
Peer Responses

Session Progress
Responses Correct:

Responses Incorrect:

179

Responses Total:

179

Responses - % Correct:

0%

Blog (https://www.pastest.com/blog) About Pastest (https://www.pastest.com/about-us)


Contact Us (https://www.pastest.com/contact-us) Help (https://www.pastest.com/help)
Pastest 2016

https://mypastest.pastest.com/Secure/TestMe/Browser/429893#Top

2/2

8/13/2016

MyPastest

Back to Filters (/Secure/TestMe/Filter/429893/QA)

Question 116 of 179

A 52-year-old man with disseminated prostatic carcinoma comes to the Emergency


Department after his family called an ambulance. They are very concerned as he has become
increasingly drowsy and they are now unable to rouse him from sleep. He is managed with
prolonged-release morphine but his dose has remained unchanged for the past 4 weeks. It is
only over the past 3 days that he has deteriorated. On examination he is unconscious and
groans in response to vigorous stimulation. His blood pressure is 100/50 mmHg and his
respiratory rate is 9 breaths per minute.
Investigations:
Hb

10.2 g/dl

White cell count

6.2 109/l

Platelets

139 109/l

Na+

142 mmol/l

K+

6.1 mmol/l

Urea

35.2 mmol/l

Creatinine

720 mol/l

ALT

1024 U/l

Albumin

32 g/l

Alkaline phosphatase

623 U/l

Urine on suprapubic catheterisation blood++

Which one of the following is most likely to be responsible for his impaired conscious level?
Your answer was incorrect

Hepatic failure

Renal failure

Cerebral metastases

https://mypastest.pastest.com/Secure/TestMe/Browser/429893#Top

1/3

8/13/2016

MyPastest

Stroke

Urinary sepsis

Explanation
Drug efficacy in renal failure
It is clear that this unrousable patient with disseminated prostate cancer has severe
renal failure
Whilst his liver function test results are in keeping with hepatic metastases, the relative
preservation of albumin would indicate that his liver function is reasonably intact
Many of the hepatic opiate metabolites are biologically active, which means that dose
adjustment of opiates is recommended in both renal and hepatic failure
Given that this patient has been stable for a prolonged period of time, and there is no
history suggestive of focal neurological impairment, cerebral metastases seem less
likely as the cause of his presentation
21365

Next Question

Previous Question

Tag Question

Feedback

End Review

Difficulty: Average
Peer Responses

Session Progress
Responses Correct:

Responses Incorrect:

179

Responses Total:

179

Responses - % Correct:

0%

https://mypastest.pastest.com/Secure/TestMe/Browser/429893#Top

2/3

8/13/2016

MyPastest

Blog (https://www.pastest.com/blog) About Pastest (https://www.pastest.com/about-us)


Contact Us (https://www.pastest.com/contact-us) Help (https://www.pastest.com/help)
Pastest 2016

https://mypastest.pastest.com/Secure/TestMe/Browser/429893#Top

3/3

8/13/2016

MyPastest

Back to Filters (/Secure/TestMe/Filter/429893/QA)

Question 117 of 179

Renal extraction ratio is a term that may be used to describe drug handling.
Which one of the following terms most accurately describes how renal drug elimination can
be defined?
A

A measure of the time during which the concentration of drug in the plasma falls by
50%

Decline of drug concentration in the plasma from the arterial to the venous side of
the kidney

The concentration of a drug in the urine divided by the concentration in the plasma

The proportion of an orally administered drug reaching the circulation

The ratio of drug concentration in the urine to drug concentration in the bile

Explanation

The answer is Decline of drug concentration in the plasma from the arterial to the venous
side of the kidney The extraction ratio is a measure of how much drug is extracted from the plasma by the
kidney, which determines the clearance (clearance = renal plasma flow extraction ratio).

A measure of the time during which the concentration of drug in the plasma falls by 50%
(Option A) is incorrect. The time it takes for the concentration of a drug in the plasma to fall
by 50% is the half-life.
The concentration of a drug in the urine divided by the concentration in the plasma (Option
C) is incorrect. The extraction ratio is determined by assessing drug concentration on the
arterial and venous sides of the renal circulation rather than assessing plasma versus urine
concentration.
The proportion of an orally administered drug reaching the circulation (Option D) is
incorrect. The proportion of orally administered drug reaching the circulation is the
bioavailability.
https://mypastest.pastest.com/Secure/TestMe/Browser/429893#Top

1/2

8/13/2016

MyPastest

The ratio of drug concentration in the urine to drug concentration in the bile (Option E) is
incorrect. The ratio of drug concentration in the urine to drug concentration in the bile gives
an idea of the percent renal excretion versus biliary excretion; this is not the extraction ratio.
46731

Next Question

Previous Question

Tag Question

Feedback

End Review

Difficulty: Average
Peer Responses

Session Progress
Responses Correct:

Responses Incorrect:

179

Responses Total:

179

Responses - % Correct:

0%

Blog (https://www.pastest.com/blog) About Pastest (https://www.pastest.com/about-us)


Contact Us (https://www.pastest.com/contact-us) Help (https://www.pastest.com/help)
Pastest 2016

https://mypastest.pastest.com/Secure/TestMe/Browser/429893#Top

2/2

8/13/2016

MyPastest

Back to Filters (/Secure/TestMe/Filter/429893/QA)

Question 118 of 179

A 32-year-old woman presents to the Emergency Department with jaundice. She has been
taking a 2-week course of antibiotics prescribed by her doctor for recurrent urinary tract
infection, but cannot remember their name. On examination she is apyrexial, her blood
pressure is 132/78 mmHg, her BMI is 24 kg/m2 and she has jaundiced sclerae. There is no
tenderness on abdominal examination. Investigations show alanine transaminase 102 IU/l,
alkaline phosphatase 394 IU/l, and bilirubin 160 mol/l.
Which one of the following antibiotics is most likely to have caused this biochemical picture?
A

Ciprofloxacin

Co-amoxiclav

Erythromycin

Rifampicin

Trimethoprim

Explanation

The answer is Co-amoxiclav


The blood picture seen here is most consistent with obstructive jaundice of cholestasis
because of the larger rise in alkaline phosphatase compared with the modest elevation in
alanine aminotransferase (ALT).

Ciprofloxacin (Option A) is incorrect. Although ciprofloxacin is excreted in bile, it is very


much less likely to cause cholestatic jaundice than co-amoxiclav.
Erythromycin (Option C) is incorrect. Erythromycin estolate preparations may cause
cholestatic jaundice, but these formulations are now uncommon.
Rifampicin (Option D) is incorrect. Rifampicin may be associated with hepatic dysfunction,
but the picture seen is usually one of hepatocellular dysfunction and a prominent
abnormality of transaminases.

https://mypastest.pastest.com/Secure/TestMe/Browser/429893#Top

1/2

8/13/2016

MyPastest

Trimethoprim (Option E) is incorrect. Trimethoprim may cause hyperkalaemia and a transient


rise in serum creatinine concentrations.
46994

Next Question

Previous Question

Tag Question

Feedback

End Review

Difficulty: Average
Peer Responses

Session Progress
Responses Correct:

Responses Incorrect:

179

Responses Total:

179

Responses - % Correct:

0%

Blog (https://www.pastest.com/blog) About Pastest (https://www.pastest.com/about-us)


Contact Us (https://www.pastest.com/contact-us) Help (https://www.pastest.com/help)
Pastest 2016

https://mypastest.pastest.com/Secure/TestMe/Browser/429893#Top

2/2

8/13/2016

MyPastest

Back to Filters (/Secure/TestMe/Filter/429893/QA)

Question 119 of 179

A 29-year-old woman has been receiving antituberculous treatments for the past 3 months
with rifampicin, isoniazid, pyrazinamide, ethambutol and pyridoxine. She has been taking the
oral contraceptive pill regularly but now suddenly finds she is pregnant.
Which drug is most likely to have resulted in failure of the oral contraceptive treatment?
A

Ethambutol

Isoniazid

Pyrazinamide

Pyridoxine

Rifampicin

Explanation

The answer is Rifampicin


The oral contraceptive pill is highly effective, but may fail if the circulating oestrogen
concentrations are insufficient. Often this is related to inconsistent patient compliance,
impaired gastrointestinal absorption, or the co-administration of enzyme inducing drugs
that accelerate hepatic metabolism. Rifampicin is a potent hepatic enzyme inducer that
increases the metabolism of many drugs, including steroid hormones; barrier
contraceptives must be used during rifampicin treatment, and for 48 weeks after
completing a course of rifampicin.

Ethambutol (Option A) is incorrect. There is no clinically significant interaction between


ethambutol and the oral contraceptive pill.
Isoniazid (Option B) is incorrect. There is no clinically significant interaction between
isoniazid and the oral contraceptive pill.
Pyrazinamide (Option C) is incorrect. There is no clinically significant interaction between
pyrazinamide and the oral contraceptive pill.

https://mypastest.pastest.com/Secure/TestMe/Browser/429893#Top

1/2

8/13/2016

MyPastest

Pyridoxine (Option D) is incorrect. There is no clinically significant interaction between


pyridoxine and the oral contraceptive pill.
46621

Next Question

Previous Question

Tag Question

Feedback

End Review

Difficulty: Average
Peer Responses

Session Progress
Responses Correct:

Responses Incorrect:

179

Responses Total:

179

Responses - % Correct:

0%

Blog (https://www.pastest.com/blog) About Pastest (https://www.pastest.com/about-us)


Contact Us (https://www.pastest.com/contact-us) Help (https://www.pastest.com/help)
Pastest 2016

https://mypastest.pastest.com/Secure/TestMe/Browser/429893#Top

2/2

8/13/2016

MyPastest

Back to Filters (/Secure/TestMe/Filter/429893/QA)

Question 120 of 179

A 45-year-old asthmatic patient with a history of myocardial infarction presents to the


Emergency Department with palpitations. An ECG shows supraventricular tachycardia.
Carotid sinus massage has no effect on symptoms or heart rate. BP is 135/72 mmHg.
What would be the most appropriate treatment to consider administering next?
A

Adenosine

Digoxin

Flecainide

Lignocaine

Verapamil

Explanation

The answer is Verapamil


Verapamil is the drug of choice in a patient with asthma. Adenosine should normally be
avoided in patients with asthma because of the risk of bronchospasm. Note that
verapamil should not be used to treat tachyarrhythmias where the QRS complex is wide
because this may precipitate cardiovascular collapse. Verapamil is also contraindicated in
patients with the WolffParkinsonWhite syndrome.

Adenosine (Option A) is incorrect. Although adenosine is the drug of choice for terminating
paroxysmal supraventricular tachycardia, it can cause bronchospasm and is thus
contraindicated in patients with asthma.
Digoxin (Option B) is incorrect. Digoxin tends to shorten refractory periods and enhance
excitability and conduction in other parts of the heart; it may therefore cause more complex
atrial and ventricular tachyarrhythmias.
Flecainide (Option C) is incorrect. Flecainide may be hazardous in patients with a history of
myocardial infarction as it may precipitate ventricular fibrillation.

https://mypastest.pastest.com/Secure/TestMe/Browser/429893#Top

1/2

8/13/2016

MyPastest

Lignocaine (Option D) is incorrect. Lignocaine is used in patients with recurrent VT, not in the
treatment of paroxysmal SVT.
46742

Next Question

Previous Question

Tag Question

Feedback

End Review

Difficulty: Average
Peer Responses

Session Progress
Responses Correct:

Responses Incorrect:

179

Responses Total:

179

Responses - % Correct:

0%

Blog (https://www.pastest.com/blog) About Pastest (https://www.pastest.com/about-us)


Contact Us (https://www.pastest.com/contact-us) Help (https://www.pastest.com/help)
Pastest 2016

https://mypastest.pastest.com/Secure/TestMe/Browser/429893#Top

2/2

8/13/2016

MyPastest

Back to Filters (/Secure/TestMe/Filter/429893/QA)

Question 121 of 179

A 55-year-old woman has had a history of hypertension over several years. Over the past 3
weeks she has experienced symptoms of fever and malaise. Her temperature is 38C, and she
is noted to have a facial rash that she says is new. There is also swelling and tenderness
affecting her wrists and knees. Blood pressure is normal.
Which of the following drugs is most likely to have caused these findings?
A

Amlodipine

Enalapril

Hydralazine

Hydrochlorothiazide

Propranolol

Explanation

The answer is Hydralazine


Drug-induced lupus is a syndrome mimicking systemic lupus erythematosus, often
involving the joints and generally without renal involvement. It is typically caused by
treatment with hydralazine, procainamide, phenytoin or ethosuximide. Risk is associated
with high doses, and with hydralazine and procainamide it is more common in slow
acetylators. Drug fever as an isolated phenomenon can occur with penicillins, phenytoin,
hydralazine and quinidine; fever is usually of low grade and the patient is generally well;
the fever normally subsides within a few days of stopping the causative drug.

Amlodipine (Option A) is incorrect. Amlodipine may cause peripheral oedema but not fever
or the other features of lupus.
Enalapril (Option B) is incorrect. Enalapril does not cause fever or lupus.
Hydrochlorothiazide (Option D) is incorrect. Thiazides typically cause hypokalaemia and
increased risk of gout.
Propranolol (Option E) is incorrect. Propranolol is not a recognised cause of lupus.
46818

https://mypastest.pastest.com/Secure/TestMe/Browser/429893#Top

1/2

8/13/2016

MyPastest

46818

Next Question

Previous Question

Tag Question

Feedback

End Review

Difficulty: Average
Peer Responses

Session Progress
Responses Correct:

Responses Incorrect:

179

Responses Total:

179

Responses - % Correct:

0%

Blog (https://www.pastest.com/blog) About Pastest (https://www.pastest.com/about-us)


Contact Us (https://www.pastest.com/contact-us) Help (https://www.pastest.com/help)
Pastest 2016

https://mypastest.pastest.com/Secure/TestMe/Browser/429893#Top

2/2

8/13/2016

MyPastest

Back to Filters (/Secure/TestMe/Filter/429893/QA)

Question 122 of 179

You are involved in selecting a candidate drug for further clinical development as an
antihypertensive agent. You have been provided with data concerning a number of different
compounds under investigation.
Which of the following compounds, according to its mode of clearance, is most likely to show
stable pharmacokinetic properties when tested between patients?
A

Compound A is predominantly cleared via the kidneys

Compound B is predominantly cleared via the CYP2D6 route

Compound C is one-third cleared by the kidneys and two-thirds by two different


P450 isoforms, neither by CYP2D6

Compound D is 50% cleared by CYP2D6, 50% by another P450 isoform

Compound E is predominantly cleared via CYP3A4

Explanation

The answer is Compound C is one-third cleared by the kidneys and two-thirds by two
different P450 isoforms, neither by CYP2D6
To maximise drug absorption, the ideal compound should be small (molecular weight of
less than 300 kDa) and have intermediate lipophilicity and hydrophilicity. Clearance by
renal and hepatic excretion renders the drug less susceptible to variation in renal or
hepatic metabolism, particularly with regard to genetic factors that alter liver clearance
between individuals (eg CYP2D6 or CYP3A4 variations). In addition, the ideal profile of
our candidate drug should be neither an inhibitor nor an inducer of the P450 system to
avoid interactions with other medications.

Compound A is predominantly cleared via the kidneys (Option A) is incorrect. Drugs that
depend solely upon renal elimination are susceptible to inter-individual pharmacokinetic
variations due to variable renal function.
Compound B is predominantly cleared via the CYP2D6 route (Option B) is incorrect. CYP2D6
is a hepatic enzymatic pathway that shows the greatest genetic variability, and hence there
https://mypastest.pastest.com/Secure/TestMe/Browser/429893#Top

1/2

8/13/2016

MyPastest

are likely to be significant and unpredictable pharmacokinetic differences between patients


in drugs metabolised down this route.
Compound D is 50% cleared by CYP2D6, 50% by another P450 isoform (Option D) is
incorrect. Drugs that are eliminated solely by hepatic elimination are prone to inter-individual
variations in liver function, particularly when specific pathways such as the CYP2D6 or
CYP3A4 isoenzymes are involved (these are susceptible to genetic variation and altered drug
clearance).
Compound E is predominantly cleared via CYP3A4 (Option E) is incorrect. The CYP3A4
isoenzyme is subject to genetic variation, and likely to result in significant differences in drug
handling between individuals.
46840

Next Question

Previous Question

Tag Question

Feedback

End Review

Difficulty: Average
Peer Responses

Session Progress
Responses Correct:

Responses Incorrect:

179

Responses Total:

179

Responses - % Correct:

0%

Blog (https://www.pastest.com/blog) About Pastest (https://www.pastest.com/about-us)


Contact Us (https://www.pastest.com/contact-us) Help (https://www.pastest.com/help)
Pastest 2016

https://mypastest.pastest.com/Secure/TestMe/Browser/429893#Top

2/2

8/13/2016

MyPastest

Back to Filters (/Secure/TestMe/Filter/429893/QA)

Question 123 of 179

You are asked to review a 72-year-old man who is being treated with warfarin for chronic
atrial fibrillation. His cholesterol is 6.5 mmol/l with triglycerides of 3.1 mmol/l. You want to
start him on some lipid-lowering therapy.
Which would be the best drug to prescribe with respect to balancing efficacy versus
potentiating the effects of his warfarin treatment?
A

Atorvastatin

Bezafibrate

Chlolestyramine

Rosuvastatin

Simvastatin

Explanation

The answer is Atorvastatin


Atorvastatin and pravastan do not appreciably interfere with warfarin and may safely be
added with little impact upon warfarin monitoring.

Bezafibrate (Option B) is incorrect. Fibrates can increase the anticoagulant effects of


warfarin.
Chlolestyramine (Option C) is incorrect. Cholestyramine would be ineffective in lowering
cholesterol in a meaningful way, and can significantly increase warfarin effects.
Rosuvastatin (Option D) is incorrect. Rosuvastatin may potentiate the anticoagulant effects
of warfarin, although there is less evidence of this than for simvastatin or fibrates.
Simvastatin (Option E) is incorrect. Simvastatin may potentiate the anticoagulant effects of
warfarin therapy.
46825

Next Question
https://mypastest.pastest.com/Secure/TestMe/Browser/429893#Top

1/2

8/13/2016

MyPastest

Previous Question

Tag Question

Feedback

End Review

Difficulty: Average
Peer Responses

Session Progress
Responses Correct:

Responses Incorrect:

179

Responses Total:

179

Responses - % Correct:

0%

Blog (https://www.pastest.com/blog) About Pastest (https://www.pastest.com/about-us)


Contact Us (https://www.pastest.com/contact-us) Help (https://www.pastest.com/help)
Pastest 2016

https://mypastest.pastest.com/Secure/TestMe/Browser/429893#Top

2/2

8/13/2016

MyPastest

Back to Filters (/Secure/TestMe/Filter/429893/QA)

Question 124 of 179

A 48-year-old man is reviewed in the outpatient medical clinic. He has a past history of
angina, hypertension and type-2 diabetes and is receiving a number of regular medications.
He mentions that he would like to receive drug therapy for impotence, and you consider
prescribing sildenafil.
Which one of the following medications is likely to pose the greatest risk if taken alongside
sildenafil treatment?
A

Aspirin

Bendroflumethiazide

Isosorbide mononitrate

Lisinopril

Metformin

Explanation

The answer is Isosorbide mononitrate


Sildenafil is a phosphodiesterase (PDE-5) inhibitor indicated for the treatment of erectile
dysfunction. It is contraindicated in patients with proven coronary artery disease and in
patients who are taking oral nitrate therapy due to an increased risk of systemic
vasodilatation and profound hypotension. A common side-effect of sildenafil includes
headache, predominantly through cerebral vasodilation. Alternatives to sildenafil may
include prostaglandins given via pessary or injection into the penis; apomorphine may
also be considered although a number of important adverse effects are recognised.

Aspirin (Option A) is incorrect. There is no significant interaction between sildenafil and


aspirin.
Bendroflumethiazide (Option B) is incorrect. There is no significant interaction between
sildenafil and bendrofluazide.
Lisinopril (Option D) is incorrect. There is a potential for the combination of sildenafil and
lisinopril to cause significant hypotension. However, the risks are greater for the combination
https://mypastest.pastest.com/Secure/TestMe/Browser/429893#Top

1/2

8/13/2016

MyPastest

of sildenafil and nitrate therapy, hence option C is the preferred answer.


Metformin (Option E) is incorrect. There is no significant interaction between sildenafil and
metformin.
46920

Next Question

Previous Question

Tag Question

Feedback

End Review

Difficulty: Average
Peer Responses

Session Progress
Responses Correct:

Responses Incorrect:

179

Responses Total:

179

Responses - % Correct:

0%

Blog (https://www.pastest.com/blog) About Pastest (https://www.pastest.com/about-us)


Contact Us (https://www.pastest.com/contact-us) Help (https://www.pastest.com/help)
Pastest 2016

https://mypastest.pastest.com/Secure/TestMe/Browser/429893#Top

2/2

8/13/2016

MyPastest

Back to Filters (/Secure/TestMe/Filter/429893/QA)

Question 125 of 179

A 64-year-old man presents to the Emergency Department complaining of marked muscle


aches and tenderness. He has a history of previous myocardial infarction and takes a number
of medications, including simvastatin. Recently he has made a number of changes to his diet
to try to improve his health. Plasma creatine kinase in the Emergency Department was
measured at 4800 U/l and you suspect rhabdomyolysis.
Which one of the following interactions with simvastatin is most likely to be responsible for
his presentation?
A

Cod liver oil capsules

Cranberry juice

Grapefruit juice

St John s wort

Vitamin C

Explanation

The answer is Grapefruit juice


Grapefruit juice is a potent inhibitor of CYP3A4 metabolism, which affects levels of a
number of drugs, including ciclosporin and simvastatin.

Cod liver oil capsules (Option A) is incorrect. Cod liver oil capsules are not known to interact
with P450 enzymes.
Cranberry juice (Option B) is incorrect. Cranberry juice contains bioflavonoids that inhibit the
CYP2C9 isoenzyme, thereby inhibiting metabolism of warfarin.
St John s wort (Option D) is incorrect. St Johns wort is a cytochrome P450 enzyme inducer
and as such may reduce the effectiveness of drugs that are metabolised by the P450 route,
including simvastatin and warfarin.
Vitamin C (Option E) is incorrect. Vitamin C is not known to affect simvastatin metabolism.
46972

https://mypastest.pastest.com/Secure/TestMe/Browser/429893#Top

1/2

8/13/2016

MyPastest

Next Question

Previous Question

Tag Question

Feedback

End Review

Difficulty: Average
Peer Responses

Session Progress
Responses Correct:

Responses Incorrect:

179

Responses Total:

179

Responses - % Correct:

0%

Blog (https://www.pastest.com/blog) About Pastest (https://www.pastest.com/about-us)


Contact Us (https://www.pastest.com/contact-us) Help (https://www.pastest.com/help)
Pastest 2016

https://mypastest.pastest.com/Secure/TestMe/Browser/429893#Top

2/2

8/13/2016

MyPastest

Back to Filters (/Secure/TestMe/Filter/429893/QA)

Question 126 of 179

A 62-year-old woman is treated with NSAIDs and long term Methotrexate for rheumatoid
arthritis. She presents to the clinic for review and complains of increasing nausea and
indigestion. An additional finding is increased BP (now 157/72 mmHg), and she is started on
new medications for both problems by her doctor. A recent GFR is measured at 28ml/min.
Which of the following medications should be used with caution in this situation?
A

Amlodipine

Bisoprolol

Gaviscon

Omeprazole

Ranitidine

Explanation
The answer is Omeprazole
Methotrexate is a substrate for the OAT-1 renal transporter and levels of methotrexate are
therefore affected by decreased renal function. OAT-1 inhibitors include drugs such as
Probenecid, and therefore should not be used in conjunction with Methotrexate. Omeprazole
is also known to affect clearance of Methotrexate, this interaction is not thought to be via
OAT-1, but is thought to be related to inhibition of breast cancer resistance protein, which is
responsible for Methotrexate transport.
The other options given are not thought to affect Methotrexate levels. In this situation with a
GFR of 28ml/min an alginate preparation or Ranitidine may therefore be better options for
initial therapy for indigestion in this situation.
37299

Next Question

Previous Question

Tag Question

https://mypastest.pastest.com/Secure/TestMe/Browser/429893#Top

Feedback

End Review

1/2

8/13/2016

MyPastest

Difficulty: Difficult
Peer Responses

Session Progress
Responses Correct:

Responses Incorrect:

179

Responses Total:

179

Responses - % Correct:

0%

Blog (https://www.pastest.com/blog) About Pastest (https://www.pastest.com/about-us)


Contact Us (https://www.pastest.com/contact-us) Help (https://www.pastest.com/help)
Pastest 2016

https://mypastest.pastest.com/Secure/TestMe/Browser/429893#Top

2/2

8/13/2016

MyPastest

Back to Filters (/Secure/TestMe/Filter/429893/QA)

Question 127 of 179

A pharmaceutical company wants to bring a generic formulation of candesartanranitidine to


the market after the patent has expired.
What kind of study is needed to obtain approval to market the drug?
A

Phase-I bioequivalence study

Phase-II proof-of-concept study

Phase-III study in patients

Phase-IV study in patients

Preclinical toxicology study

Explanation

The answer is Phase-I bioequivalence study


Generic medicines are produced by companies who are subject to the same tight
manufacturing quality controls as those who make branded products. Often the same
company makes both branded and generic medicines. A phase-I bioequivalence study
ensures that the pharmacokinetic properties of the drug are acceptably similar to the
original product, including the maximum concentration (Cmax), the area under the curve
(AUC) and the time of maximum concentration (tmax). There is no need to submit the
entire body of pre-clinical research required for a new agent.

Phase-II proof-of-concept study (Option B) is incorrect. There is no need to repeat phase-II


proof-of-concept studies.
Phase-III study in patients (Option C) is incorrect. There is no need to repeat phase-III clinical
studies.
Phase-IV study in patients (Option D) is incorrect. There is no need to repeat phase-IV or
post-marketing studies.
Preclinical toxicology study (Option E) is incorrect. Preclinical toxicology studies are
performed before and during phase-1 clinical research to identify possible adverse effects
https://mypastest.pastest.com/Secure/TestMe/Browser/429893#Top

1/2

8/13/2016

MyPastest

that might be relevant in people.


46809

Next Question

Previous Question

Tag Question

Feedback

End Review

Difficulty: Average
Peer Responses

Session Progress
Responses Correct:

Responses Incorrect:

179

Responses Total:

179

Responses - % Correct:

0%

Blog (https://www.pastest.com/blog) About Pastest (https://www.pastest.com/about-us)


Contact Us (https://www.pastest.com/contact-us) Help (https://www.pastest.com/help)
Pastest 2016

https://mypastest.pastest.com/Secure/TestMe/Browser/429893#Top

2/2

8/13/2016

MyPastest

Back to Filters (/Secure/TestMe/Filter/429893/QA)

Question 128 of 179

Tamoxifen has been prescribed for a woman who is undergoing treatment for breast
carcinoma.
What is the most significant characteristic of this drug?
A

A yearly PAP smear is recommended for women on tamoxifen

It functions as a selective oestrogen-receptor modulator

It has no effect on tumours that are oestrogen-receptor negative

It increases low-density lipoprotein cholesterol levels

It is recommended mainly in women who have undergone hysterectomy

Explanation

The answer is It functions as a selective oestrogen-receptor modulator


Tamoxifen is a selective oestrogen-receptor modulator (SERM). It acts both as a potent
oestrogen antagonist in some areas and as a weak oestrogen agonist in other places. In
patients with advanced metastatic breast cancer, oestrogen deprivation causes tumour
regression in 30% of unselected women and in more than 60% of those with oestrogenreceptor-positive tumours for a median duration of 20 months. In the adjuvant setting,
ovarian ablation or tamoxifen results in a 25% reduction in the relative risk of dying from
metastatic disease.

A yearly PAP smear is recommended for women on tamoxifen (Option A) is incorrect.


Tamoxifen does not increase the risk of cervical carcinoma, so there is no need for increased
frequency of PAP smear screening.
It has no effect on tumours that are oestrogen-receptor negative (Option C) is incorrect.
Benefits are greatest in patients with tumours that express the oestrogen receptor, but are
not exclusive to these patients.
It increases low-density lipoprotein cholesterol levels (Option D) is incorrect. Tamoxifen
decreases low-density lipoprotein cholesterol with no change in high-density lipoprotein
cholesterol, and thus decreases the risk of coronary artery disease.
https://mypastest.pastest.com/Secure/TestMe/Browser/429893#Top

1/2

8/13/2016

MyPastest

It is recommended mainly in women who have undergone hysterectomy (Option E) is


incorrect. Endometrial sampling is recommended for women on tamoxifen as the risk of
endometrial cancer is increased 23 times with long-term use, but prior hysterectomy is not a
prerequisite for treatment with tamoxifen.
46872

Next Question

Previous Question

Tag Question

Feedback

End Review

Difficulty: Average
Peer Responses

Session Progress
Responses Correct:

Responses Incorrect:

179

Responses Total:

179

Responses - % Correct:

0%

Blog (https://www.pastest.com/blog) About Pastest (https://www.pastest.com/about-us)


Contact Us (https://www.pastest.com/contact-us) Help (https://www.pastest.com/help)
Pastest 2016

https://mypastest.pastest.com/Secure/TestMe/Browser/429893#Top

2/2

8/13/2016

MyPastest

Back to Filters (/Secure/TestMe/Filter/429893/QA)

Question 129 of 179

You are asked to review a 36-year-old man who suffers from partial epilepsy. He has
presented to the Emergency Department after a fall, and a visual-field defect has been
identified. He has been on vigabatrin combination therapy for 3 years and his family state
that his epilepsy had been poorly managed up until then.
What type of adverse drug reaction is most likely to be responsible for his visual-field
defect?
A

Type A

Type B

Type C

Type D

Type E

Explanation

The answer is Type C


Type C reactions occur in the setting of prolonged drug use (eg analgesic nephropathy
or visual-field defects with vigabatrin, as in this case). Visual-field defects due to
vigabatrin use are unpredictable and may occur any time between 1 month and several
years after the commencement of therapy. Visual-field defects are an absolute
contraindication to vigabatrin use.

Type A (Option A) is incorrect. Type A adverse drug reactions occur as an anticipated effect
arising from a particular pharmacological property of the drug; they are dose-related, eg
bradycardia as an adverse effect of -adrenoceptor blocking drugs.
Type B (Option B) is incorrect. Type B adverse drug reactions are unpredictable and not
related to a predictable pharmacological effect of the drug. They are idiosyncratic,
unexpected, usually rare, and may be serious.
Type D (Option D) is incorrect. Type D adverse drug reactions emerge late after treatment,
e.g. teratogenic or carcinogenic reactions, such as phocomyelia after administration of
https://mypastest.pastest.com/Secure/TestMe/Browser/429893#Top

1/2

8/13/2016

MyPastest

thalidomide in pregnancy.
Type E (Option E) is incorrect. Type E adverse drug reactions are related to drug withdrawal
phenomena observed after a drug is discontinued, e.g. SSRI cessation and agitation, or
opioid withdrawal syndrome.
46848

Next Question

Previous Question

Tag Question

Feedback

End Review

Difficulty: Average
Peer Responses

Session Progress
Responses Correct:

Responses Incorrect:

179

Responses Total:

179

Responses - % Correct:

0%

Blog (https://www.pastest.com/blog) About Pastest (https://www.pastest.com/about-us)


Contact Us (https://www.pastest.com/contact-us) Help (https://www.pastest.com/help)
Pastest 2016

https://mypastest.pastest.com/Secure/TestMe/Browser/429893#Top

2/2

8/13/2016

MyPastest

Back to Filters (/Secure/TestMe/Filter/429893/QA)

Question 130 of 179

You are asked to give advice to a GP regarding a family with concerns that they may have
been exposed to high lead concentrations in their local water supply; they are worried that
they might be suffered from lead poisoning.
Which one of the following features would most strongly support a diagnosis of lead
toxicity?
A

Basophilic stippling of red blood cells on peripheral blood film

Chronic daily headache

Development of membranous glomerulonephritis

Posterior uveitis

Sensory neuropathy in a glove and stocking distribution

Explanation

The answer is Basophilic stippling of red blood cells on peripheral blood film
Lead poisoning may arise from occupational exposure (eg miners, welders, storage
battery workers and pottery glaze workers) or environmental exposure. Accumulation
may occur as a result of lead ingestion, inhalation and direct skin contact. The most
common clinical feature is abdominal pain, possibly due to a direct effect of lead on
intestinal smooth muscle. Lead interferes with a variety of red cell enzymes and causes
characteristic red cell abnormalities, including punctate basophilic stippling and clover
leaf morphology. Other features include peripheral neuropathy, which is almost
exclusively motor. Interstitial nephritis is a characteristic lesion affecting the kidneys, and
a gingival blue-black or grey line may be found in up to 20% of adults.

Chronic daily headache (Option B) is incorrect. Headache is not a prominent feature.


Development of membranous glomerulonephritis (Option C) is incorrect. Lead toxicity may
cause interstitial nephritis and cause proximal tubular dysfunction.
Posterior uveitis (Option D) is incorrect. Lead poisoning may cause anterior uveitis or iritis,
rather than posterior uveitis.
https://mypastest.pastest.com/Secure/TestMe/Browser/429893#Top

1/2

8/13/2016

MyPastest

Sensory neuropathy in a glove and stocking distribution (Option E) is incorrect. Lead


neuropathy normally involves motor deficits.
46623

Next Question

Previous Question

Tag Question

Feedback

End Review

Difficulty: Average
Peer Responses

Session Progress
Responses Correct:

Responses Incorrect:

179

Responses Total:

179

Responses - % Correct:

0%

Blog (https://www.pastest.com/blog) About Pastest (https://www.pastest.com/about-us)


Contact Us (https://www.pastest.com/contact-us) Help (https://www.pastest.com/help)
Pastest 2016

https://mypastest.pastest.com/Secure/TestMe/Browser/429893#Top

2/2

8/13/2016

MyPastest

Back to Filters (/Secure/TestMe/Filter/429893/QA)

Question 131 of 179

Iron deficiency anaemia has been diagnosed in an 80-year-old woman suffering from
osteoarthritis. She also has a history of cardiovascular disease. You suspect a cause related to
her medication.
Which of the following medications is most likely to have caused the iron deficiency
anaemia?
A

Aspirin

Celecoxib

Diclofenacmisoprostol combination

Lactulose

Metoclopramide

Explanation

The answer is Aspirin


Chronic, low-volume blood loss is usually subclinical until such time as the patient
presents with iron deficiency anaemia. Equally, the haemoglobin may be low in a patient
with iron deficiency anaemia resulting from chronic haemorrhage, who presents with a
small, acute bleed. Many gastrointestinal lesions can result in haemorrhage but peptic
ulcers are the most frequent in the UK. Aspirin and non-steroidal anti-inflammatory drugs
(NSAIDs) may cause peptic ulceration affecting any part of the bowel.

Celecoxib (Option B) is incorrect. Cyclo-oxygenase-II selective inhibitors produce fewer


gastrointestinal side-effects compared with NSAIDs. Rofecoxib (Vioxx) was withdrawn from
the market worldwide in 2004 because of an increased risk of myocardial infarction and
cerebrovascular events after long-term use.
Diclofenacmisoprostol combination (Option C) is incorrect. Compared with placebo,
misoprostol significantly reduces NSAID-associated gastric and duodenal ulcers found on
endoscopy and reduces serious upper gastrointestinal (GI) complications. Bleeding may still

https://mypastest.pastest.com/Secure/TestMe/Browser/429893#Top

1/2

8/13/2016

MyPastest

occur despite misoprostol but the occurrence is likely to be less than that in the case of
NSAIDs alone or aspirin.
Lactulose (Option D) is incorrect. Lactulose does not increase the risk of peptic ulceration or
GI bleeding.
Metoclopramide (Option E) is incorrect. Metoclopramide increases gastric emptying.
46810

Next Question

Previous Question

Tag Question

Feedback

End Review

Difficulty: Average
Peer Responses

Session Progress
Responses Correct:

Responses Incorrect:

179

Responses Total:

179

Responses - % Correct:

0%

Blog (https://www.pastest.com/blog) About Pastest (https://www.pastest.com/about-us)


Contact Us (https://www.pastest.com/contact-us) Help (https://www.pastest.com/help)
Pastest 2016

https://mypastest.pastest.com/Secure/TestMe/Browser/429893#Top

2/2

8/13/2016

MyPastest

Back to Filters (/Secure/TestMe/Filter/429893/QA)

Question 132 of 179

A 31-year-old woman is found unconscious with empty bottles of pills and an empty vodka
bottle, and brought to the Emergency Department. An ECG shows rate of 140 bpm with wide
QRS (115 ms).
Which one of the following is the best first-line management?
A

Atenolol

Bicarbonate

Lidocaine

Naloxone

Verapamil

Explanation

The answer is Bicarbonate


This lady is unconscious and in ventricular tachycardia, and the most likely explanation in
this context is tricyclic overdose. Intravenous sodium bicarbonate reduces sodium
channel blockade, and makes arrhythmia and seizures less likely to occur. Correction of
acidosis often resolves the arrhythmia.

Atenolol (Option A) is incorrect. Beta-blockers may be used to aid rate control in atrial
fibrillation and to control supraventricular tachycardias, but would do little to alter sodium
channel conductance, which is the main mechanism by which tricyclics cause arrhythmia and
seizure.
Lidocaine (Option C) is incorrect. Lidocaine and other class I agents are ineffective in
tricyclic poisoning, and may worsen the situation by additional sodium channel blockade.
Naloxone (Option D) is incorrect. Naloxone is used as a competitive antagonist to the effects
of opiate toxicity.
Verapamil (Option E) is incorrect. Verapamil should be avoided in tachycardia unless a
supraventricular origin is established.
46965

https://mypastest.pastest.com/Secure/TestMe/Browser/429893#Top

1/2

8/13/2016

MyPastest

46965

Next Question

Previous Question

Tag Question

Feedback

End Review

Difficulty: Average
Peer Responses

Session Progress
Responses Correct:

Responses Incorrect:

179

Responses Total:

179

Responses - % Correct:

0%

Blog (https://www.pastest.com/blog) About Pastest (https://www.pastest.com/about-us)


Contact Us (https://www.pastest.com/contact-us) Help (https://www.pastest.com/help)
Pastest 2016

https://mypastest.pastest.com/Secure/TestMe/Browser/429893#Top

2/2

8/13/2016

MyPastest

Back to Filters (/Secure/TestMe/Filter/429893/QA)

Question 133 of 179

A 43-year-old woman presents to the Emergency Department after an overdose involving a


mixture of medications including paracetamol, citalopram, sodium valproate, fluoxetine and
ferrous sulfate. You are contemplating administering oral activated charcoal.
Which of the following factors would be most strongly influence your decision to administer
or avoid oral activated charcoal?
A

Absence of bowel sounds

Administration is less effective after mixed overdose

Charcoal minimises toxicity if given within 1 hour of iron overdose

Charcoal should be used only for modified-release preparations

The patient is asymptomatic

Explanation

The answer is Absence of bowel sounds


Oral activated charcoal adsorbs many drugs and complex chemicals, thereby reducing
drug absorption from the gastrointestinal tract, and interrupting enterohepatic
recirculation. It is generally safe, but should be administered only in patients who are
able to protect their airway. The absence of bowel sounds may indicate a paralytic ileus,
which is surprisingly common after overdose, and which is associated with an increased
risk of charcoal aspiration and pneumonitis.

Administration is less effective after mixed overdose (Option B) is incorrect. Activated


charcoal is capable of adsorbing around 10% of its own weight, so administration of charcoal
50 g might be expected to adsorb around 5 g of drug.
Charcoal minimises toxicity if given within 1 hour of iron overdose (Option C) is incorrect.
Iron, lithium and other cations are not adsorbed by charcoal; alcohols including ethanol,
methanol and ethylene glycol are not adsorbed either.

https://mypastest.pastest.com/Secure/TestMe/Browser/429893#Top

1/2

8/13/2016

MyPastest

Charcoal should be used only for modified-release preparations (Option D) is incorrect.


Activated charcoal should normally be administered within 1 hour of drug overdose, but may
be effective when administered after a longer interval, particularly after modified-release
preparations.
The patient is asymptomatic (Option E) is incorrect. The absence of symptoms does not help
determine whether patients are at risk of delayed toxicity, and charcoal should normally be
administered within 1 hour of overdose to minimise absorption and prevent toxicity.
45804

Next Question

Previous Question

Tag Question

Feedback

End Review

Difficulty: Average
Peer Responses

Session Progress
Responses Correct:

Responses Incorrect:

179

Responses Total:

179

Responses - % Correct:

0%

Blog (https://www.pastest.com/blog) About Pastest (https://www.pastest.com/about-us)


Contact Us (https://www.pastest.com/contact-us) Help (https://www.pastest.com/help)
Pastest 2016

https://mypastest.pastest.com/Secure/TestMe/Browser/429893#Top

2/2

8/13/2016

MyPastest

Back to Filters (/Secure/TestMe/Filter/429893/QA)

Question 134 of 179

A 25-year-old man presents to the Emergency Department with an acute exacerbation of


asthma that is failing to respond to increased use of his usual inhaled salbutamol inhaler. You
are asked to review him in the resuscitation area and diagnose acute severe asthma; you
decide to treat him with intravenous magnesium.
Which of the following statements is true?
A

A rise in systemic blood pressure occurs commonly

Drowsiness and coma in severe asthma are normally caused by hypomagnesaemia

Magnesium increases bronchial acetylcholine release

Magnesium relaxes bronchial smooth muscle

The usual dose is 2 mg intravenously

Explanation

The answer is Magnesium relaxes bronchial smooth muscle


Intravenous magnesium is now indicated in the management of severe life-threatening
acute asthma attacks; the normal adult dose is 1.22 g given over 20 min. Its principal
pharmacological actions are inhibition of acetylcholine release at the neuromuscular
junction, relaxation of bronchial smooth muscle and stabilisation of mast cells. Adverse
effects are uncommon after a single administration, although a slight decrease in blood
pressure and flushing can occur. Symptoms of chronic hypermagnesaemia may include
nausea, diarrhoea, flushing, hypertension, confusion, loss of deep tendon reflexes and
coma.

A rise in systemic blood pressure occurs commonly (Option A) is incorrect. A slight fall in
blood pressure may occur.
Drowsiness and coma in severe asthma are normally caused by hypomagnesaemia (Option
B) is incorrect. Drowsiness and coma in severe asthma may be caused by hypercapnoea and
respiratory acidosis, and/or hypoxia.

https://mypastest.pastest.com/Secure/TestMe/Browser/429893#Top

1/2

8/13/2016

MyPastest

Magnesium increases bronchial acetylcholine release (Option C) is incorrect. Magnesium


reduces bronchial acetylcholine release.
The usual dose is 2 mg intravenously (Option E) is incorrect. The usual dose is 1.22 g. It is
reasonable to expect candidates to know dosages for medications required in emergency
situations such as this.
46377

Next Question

Previous Question

Tag Question

Feedback

End Review

Difficulty: Average
Peer Responses

Session Progress
Responses Correct:

Responses Incorrect:

179

Responses Total:

179

Responses - % Correct:

0%

Blog (https://www.pastest.com/blog) About Pastest (https://www.pastest.com/about-us)


Contact Us (https://www.pastest.com/contact-us) Help (https://www.pastest.com/help)
Pastest 2016

https://mypastest.pastest.com/Secure/TestMe/Browser/429893#Top

2/2

8/13/2016

MyPastest

Back to Filters (/Secure/TestMe/Filter/429893/QA)

Question 135 of 179

A 25-year-old woman who has a history of asthma attends for review. She is currently taking
fluticasone dipropionate 250 g inhaled twice daily. You plan to add salmeterol.
Which of the following statements best fits the pharmacological characteristics of
salmeterol?
A

It may cause paradoxical bronchospasm

It is a long-acting 1-agonist

It is a short-acting 1-agonist

It is a short-acting 2-agonist

Tachycardias are not associated with its use

Explanation

The answer is It may cause paradoxical bronchospasm


Salmeterol is a long-acting 2-agonist indicated for the treatment of bronchial asthma in
combination with inhaled corticosteroid therapy. It should not be used as first-line
therapy because clinical studies suggest increased mortality and morbidity in patients
treated with long-acting 2-agonists monotherapy; they are normally used after inhaled
corticosteroids. Bronchospasm is a rare but recognised complication of salmeterol and
other 2-agonist treatments; bronchospasm is a more common complication of inhaled
disodium chromoglycate. Recognised effects of salmeterol and formoterol (long-acting
2-agonists) include hypokalaemia; they should be used with caution in severe liver
cirrhosis, breast feeding and pregnancy.

It is a long-acting 1-agonist (Option B) is incorrect. Although salmeterol is 2-selective, it is


not 100% specific and there is some crossover of receptor expression between cardiac and
pulmonary tissue so that 1- and 2-agonist effects are recognised.
It is a short-acting 1-agonist (Option C) is incorrect. Although salmeterol is 2-selective, it is
not 100% specific and there is some crossover of receptor expression between cardiac and
pulmonary tissue so that 1- and 2-agonist effects are recognised.
https://mypastest.pastest.com/Secure/TestMe/Browser/429893#Top

1/2

8/13/2016

MyPastest

It is a short-acting 2-agonist (Option D) is incorrect. Salmeterol and formoterol are longacting agents; salbutamol and terbutaline are short-acting 2-agonists.
Tachycardias are not associated with its use (option E) is incorrect. Although delivered in
highest concentration to the lungs by inhalation, there may be some systemic effects,
including tachycardia due to direct stimulation of 1- and 2-adrenoceptors in the heart. It is a
long-acting agent.
46882

Next Question

Previous Question

Tag Question

Feedback

End Review

Difficulty: Average
Peer Responses

Session Progress
Responses Correct:

Responses Incorrect:

179

Responses Total:

179

Responses - % Correct:

0%

Blog (https://www.pastest.com/blog) About Pastest (https://www.pastest.com/about-us)


Contact Us (https://www.pastest.com/contact-us) Help (https://www.pastest.com/help)
Pastest 2016

https://mypastest.pastest.com/Secure/TestMe/Browser/429893#Top

2/2

8/13/2016

MyPastest

Back to Filters (/Secure/TestMe/Filter/429893/QA)

Question 136 of 179

A 27-year-old homeless man is admitted having ingested a large quantity of methanol which
he found on a garage forecourt. He is inebriated with a deteriorating conscious level. He has a
pH of 7.15 with a serum bicarbonate concentration of 14 mmol/l.
What is the most appropriate next step in his management?
A

Administration of ethanol

Administration of ethylene glycol

Administration of folinic acid

Administration of formaldehyde

Administration of formate

Explanation

The answer is Administration of ethanol


Methanol is subject to hepatic metabolism, initially by alcohol dehydrogenase and
additional enzymatic steps that result in formation of toxic metabolites. The metabolites
may cause severe nephrotoxicity and neurotoxicity, and provoke a number of metabolic
derangements including profound hypocalcaemia. Administration of fomepizole, a
competitive alcohol dehydrogenase inhibitor, prevents formation of methanol
metabolites and thereby minimises toxicity. An alternative is administration of ethanol
which competes with methanol for metabolism, thereby lessening but not fully
preventing methanol metabolism. Haemodialysis may be considered when methanol
levels are particularly high or when metabolic acidosis cannot be corrected by
bicarbonate administration.

Administration of ethylene glycol (Option B) is incorrect. Ethylene glycol would contribute to


toxicity.
Administration of folinic acid (Option C) is incorrect. Administration of folinic acid may
protect against ocular toxicity but is a less good option here because it is an adjunctive
treatment that may be considered in addition to fomepizole or ethanol administration.
https://mypastest.pastest.com/Secure/TestMe/Browser/429893#Top

1/2

8/13/2016

MyPastest

Administration of formaldehyde (Option D) is incorrect. Formaldehyde is incorrect; this is one


of the metabolites that contributes to metabolic acidosis.
Administration of formate (Option E) is incorrect. Formate is one of the metabolic substrates
arising from metabolism of formaldehyde by formaldehyde dehydrogenase.
46847

Next Question

Previous Question

Tag Question

Feedback

End Review

Difficulty: Average
Peer Responses

Session Progress
Responses Correct:

Responses Incorrect:

179

Responses Total:

179

Responses - % Correct:

0%

Blog (https://www.pastest.com/blog) About Pastest (https://www.pastest.com/about-us)


Contact Us (https://www.pastest.com/contact-us) Help (https://www.pastest.com/help)
Pastest 2016

https://mypastest.pastest.com/Secure/TestMe/Browser/429893#Top

2/2

8/13/2016

MyPastest

Back to Filters (/Secure/TestMe/Filter/429893/QA)

Question 137 of 179

A 32-year-old former heroin user is on a methadone programme. He is admitted to hospital


after a motor vehicle accident and has sustained multiple pelvic fractures.
Which one of the following is the most appropriate approach to his analgesic medications?
A

Continue on methadone and add diclofenac

Continue methadone and titrate to pain requirement

Discontinue methadone

Discontinue methadone and start on parenteral morphine

Discontinue methadone and substitute diclofenac

Explanation

The answer is Continue on methadone and add diclofenac


Continuation of methadone and consideration of analgesics with a different mode of
action (ie non-steroidals such as parenteral diclofenac) is recommended. Despite
concerns about cardiovascular risk associated with NSAIDs, the balance of benefitrisk
with respect to short-term use of parenteral diclofenac is favourable in this situation.
Methadone can then be titrated over a number of days to relieve the extra pain owing to
the pelvic fractures in this particular patient.

Continue methadone and titrate to pain requirement (Option B) is incorrect. Titration of


methadone every 68 h for acute pain is not recommended in chronic users because there is
a significant risk of accumulation.
Discontinue methadone (Option C) is incorrect. Discontinuation of methadone may result in
symptoms of acute opiate withdrawal and is not recommended.
Discontinue methadone and start on parenteral morphine (Option D) is incorrect.
Discontinuation of methadone and substitution with morphine would be complex and
difficult to manage safely; patients would be at high risk of opioid intoxication or withdrawal
features.
https://mypastest.pastest.com/Secure/TestMe/Browser/429893#Top

1/2

8/13/2016

MyPastest

Discontinue methadone and substitute diclofenac (Option E) is incorrect. Discontinuation of


methadone may result in symptoms of acute opioid withdrawal.
46927

Next Question

Previous Question

Tag Question

Feedback

End Review

Difficulty: Average
Peer Responses

Session Progress
Responses Correct:

Responses Incorrect:

179

Responses Total:

179

Responses - % Correct:

0%

Blog (https://www.pastest.com/blog) About Pastest (https://www.pastest.com/about-us)


Contact Us (https://www.pastest.com/contact-us) Help (https://www.pastest.com/help)
Pastest 2016

https://mypastest.pastest.com/Secure/TestMe/Browser/429893#Top

2/2

8/13/2016

MyPastest

Back to Filters (/Secure/TestMe/Filter/429893/QA)

Question 138 of 179

A 35-year-old woman, who has been on antiepileptic medication for many years, presents
complaining of fatigue, lethargy, bone pain, tingling and numbness in her lower limbs, and
swelling of her gums. Investigations reveal Hb 8.4 g/dl, MCV 106 fl, WCC 7.2 109/l, platelets
170 109/l, alkaline phosphatase 534 IU/l, parathyroid hormone 10.4 pmol/l.
Which antiepileptic medication is most likely to cause these problems?
A

Carbamazepine

Phenobarbital

Phenytoin

Primidone

Sodium valproate

Explanation

The answer is Phenytoin


Both phenytoin and phenobarbital are associated with symptoms of fatigue, lethargy,
bone pain, tingling and numbness in the lower limbs; however, phenytoin is the only one
that causes gum hypertrophy. Folate deficiency and megaloblastic anaemia may arise
due to enzyme induction and rapid folate metabolism, and due to phenytoin mediated
inhibition of intestinal conjugase. Osteomalacia and neuropathy are long-term sideeffects of both phenytoin and phenobarbital. Primidone is a pro-drug that is converted in
the liver to phenobarbital.

Carbamazepine (Option A) is incorrect. Adverse effects of carbamazepine include


drowsiness, ataxia, nystagmus, diplopia, rash, and thrombocytopaenia and other blood
dyscrasias.
Phenobarbital (Option B) is incorrect. Phenobarbital may cause many of the presented
symptoms, but gum hypertrophy is not a recognised adverse effect.
Primidone (Option D) is incorrect. Primidone is a pro-drug of phenobarbital and may cause
many of the presented symptoms, but gum hypertrophy is not a recognised adverse effect.
https://mypastest.pastest.com/Secure/TestMe/Browser/429893#Top

1/2

8/13/2016

MyPastest

Sodium valproate (Option E) is incorrect. Sodium valproate is associated with alopecia, rash,
blood dyscrasias, liver damage, pancreatitis and hyperammonaemia.
46738

Next Question

Previous Question

Tag Question

Feedback

End Review

Difficulty: Average
Peer Responses

Session Progress
Responses Correct:

Responses Incorrect:

179

Responses Total:

179

Responses - % Correct:

0%

Blog (https://www.pastest.com/blog) About Pastest (https://www.pastest.com/about-us)


Contact Us (https://www.pastest.com/contact-us) Help (https://www.pastest.com/help)
Pastest 2016

https://mypastest.pastest.com/Secure/TestMe/Browser/429893#Top

2/2

8/13/2016

MyPastest

Back to Filters (/Secure/TestMe/Filter/429893/QA)

Question 139 of 179

A 68-year-old male patient with disseminated carcinoma of the prostate is being treated with
buprenorphine, which until recently has controlled his bone pain well. Other significant
history of note includes chronic renal failure; his creatinine is 205 mol/l. More recently, he
has complained of increasing pain in the hip, but maintains a good functional status and
wants to continue to tend his garden at home.
Which of the following measures would be most appropriate to optimise his pain control?
A

Add amitriptyline to his therapy

Add diclofenac to his therapy

Add morphine elixir to his therapy

Change the buprenorphine to morphine elixir

Substitute sustained-relase morphine tablets for buprenorphine

Explanation

The answer is Change the buprenorphine to morphine elixir


Buprenorphine is a partial agonist at opioid receptors; it will antagonise the action of a
full agonist such as morphine. Therefore, the addition of morphine or codeine to
buprenorphine is illogical. The main options here may be to increase the buprenorphine
dose or to switch to an alternative such as morphine or oxycodone. Titrating the dose of
morphine should be done with short-lived formulations of morphine, and once adequate
analgesia is obtained, then switch to sustained-release morphine.

Add amitriptyline to his therapy (Option A) is incorrect. Amitriptyline is an effective


treatment for neuropathic pain, but much less effective for bony pain. It may be a helpful
adjunct to allow normal sleep patterns.
Add diclofenac to his therapy (Option B) is incorrect. Non-steroidal anti-inflammatory drugs
(NSAIDs) are useful if there is a significant inflammatory component to pain, but would
probably be best avoided here in the presence of such a degree of renal impairment.

https://mypastest.pastest.com/Secure/TestMe/Browser/429893#Top

1/2

8/13/2016

MyPastest

Add morphine elixir to his therapy (Option C) is incorrect. It would be illogical to add
morphine to buprenorphine, because buprenorphine effectively acts as a morphine
antagonist.
Substitute sustained-relase morphine tablets for buprenorphine (Option E) is incorrect. This
may be a reasonable option if the morphine dose can be estimately safely, but this is a less
preferred option than administering short-acting morphine first until the daily requirements
can be ascertained.
46895

Next Question

Previous Question

Tag Question

Feedback

End Review

Difficulty: Average
Peer Responses

Session Progress
Responses Correct:

Responses Incorrect:

179

Responses Total:

179

Responses - % Correct:

0%

Blog (https://www.pastest.com/blog) About Pastest (https://www.pastest.com/about-us)


Contact Us (https://www.pastest.com/contact-us) Help (https://www.pastest.com/help)
Pastest 2016

https://mypastest.pastest.com/Secure/TestMe/Browser/429893#Top

2/2

8/13/2016

MyPastest

Back to Filters (/Secure/TestMe/Filter/429893/QA)

Question 140 of 179

You review a 76-year-old man in the cardiovascular clinic. He has a history of congestive heart
failure, and is receiving a number of different medications. He describes feeling dizzy, and
blood pressure is 98/62 mmHg. In view of his low blood pressure, you wish to stop one of his
existing medications.
Which one of the following drugs if stopped would have the least impact on long-term
survival in this patient?
A

Bisoprolol

Candesartan

Digoxin

Enalapril

Spironolactone

Explanation

The answer is Digoxin


All the listed drugs other than digoxin have been shown to improve mortality rates in
congestive heart failure. Digoxin may be helpful in reducing cardiac symptoms, and may
reduce heart failure deaths, but has no significant effect on cardiovascular and overall
mortality.

Bisoprolol (Option A) is incorrect. Bisoprolol, metoprolol and carvedilol have been shown to
improve survival in heart failure patients.
Candesartan (Option B) is incorrect. Candesartan and other angiotensin receptor antagonists
have been shown to improve survival in heart failure patients.
Enalapril (Option D) is incorrect. Enalapril and other ACE inhibitors have been shown to
improve survival in heart failure patients.
Spironolactone (Option E) is incorrect. Spironolactone has been shown to improve survival in
heart failure patients when added to other conventional medications.
46599

https://mypastest.pastest.com/Secure/TestMe/Browser/429893#Top

1/2

8/13/2016

MyPastest

46599

Next Question

Previous Question

Tag Question

Feedback

End Review

Difficulty: Average
Peer Responses

Session Progress
Responses Correct:

Responses Incorrect:

179

Responses Total:

179

Responses - % Correct:

0%

Blog (https://www.pastest.com/blog) About Pastest (https://www.pastest.com/about-us)


Contact Us (https://www.pastest.com/contact-us) Help (https://www.pastest.com/help)
Pastest 2016

https://mypastest.pastest.com/Secure/TestMe/Browser/429893#Top

2/2

8/13/2016

MyPastest

Back to Filters (/Secure/TestMe/Filter/429893/QA)

Question 141 of 179

An 81-year-old woman is admitted to the Emergency Department with a severe pneumonia,


hypotension and signs of sepsis. There is a history of previous GP attendance for a urinary
tract infection. According to her husband, she has been prescribed a prolonged course of cotrimoxazole, as this has been her third urinary infection in the past 2 months. She has had
diarrhoea and vomiting for the past 24 h. Her total white blood count is 1.8 109/litre, and
neutrophil count is 0.2 109/litre.
What is the most likely explanation for these laboratory findings?
A

Chronic myeloid leukaemia

Neutropaenia secondary to co-trimoxazole therapy

Salmonella infection

Severe pneumonia

Urinary tract infection with sepsis

Explanation

The answer is Neutropaenia secondary to co-trimoxazole therapy


Co-trimoxazole is now only indicated for oral prophylaxis against pneumocystis
pneumonia, toxoplasmosis and nocardiosis due to the risk of neutropaenia. It is effective
against a number of organisms that cause respiratory and urinary infections, but should
normally only be considered where there is no other safer alternative. Adverse effects
include nausea, rash, StephensJohnson syndrome, toxic epidermal necrolysis,
neutropaenia, thrombocytopaenia and photosensitivity. It should be avoided or used with
caution in patients with renal or hepatic impairment.

Chronic myeloid leukaemia (Option A) is incorrect. A hallmark of chronic myeloid leukaemia


is production of high numbers of leukocytes, not neutropaenia, albeit that they are often
dysfunctional and render patients susceptible to infection.
Salmonella infection (Option C) is incorrect. Salmonella infection may be associated with
leukopaenia and neutropaenia, although it would be very rare for the white count to be
https://mypastest.pastest.com/Secure/TestMe/Browser/429893#Top

1/2

8/13/2016

MyPastest

suppressed to the low levels reported here.


Severe pneumonia (Option D) is incorrect. Overwhelming bacterial infection may be
associated with leukopaenia and neutropaenia, although it would be very rare for the white
count to be suppressed to the low levels reported here.
Urinary tract infection with sepsis (Option E) is incorrect. Overwhelming bacterial infection
may be associated with leukopaenia and neutropaenia, although it would be very rare for the
white count to be suppressed to the low levels reported here.
46886

Next Question

Previous Question

Tag Question

Feedback

End Review

Difficulty: Average
Peer Responses

Session Progress
Responses Correct:

Responses Incorrect:

179

Responses Total:

179

Responses - % Correct:

0%

Blog (https://www.pastest.com/blog) About Pastest (https://www.pastest.com/about-us)


Contact Us (https://www.pastest.com/contact-us) Help (https://www.pastest.com/help)
Pastest 2016

https://mypastest.pastest.com/Secure/TestMe/Browser/429893#Top

2/2

8/13/2016

MyPastest

Back to Filters (/Secure/TestMe/Filter/429893/QA)

Question 142 of 179

A 72-year-old man is managed on the elderly care ward where he was admitted with terminal
bronchial carcinoma because his wife was unable to manage at home. He has known cerebral
metastases and is controlled with regular morphine sulphate (MST) and top-up oral
morphine. You are asked to see him by the on-call FY2 who gave him an injection of
haloperidol because the nurses were unable to manage his agitation. On examination his eyes
are deviated to the left and he seems unable to correct this. His tongue is also protruding and
his neck is in spasm, with his head forcibly rotated to the left-hand side.
Which one of the following is the most appropriate treatment?
A

Further oral morphine

Oral diazepam

IV midazolam

Trihexiphenidyl hydrochloride syrup

S/c diamorphine

Explanation
Oculogyric crisis
The history given here is consistent with an oculogyric crisis, the treatment of which is
with an anti-cholinergic such as trihexiphenidyl hydrochloride
It is important to recognise the condition, which is known to occur with anti-psychotics
and anti-emetics, as prompt intervention can lead to rapid resolution of the condition
Other notes

Because of increased risk of acute dystonia, use of metoclopramide and


prochlorperazine is not recommended, particularly in young women
22512

Next Question
https://mypastest.pastest.com/Secure/TestMe/Browser/429893#Top

1/2

8/13/2016

MyPastest

Previous Question

Tag Question

Feedback

End Review

Difficulty: Average
Peer Responses

Session Progress
Responses Correct:

Responses Incorrect:

179

Responses Total:

179

Responses - % Correct:

0%

Blog (https://www.pastest.com/blog) About Pastest (https://www.pastest.com/about-us)


Contact Us (https://www.pastest.com/contact-us) Help (https://www.pastest.com/help)
Pastest 2016

https://mypastest.pastest.com/Secure/TestMe/Browser/429893#Top

2/2

8/13/2016

MyPastest

Back to Filters (/Secure/TestMe/Filter/429893/QA)

Question 143 of 179

A 39-year-old woman underwent excision of a meningioma 9 months ago, and has been
maintained on regular antiepileptic medication since that time. Over the past few months, she
has complained of facial acne, something she has not suffered from previously, and she has
developed painless swelling around her gums.
Which of the following medicaitons is most likely to account for her symptoms?
A

Carbamazepine

Oxcarbazepine

Phenytoin

Sodium valproate

Topiramate

Explanation

The answer is Phenytoin


Patients may be treated with any one of a number of antiepileptic agents after
neurosurgery, including carbamazepine, sodium valproate and lamotrigine. Phenytoin
may be effective, but has many adverse effects associated with long-term use, and is less
preferred; these include gum hypertrophy, folate deficiency, osteomalacia, coarse facial
features, neuropathy and aplastic anaemia (rarely).

Carbamazepine (Option A) is incorrect. Carbamazepine may cause drowsiness, blurred vision


and ataxia.
Oxcarbazepine (Option B) is incorrect. Oxcarbazepine may cause drowsiness, blurred vision
and ataxia.
Sodium valproate (Option D) is incorrect. Sodium valproate may cause drowsiness, mood
changes, alopecia, thrombocytopaenia, nausea and liver impairment.
Topiramate (Option E) is incorrect. Topiramate may cause drowsiness, speech disturbance,
impaired concentration and mood changes.
46617

https://mypastest.pastest.com/Secure/TestMe/Browser/429893#Top

1/2

8/13/2016

MyPastest

46617

Next Question

Previous Question

Tag Question

Feedback

End Review

Difficulty: Average
Peer Responses

Session Progress
Responses Correct:

Responses Incorrect:

179

Responses Total:

179

Responses - % Correct:

0%

Blog (https://www.pastest.com/blog) About Pastest (https://www.pastest.com/about-us)


Contact Us (https://www.pastest.com/contact-us) Help (https://www.pastest.com/help)
Pastest 2016

https://mypastest.pastest.com/Secure/TestMe/Browser/429893#Top

2/2

8/13/2016

MyPastest

Back to Filters (/Secure/TestMe/Filter/429893/QA)

Question 144 of 179

You review a 68-year-old man who attends the Ambulatory Care Unit with a swollen left calf
and undergoes a Doppler scan. This confirms a diagnosis of deep vein thrombosis and you
decide to administer warfarin. The patient is receiving a number of other medications.
Which of the following substances is most likely to reduce the effect of warfarin therapy?
A

Acute alcohol ingestion

Cholestyramine

Cimetidine

Co-trimoxazole

Metronidazole

Explanation

The answer is Cholestyramine


Cholestyramine is a resin used for binding cholesterol in the gut and may be used to
treat familial hypercholesterolaemia. Cholestyramine reduces absorption of a number of
drugs including warfarin. Warfarin has a half-life of around 36 h and is eliminated by
hepatic metabolism; it is contraindicated in patients with severe hepatic disease. Coadministration of enzyme inducing drugs including phenytoin and carbamazepine may
increase the extent of warfarin metabolism, thereby reducing INR.

Acute alcohol ingestion (Option A) is incorrect. Acute ethanol intake transiently inhibits
hepatic enzyme activity, thereby increasing INR and bleeding risk. In contrast, chronic
ethanol intake may upregulate hepatic enzyme activity.
Cimetidine (Option C) is incorrect. Cimetidine is an anti-H2 histamine antagonist that inhibits
hepatic enzyme activity, thereby increasing INR.
Co-trimoxazole (Option D) is incorrect. Co-trimoxazole may increase warfarin action due to a
protein-binding interaction.

https://mypastest.pastest.com/Secure/TestMe/Browser/429893#Top

1/2

8/13/2016

MyPastest

Metronidazole (Option E) is incorrect. Metronidazole is a hepatic enzyme inhibitor that may


increase warfarin activity and increase INR.
46892

Next Question

Previous Question

Tag Question

Feedback

End Review

Difficulty: Average
Peer Responses

Session Progress
Responses Correct:

Responses Incorrect:

179

Responses Total:

179

Responses - % Correct:

0%

Blog (https://www.pastest.com/blog) About Pastest (https://www.pastest.com/about-us)


Contact Us (https://www.pastest.com/contact-us) Help (https://www.pastest.com/help)
Pastest 2016

https://mypastest.pastest.com/Secure/TestMe/Browser/429893#Top

2/2

8/13/2016

MyPastest

Back to Filters (/Secure/TestMe/Filter/429893/QA)

Question 145 of 179

A retired salesman attends the cardiology outpatient clinic for review of his hypertension and
moderate heart failure. He is complaining of a persistent dry cough that developed soon after
his previous clinic appointment 6 months ago.
Which medication is most likely to cause this symptom?
A

Candesartan

Doxazosin

Lisinopril

Methyldopa

Propranolol

Explanation

The answer is Lisinopril


Angiotensin-converting enzyme (ACE) inhibitors such as lisinopril interfere with the
breakdown of bradykinin in the lungs, and high bradykinin concentrations appear
capable of causing a persistent dry cough in 1020% of patients.

Candesartan (Option A) is incorrect. Angiotensin II receptor antagonists (losartan,


candesartan) are useful alternatives as they do not potentiate bradykinin, and cough is
reported in only 12% of patients.
Doxazosin (Option B) is incorrect. Doxazosin typically causes postural hypotension, fluid
retention and heart failure; less commonly it may cause rhinitis.
Methyldopa (Option D) is incorrect. Methyldopa is usually associated with nasal congestion
and a lupus-like syndrome.
Propranolol (Option E) is incorrect. Propranolol is a non-selective 1- and 2-blocker and may
precipitate bronchospasm in asthmatic individuals, or prevent 2-agonist bronchodilatation.
46756

https://mypastest.pastest.com/Secure/TestMe/Browser/429893#Top

1/2

8/13/2016

MyPastest

Next Question

Previous Question

Tag Question

Feedback

End Review

Difficulty: Average
Peer Responses

Session Progress
Responses Correct:

Responses Incorrect:

179

Responses Total:

179

Responses - % Correct:

0%

Blog (https://www.pastest.com/blog) About Pastest (https://www.pastest.com/about-us)


Contact Us (https://www.pastest.com/contact-us) Help (https://www.pastest.com/help)
Pastest 2016

https://mypastest.pastest.com/Secure/TestMe/Browser/429893#Top

2/2

8/13/2016

MyPastest

Back to Filters (/Secure/TestMe/Filter/429893/QA)

Question 146 of 179

A 56-year-old man has been admitted with lower abdominal pain and difficulty urinating. A
bedside bladder scan indicates a residual bladder volume of around 580 ml. He has been
receiving a number of different medications.
Which of the following drugs is most likely to cause urinary retention?
A

Amitriptyline

Diazepam

Fluoxetine

Venlafaxine

Zopiclone

Explanation

The answer is Amitriptyline


Amitriptyline causes a number of effects, including anticholinergic effects. Common
anticholinergic effects include dry mouth, blurred vision, urinary retention, constipation,
palpitations and tachycardia.

Diazepam (Option B) is incorrect. Diazepam does not provoke any anticholinergic effects.
Fluoxetine (Option C) is incorrect. Fluoxetine potentiates the effects of serotonin and has no
direct effect on cholinergic pathways.
Venlafaxine (Option D) is incorrect. Venlafaxine possesses a mixture of enhanced serotonin
and norepinephrine effects, the latter more prominent after high doses.
Zopiclone (Option E) is incorrect. Zopiclone exerts effects similar to benzodiazepines, and
has no known anticholinergic properties.
46937

Next Question

https://mypastest.pastest.com/Secure/TestMe/Browser/429893#Top

1/2

8/13/2016

MyPastest

Previous Question

Tag Question

Feedback

End Review

Difficulty: Average
Peer Responses

Session Progress
Responses Correct:

Responses Incorrect:

179

Responses Total:

179

Responses - % Correct:

0%

Blog (https://www.pastest.com/blog) About Pastest (https://www.pastest.com/about-us)


Contact Us (https://www.pastest.com/contact-us) Help (https://www.pastest.com/help)
Pastest 2016

https://mypastest.pastest.com/Secure/TestMe/Browser/429893#Top

2/2

8/13/2016

MyPastest

Back to Filters (/Secure/TestMe/Filter/429893/QA)

Question 147 of 179

A patient with haematemesis is stabilised and undergoes urgent upper gastrointestinal


endoscopy. He is started on omeprazole, with good response.
Which of the following best explains the mechanism of action of omeprazole that is likely to
promote ulcer healing?
A

Gastric prostaglandin secretion

H2-receptor blockade

Helicobacter pylori eradication

Increase in gastric emptying

Inhibition of H+/K+-ATPase

Explanation

The answer is Inhibition of H+/K+-ATPase


Omeprazole and other members of this class of drugs (proton pump inhibitors (PPIs))
act by inhibiting the hydrogen/potassium adenosine triphosphatase enzyme system
(H+/K+-ATPase or proton pump), which secretes hydrogen ions into the gastric lumen.
They are prodrugs that are taken up into the parietal cells and metabolised into an active
drug that interferes with proton pump activity. Acid secretion is only restored when the
cell synthesises new proton-pump protein, so that the effects last much longer than the
drugs, which are rapidly cleared from the circulation; they are longer acting than H2receptor antagonists. Once-daily dosing is normally sufficient to allow healing of peptic
ulcers and prevent gastro-oesophageal acid reflux.

Gastric prostaglandin secretion (Option A) is incorrect. One of the mechanisms of peptic


ulcer in patients treated with aspirin or non-steroidal anti-inflammatory drugs (NSAIDs) is
altered prostaglandin synthesis. Drugs that specifically target prostaglandin synthesis have
not been proven to improve ulcer healing.
H2-receptor blockade (Option B) is incorrect. Histamine subtype-2 receptor antagonists, eg
ranitidine, are less effective in suppressing gastric acid secretion and not as effective in
https://mypastest.pastest.com/Secure/TestMe/Browser/429893#Top

1/2

8/13/2016

MyPastest

promoting ulcer healing.


Helicobacter pylori eradication (Option C) is incorrect. PPI therapy is only one part of H.
pylori eradication regimens, which require the addition of antibiotics to be effective.
Eradication is valuable in accelerating healing of acute and chronic peptic ulcers in patients
with proven infection.
Increase in gastric emptying (Option D) is incorrect. Certain drugs, particularly
metoclopramide or macrolide antibiotics, may promote gastric emptying but this mechanism
has no direct effect on ulcer healing.
46804

Next Question

Previous Question

Tag Question

Feedback

End Review

Difficulty: Average
Peer Responses

Session Progress
Responses Correct:

Responses Incorrect:

179

Responses Total:

179

Responses - % Correct:

0%

Blog (https://www.pastest.com/blog) About Pastest (https://www.pastest.com/about-us)


Contact Us (https://www.pastest.com/contact-us) Help (https://www.pastest.com/help)
Pastest 2016

https://mypastest.pastest.com/Secure/TestMe/Browser/429893#Top

2/2

8/13/2016

MyPastest

Back to Filters (/Secure/TestMe/Filter/429893/QA)

Question 148 of 179

A 55-year-old man is on long-term phenytoin therapy for control of generalised seizures. He


is admitted to hospital after a further generalised seizure, and the dose of his therapy has
been increased.
Which of the following adverse effects caused by phenytoin is most likely to be provoked by
the increased drug dose?
A

Agitation

Bradycardia

Hemiparesis

Hypotension

Osteoporosis

Explanation

The answer is Agitation


Phenytoin may give rise to dose-dependent central nervous system adverse effects such
as agitation, ataxia, nystagmus and drowsiness. These are generally short-term adverse
effects that resolve after drug cessation or dose reduction. Adverse effects related to
long-term therapy include altered vitamin D metabolism and osteomalacia, selective
immunoglobulin A deficiency, and Dupuytrens contracture. Idiosyncratic adverse effects
include hepatitis.

Bradycardia (Option B) is incorrect. Parenteral administration has been linked to atrial and
ventricular conduction depression and ventricular fibrillation; these are not seen after regular
oral treatment.
Hemiparesis (Option C) is incorrect. Hemiparesis would suggest a focal neurological defect.
Hypotension (Option D) is incorrect. Hypotension does not normally occur after therapeutic
doses of phenytoin.

https://mypastest.pastest.com/Secure/TestMe/Browser/429893#Top

1/2

8/13/2016

MyPastest

Osteoporosis (Option E) is incorrect. This is a recognised adverse effect of long-term


treatment and unlikely to be provoked by a short-term dose increase.
46914

Next Question

Previous Question

Tag Question

Feedback

End Review

Difficulty: Average
Peer Responses

Session Progress
Responses Correct:

Responses Incorrect:

179

Responses Total:

179

Responses - % Correct:

0%

Blog (https://www.pastest.com/blog) About Pastest (https://www.pastest.com/about-us)


Contact Us (https://www.pastest.com/contact-us) Help (https://www.pastest.com/help)
Pastest 2016

https://mypastest.pastest.com/Secure/TestMe/Browser/429893#Top

2/2

8/13/2016

MyPastest

Back to Filters (/Secure/TestMe/Filter/429893/QA)

Question 149 of 179

You are reviewing a 78-year-old woman with leg weakness and impaired mobility. Initial
investigations show significant osteoporosis.
Which one of the following would be most effective in treatment of established
osteoporosis?
A

800 mg/day of calcium, 100 units/day of vitamin D

900 mg/day of calcium, 200 units/day of vitamin D

1200 mg/day of calcium, 200400 units/day of vitamin D

1500 mg/day of calcium, 400800 units/day of vitamin D

1800 mg/day of calcium, 8001000 units/day of vitamin D

Explanation

The answer is option 1500 mg/day of calcium, 400-800 units/day of vitamin D


Adequate calcium and vitamin D intake should be part of the prevention and treatment
of osteoporosis. Recommmended dietary intake of calcium should be:
8001000 mg/day in childhood through early adulthood
10001200 mg/day in middle adulthood
1500 mg/day in older adults
If osteoporosis is established, the treatment includes 1500 mg/day of calcium and 400
800 units/day of vitamin D (1000 units is equivalent to 25 micrograms).

800 mg/day of calcium, 100 units/day of vitamin D (Option A) is incorrect. Optimal is 1500
mg/day of calcium, 400800 units/day of vitamin D.
900 mg/day of calcium, 200 units/day of vitamin D (Option B) is incorrect. Optimal is 1500
mg/day of calcium, 400800 units/day of vitamin D.

https://mypastest.pastest.com/Secure/TestMe/Browser/429893#Top

1/2

8/13/2016

MyPastest

1200 mg/day of calcium, 200400 units/day of vitamin D (Option C) is incorrect. Optimal is


1500 mg/day of calcium, 400800 units/day of vitamin D.
1800 mg/day of calcium, 8001000 units/day of vitamin D (Option E) is incorrect. Optimal is
1500 mg/day of calcium, 400800 units/day of vitamin D.
46734

Next Question

Previous Question

Tag Question

Feedback

End Review

Difficulty: Average
Peer Responses

Session Progress
Responses Correct:

Responses Incorrect:

179

Responses Total:

179

Responses - % Correct:

0%

Blog (https://www.pastest.com/blog) About Pastest (https://www.pastest.com/about-us)


Contact Us (https://www.pastest.com/contact-us) Help (https://www.pastest.com/help)
Pastest 2016

https://mypastest.pastest.com/Secure/TestMe/Browser/429893#Top

2/2

8/13/2016

MyPastest

Back to Filters (/Secure/TestMe/Filter/429893/QA)

Question 150 of 179

A 65-year-old woman has been receiving chemotherapy treatment and asks you what you
know about the possible adverse effects of bleomycin.
What is the most likely serious adverse effect of bleomycin that she is at increased risk of
developing?
A

Acute kidney injury

Cardiomyopathy

Neuropathy

Pneumonitis

Retinopathy

Explanation

The answer is Pneumonitis


Bleomycin causes lung toxicity, including pneumonitis, which can progress to interstitial
fibrosis and diminished gas transfer. The risks increase with cumulative bleomycin doses.

Acute kidney injury (Option A) is incorrect. A number of chemotherapy agents may provoke
tumour lysis syndrome and acute kidney injury, particularly in treatment of leukaemia and
other haematological malignancies. Bleomycin is not particularly associated with this.
Methotrexate in high doses can cause acute renal failure due to precipitation of the drug in
the renal tubules.
Cardiomyopathy (Option B) is incorrect. Doxorubicin can cause cardiac toxicity and
cardiomyopathy.
Neuropathy (Option C) is incorrect. Cisplatin characteristically causes peripheral motor and
sensory neuropathy; cytarabine administered in high single doses can cause irreversible
cerebellar damage.
Retinopathy (Option E) is incorrect. Ocular toxicity may characteristically occur after
platinum-based agents and pyrimidine analogues.
46793

https://mypastest.pastest.com/Secure/TestMe/Browser/429893#Top

1/2

8/13/2016

MyPastest

46793

Next Question

Previous Question

Tag Question

Feedback

End Review

Difficulty: Average
Peer Responses

Session Progress
Responses Correct:

Responses Incorrect:

179

Responses Total:

179

Responses - % Correct:

0%

Blog (https://www.pastest.com/blog) About Pastest (https://www.pastest.com/about-us)


Contact Us (https://www.pastest.com/contact-us) Help (https://www.pastest.com/help)
Pastest 2016

https://mypastest.pastest.com/Secure/TestMe/Browser/429893#Top

2/2

8/13/2016

MyPastest

Back to Filters (/Secure/TestMe/Filter/429893/QA)

Question 151 of 179

You are reviewing a patient in the outpatient department who has recently been diagnosed
with tuberculosis and has been started on combination therapy with isoniazid, rifampicin,
pyrazinamide and ethambutol. Your consultant asks you to arrange for tests to determine
acetylator status.
Which adverse effects would be expected to be more likely to occur if the patient was found
to be a slow acetylator?
A

Cranial nerve palsy

Hepatitis

Peripheral neuropathy

Renal toxicity

Visual disturbances

Explanation

The answer is Peripheral neuropathy


The patterns of drug efficacy and adverse effects may be predicted by genetic variation.
For example, slow acetylators of isoniazid are more likely to be exposed to high
concentrations of isoniazid and develop adverse effects including peripheral neuropathy,
which is a direct drug adverse effect. Rapid acetylators are associated with lower drug
concentrations and a greater risk of treatment failure; rapid acetylators are more prone
to isoniazid-induced hepatitis, which is not caused by isoniazid but one of its
metabolites.

Cranial nerve palsy (Option A) is incorrect. Slow acetylator status increases the likelihood of
isoniazid-induced neuropathy, but this more commonly involves a peripheral sensorimotor
neuropathy.
Hepatitis (Option B) is incorrect. Isoniazid-induced hepatitis is less likely to occur in slow
acetylators, and more likely in rapid acetylators because it is mediated by an isoniazid
metabolite.
https://mypastest.pastest.com/Secure/TestMe/Browser/429893#Top

1/2

8/13/2016

MyPastest

Renal toxicity (Option D) is incorrect. The risk of renal toxicity is not influenced acetylator
status.
Visual disturbances (Option E) is incorrect. Visual disturbance may represent a serious
adverse effect of ethambutol, but this is not influenced by acetylator status.
46790

Next Question

Previous Question

Tag Question

Feedback

End Review

Difficulty: Average
Peer Responses

Session Progress
Responses Correct:

Responses Incorrect:

179

Responses Total:

179

Responses - % Correct:

0%

Blog (https://www.pastest.com/blog) About Pastest (https://www.pastest.com/about-us)


Contact Us (https://www.pastest.com/contact-us) Help (https://www.pastest.com/help)
Pastest 2016

https://mypastest.pastest.com/Secure/TestMe/Browser/429893#Top

2/2

8/13/2016

MyPastest

Back to Filters (/Secure/TestMe/Filter/429893/QA)

Question 152 of 179

You review a 72-year-old woman who has suffered from agitation and extreme nausea after
previous general anaesthetics. You are on call for the surgical wards and note that the
anaesthetists have prescribed haloperidol for nausea.
Which of the following best describes the main pharmacological site of antiemetic action of
haloperidol?
A

Adrenergic receptors

Chemoreceptor trigger zone

Cholinergic receptors

It is a prokinetic agent

Vestibular system

Explanation

The answer is Chemoreceptor trigger zone


Haloperidol is an anti-dopaminergic that is used mainly as an antipsychotic; it causes
more extrapyramidal adverse effects than phenothiazines, but a lower risk of
hypotension. Its main site of antiemetic action is the chemoreceptor trigger zone. Other
drugs with antiemetic properties through an action on the chemoreceptor trigger zone
include phenothiazines (e.g. promethazine) and domperidone. Newer agents, such as the
5-hydroxytryptamine (5-HT3) inhibitors (e.g. ondansetron), are associated with fewer
side-effects, and act directly on 5-HT receptors in the gut and central nervous system.

Adrenergic receptors (Option A) is incorrect. Adrenergic pathways are not a particularly


important target for control of nausea.
Cholinergic receptors (Option C) is incorrect. Haloperidol exerts little if any cholinergic
effects. Cyclizine exerts its antiemetic effects by blocking cholinergic pathways.
It is a prokinetic agent (Option D) is incorrect. Metoclopramide has prokinetic actions and
may be more useful in nausea and vomiting related to gastrointestinal causes.
https://mypastest.pastest.com/Secure/TestMe/Browser/429893#Top

1/2

8/13/2016

MyPastest

Vestibular system (Option E) is incorrect. Vestibular suppressants include low-dose


benzodiazepines, anticholinergic agents (eg cyclizine, hyoscine).
46917

Next Question

Previous Question

Tag Question

Feedback

End Review

Difficulty: Average
Peer Responses

Session Progress
Responses Correct:

Responses Incorrect:

179

Responses Total:

179

Responses - % Correct:

0%

Blog (https://www.pastest.com/blog) About Pastest (https://www.pastest.com/about-us)


Contact Us (https://www.pastest.com/contact-us) Help (https://www.pastest.com/help)
Pastest 2016

https://mypastest.pastest.com/Secure/TestMe/Browser/429893#Top

2/2

8/13/2016

MyPastest

Back to Filters (/Secure/TestMe/Filter/429893/QA)

Question 153 of 179

You are reviewing a phase 2 trial of a new agent for treating diabetes. You are aware from
some phase 1 studies that the half-life of the agent is around 4 h. When deciding what dosing
schedule would be appropriate in the next phase 2 study, you consider whether this drug
should be administered once, twice or thrice daily.
Given the half-life of this agent, approximately what percentage of the drug will have been
eliminated after 20 h?
A

3%

50%

75%

87.5%

97%

Explanation

The answer is 97% The concentration of the drug reduces by 50% over each half-life. Therefore, after 4 h
50%, after 8 h 25%, after 12 h 12.5%, after 16 h 6.25%, and after 20 h 3.125%. That is to say,
at 20 h, the drug concentration will have fallen by 97%.

3% (Option A) is incorrect. Concentration will have fallen by 97%, so that it would be at


around 3% of original level at five half-lives, 20 h.
50% (Option B) is incorrect. Concentration will have fallen by 50% at one half-life, 4 h.
75% (Option C) is incorrect. Concentration will have fallen by 75% after two half-lives, 8 h.
87.5% (Option D) is incorrect. Concentration will have fallen by 87.5% after three half-lives, 12
h.
46983

Next Question
https://mypastest.pastest.com/Secure/TestMe/Browser/429893#Top

1/2

8/13/2016

MyPastest

Previous Question

Tag Question

Feedback

End Review

Difficulty: Average
Peer Responses

Session Progress
Responses Correct:

Responses Incorrect:

179

Responses Total:

179

Responses - % Correct:

0%

Blog (https://www.pastest.com/blog) About Pastest (https://www.pastest.com/about-us)


Contact Us (https://www.pastest.com/contact-us) Help (https://www.pastest.com/help)
Pastest 2016

https://mypastest.pastest.com/Secure/TestMe/Browser/429893#Top

2/2

8/13/2016

MyPastest

Back to Filters (/Secure/TestMe/Filter/429893/QA)

Question 154 of 179

You have investigated a 23-year-old woman with morning headaches and made a diagnosis
of benign intracranial hypertension. You have prescribed a course of acetazolamide
treatment.
Which of the following adverse effects would you be most likely to anticipate?
A

Acute interstitial nephritis

Hyperkalaemia

Macrocytic hypochromic anaemia

Membranous glomerulonephropathy

Metabolic alkalosis

Explanation

The answer is Acute interstitial nephritis


Acetazolamide is an inhibitor of carbonic anhydrase. It may be used to treat patients with
hydrocephalus after intracerebral haemorrhage, to reduce intracranial pressure in benign
intracranial hypertension, and to reduce intraocular pressure in glaucoma. It causes
metabolic acidosis, due to bicarbonate loss in the proximal and distal tubules through
inhibition of reabsorption, and hypokalaemia. Recognised adverse effects include acute
interstitial nephritis (AIN), agranulocytosis and thrombocytopaenia.

Hyperkalaemia (Option B) is incorrect. Hypokalaemia is recognised rather than


hyperkalaemia.
Macrocytic hypochromic anaemia (Option C) is incorrect. Macrocytic hypochromic anaemia
is not a recognised feature.
Membranous glomerulonephropathy (Option D) is incorrect. Membranous
glomerulonephropathy is not a recognised complication. Acetazolamide has been explored
as a means of preventing contrast-induced nephropathy, although there is insufficient
evidence of efficacy.
https://mypastest.pastest.com/Secure/TestMe/Browser/429893#Top

1/2

8/13/2016

MyPastest

Metabolic alkalosis (Option E) is incorrect. Metabolic acidosis is a characteristic feature.


46605

Next Question

Previous Question

Tag Question

Feedback

End Review

Difficulty: Average
Peer Responses

Session Progress
Responses Correct:

Responses Incorrect:

179

Responses Total:

179

Responses - % Correct:

0%

Blog (https://www.pastest.com/blog) About Pastest (https://www.pastest.com/about-us)


Contact Us (https://www.pastest.com/contact-us) Help (https://www.pastest.com/help)
Pastest 2016

https://mypastest.pastest.com/Secure/TestMe/Browser/429893#Top

2/2

8/13/2016

MyPastest

Back to Filters (/Secure/TestMe/Filter/429893/QA)

Question 155 of 179

A 19-year-old woman is admitted via the Emergency Department in the early hours of the
morning. She presented due to nausea, vomiting and dizziness. Her friend states that she had
taken some ecstasy tablets (MDMA) while out clubbing a few hours earlier.
Which of the following provides the best explanation for the mechanism of toxicity of
ecstasy (MDMA)?
A

Alpha-adrenoceptor stimulation within the central nervous system

Cerebellar hypoxia causing nystagmus

Inappropriate ADH release may cause hyponatraemia

Stimulation of the parasympathetic nervous system

Suppression of serotonergic neurotransmitter activity

Explanation

The answer is Inappropriate ADH release may cause hyponatraemia


Ecstasy is an amphetamine derivative (3,4-methylenedioxymethamphetamine (MDMA)).
Its principal effects are stimulation of release of serotonin within the central nervous
system and activation of the sympathetic nervous system. Onset is usually within 1 hour
and effects typically last 46 hours, although toxic effects may persist for up to several
days after ingestion. SIADH and hyponatraemia are well characterised features, and
hyponatraemia may be made worse by excessive sweating and fluid intake. Adverse
effects include arrhythmia (the main mechanism of MDMA fatalaties), seizures,
hypertension and hyponatraemia. Treatment includes benzodiazepines to reduce
agitation and minimise seizure risk, active cooling and supportive care.

Alpha-adrenoceptor stimulation within the central nervous system (Option A) is incorrect.


This is not a predominant feature of MDMA; the clinical features are more typically related to
serotonin and norepinephrine activity.

https://mypastest.pastest.com/Secure/TestMe/Browser/429893#Top

1/2

8/13/2016

MyPastest

Cerebellar hypoxia causing nystagmus (Option B) is incorrect. This is not typical in MDMA
toxicity. Cerebellar toxicity is a characteristic adverse effect of methoxetamine toxicity, also
seen after ingestion of ethanol, carbamazepine or phenytoin.
Stimulation of the parasympathetic nervous system (Option D) is incorrect. MDMA
predominantly activates the sympathetic nervous system.
Suppression of serotonergic neurotransmitter activity (Option E) is incorrect. One of the key
mechanisms of MDMA is activation of serotonergic neurotransmission within the central
nervous system.
45793

Next Question

Previous Question

Tag Question

Feedback

End Review

Difficulty: Average
Peer Responses

Session Progress
Responses Correct:

Responses Incorrect:

179

Responses Total:

179

Responses - % Correct:

0%

Blog (https://www.pastest.com/blog) About Pastest (https://www.pastest.com/about-us)


Contact Us (https://www.pastest.com/contact-us) Help (https://www.pastest.com/help)
Pastest 2016

https://mypastest.pastest.com/Secure/TestMe/Browser/429893#Top

2/2

8/13/2016

MyPastest

Back to Filters (/Secure/TestMe/Filter/429893/QA)

Question 156 of 179

A 67-year-old man presents with weakness and muscle aches. He has a history of
hypertension and dyslipidaemia and is managed with ramipril and simvastatin. He also has
chronic obstructive pulmonary disease and is treated with a high-dose seretide inhaler. You
understand he was started by his GP on antibiotics a few days earlier for a lower respiratory
tract infection.
Investigations:
Hb

12.1 g/dl

White cell count 9.4 109/l


Platelets

272 109/l

Na+

141 mmol/l

K+

5.9 mmol/l

Creatinine

190 mol/l

CK

890 U/l (24195)

Which one of the following is the antibiotic he is most likely to have been prescribed?
A

Doxycycline

Ciprofloxacin

Amoxicillin

Co-amoxiclav

Clarithromycin

Explanation
Drug interactions
Simvastatin is metabolised by CYP3A4
https://mypastest.pastest.com/Secure/TestMe/Browser/429893#Top

1/2

8/13/2016

MyPastest

Antibiotics in the macrolide class, including clarithromycin and azithromycin, are potent
inhibitors of CYP3
This leads to simvastatin accumulation and possible rhabdomyolysis
The picture seen here, with raised potassium, creatinine and creatine kinase, fits with
that picture
Because of this interaction, caution is recommended when considering macrolides in
conjunction with simvastatin at higher doses, and another antibiotic should be used if
possible
21362

Next Question

Previous Question

Tag Question

Feedback

End Review

Difficulty: Average
Peer Responses

Session Progress
Responses Correct:

Responses Incorrect:

179

Responses Total:

179

Responses - % Correct:

0%

Blog (https://www.pastest.com/blog) About Pastest (https://www.pastest.com/about-us)


Contact Us (https://www.pastest.com/contact-us) Help (https://www.pastest.com/help)
Pastest 2016

https://mypastest.pastest.com/Secure/TestMe/Browser/429893#Top

2/2

8/13/2016

MyPastest

Back to Filters (/Secure/TestMe/Filter/429893/QA)

Question 157 of 179

A 39-year-old woman with a history of bipolar disorder visits her GP for review. She is
currently treated with lithium therapy. The GP has been monitoring her blood pressure for the
last few months. It is 155/105 mmHg in the clinic and the GP contacts you for advice
regarding the choice of antihypertensive agents available.
Which one of the following statements is most accurate concerning the pharmacological
interaction between antihypertensive agents and lithium?
A

Acetazolamide decreases lithium concentrations

Angiotensin converting enzyme (ACE) inhibitors lead to decreased lithium


concentration

Bendroflumethiazide decreases lithium concentration

Calcium channel blockers lessen the risk of lithium neurotoxicity

Methyldopa leads to decreased neurotoxicity

Explanation

The answer is Acetazolamide decreases lithium concentrations


Osmotic diuretics and carbonic anhydrase inhibitors such as acetazolamide lead to
increased renal excretion of sodium and lithium; lithium concentrations would fall.

Angiotensin converting enzyme (ACE) inhibitors lead to decreased lithium concentration


(Option B) is incorrect. ACE inhibitors lead to increased lithium concentration because of
decreased excretion.
Bendroflumethiazide decreases lithium concentration (Option C) is incorrect. Use of thiazide
diuretics may result in paradoxical increased proximal tubular sodium and water retention,
and increased lithium reabsorption. Thiazides may cause lithium intoxication.
Calcium channel blockers lessen the risk of lithium neurotoxicity (Option D) is incorrect.
Calcium channel blockers combined with lithium may cause a syndrome of ataxia, confusion
and sleepiness, which is reversible on stopping the drug.
https://mypastest.pastest.com/Secure/TestMe/Browser/429893#Top

1/2

8/13/2016

MyPastest

Methyldopa leads to decreased neurotoxicity (Option E) is incorrect. Methyldopa also leads


to increased risk of neurotoxicity.
46955

Next Question

Previous Question

Tag Question

Feedback

End Review

Difficulty: Average
Peer Responses

Session Progress
Responses Correct:

Responses Incorrect:

179

Responses Total:

179

Responses - % Correct:

0%

Blog (https://www.pastest.com/blog) About Pastest (https://www.pastest.com/about-us)


Contact Us (https://www.pastest.com/contact-us) Help (https://www.pastest.com/help)
Pastest 2016

https://mypastest.pastest.com/Secure/TestMe/Browser/429893#Top

2/2

8/13/2016

MyPastest

Back to Filters (/Secure/TestMe/Filter/429893/QA)

Question 158 of 179

A 56-year-old man is undergoing acute intervention in the cardiac catheterisation laboratory


for unstable angina. The cardiologist decides to employ a glycoprotein IIb/IIIa inhibitor.
Which of the following compounds would be most suitable?
A

Abciximab

Clopidogrel

Eptifibatide plus abciximab

Ticlopidine

Tirofiban

Explanation

The answer is Abciximab


Tirofiban, eptifibatide and abciximab are all glycoprotein IIb/IIIa inhibitors, but only
eptifibatide and abciximab are licensed as adjunctive therapy in acute coronary
intervention. Glycoprotein IIb/IIIa inhibitors act by blocking the final common pathway
involved in platelet aggregation; the main side-effects include bleeding and
thrombocytopaenia. Thrombocytopaenia may be particularly severe after abciximab,
which contains murine antibody components.

Clopidogrel (Option B) is incorrect. Clopidogrel is an adjunctive therapy in conjunction with


heparin and aspirin to prevent early myocardial infarction in patients with unstable angina.
Eptifibatide plus abciximab (Option C) is incorrect. Eptifibatide or abciximab may be used
but not in combination.
Ticlopidine (Option D) is incorrect. Ticlopidine is licenced on a named patient basis for use in
a coronary care unit, and is an adjunctive therapy in conjunction with heparin and aspirin to
prevent early myocardial infarction in patients with unstable angina.
Tirofiban (Option E) is incorrect. Tirofiban is used in patients with non-ST elevation
myocardial infarction, but is not licenced for treatment in acute coronary intervention.
46849

https://mypastest.pastest.com/Secure/TestMe/Browser/429893#Top

1/2

8/13/2016

MyPastest

46849

Next Question

Previous Question

Tag Question

Feedback

End Review

Difficulty: Average
Peer Responses

Session Progress
Responses Correct:

Responses Incorrect:

179

Responses Total:

179

Responses - % Correct:

0%

Blog (https://www.pastest.com/blog) About Pastest (https://www.pastest.com/about-us)


Contact Us (https://www.pastest.com/contact-us) Help (https://www.pastest.com/help)
Pastest 2016

https://mypastest.pastest.com/Secure/TestMe/Browser/429893#Top

2/2

8/13/2016

MyPastest

Back to Filters (/Secure/TestMe/Filter/429893/QA)

Question 159 of 179

A 50-year-old truck driver with type-2 diabetes for the past 5 years, BMI 35 kg/m2 and HbA1c
of 62 mmol/mol (7.8%), is taking 160 mg gliclazide twice a day and metformin 2 g daily.
Which drug would be the most suitable additional therapy in this patient?
A

Acarbose

A mixture of short- and medium-acting insulins

Pioglitazone

Repaglinide

Sitagliptin

Explanation

The answer is Sitagliptin


There are three key options here: commence insulin, a dipeptidyl peptidase IV (DPPIV)
inhibitor, or glitazone such as pioglitazone. Addition of insulin treatment would
necessitate rescinding his HGV licence. The HbA1c is close to target, therefore a glitazone
or DPPIV inhibitor would be preferred. Sitagliptin would be the best option because it is
associated with weight maintenance or slight loss, whereas pioglitazone would promote
weight gain. Sitagliptin belongs to the DPPIV inhibitor class, leading to increased plasma
levels of the naturally occurring incretin hormone, GLP-1, by preventing its breakdown.

Acarbose (Option A) is incorrect. Acarbose reduces intestinal absorption of starch, but may
be associated with significant GI upset including diarrhoea and flatulence.
A mixture of short- and medium-acting insulins (Option B) is incorrect. Addition of insulin
would be an effective means of achieving glycaemic control, but this would mean the patient
would lose his HGV licence and livelihood.
Pioglitazone (Option C) is incorrect. Pioglitazone belongs to the thiazolidinedione group of
drugs, which decrease insulin resistance by decreasing leptin expression and increasing
p85-P13K gene expression (a gene that allows insulin to work). The reason that this is a less
preferred option to sitagliptin is because pioglitazone may cause weight gain. Pioglitazone
https://mypastest.pastest.com/Secure/TestMe/Browser/429893#Top

1/2

8/13/2016

MyPastest

leads to weight gain, increased risk of fluid retention, and is possibly associated with bladder
carcinoma, although it is effective at controlling blood glucose.
Repaglinide (Option D) is incorrect. Although repaglinide is shorter acting compared with a
traditional SU, it still has at least some potential to cause hypoglycaemia, and is unlikely to
offer significant advantages over and above the existing regimen.
46754

Next Question

Previous Question

Tag Question

Feedback

End Review

Difficulty: Average
Peer Responses

Session Progress
Responses Correct:

Responses Incorrect:

179

Responses Total:

179

Responses - % Correct:

0%

Blog (https://www.pastest.com/blog) About Pastest (https://www.pastest.com/about-us)


Contact Us (https://www.pastest.com/contact-us) Help (https://www.pastest.com/help)
Pastest 2016

https://mypastest.pastest.com/Secure/TestMe/Browser/429893#Top

2/2

8/13/2016

MyPastest

Back to Filters (/Secure/TestMe/Filter/429893/QA)

Question 160 of 179

Which one of the following antihypertensive agents exerts its pharmacological effects
predominantly through blockade of peripheral 1-adrenoceptors?
A

Clonidine

Doxazosin

Losartan

Methyldopa

Minoxidil

Explanation

The answer is Doxazosin


Prazosin, terazosin and doxazosin are peripheral postsynaptic 1-adrenergic blockers that
act on veins and arterioles.

Clonidine (Option A) is incorrect. Clonidine is a centrally acting agent that reduces


sympathetic outflow.
Losartan (Option C) is incorrect. Losartan blocks angiotensin II receptors and therefore
interferes with the reninangiotensin system, perhaps more completely than the angiotensin
converting enzyme inhibitors. Angiotensin II receptor blockers are less likely to provoke
cough than ace inhibitors, presumably because they do not block the degradation of
bradykinin.
Methyldopa (Option D) is incorrect. Methyldopa, clonidine, guanabenz and guanfacine
reduce sympathetic nervous activity by stimulating presynaptic 2-adrenergic receptors in
the brainstem. Methyldopa acts primarily on the brainstem vasomotor centre causing the
release of -methylnoradrenaline, which enhances central nervous system 2-adrenoceptor
activity (this inhibits sympathetic outflow). Methyldopa reduces peripheral vascular
resistance. It causes less peripheral adrenergic blockade and therefore less postural
hypotension.

https://mypastest.pastest.com/Secure/TestMe/Browser/429893#Top

1/2

8/13/2016

MyPastest

Minoxidil (Option E) is incorrect. Minoxidil causes direct relaxation of vascular smooth muscle
and affects mainly arterial resistance as opposed to venous capacitance vessels, as
evidenced by lack of postural effects. Minoxidil may produce significant hypertrichosis and
fluid retention and may provoke congestive heart failure.
46726

Next Question

Previous Question

Tag Question

Feedback

End Review

Difficulty: Average
Peer Responses

Session Progress
Responses Correct:

Responses Incorrect:

179

Responses Total:

179

Responses - % Correct:

0%

Blog (https://www.pastest.com/blog) About Pastest (https://www.pastest.com/about-us)


Contact Us (https://www.pastest.com/contact-us) Help (https://www.pastest.com/help)
Pastest 2016

https://mypastest.pastest.com/Secure/TestMe/Browser/429893#Top

2/2

8/13/2016

MyPastest

Back to Filters (/Secure/TestMe/Filter/429893/QA)

Question 161 of 179

A 40-year-old man with polycystic kidney disease is under follow-up by the Renal Clinic for
progressively declining renal function. His general practitioner has asked for some advice
regarding drug prescribing, given his renal failure.
Which of the following is the single most important factor that determines the effect of renal
function on drug handling?
A

Age

Blood urea levels

Body weight

Glomerular filtration rate

Molecular weight of the drug

Explanation

The answer is Glomerular filtration rate


In renal failure, there is alteration of drug absorption, distribution, protein binding,
metabolism and pharmacodynamic effects. Renal elimination is the most fundamentally
altered pharmacokinetic factor, and the extent of drug clearance normally depends more
on glomerular filtration rate (GFR) than other aspects of renal elimination; tubular
secretion or reabsorption are more difficult to measure in clinical practice.

Age (Option A) is incorrect. Age is one factor that contributes to GFR.


Blood urea levels (Option B) is incorrect. These provide a less reliable measure of drug
elimination than GFR.
Body weight (Option C) is incorrect. Body weight provides a poor estimate of renal function
and GFR.
Molecular weight of the drug (Option E) is incorrect. This will have some impact on whether
the drug is subject to renal or hepatic elimination, but does not provide a measure of the
effectiveness of elimination.
46905

https://mypastest.pastest.com/Secure/TestMe/Browser/429893#Top

1/2

8/13/2016

MyPastest

46905

Next Question

Previous Question

Tag Question

Feedback

End Review

Difficulty: Average
Peer Responses

Session Progress
Responses Correct:

Responses Incorrect:

179

Responses Total:

179

Responses - % Correct:

0%

Blog (https://www.pastest.com/blog) About Pastest (https://www.pastest.com/about-us)


Contact Us (https://www.pastest.com/contact-us) Help (https://www.pastest.com/help)
Pastest 2016

https://mypastest.pastest.com/Secure/TestMe/Browser/429893#Top

2/2

8/13/2016

MyPastest

Back to Filters (/Secure/TestMe/Filter/429893/QA)

Question 162 of 179

A 68-year-old man with known ischaemic heart disease, on regular atenolol and aspirin as
well as glyceryl trinitrate therapy as required, is admitted to hospital with unstable angina.
Regular oral clopidogrel and nifedipine are added, and he is placed on regular subcutaneous
enoxaparin and an intravenous infusion of glyceryl trinitrate. Three days later, he is noted to
be jaundiced.
What is the most likely explanation?
A

Clopidogrel therapy

Enoxaparin therapy

He has co-existent viral hepatitis

High-dose glyceryl trinitrate therapy

Nifedipine therapy

Explanation

The answer is Nifedipine therapy


Nifedipine causes jaundice due to cholestasis or, less commonly, drug-induced hepatitis.

Clopidogrel therapy (Option A) is incorrect. Clopidogrel is not expected to cause liver


disease.
Enoxaparin therapy (Option B) is incorrect. Enoxaparin may cause bruising and increased
bleeding risk, but is not normally associated with jaundice.
He has co-existent viral hepatitis (Option C) is incorrect. Viral hepatitis is possible, but fails to
take account of all the information provided.
High-dose glyceryl trinitrate therapy (Option D) is incorrect. Glyceryl trinitrate does not
cause jaundice.
46906

Next Question
https://mypastest.pastest.com/Secure/TestMe/Browser/429893#Top

1/2

8/13/2016

MyPastest

Previous Question

Tag Question

Feedback

End Review

Difficulty: Average
Peer Responses

Session Progress
Responses Correct:

Responses Incorrect:

179

Responses Total:

179

Responses - % Correct:

0%

Blog (https://www.pastest.com/blog) About Pastest (https://www.pastest.com/about-us)


Contact Us (https://www.pastest.com/contact-us) Help (https://www.pastest.com/help)
Pastest 2016

https://mypastest.pastest.com/Secure/TestMe/Browser/429893#Top

2/2

8/13/2016

MyPastest

Back to Filters (/Secure/TestMe/Filter/429893/QA)

Question 163 of 179

A 30-year-old sales executive is admitted for an operative procedure requiring general


anaesthesia. He regularly drinks red wine, equivalent to 60 units of alcohol per week. You are
considering treatments to prevent the occurrence of alcohol withdrawal symptoms
postoperatively.
Which drug would be most appropriate in preventing and alleviating this problem?
A

Chlordiazepoxide

Chlorpromazine

Clomethiazole

Lorazepam

Temazepam

Explanation

The answer is Chlordiazepoxide


Diazepam and chlordiazepoxide are routinely used to prevent seizures in recently
abstinent chronic alcoholics and, perhaps less importantly, acute withdrawal symptoms.

Chlorpromazine (Option B) is incorrect. Phenothiazines (chlorpromazine) may be a helpful


adjunctive therapy in some patients, but have been associated with significant respiratory
depression.
Clomethiazole (Option C) is incorrect. Clomethiazole readily causes addiction and has been
associated with respiratory depression; it is rarely used now.
Lorazepam (Option D) is incorrect. Lorazepam may be effective when urgent, intravenous
administration is required. However, it is comparatively short acting and less preferred as oral
therapy compared to chlordiazepoxide or diazepam.
Temazepam (Option E) is incorrect. Temazepam may be effective, but it has a shorter
duration of action so that repeated dose administration is needed.
46755

https://mypastest.pastest.com/Secure/TestMe/Browser/429893#Top

1/2

8/13/2016

MyPastest

Next Question

Previous Question

Tag Question

Feedback

End Review

Difficulty: Average
Peer Responses

Session Progress
Responses Correct:

Responses Incorrect:

179

Responses Total:

179

Responses - % Correct:

0%

Blog (https://www.pastest.com/blog) About Pastest (https://www.pastest.com/about-us)


Contact Us (https://www.pastest.com/contact-us) Help (https://www.pastest.com/help)
Pastest 2016

https://mypastest.pastest.com/Secure/TestMe/Browser/429893#Top

2/2

8/13/2016

MyPastest

Back to Filters (/Secure/TestMe/Filter/429893/QA)

Question 164 of 179

A 73-year-old man presents to the Emergency Department with a 2-week history of diarrhoea
and reduced oral fluid intake. He has been taking digoxin for atrial fibrillation for several
years, and is found to have a serum digoxin concentration more than double the local
therapeutic reference range.
Which of the following statements best describes the management of digoxin poisoning?
A

Bradycardia with hypotension is an indication for initial use of digoxin-specific


antibodies

Calcium gluconate should be avoided in patients with hyperkalaemia

Digoxin-specific antibodies may cause serum sickness

Furosemide is often helpful in minimising toxicity

Oral activated charcoal should be avoided

Explanation

The answer is Digoxin-specific antibodies may cause serum sickness


Digoxin-specific antibodies are indicated for severe digoxin toxicity, namely where there
is severe hyperkalaemia that has failed to respond to fluid resuscitation and
administration of insulin and dextrose. They are also indicated in patients with significant
arrhythmias and hypotension, most commonly bradycardia that fails to respond to
atropine or other measures. Digoxin-specific antibodies offer temporary reversal of
toxicity, but ultimately patients require renal excretion of digoxin and, if there is poor
urine output and renal impairment, then haemodialysis may be required. Once
administered, it is not possible to reliably measure circulating digoxin concentrations.
Serum sickness is an uncommon but recognised and potentially life-threatening reaction.

Bradycardia with hypotension is an indication for initial use of digoxin-specific antibodies


(Option A) is incorrect. The initial approach would be to administer fluids and correct
electrolyte disturbances.

https://mypastest.pastest.com/Secure/TestMe/Browser/429893#Top

1/2

8/13/2016

MyPastest

Calcium gluconate should be avoided in patients with hyperkalaemia (Option B) is incorrect.


Correction of calcium or magnesium may be required to minimise risk of arrhythmia.
Furosemide is often helpful in minimising toxicity (Option D) is incorrect. Furosemide should
not routinely be administered.
Oral activated charcoal should be avoided (Option E) is incorrect. Charcoal may be helpful if
given sufficiently early after acute digoxin ingestion. Repeated oral charcoal administration
may enhance digoxin clearance, although patients with severe toxicity are often vomiting, so
charcoal may not be administered.
45808

Next Question

Previous Question

Tag Question

Feedback

End Review

Difficulty: Average
Peer Responses

Session Progress
Responses Correct:

Responses Incorrect:

179

Responses Total:

179

Responses - % Correct:

0%

Blog (https://www.pastest.com/blog) About Pastest (https://www.pastest.com/about-us)


Contact Us (https://www.pastest.com/contact-us) Help (https://www.pastest.com/help)
Pastest 2016

https://mypastest.pastest.com/Secure/TestMe/Browser/429893#Top

2/2

8/13/2016

MyPastest

Back to Filters (/Secure/TestMe/Filter/429893/QA)

Question 165 of 179

A 72-year-old woman with metastatic carcinoma of the breast is admitted to the local
hospice for management of her pain. The palliative care team believes she only has a matter
of a few weeks left. She is currently managed with 120 mg bd of Morphine slow release
(MST), 1020 mg of oral morphine prn and regular paracetamol. She complains of worsening
pain and yet is worried about the morphine making her drowsy and confused.
Investigations:
Hb

10.9 g/dl

White cell count 5.9 109/l


Platelets

191 109/l

Na+

139 mmol/l

K+

5.0 mmol/l

Creatinine

146 mol/l

Which one of the following would be the most appropriate plan to manage her pain?
A

Reduce her MST and add naproxen

Keep her MST stable and stop her oral morphine

Stop her MST and start naproxen

Keep her MST stable, continue to allow PRN oral morphine and add naproxen

Transfer her pain relief to fentanyl patches

Explanation
Pain relief

https://mypastest.pastest.com/Secure/TestMe/Browser/429893#Top

1/2

8/13/2016

MyPastest

Reducing this patients morphine sulphate (MST) is likely to lead to a significant


worsening in her level of pain and agitation
However, a significant degree of confusion and drowsiness is likely to result from the
intermittent dosing of oral morphine
Management of choice is to mix multiple modalities for pain relief, and as such addition
of naproxen is the most appropriate next step, she may then be able to reduce her
intermittent oral morphine
Transition to fentanyl patches, without establishing a stable dose of morphine, is
absolutely not recommended and is associated with a significant risk of over or under
dosing
21259

Next Question

Previous Question

Tag Question

Feedback

End Review

Difficulty: Average
Peer Responses

Session Progress
Responses Correct:

Responses Incorrect:

179

Responses Total:

179

Responses - % Correct:

0%

Blog (https://www.pastest.com/blog) About Pastest (https://www.pastest.com/about-us)


Contact Us (https://www.pastest.com/contact-us) Help (https://www.pastest.com/help)
Pastest 2016

https://mypastest.pastest.com/Secure/TestMe/Browser/429893#Top

2/2

8/13/2016

MyPastest

Back to Filters (/Secure/TestMe/Filter/429893/QA)

Question 166 of 179

A 25-year-old patient presents to the Emergency Department suffering from bloody


diarrhoea and pain in his right abdomen for the past 2 weeks. He just returned from a holiday
in the tropics. Entamoeba histolytica is detected in a stool sample.
Which of the following medications would be most appropriate therapy?
A

Erythromycin

Mebendazole

Mefloquine

Metronidazole

Vancomycin

Explanation

The answer is Metronidazole


Metronidazole is active against a number of protozoa including Entamoeba histolytica as
well as anaerobic bacteria. Entamoeba histolytica is by far the most common parasitic
cause of dysenteric illness worldwide, and associated with around 50 million cases of
invasive colitis each year, primarily in developing countries. Within Westernised nations,
those at highest risk of infection include institutionalised people, recent immigrants and
returning travellers. Infection may be more severe in the context of malnutrition,
malignancy, pregnancy and childhood.

Erythromycin (Option A) is incorrect. Erythromycin is inactive against E. histolytica.


Mebendazole (Option B) is incorrect. Mebednazole is active against helminithic infections,
including ascariasis, pinworm and hookworm.
Mefloquine (Option C) is incorrect. Mefloquine is active against certain malarial infections.
Vancomycin (Option E) is incorrect. Vancomycin is active against certain Gram-positive
infections, but is inactive against E. histolytica.
46799

https://mypastest.pastest.com/Secure/TestMe/Browser/429893#Top

1/2

8/13/2016

MyPastest

Next Question

Previous Question

Tag Question

Feedback

End Review

Difficulty: Average
Peer Responses

Session Progress
Responses Correct:

Responses Incorrect:

179

Responses Total:

179

Responses - % Correct:

0%

Blog (https://www.pastest.com/blog) About Pastest (https://www.pastest.com/about-us)


Contact Us (https://www.pastest.com/contact-us) Help (https://www.pastest.com/help)
Pastest 2016

https://mypastest.pastest.com/Secure/TestMe/Browser/429893#Top

2/2

8/13/2016

MyPastest

Back to Filters (/Secure/TestMe/Filter/429893/QA)

Question 167 of 179

A diabetic patient is currently being treated with metformin and sulfonylurea. However, his
HbA1C is still suboptimally controlled. You decide to add pioglitazone therapy.
Which of the following best describes the pharmacological action of this drug?
A

Increases insulin secretion

Increases insulin sensitivity

Increases glucose elimination

Inhibits gluconeogenesis

Inhibits glucose absorption

Explanation

The answer is Increases insulin sensitivity


Thiazolidinediones bind to specific receptors in the nucleus, peroxisome proliferatoractivating receptor- (PPAR-). They do not cause hypoglycaemia when used alone, but
can exaggerate the hypoglycaemic effects of insulin or sulfonylureas. Thiazolidinediones
lower glucose by 23 mmol/l and HbA1c by 1%, somewhat less than the sulfonylureas.
Adverse effects include weight gain, fluid retention and decreased bone mineral density.

Increases insulin secretion (Option A) is incorrect. Thiazolidinediones do not increase insulin


secretion.
Increases glucose elimination (Option C) is incorrect. SGLT2 inhibitors, so-called gliflozins,
decrease the renal threshold for glucose elimination.
Inhibits gluconeogenesis (Option D) is incorrect. Thiazolidinediones do not alter
gluconeogenesis.
Inhibits glucose absorption (Option E) is incorrect. Acarbose delays gastrointestinal glucose
absorption and reduces post-prandial glucose concentrations.
46823

https://mypastest.pastest.com/Secure/TestMe/Browser/429893#Top

1/2

8/13/2016

MyPastest

Next Question

Previous Question

Tag Question

Feedback

End Review

Difficulty: Average
Peer Responses

Session Progress
Responses Correct:

Responses Incorrect:

179

Responses Total:

179

Responses - % Correct:

0%

Blog (https://www.pastest.com/blog) About Pastest (https://www.pastest.com/about-us)


Contact Us (https://www.pastest.com/contact-us) Help (https://www.pastest.com/help)
Pastest 2016

https://mypastest.pastest.com/Secure/TestMe/Browser/429893#Top

2/2

8/13/2016

MyPastest

Back to Filters (/Secure/TestMe/Filter/429893/QA)

Question 168 of 179

A 48-year-old man has cirrhosis of the liver secondary to former alcohol abuse.
Which of the following drugs undergoes high first-pass metabolism and should be used with
caution in patients with mild to moderate liver disease?
A

Amoxicillin

Atenolol

Ciprofloxacin

Omeprazole

Verapamil

Explanation

The answer is Verapamil


Good examples of drugs that undergo high rates of first-pass metabolism (ie have high
liver extraction) include propranolol, verapamil and morphine. All should be used with
caution in liver disease, as poor hepatic function may lead to their accumulation because
of increased bioavailability. The handling of many drugs may be affected by liver disease
owing to reduced hepatic conjugation, reduced activity of the cytochrome P450 system
and changes in plasma protein binding. It is important to check the liver disease section
of the British National Formulary before considering therapies in patients with known
chronic liver disease.

Amoxicillin (Option A) is incorrect. Amoxicillin is excreted largely unchanged in the urine.


Atenolol (Option B) is incorrect. Atenolol is subject to renal elimination, and the dose must
be adjusted in patients with renal impairment.
Ciprofloxacin (Option C) is incorrect. Ciprofloxacin is subject to renal elimination;
ciprofloxacin exerts an inhibitory effect on P450 enzyme activity.
Omeprazole (Option D) is incorrect. Omeprazole is completely metabolised by the P450
system, but accumulation is less likely to cause toxicity than verapamil, hence it is not the
https://mypastest.pastest.com/Secure/TestMe/Browser/429893#Top

1/2

8/13/2016

MyPastest

preferred answer here.


46850

Next Question

Previous Question

Tag Question

Feedback

End Review

Difficulty: Average
Peer Responses

Session Progress
Responses Correct:

Responses Incorrect:

179

Responses Total:

179

Responses - % Correct:

0%

Blog (https://www.pastest.com/blog) About Pastest (https://www.pastest.com/about-us)


Contact Us (https://www.pastest.com/contact-us) Help (https://www.pastest.com/help)
Pastest 2016

https://mypastest.pastest.com/Secure/TestMe/Browser/429893#Top

2/2

8/13/2016

MyPastest

Back to Filters (/Secure/TestMe/Filter/429893/QA)

Question 169 of 179

An 81-year-old woman with advanced chronic obstructive pulmonary disease is admitted in


an unconscious state. She appears to have an acute lower respiratory tract infection. The
anaesthesiologist feels that she is not a candidate for Intensive Therapy Unit admission due to
poor premorbid functional state. After discussing care with the family, you decide to
prescribe doxapram therapy.
Which of the following pharmacological properties is most closely associated with
doxapram?
A

It causes hypotension

It is contraindicated in hyperthyroidism

It is safe in phaeochromocytoma

It is compatible in infusion with aminophylline

It has a half-life of 12 h

Explanation

The answer is It is contraindicated in hyperthyroidism


Doxapram is a centrally acting respiratory stimulant, which is generally used in patients
with severe respiratory disease where it may improve respiratory function sufficiently for
survival. It has a limited clinical role, and may be considered in patients who are deemed
unsuitable for invasive ventilatory support. The usual dosing regimen is 14 mg/min given
as an intravenous infusion.

It causes hypotension (Option A) is incorrect.Side-effects of therapy include hypertension,


exacerbation of apparent dyspnoea, agitation, confusion, sweating, cough, headache,
dizziness, nausea, vomiting and urinary retention.
It is safe in phaeochromocytoma (Option C) is incorrect. Doxapram is contraindicated in
patients with heart disease, epilepsy, cerebral oedema, stroke, status asthmaticus,
hypertension, hyperthyroidism and phaeochromocytoma.

https://mypastest.pastest.com/Secure/TestMe/Browser/429893#Top

1/2

8/13/2016

MyPastest

It is compatible in infusion with aminophylline (Option D) is incorrect. Doxapram solution has


pH 3.55.0. Infusion is incompatible with alkaline solutions including sodium bicarbonate,
furosemide, or aminophylline due to a risk of gas formation or precipitation.
It has a half-life of 12 h (Option E) is incorrect. The half-life is short, and the duration of
action is between 5 and 10 minutes after intravenous administration.
46893

Next Question

Previous Question

Tag Question

Feedback

End Review

Difficulty: Average
Peer Responses

Session Progress
Responses Correct:

Responses Incorrect:

179

Responses Total:

179

Responses - % Correct:

0%

Blog (https://www.pastest.com/blog) About Pastest (https://www.pastest.com/about-us)


Contact Us (https://www.pastest.com/contact-us) Help (https://www.pastest.com/help)
Pastest 2016

https://mypastest.pastest.com/Secure/TestMe/Browser/429893#Top

2/2

8/13/2016

MyPastest

Back to Filters (/Secure/TestMe/Filter/429893/QA)

Question 170 of 179

You are asked by the FY1 doctor for advice on therapeutic drug monitoring for a patient
admitted to hospital for investigation of abdominal pain.
When would be the best to take a blood level of lithium to provide optimum monitoring?
A

2 hours after dose

12 hours after dose

Any time

Before dose

Immediately after dose

Explanation

The answer is 12 hours after dose


Therapeutic drug monitoring is appropriate for drugs with a narrow therapeutic index
and where plasma concentrations give a reliable indicator of likely toxicity. Lithium is
used in the treatment and prophylaxis of mania, manic-depressive illness and recurrent
depression, and the ideal therapeutic range is 0.41.0 mmol/l on samples taken at 12 h
after the last dose. Toxic effects of lithium include tremor, ataxia and renal impairment,
and are more likely to occur at concentrations above the therapeutic range, especially if
>1.5 mmol/l.

2 hours after dose (Option A) is incorrect. This would not allow sufficient time for lithium
absorption.
Any time (Option C) is incorrect. Standardisation of the interval between dose and blood
levels is important.
Before dose (Option D) is incorrect. The trough levels are a less reliable measure of lithium
exposure and risk of toxicity (trough levels may be used for other drugs, eg vancomycin,
because they may correspond with efficacy and risk of renal failure).

https://mypastest.pastest.com/Secure/TestMe/Browser/429893#Top

1/2

8/13/2016

MyPastest

Immediately after dose (Option E) is incorrect. This would not allow sufficient time for
lithium absorption.
46967

Next Question

Previous Question

Tag Question

Feedback

End Review

Difficulty: Average
Peer Responses

Session Progress
Responses Correct:

Responses Incorrect:

179

Responses Total:

179

Responses - % Correct:

0%

Blog (https://www.pastest.com/blog) About Pastest (https://www.pastest.com/about-us)


Contact Us (https://www.pastest.com/contact-us) Help (https://www.pastest.com/help)
Pastest 2016

https://mypastest.pastest.com/Secure/TestMe/Browser/429893#Top

2/2

8/13/2016

MyPastest

Back to Filters (/Secure/TestMe/Filter/429893/QA)

Question 171 of 179

You are reviewing a 54-year-old man with a phaeochromocytoma.


Which one of the following is most likely to exert -adrenoreceptor antagonist properties
that might be effective in the presurgical management of his hypertension?
A

Atenolol

Nebivolol

Phenoxybenzamine

Propranolol

Salbutamol

Explanation

The answer is Phenoxybenzamine


Non-competitive antagonists at the -adrenoceptor (e.g. phenoxybenzamine) cannot be
displaced or have their effects diminished by an endogenous receptor ligand, namely
norepinephrine. Competitive antagonists bind to the site of action for the endogenous
receptor ligand and can be displaced (e.g. prazosin).

Atenolol (Option A) is incorrect. Atenolol is a -1 selective -receptor antagonist.


Nebivolol (Option B) is incorrect. Nebivolol is a racemic compound; one enantiomer is a
selective -1 receptor antagonist, the other enantiomer is a nitric oxide donor that causes
vasodilatation.
Propranolol (Option D) is incorrect. Propranolol is a non-selective -1/-2 receptor antagonist.
Salbutamol (Option E) is incorrect. Salbutamol is -2 receptor agonist.
46844

Next Question

https://mypastest.pastest.com/Secure/TestMe/Browser/429893#Top

1/2

8/13/2016

MyPastest

Previous Question

Tag Question

Feedback

End Review

Difficulty: Average
Peer Responses

Session Progress
Responses Correct:

Responses Incorrect:

179

Responses Total:

179

Responses - % Correct:

0%

Blog (https://www.pastest.com/blog) About Pastest (https://www.pastest.com/about-us)


Contact Us (https://www.pastest.com/contact-us) Help (https://www.pastest.com/help)
Pastest 2016

https://mypastest.pastest.com/Secure/TestMe/Browser/429893#Top

2/2

8/13/2016

MyPastest

Back to Filters (/Secure/TestMe/Filter/429893/QA)

Question 172 of 179

You are asked for advice on how to manage postural hypotension symptoms in a 76-year-old
man who is taking simvastatin 20 mg, ramipril 2.5 mg, nifedipine 20 mg, bendroflumethiazide
2.5 mg, doxazocin 8 mg and aspirin 75 mg?
Which drug is most likely responsible for postural hypotension symptoms?
A

Bendroflumethiazide

Doxazosin

Nifedipine

Ramipril

Simvastatin

Explanation

The answer is Doxazosin


Doxazosin and -blockers often cause postural hypotension, particularly in older adults.

Bendroflumethiazide (Option A) is incorrect. Bendroflumethazide and other diuretics may


cause significant volume depletion, especially in chronic use. Symptomatic hypotension is
most likely to occur in patients who have been volume- and/or salt-depleted as a result of
co-existent dietary salt restriction, dialysis, diarrhoea, or vomiting.
Nifedipine (Option C) is incorrect. Nifedipine may cause hypotension, although overall there
is less risk of postural hypotension than ACE inhibitors and -blockers, which is why option B
is the preferred answer.
Ramipril (Option D) is incorrect. Ramipril can cause symptomatic postural hypotension, after
either the initial dose or a later dose when the dosage has been increased. This is less likely
to occur than after -blocker treatment, particularly when low doses of ACE inhibitors are
involved.
Simvastatin (Option E) is incorrect. Simvastatin is not expected to have any significant effect
on blood pressure.
46947

https://mypastest.pastest.com/Secure/TestMe/Browser/429893#Top

1/2

8/13/2016

MyPastest

46947

Next Question

Previous Question

Tag Question

Feedback

End Review

Difficulty: Average
Peer Responses

Session Progress
Responses Correct:

Responses Incorrect:

179

Responses Total:

179

Responses - % Correct:

0%

Blog (https://www.pastest.com/blog) About Pastest (https://www.pastest.com/about-us)


Contact Us (https://www.pastest.com/contact-us) Help (https://www.pastest.com/help)
Pastest 2016

https://mypastest.pastest.com/Secure/TestMe/Browser/429893#Top

2/2

8/13/2016

MyPastest

Back to Filters (/Secure/TestMe/Filter/429893/QA)

Question 173 of 179

You are reviewing your patients blood results before the ward round and notice that a 53year-old woman has hyponatraemia. You note that she is receiving multiple medications that
are capable of provoking hyponatraemia.
Which one of the following drugs is most likely to cause hyponatraemia brought about by
syndrome of inappropriate antidiuretic hormone (SIADH)?
A

Amitriptyline

Chlorpheniramine

Furosemide

Haloperidol

Thyroxine

Explanation

The answer is Amitriptyline


Hyponatraemia is commonly seen in medical practice and has many causes. SIADH is
characterised by an increase in total body water with no oedema. There are a number of
drug causes of SIADH. Some drugs that stimulate the release of ADH from the posterior
pituitary gland include nicotine, phenothiazines and tricyclics. Certain drugs increase the
renal response to ADH, including desmopressin, oxytocin and prostaglandin synthesis
inhibitors. Drugs that cause SIADH by means of mixed or uncertain mechanism include
chlorpropamide, carbamazepine, cyclophosphamide, vincristine and omeprazole.
Hyponatraemia unrelated to SIADH includes circumstances related to decreased total
body water with oedema (e.g. nephrotic syndrome, cirrhosis, congestive cardiac failure)
and decreased body water without oedema (eg diuretics, Addisons disease, diarrhoea).

Chlorpheniramine (Option B) is incorrect. This is not a recognised adverse effect of


chlorpheniramine.
Furosemide (Option C) is incorrect. Diuretics cause hyponatraemia and total body water
depletion, not caused by SIADH.
https://mypastest.pastest.com/Secure/TestMe/Browser/429893#Top

1/2

8/13/2016

MyPastest

Haloperidol (Option D) is incorrect. SIADH may occur after phenothiazine antipsychotics, e.g.
chlorpromazine.
Thyroxine (Option E) is incorrect. Hypothyroidism may be associated with SIADH, but not
thyroxine replacement.
46384

Next Question

Previous Question

Tag Question

Feedback

End Review

Difficulty: Average
Peer Responses

Session Progress
Responses Correct:

Responses Incorrect:

179

Responses Total:

179

Responses - % Correct:

0%

Blog (https://www.pastest.com/blog) About Pastest (https://www.pastest.com/about-us)


Contact Us (https://www.pastest.com/contact-us) Help (https://www.pastest.com/help)
Pastest 2016

https://mypastest.pastest.com/Secure/TestMe/Browser/429893#Top

2/2

8/13/2016

MyPastest

Back to Filters (/Secure/TestMe/Filter/429893/QA)

Question 174 of 179

You are asked to review a 73-year-old man with chronic obstructive pulmonary disease
(COPD). He is maintained by his GP on combined inhalers and theophylline tablets, and has
recently been treated with antibiotics for a COPD exacerbation. He describes feeling intense
nausea and vomiting, and you find he has tachycardia. Investigations show serum potassium
3.1 mmol/l.
Which of the following antibiotics is most likely to have caused these symptoms and
findings?
A

Amoxicillin

Ceftriaxone

Clarithromycin

Penicillin V

Rifampicin

Explanation

The answer is Clarithromycin


Macrolides and quinolones inhibit metabolism of theophylline metabolism and may cause
theophylline toxicity. Quinolones may also increase the risk of convulsions if given
concurrently with theophylline therapy.

Amoxicillin (Option A) is incorrect. Amoxicillin is unlikely to have any significant effect on


theophylline metabolism.
Ceftriaxone (Option B) is incorrect. Ceftriaxone is a long-acting cephalosporin antibiotic that
would not be expected to interfere with theophylline metabolism.
Penicillin V (Option D) is incorrect. Penicillin V is unlikely to interact with theophylline.
Rifampicin (Option E) is incorrect. Rifampicin leads to the induction of enzymes involved in
theophylline metabolism, resulting in a reduced plasma concentration.
46828

https://mypastest.pastest.com/Secure/TestMe/Browser/429893#Top

1/2

8/13/2016

MyPastest

Next Question

Previous Question

Tag Question

Feedback

End Review

Difficulty: Average
Peer Responses

Session Progress
Responses Correct:

Responses Incorrect:

179

Responses Total:

179

Responses - % Correct:

0%

Blog (https://www.pastest.com/blog) About Pastest (https://www.pastest.com/about-us)


Contact Us (https://www.pastest.com/contact-us) Help (https://www.pastest.com/help)
Pastest 2016

https://mypastest.pastest.com/Secure/TestMe/Browser/429893#Top

2/2

8/13/2016

MyPastest

Back to Filters (/Secure/TestMe/Filter/429893/QA)

Question 175 of 179

One of your patients has been referred to clinic for investigation of abnormal liver
biochemistry tests. These show alanine transaminase activity of 340 U/l, bilirubin 32
millimoles per litre, and alkaline phosphatase 187 U/l. Hepatitis viral serology tests are
negative. He is taking a number of medications.
Which of the following drugs would be most likely to account for the abnormal liver
biochemistry?
A

Bendroflumethiazide

Gliclazide

Nifedipine

Paracetamol

Ramipril

Explanation

The answer is Nifedipine


Nifedipine is a recognised cause of drug-induced hepatitis and cholestatic jaundice,
albeit an uncommon cause. Other more common causes of drug-induced hepatitis
include phenytoin, carbamazepine and rifampicin.

Bendroflumethiazide (Option A) is incorrect. This is not expected to cause hepatitis.


Gliclazide (Option B) is incorrect. This is not expected to cause hepatitis.
Paracetamol (Option D) is incorrect. Paracetamol toxicity characteristically causes delayed
hepatitis, but this is not expected to occur in association with therapeutic paracetamol
doses.
Ramipril (Option E) is incorrect. This is not expected to cause hepatitis.
45811

Next Question
https://mypastest.pastest.com/Secure/TestMe/Browser/429893#Top

1/2

8/13/2016

MyPastest

Previous Question

Tag Question

Feedback

End Review

Difficulty: Average
Peer Responses

Session Progress
Responses Correct:

Responses Incorrect:

179

Responses Total:

179

Responses - % Correct:

0%

Blog (https://www.pastest.com/blog) About Pastest (https://www.pastest.com/about-us)


Contact Us (https://www.pastest.com/contact-us) Help (https://www.pastest.com/help)
Pastest 2016

https://mypastest.pastest.com/Secure/TestMe/Browser/429893#Top

2/2

8/13/2016

MyPastest

Back to Filters (/Secure/TestMe/Filter/429893/QA)

Question 176 of 179

A patient on treatment for a psychiatric disorder was noted to have hypertension and was
prescribed bendroflumethiazide. Within a week she developed tremor and agitation. Now, 4
weeks later, she presents with heart block, seizures and a raised creatinine concentration of
400 mol/l.
What psychotropic drug is most likely to have interacted with bendroflumethiazide to cause
these clinical features?
A

Amitriptyline

Chlordiazepoxide

Chlorpromazine

Fluoxetine

Lithium

Explanation

The answer is Lithium


Lithium is prescribed for bipolar disorders and schizoaffective psychosis. The patient
presents with heart block, seizures and a raised creatinine concentration of 400 mol/l,
suggestive of lithium toxicity. Other features of toxicity include blurred vision,
hypokalaemia, drowsiness, ataxia, tremor, dysarthria, arrhythmias, seizures,
hypothyroidism, psychosis and coma. Lithium toxicity may be caused by the concomitant
use of diuretics due to increased renal lithium reabsorption.

Amitriptyline (Option A) is incorrect. Amitriptyline may cause sedation, dry mouth, urinary
retention, blurred vision, acute delirium and postural hypotension.
Chlordiazepoxide (Option B) is incorrect. Adverse effects of chlordiazepoxide include
drowsiness, nausea, headache and skin rash.
Chlorpromazine (Option C) is incorrect. Chlorpromazine may cause cholestatic jaundice,
drowsiness, respiratory depression, leukopenia, tardive dyskinesia and QT prolongation.
https://mypastest.pastest.com/Secure/TestMe/Browser/429893#Top

1/2

8/13/2016

MyPastest

Fluoxetine (Option D) is incorrect. Fluoxetine is also associated with nausea, dry mouth,
blurred vision, seizures, tremor, myoclonus and decreased libido.
46771

Next Question

Previous Question

Tag Question

Feedback

End Review

Difficulty: Average
Peer Responses

Session Progress
Responses Correct:

Responses Incorrect:

179

Responses Total:

179

Responses - % Correct:

0%

Blog (https://www.pastest.com/blog) About Pastest (https://www.pastest.com/about-us)


Contact Us (https://www.pastest.com/contact-us) Help (https://www.pastest.com/help)
Pastest 2016

https://mypastest.pastest.com/Secure/TestMe/Browser/429893#Top

2/2

8/13/2016

MyPastest

Back to Filters (/Secure/TestMe/Filter/429893/QA)

Question 177 of 179

A 30-year-old woman is receiving treatment for secondary generalised tonicclonic seizures


and complains of anorexia, nausea and general fatigue. On examination, there is no peripheral
oedema and respiratory examination is normal. Investigations show Na+ 124 mmol/l, K+ 4.0
mmol/l and plasma osmolality 220 mosmol/kg.
Which of the following antiepileptic drugs is most likely to explain these abnormalities?
A

Acetazolamide

Carbamazepine

Lamotrigine

Levetiracetam

Topiramate

Explanation

The answer is Carbamazepine


Carbamazepine is a well-recognised cause of hyponatraemia, which occurs in a doserelated manner and is reversible after drug cessation.

Acetazolamide (Option A) is incorrect. Acetazolamide may cause a significant metabolic


acidosis.
Lamotrigine (Option C) is incorrect. Lamotrigine would not be expected to cause
hyponatraemia.
Levetiracetam (Option D) is incorrect. Levetiracetam would not be expected to cause
hyponatraemia.
Topiramate (Option E) is incorrect. Topirimate would not be expected to cause
hyponatraemia.
46759

Next Question
https://mypastest.pastest.com/Secure/TestMe/Browser/429893#Top

1/2

8/13/2016

MyPastest

Previous Question

Tag Question

Feedback

End Review

Difficulty: Average
Peer Responses

Session Progress
Responses Correct:

Responses Incorrect:

179

Responses Total:

179

Responses - % Correct:

0%

Blog (https://www.pastest.com/blog) About Pastest (https://www.pastest.com/about-us)


Contact Us (https://www.pastest.com/contact-us) Help (https://www.pastest.com/help)
Pastest 2016

https://mypastest.pastest.com/Secure/TestMe/Browser/429893#Top

2/2

8/13/2016

MyPastest

Back to Filters (/Secure/TestMe/Filter/429893/QA)

Question 178 of 179

An 18-year-old woman presents to the Emergency Department with symptoms of nausea and
vomiting. She states that she had been feeling very frustrated and upset and had taken an
intentional overdose involving 50 paracetamol tablets 3 h earlier.
Which of the following would be the most effective treatment at this time?
A

Acetylcysteine

Methionine

Metoclopramide

Naloxone

Oral activated charcoal

Explanation

The answer is Acetylcysteine


As little as 1015 g (2030 tablets) in an adult or 150 mg/kg of paracetamol taken within
24 h may cause severe hepatocellular necrosis and, less frequently, renal tubular necrosis.
Early features include nausea and vomiting, and as liver damage progresses over the
next 24 h there may be right subcostal pain and tenderness. In patients that recover, liver
damage is usually maximal 23 days after the ingestion. In patients that do not recover,
there is progressive liver damage, encephalopathy, haemorrhage, hypoglycaemia,
cerebral oedema, coma and death. Acetylcysteine is a highly effective antidote that
prevents fatal liver toxicity if administered within 8 h of overdose, with diminishing
effectiveness after longer intervals between ingestion and treatment.

Methionine (Option B) is incorrect. Although methionine may protect the liver against
paracetamol-induced toxicity, there is a stronger evidence base in support of acetylcysteine.
No comparative efficacy data are available for methionine and acetylcysteine, but
acetylcysteine is the standard treatment in most international guidelines.
Metoclopramide (Option C) is incorrect. Antiemetics are important in controlling symptoms
of nausea and vomiting, but metoclopramide is associated with a high risk of acute dystonia
https://mypastest.pastest.com/Secure/TestMe/Browser/429893#Top

1/2

8/13/2016

MyPastest

in young female patients, and alternatives such as ondansetron are preferred.


Naloxone (Option D) is incorrect. Naloxone is used for the treatment of opiate overdose, and
is normally administered in multiple repeated dosages titrated against clinical response to
achieve sufficiently improved conscious level that patients are able to protect their own
airway.
Oral activated charcoal (Option E) is incorrect. Oral activated charcoal is effective in
reducing paracetamol absorption, and may lessen the need for acetylcysteine antidote.
However, it should be administered within 1 h of ingestion, and is contraindicated if patients
are actively vomiting.
46800

Next Question

Previous Question

Tag Question

Feedback

End Review

Difficulty: Average
Peer Responses

Session Progress
Responses Correct:

Responses Incorrect:

179

Responses Total:

179

Responses - % Correct:

0%

Blog (https://www.pastest.com/blog) About Pastest (https://www.pastest.com/about-us)


Contact Us (https://www.pastest.com/contact-us) Help (https://www.pastest.com/help)
Pastest 2016

https://mypastest.pastest.com/Secure/TestMe/Browser/429893#Top

2/2

8/13/2016

MyPastest

Back to Filters (/Secure/TestMe/Filter/429893/QA)

Question 179 of 179

A 58-year-old man with chronic upper gastrointestinal symptoms has recently been
diagnosed as having a duodenal ulcer at endoscopy. No evidence of oesophageal or gastric
pathology was found. Helicobacter pylori testing was positive.
Which of the following treatment regimens would be most appropriate?
A

H2 receptor antagonist and antacids long term

Proton-pump inhibitor and sucralfate

Proton-pump inhibitor, bismuth, metronidazole and tetracycline, followed by


antacids long term

Proton-pump inhibitor, clarithromycin and metronidazole, followed by a protonpump inhibitor long term

Proton-pump inhibitor long term and dietary advice

Explanation

The answer is Proton-pump inhibitor, clarithromycin and metronidazole, followed by a


proton-pump inhibitor long term
All patients with proven acute or chronic duodenal ulcer disease and those with gastric
ulcer who are Helicobacter pylori-positive should be offered eradication therapy as
primary treatment. A number of options are available depending on patient factors,
including allergy status. The recommended first-line therapy should include a protonpump inhibitor (PPI) at the standard dose 12 hourly. In addition, antimicrobial treatment
normally includes clarithromycin 500 mg 12 hourly, plus either amoxicillin 1 g 12 hourly or
metronidazole 400 mg 12 hourly.

H2 receptor antagonist and antacids long term (Option A) is incorrect. PPIs are the most
powerful inhibitors of gastric acid secretion with maximal inhibition occurring 36 h after an
oral dose. They have an excellent safety profile, and allow rapid healing of both gastric and
duodenal ulcers. Antacids are insufficiently effective in increasing gastric pH to allow
effective healing.
https://mypastest.pastest.com/Secure/TestMe/Browser/429893#Top

1/2

8/13/2016

MyPastest

Proton-pump inhibitor and sucralfate (Option B) is incorrect. This regimen would not allow
eradication of H. pylori.
Proton-pump inhibitor, bismuth, metronidazole and tetracycline, followed by antacids long
term (Option C) is incorrect. Bismuth is no longer routinely recommended.
Proton-pump inhibitor long term and dietary advice (Option E) is incorrect. This regimen
would not allow eradication of H. pylori, and no special dietary advice is required.
46609

End Session

Previous Question

Tag Question

Feedback

End Review

Difficulty: Average
Peer Responses

Session Progress
Responses Correct:

Responses Incorrect:

179

Responses Total:

179

Responses - % Correct:

0%

Blog (https://www.pastest.com/blog) About Pastest (https://www.pastest.com/about-us)


Contact Us (https://www.pastest.com/contact-us) Help (https://www.pastest.com/help)
Pastest 2016

https://mypastest.pastest.com/Secure/TestMe/Browser/429893#Top

2/2

8/11/2016

MyPastest

Back to Filters (/Secure/TestMe/Filter/429893/QA)

Question 1 of 207

A 56-year-old worker is brought into the Emergency Department from a nearby


electroplating company. Two of his colleagues are present, and advise that he may have been
accidentally exposed to hydrogen cyanide gas.
Which of the following steps best describes the treatment of suspected cyanide poisoning?
A

Amyl nitrite may be a useful antidote therapy

Await formal laboratory cyanide confirmation before administering antidote

Oral activated charcoal should be administered within 1 h of exposure

Patients with generalised headache should be treated with dicobalt edetate

Supplementary oxygen may increase pulmonary toxicity and should be avoided

Explanation

The answer is Amyl nitrite may be a useful antidote therapy


Cyanide can be found in certain plants, such as cyanogenic glycosides (eg bitter
almonds, apricot kernels). It is rapidly absorbed through the respiratory and
gastrointestinal tracts and the skin. Effects are more rapid onset after inhalation than
ingestion, death may occur within minutes. Immediate management of suspected
cyanide poisoning may consist of oxygen administration and administration of inhaled
nitrites, eg amyl nitrite.

Await formal laboratory cyanide confirmation before administering antidote (Option B) is


incorrect. Although laboratory confirmation may be helpful in confirming the cyanide
exposure and extent of poisoning, treatment should not be delayed.
Oral activated charcoal should be administered within 1 h of exposure (Option C) is incorrect.
Charcoal is unlikely to be effective.
Patients with generalised headache should be treated with dicobalt edetate (Option D) is
incorrect. Intravenous dicobalt edetate is reserved for patients with signs of severe poisoning
(coma, metabolic acidosis, respiratory depression). There should be a clear history of
https://mypastest.pastest.com/Secure/TestMe/Browser/429893

1/2

8/11/2016

MyPastest

exposure, since the potential serious adverse effects (bronchospasm, laryngeal oedema) are
more likely if no cyanide ions are present.
Supplementary oxygen may increase pulmonary toxicity and should be avoided (Option E) is
incorrect. This is true of Paraquat, but not of cyanide poisoning.
46492

Next Question

Previous Question

Tag Question

Feedback

End Review

Difficulty: Average
Peer Responses

Session Progress
Responses Correct:

Responses Incorrect:

207

Responses Total:

207

Responses - % Correct:

0%

Blog (https://www.pastest.com/blog) About Pastest (https://www.pastest.com/about-us)


Contact Us (https://www.pastest.com/contact-us) Help (https://www.pastest.com/help)
Pastest 2016

https://mypastest.pastest.com/Secure/TestMe/Browser/429893

2/2

8/11/2016

MyPastest

Back to Filters (/Secure/TestMe/Filter/429893/QA)

Question 2 of 207

A 68-year-old woman with severe rheumatoid arthritis presents for review. She complains of
increasing tiredness and has multiple swollen lymph nodes. There is mild normochromic,
normocytic anaemia and her erythrocyte sedimentation rate (ESR) is markedly raised despite
quiescent rheumatoid disease at present. She is currently managed with methotrexate and
infliximab.
What diagnosis fits best with this clinical picture?
A

Carcinomatosis

Chronic myeloid leukaemia secondary to methotrexate

Lymphoma secondary to immune-modulation therapy

Reactivation of rheumatoid disease

SLE-type syndrome related to immune-modulation therapy

Explanation

The answer is Lymphoma secondary to immune-modulation therapy


Various studies have attempted to estimate the long-term risk of lymphoma after
treatment with antitumour necrosis factor-alpha (TNF-) immune-modulation therapy
(infliximab). The Food and Drug Administration (FDA) estimate a two to six-fold
increased risk of lymphoma, and anti-TNF- agents carry a warning label concerning the
risk of malignancy.

Carcinomatosis (Option A) is incorrect. The possible increased risk of solid tumours is less
striking than the increased risk of lymphoma, hence this answer is a less preferred option
than lymphoma secondary to immune-modulation therapy.
Chronic myeloid leukaemia secondary to methotrexate (Option B) is incorrect. Methotrexate
is associated with an increased risk of melanoma and other cancers, but less strongly
associated with lymphoma than anti-TNF- immune-modulators.
Reactivation of rheumatoid disease (Option D) is incorrect. This cannot explain the
occurrence of lymphadenopathy, and is less likely given the absence of active joint
https://mypastest.pastest.com/Secure/TestMe/Browser/429893

1/2

8/11/2016

MyPastest

symptoms.
SLE-type syndrome related to immune-modulation therapy (Option E) is incorrect. There is
an increased occurrence of antinuclear antibody positivity, although the occurrences of
clinical features of systemic lupus erythematosus (SLE) are rare, and resolve after drug
discontinuation.
46889

Next Question

Previous Question

Tag Question

Feedback

End Review

Difficulty: Average
Peer Responses

Session Progress
Responses Correct:

Responses Incorrect:

207

Responses Total:

207

Responses - % Correct:

0%

Blog (https://www.pastest.com/blog) About Pastest (https://www.pastest.com/about-us)


Contact Us (https://www.pastest.com/contact-us) Help (https://www.pastest.com/help)
Pastest 2016

https://mypastest.pastest.com/Secure/TestMe/Browser/429893

2/2

8/11/2016

MyPastest

Back to Filters (/Secure/TestMe/Filter/429893/QA)

Question 3 of 207

A 66-year-old woman attends the cardiology outpatient clinic for review. She has been
receiving a number of medications, including a -blocker (-adrenergic antagonist) for
treatment of angina, and asks you for information about this medication.
Which of the following statements best describes the therapeutic action of -blockers in
angina?
A

Decrease the heart rate and myocardial contractility

Increase the cardiac preload

Increase the sinus node automaticity

Increase the left atrial volume and pressure

Increase the peripheral vascular resistance

Explanation

The answer is Decrease the heart rate and myocardial contractility


The cardiovascular effects of -adrenoceptor block depend on the amount of
sympathetic tone present, which normally increases during exercise or activity. The
primary pharmacological action of -blockers is to reduce sympathetic drive, which
reduces heart rate (automaticity) and lessens myocardial contractility (rate of rise of
pressure in the ventricle). These effects result in reduced cardiac output and an overall
fall in myocardial oxygen consumption.

Increase the cardiac preload (Option B) is incorrect. Beta-blockers tend to increase afterload
due to increased peripheral vascular resistance, but do not have any significant effect on
cardiac preload.
Increase the sinus node automaticity (Option C) is incorrect. Beta-blockers reduce sinus
node automaticity, thereby lowering heart rate.
Increase the left atrial volume and pressure (Option D) is incorrect. Beta-blockers decrease
left atrial volume and pressure.
https://mypastest.pastest.com/Secure/TestMe/Browser/429893

1/2

8/11/2016

MyPastest

Increase the peripheral vascular resistance (Option E) is incorrect. It is true that -blockers
cause a rise in peripheral vascular resistance due to the unopposed action of catecholamines
on -adrenoceptors in the periphery. However, this gives rise to the characteristic adverse
effects (vasoconstriction, muscle cramps and cold peripheries) but not the intended
pharmacological effects.
46629

Next Question

Previous Question

Tag Question

Feedback

End Review

Difficulty: Average
Peer Responses

Session Progress
Responses Correct:

Responses Incorrect:

207

Responses Total:

207

Responses - % Correct:

0%

Blog (https://www.pastest.com/blog) About Pastest (https://www.pastest.com/about-us)


Contact Us (https://www.pastest.com/contact-us) Help (https://www.pastest.com/help)
Pastest 2016

https://mypastest.pastest.com/Secure/TestMe/Browser/429893

2/2

8/11/2016

MyPastest

Back to Filters (/Secure/TestMe/Filter/429893/QA)

Question 4 of 207

A patient is brought into hospital at 0700 h after accidentally drinking 300 ml diethylene
glycol from an unmarked bottle on the previous night. He appears intoxicated with slurred
speech and drowsiness, and initial investigations show metabolic acidosis and acute renal
failure.
What is the most appropriate next step in his management?
A

Forced diuresis

Gastric lavage

Haemodialysis and intravenous ethanol

Intravenous insulin

Oral methanol

Explanation

The answer is Haemodialysis and intravenous ethanol


Diethylene glycol is used mainly in an industrial setting as a solvent used in manufacture
of polyester resins and other chemicals. Several instances of fatal outbreaks have
occurred where diethylene glycol has been inadvertently added to pharmaceutical
preparations. Features of toxicity include nausea, vomiting and abdominal pain. As
toxicity develops, there may be jaundice, pulmonary oedema, severe metabolic acidosis,
renal failure, coma and death. Management involves supportive measures to correct
dehydration and metabolic distrurbances. Toxicity is mediated by the metabolites of
diethylene glycol, and so fomepizole (4-methylpyrazole) should be administered to block
diethylene glycol metabolism by alcohol dehydrogenase. Haemodialysis should be
considered if renal failure develops or there is severe acidosis or metabolic disturbance.
An alternative to fomepizole is ethanol, which competes with diethylene glycol for
metabolism and minimises formation of toxic metabolites. Fomepizole or ethanol should
normally be continued until diethylene glycol is no longer detectable in the blood.

Forced diuresis (Option A) is incorrect. Forced diuresis is unhelpful and potentially


hazardous.
https://mypastest.pastest.com/Secure/TestMe/Browser/429893

1/2

8/11/2016

MyPastest

Gastric lavage (Option B) is incorrect. This is rarely helpful in clinical practice and potentially
hazardous, and too late to be of any benefit in this case.
Intravenous insulin (Option D) is incorrect. The metabolic acidosis is attributable to build-up
of metabolites and renal failure, not diabetic ketoacidosis.
Oral methanol (Option E) is incorrect. Methanol would compete with alcohol dehydrogenase
but would add to the burden of toxic metabolites and worsen the clinical outcome.
46791

Next Question

Previous Question

Tag Question

Feedback

End Review

Difficulty: Average
Peer Responses

Session Progress
Responses Correct:

Responses Incorrect:

207

Responses Total:

207

Responses - % Correct:

0%

Blog (https://www.pastest.com/blog) About Pastest (https://www.pastest.com/about-us)


Contact Us (https://www.pastest.com/contact-us) Help (https://www.pastest.com/help)
Pastest 2016

https://mypastest.pastest.com/Secure/TestMe/Browser/429893

2/2

8/11/2016

MyPastest

Back to Filters (/Secure/TestMe/Filter/429893/QA)

Question 5 of 207

You are reviewing a 63-year-old man in the medical admissions unit who has been given a
diagnosis of community-acquired pneumonia. You remember from a recent teaching session
that moxifloxacin is licensed for the treatment of community-acquired pneumonia, acute
exacerbation of chronic bronchitis and acute bacterial sinusitis.
Which of the following adverse effects is most strongly associated with moxifloxacin?
A

Achilles tendon rupture

Clostridium difficile associated diarrhoea

Neutropaenia

Oesophageal erosions and perforation

Widening of the QRS duration

Explanation

The answer is Achilles tendon rupture


Moxifloxacin is a new generation of quinolone antibiotics that is bactericidal by
interfering with DNA synthesis, and is effective against Gram-positive, Gram-negative
and certain atypical respiratory pathogens. Adverse effects of quinolones include risk of
tendonitis and tendon rupture. Other adverse effects include gastrointestinal
disturbances, headache and liver dysfunction. Quinolones should not be used in
combination with other drugs that prolong the QT interval (eg erythromycin, citalopram)
because there is an increased risk of cardiac arrhythmias. Quinolones may reduce cardiac
ejection fraction and are contraindicated in left heart failure. They should not be given at
the same time as bivalent or trivalent cations (eg aluminium, iron) as these reduce
absorption.

Clostridium difficile associated diarrhoea (Option B) is incorrect. Although all broadspectrum antibiotics are capable of provoking Clostridium difficile associated diarrhea, this is
less specific to quinolones than the occurrence of tendon rupture.

https://mypastest.pastest.com/Secure/TestMe/Browser/429893

1/2

8/11/2016

MyPastest

Neutropaenia (Option C) is incorrect. This may be encountered in patients with sepsis, but is
not a commonly recognised adverse effect of quinolones.
Oesophageal erosions and perforation (Option D) is incorrect. This is a recognised
complication of bisphosphonates.
Widening of the QRS duration (Option E) is incorrect. Quinolones may cause QT
prolongation.
46380

Next Question

Previous Question

Tag Question

Feedback

End Review

Difficulty: Average
Peer Responses

Session Progress
Responses Correct:

Responses Incorrect:

207

Responses Total:

207

Responses - % Correct:

0%

Blog (https://www.pastest.com/blog) About Pastest (https://www.pastest.com/about-us)


Contact Us (https://www.pastest.com/contact-us) Help (https://www.pastest.com/help)
Pastest 2016

https://mypastest.pastest.com/Secure/TestMe/Browser/429893

2/2

8/11/2016

MyPastest

Back to Filters (/Secure/TestMe/Filter/429893/QA)

Question 6 of 207

You review a 75-year-old man in clinic with long-standing Alzheimers disease. His family
attends with him and asks you some questions about memantine, a drug that is licensed for
the management of dementia in the UK.
Which of the following statements is most appropriate regarding memantine?
A

Adverse effects are more likely if co-administered with amantadine

Concomitant baclofen doses may have to be increased

It inhibits cholinesterase activity within the central nervous system

Its pharmacological action is an NMDA-receptor agonist

Quinine serum concentrations will fall during memantine use

Explanation

The answer is Adverse effects are more likely if co-administered with amantadine Excess activation of the NMDA (N-methyl-d-aspartate) receptor may play a role in the
pathogenesis of Alzheimers disease. Memantine is an antagonist at the NMDA receptor.
Other treatment options for Alzheimers disease include cholinesterase inhibitors (e.g.
tacrine, donepezil). Other drugs with NMDA-receptor antagonist properties include
ketamine and amantadine, which increase the risk of psychosis when co-administered.

Concomitant baclofen doses may have to be increased (Option B) is incorrect. Memantine


has some antispasmodic effects, therefore the dose of baclofen may be reduced in some
cases.
It inhibits cholinesterase activity within the central nervous system (Option C) is incorrect.
Certain other drugs used in Alzheimers disease inhibit cholinesterase activity (e.g. tacrine,
donepezil).
Its pharmacological action is an NMDA-receptor agonist (Option D) is incorrect. It is an
antagonist, not an agonist at the NMDA receptor

https://mypastest.pastest.com/Secure/TestMe/Browser/429893

1/2

8/11/2016

MyPastest

Quinine serum concentrations will fall during memantine use (Option E) is incorrect.
Memantine interferes with cationic transporters in the kidney so that excretion of some drugs
is reduced (eg quinine, cimetidine, ranitidine), therefore higher plasma concentrations are
likely.
46376

Next Question

Previous Question

Tag Question

Feedback

End Review

Difficulty: Average
Peer Responses

Session Progress
Responses Correct:

Responses Incorrect:

207

Responses Total:

207

Responses - % Correct:

0%

Blog (https://www.pastest.com/blog) About Pastest (https://www.pastest.com/about-us)


Contact Us (https://www.pastest.com/contact-us) Help (https://www.pastest.com/help)
Pastest 2016

https://mypastest.pastest.com/Secure/TestMe/Browser/429893

2/2

8/11/2016

MyPastest

Back to Filters (/Secure/TestMe/Filter/429893/QA)

Question 7 of 207

You are reviewing a 74-year-old woman who has been treated with verapamil for chronic
atrial tachycardia. You note in clinic that she is hypertensive with a blood pressure of 160/85
mmHg. You want to start additional blood-pressure-lowering therapy.
Which of the following would be the most appropriate next agent to add in?
A

Atenolol

Diltiazem

Metoprolol

Ramipril

Sotalol

Explanation

The answer is Ramipril


The combination of a calcium-channel blocker with angiotensin converting enzyme
(ACE) inhibitor would be effective in achieving good blood pressure control. Ramipril is
an ACE inhibitor with good outcome data for cardiovascular risk reduction.

Atenolol (Option A) is incorrect. The combination of a non-dihydropyridine calcium


antagonist with -blockade is ill advised owing to the risk of severe bradycardia, heart block,
severe hypotension and syncope.
Diltiazem (Option B) is incorrect. Diltiazem has a similar mode of action to verapamil and as
such it may provoke heart block and severe bradycardia.
Metoprolol (Option C) is incorrect. The combination of a non-dihydropyridine calcium
antagonist with -blockade is ill advised owing to the risk of severe bradycardia, heart block,
severe hypotension and syncope.
Sotalol (Option E) is incorrect. Sotalol is unlikely to confer any meaningful additional blood
pressure control.
46826

https://mypastest.pastest.com/Secure/TestMe/Browser/429893

1/2

8/11/2016

MyPastest

Next Question

Previous Question

Tag Question

Feedback

End Review

Difficulty: Average
Peer Responses

Session Progress
Responses Correct:

Responses Incorrect:

207

Responses Total:

207

Responses - % Correct:

0%

Blog (https://www.pastest.com/blog) About Pastest (https://www.pastest.com/about-us)


Contact Us (https://www.pastest.com/contact-us) Help (https://www.pastest.com/help)
Pastest 2016

https://mypastest.pastest.com/Secure/TestMe/Browser/429893

2/2

8/11/2016

MyPastest

Back to Filters (/Secure/TestMe/Filter/429893/QA)

Question 8 of 207

A patient is referred to the Acute Medical Unit from the local cancer care centre. They have
been receiving chemotherapy treatment and have presented to hospital with increased
urinary frequency and frank haematuria. Full blood count shows a mild anaemia and
leukopenia.
Which of the following drugs is most likely to account for these findings?
A

Allopurinol

Cisplatin

Cyclophosphamide

Mithramycin

Tamsulosin

Explanation

The answer is Cyclophosphamide


Cyclophosphamide is a prodrug that is metabolised by the liver to 4-hydroxyl
cyclophosphamide, which decomposes into alkylating species as well as to
chloroacetaldehyde and acrolein. Acrolein is known to cause chemical haemorrhagic
cystitis, and therefore excellent hydration must be maintained during cyclophosphamide
therapy. Ifosamide may also cause haemorrhagic cystitis. Co-administration of mesna
may minimise the risk of bladder toxicity by binding to acrolein and the toxic metabolites
of ifosfamide.

Allopurinol (Option A) is incorrect. Allopurinol does not cause bladder irritation.


Cisplatin (Option B) is incorrect. Cisplatin characteristically causes acute kidney injury.
Mithramycin (Option D) is incorrect. Mithramycin may cause renal damage and bone marrow
suppression.
Tamsulosin (Option E) is incorrect. Tamsulosin may cause urinary incontinence.
46595

https://mypastest.pastest.com/Secure/TestMe/Browser/429893

1/2

8/11/2016

MyPastest

Next Question

Previous Question

Tag Question

Feedback

End Review

Difficulty: Average
Peer Responses

Session Progress
Responses Correct:

Responses Incorrect:

207

Responses Total:

207

Responses - % Correct:

0%

Blog (https://www.pastest.com/blog) About Pastest (https://www.pastest.com/about-us)


Contact Us (https://www.pastest.com/contact-us) Help (https://www.pastest.com/help)
Pastest 2016

https://mypastest.pastest.com/Secure/TestMe/Browser/429893

2/2

8/11/2016

MyPastest

Back to Filters (/Secure/TestMe/Filter/429893/QA)

Question 9 of 207

A 57-year-old woman who has had a previous renal transplant is being given long-term
azathioprine treatment.
Which of the following statements best describes the main mechanism of action of
azathioprine?
A

It blocks antibody formation

It interferes with T-cell activation mechanisms at an intracellular level

It interferes with T-cellmacrophage cooperation

It reduces the production of cytokines

It suppresses lymphocyte numbers and function

Explanation

The answer is It suppresses lymphocyte numbers and function


Azathioprine acts as an inhibitor of purine synthase, which is an essential step in
proliferation of leukocytes and lymphocytes.

It blocks antibody formation (Option A) is incorrect. Anti-B-cell agents reduce antibody


production, including rituximab.
It interferes with T-cell activation mechanisms at an intracellular level (Option B) is incorrect.
Ciclosporin and tacrolimus act as immunosuppressants by interfering with T-cell activation at
an intracellular level.
It interferes with T-cellmacrophage cooperation (Option C) is incorrect. Corticosteroids
interfere with T-cellmacrophage cooperation and impair macrophage responses to
cytokines.
It reduces the production of cytokines (Option D) is incorrect. Selective antibody agents may
prevent or minimise formation of cytokines, eg anti-IL1 monoclonal antibodies.
46866

https://mypastest.pastest.com/Secure/TestMe/Browser/429893

1/2

8/11/2016

MyPastest

Next Question

Previous Question

Tag Question

Feedback

End Review

Difficulty: Average
Peer Responses

Session Progress
Responses Correct:

Responses Incorrect:

207

Responses Total:

207

Responses - % Correct:

0%

Blog (https://www.pastest.com/blog) About Pastest (https://www.pastest.com/about-us)


Contact Us (https://www.pastest.com/contact-us) Help (https://www.pastest.com/help)
Pastest 2016

https://mypastest.pastest.com/Secure/TestMe/Browser/429893

2/2

8/11/2016

MyPastest

Back to Filters (/Secure/TestMe/Filter/429893/QA)

Question 10 of 207

You are examining the local protocols concerning patients treated at the local cancer unit,
and the indications for referral for pre-treatment echocardiography.
Which one of the following agents is most likely to cause cardiotoxicity?
A

Cisplatin

Cyclophosphamide

Rituximab

Sunitinib

Trastuzumab

Explanation

The answer is Trastuzumab


Trastuzumab (Herceptin) is used in the treatment of HER2-receptor-positive metastatic
breast cancer. Trastuzumab is associated with cardiomyopathy in between 2% and 7% of
users, and the risk of cardiomyopathy is increased when given in combination with
anthracycline.

Cisplatin (Option A) is incorrect. Cisplatin is associated with neurotoxicity, nephrotoxicity


and ototoxicity.
Cyclophosphamide (Option B) is incorrect. Cyclophosphamide is associated with
haemorrhagic cystitis.
Rituximab (Option C) is incorrect. Rituximab is associated with cytokine release.
Sunitinib (Option D) is incorrect. Sunitinib is associated mainly with fatigue, diarrhoea,
hypertension and hand/foot syndrome.
46986

Next Question

https://mypastest.pastest.com/Secure/TestMe/Browser/429893

1/2

8/11/2016

MyPastest

Previous Question

Tag Question

Feedback

End Review

Difficulty: Average
Peer Responses

Session Progress
Responses Correct:

Responses Incorrect:

207

Responses Total:

207

Responses - % Correct:

0%

Blog (https://www.pastest.com/blog) About Pastest (https://www.pastest.com/about-us)


Contact Us (https://www.pastest.com/contact-us) Help (https://www.pastest.com/help)
Pastest 2016

https://mypastest.pastest.com/Secure/TestMe/Browser/429893

2/2

8/11/2016

MyPastest

Back to Filters (/Secure/TestMe/Filter/429893/QA)

Question 11 of 207

An 82-year-old woman presents to the Acute Medical Unit with a history of vomiting and
fever, and has been diagnosed with a lower urinary tract infection. You decide to administer
oral trimethoprim.
Which of the following most accurately describes the pharmacological actions of
trimethoprim?
A

Hypokalaemia occurs in 2030% of patients

It inhibits dihydrofolate reductase

It is bactericidal

It may cause marrow depression and microcytic anaemia

It requires therapeutic drug monitoring to ensure efficacy

Explanation

The answer is It inhibits dihydrofolate reductase


Trimethoprim acts by inhibiting dihydrofolate reductase, an enzyme that is important in
the division of bacterial cells. It is bacteriostatic, rather than bactericidal.
Sulfamethoxazole also inhibits folate metabolism, and when combined with trimethoprim
(co-trimoxazole) this exerts bactericidal effects; co-trimoxazole is associated with
significant adverse effects including blood dyscrasias and autoimmune phenomena, and
is normally reserved for severe infections such as pneumocystis pneumonia.
Trimethoprim may cause megaloblastic anaemia due to folate deficiency where used in
long-term treatment.

Hypokalaemia occurs in 2030% of patients (Option A) is incorrect. Hyperkalaemia and a rise


in serum creatinine concentrations characteristically occur due to interference with tubular
secretion, and are fully reversible on stopping trimethoprim.
It is bactericidal (Option C) is incorrect. It is bacteriostatic. The combination with
sulfamethoxazole does confer bactericidal effects.

https://mypastest.pastest.com/Secure/TestMe/Browser/429893#Top

1/2

8/11/2016

MyPastest

It may cause marrow depression and microcytic anaemia (Option D) is incorrect. Significant
bone marrow depression occurs when used in combination with sulfamethoxazole.
Macrocytic anaemia may occur, not microcytic.
It requires therapeutic drug monitoring to ensure efficacy (Option E) is incorrect. Serum
levels are rarely used to monitor trimethoprim treatment.
46603

Next Question

Previous Question

Tag Question

Feedback

End Review

Difficulty: Average
Peer Responses

Session Progress
Responses Correct:

Responses Incorrect:

207

Responses Total:

207

Responses - % Correct:

0%

Blog (https://www.pastest.com/blog) About Pastest (https://www.pastest.com/about-us)


Contact Us (https://www.pastest.com/contact-us) Help (https://www.pastest.com/help)
Pastest 2016

https://mypastest.pastest.com/Secure/TestMe/Browser/429893#Top

2/2

8/11/2016

MyPastest

Back to Filters (/Secure/TestMe/Filter/429893/QA)

Question 12 of 207

You are reviewing a 42-year-old man in the hypertension clinic, and are considering
prescribing doxazosin.
In which circumstances would prescription of an -blocker be considered valuable in patients
with hypertension?
A

Essential hypertension

Hypertension and asthma

Hypertension due to Conn syndrome

Hypertension due to phaeochromocytoma

Hypertension with renal insufficiency

Explanation

The answer is Hypertension due to phaeochromocytoma


The mainstay of medical treatment is -blockade to minimise the effect of excess
norepinephrine concentrations acting on peripheral -receptors that cause
vasoconstriction. The -blocker of choice is phenoxybenzamine, which irreversibly blocks
the -receptor by alkylation.

Essential hypertension (Option A) is incorrect. Alpha-blockers are second- or third-line


treatments for essential hypertension, after ACE inhibitors, angiotensin-receptor blockers,
calcium-channel blockers and diuretics.
Hypertension and asthma (Option B) is incorrect. Beta-blockers should be avoided in asthma.
First-line therapies remain ACE inhibitors, angiotensin-receptor blockers and calcium-channel
blockers.
Hypertension due to Conn syndrome (Option C) is incorrect. Conn syndrome is characterised
by excess aldosterone. Spironolactone possesses anti-aldosterone properties and may be
highly effective in achieving blood pressure control.

https://mypastest.pastest.com/Secure/TestMe/Browser/429893

1/2

8/11/2016

MyPastest

Hypertension with renal insufficiency (Option E) is incorrect. Alpha-blockers are often used
in renal failure but are still regarded as second-line to alternative agents including calciumchannel blockers and ACE inhibitors.
46811

Next Question

Previous Question

Tag Question

Feedback

End Review

Difficulty: Average
Peer Responses

Session Progress
Responses Correct:

Responses Incorrect:

207

Responses Total:

207

Responses - % Correct:

0%

Blog (https://www.pastest.com/blog) About Pastest (https://www.pastest.com/about-us)


Contact Us (https://www.pastest.com/contact-us) Help (https://www.pastest.com/help)
Pastest 2016

https://mypastest.pastest.com/Secure/TestMe/Browser/429893

2/2

8/11/2016

MyPastest

Back to Filters (/Secure/TestMe/Filter/429893/QA)

Question 13 of 207

A middle-aged man is rushed in an unconscious state to the Emergency Department. It is


stated that he swallowed a large number of unknown tablets. Investigations reveal: Na+ 137
mmol/l; K+ 3.5 mmol/l; Cl 96 mmol/l; HCO3 16 mmol/l; pH 7.25; anion gap 25 mmol/l.
What medicine is he most likely to have swallowed?
A

Allopurinol

Aspirin

Benzylpenicillin

Diclofenac

d-Penicillamine

Explanation

The answer is Aspirin


The anion gap represents those negative ions not normally measured in clinical practice,
which include phosphate, sulfate, lactate, ketoacids and albumin. A number of formulae
exist for calculating the anion gap, eg:
Anion gap = plasma [Na+ + K+] - (plasma [Cl] + plasma [HCO3]) The normal range is
816 mmol/l.
The anion gap is increased when excessive acid is added to the plasma, either by
disordered metabolism or by the addition of exogenous acid or when there is failure of
acid secretion. Salicylate poisoning is a known cause of an increased anion gap. Other
causes of an increased anion gap include dehydration, lactic acidosis, diabetic and
alcoholic ketoacidosis, uraemia, poisoning by ethylene glycol and methanol.

Allopurinol (Option A) is incorrect. Allopurinol, benzylpenicillin and indometacin cause acute


interstitial nephritis, which may cause metabolic acidosis but the anion gap will be normal.
Benzylpenicillin (Option C) is incorrect. Benzylpenicillin may rarely cause acute interstitial
nephritis and metabolic acidosis, but the anion gap will be normal.
https://mypastest.pastest.com/Secure/TestMe/Browser/429893#Top

1/2

8/11/2016

MyPastest

Diclofenac (Option D) is incorrect. Diclofenac may cause acute interstitial nephritis and acute
tubular necrosis, which may be associated with metabolic acidosis, but the anion gap will be
normal.
d-Penicillamine (Option E) is incorrect. d-Penicillamine may cause membranous nephropathy,
which does not cause metabolic acidosis.
46745

Next Question

Previous Question

Tag Question

Feedback

End Review

Difficulty: Average
Peer Responses

Session Progress
Responses Correct:

Responses Incorrect:

207

Responses Total:

207

Responses - % Correct:

0%

Blog (https://www.pastest.com/blog) About Pastest (https://www.pastest.com/about-us)


Contact Us (https://www.pastest.com/contact-us) Help (https://www.pastest.com/help)
Pastest 2016

https://mypastest.pastest.com/Secure/TestMe/Browser/429893#Top

2/2

8/11/2016

MyPastest

Back to Filters (/Secure/TestMe/Filter/429893/QA)

Question 14 of 207

A 67-year-old man presents with sudden-onset atrial fibrillation (ventricular rate of 150 bpm).
His serum creatinine concentration is 250 mol/l (70110).
What is the main factor that determines the choice of loading dose of digoxin in this patient?
A

Bioavailability

First-pass metabolism

Plasma-binding proteins

Renal clearance

Volume of distribution

Explanation

The answer is Renal clearance


Digoxin is water soluble and, as such, the major determinant of any loading dose is the
patients renal function. Effective plasma levels for digoxin are thought to be between 1
and 2.6 nmol/l. In a patient with a creatinine of 250 mol/l, adjustment of both the
loading and maintenance doses is necessary because the normal half-life of 36 h is
considerably lengthened and the drug may accumulate with repeated dose
administration. Digoxin binds to a subunit of the Na+/K+ ATPase pump on the
membranes of cardiac myocytes. Adverse effects are increased in patients with
concomitant hypokalaemia.

Bioavailability (Option A) is incorrect. Drug absorption is likely to remain fairly normal, hence
bioavailability is unchanged.
First-pass metabolism (Option B) is incorrect. Digoxin is not subject to liver metabolism, so
first-pass metabolism is not relevant.
Plasma-binding proteins (Option C) is incorrect. Hypoalbuminaemia is a recognised feature
of chronic kidney disease, but is a less important determinant of digoxin toxicity than
impaired elimination.
https://mypastest.pastest.com/Secure/TestMe/Browser/429893#Top

1/2

8/11/2016

MyPastest

Volume of distribution (Option E) is incorrect. Volume of distribution is unlikely to be


significantly altered.
46996

Next Question

Previous Question

Tag Question

Feedback

End Review

Difficulty: Average
Peer Responses

Session Progress
Responses Correct:

Responses Incorrect:

207

Responses Total:

207

Responses - % Correct:

0%

Blog (https://www.pastest.com/blog) About Pastest (https://www.pastest.com/about-us)


Contact Us (https://www.pastest.com/contact-us) Help (https://www.pastest.com/help)
Pastest 2016

https://mypastest.pastest.com/Secure/TestMe/Browser/429893#Top

2/2

8/11/2016

MyPastest

Back to Filters (/Secure/TestMe/Filter/429893/QA)

Question 15 of 207

A 54-year-old woman with metastatic carcinoma of the breast presents to the clinic with very
much worsening pain in her back and ribs; she has known bony metastases in the area.
Current medication includes 100 mg of MST bd with top-up doses of 30 mg oral morphine as
required. She also takes regular paracetamol.
Which one of the following additional steps would you take next to control her pain?
A

Increase dose/frequency of oral morphine top-ups

Increase dose of MST

Continue oral morphine and prescribe diclofenac

Start diamorphine syringe driver

Start fentanyl patches

Explanation
Pain relief in metastatic disease
Given guidance on the multi-modal approach to pain relief, addition of a NSAID is the
most appropriate next step. They are thought to be particularly effective in managing
pain from bony metastases
Increasing morphine sulphate (MST) or beginning fentanyl patches is an option once
oral morphine top-ups are stable, but should not be attempted until pain is relatively
controlled, this is because of the risk of overdose due to variability in analgesic
requirements from day to day
Equally, a diamorphine syringe driver should only be commenced if the patient is
unable to take oral pain relief to manage pain
Depending on how well localised the metastases are, local radiotherapy may also be
highly effective in controlling pain
20945

https://mypastest.pastest.com/Secure/TestMe/Browser/429893#Top

1/2

8/11/2016

MyPastest

Next Question

Previous Question

Tag Question

Feedback

End Review

Difficulty: Difficult
Peer Responses

Session Progress
Responses Correct:

Responses Incorrect:

207

Responses Total:

207

Responses - % Correct:

0%

Blog (https://www.pastest.com/blog) About Pastest (https://www.pastest.com/about-us)


Contact Us (https://www.pastest.com/contact-us) Help (https://www.pastest.com/help)
Pastest 2016

https://mypastest.pastest.com/Secure/TestMe/Browser/429893#Top

2/2

8/11/2016

MyPastest

Back to Filters (/Secure/TestMe/Filter/429893/QA)

Question 16 of 207

A 12-year-old boy presents to an Out of hours service with headache, drowsiness, fever and
neck stiffness. The fundi appear normal. There is a purpuric rash on his body. Previous
medical history and family history are unremarkable. The GP phones you for advice as to
what he should do while awaiting urgent ambulance transfer to hospital.
What advice should be given?
A

Do nothing pending blood cultures and CSF analysis

Give bolus IV cefotaxime

Give intramuscular benzylpenicillin

Give single high-dose oral benzylpenicillin

Give single high-dose oral cephalosporin

Explanation

The answer is Give intramuscular benzylpenicillin


The scenario is suggestive of meningitis caused by Neisseria meningitides. Initial blind
treatment of suspected bacterial meningitis (especially meningococcal disease, as is
likely given the rash) involves urgent transfer to hospital and parenteral benzylpenicillin
before transfer (IV being preferable to intramuscular, if possible). Cefotaxime may be an
alternative in penicillin allergy, and chloramphenicol may be used if there is a history of
anaphylaxis to penicillin or cephalosporins.
In hospital the current guideline is to give IV cefotaxime pending confirmation of the
organism and antibiotic sensitivities; thereafter the regimen of choice for confirmed
meningococcal meningitis is intravenous benzylpenicillin, 2.4 g every 4 h for 57 days or
IV cefotaxime.

Do nothing pending blood cultures and CSF analysis (Option A) is incorrect. Where there is a
non-blanching rash, current guidelines advocate pre-hospital immediate antibiotic
administration.

https://mypastest.pastest.com/Secure/TestMe/Browser/429893#Top

1/2

8/11/2016

MyPastest

Give bolus IV cefotaxime (Option B) is incorrect. Although this may be effective, current
guidelines advocate benzylpenicillin as first-line therapy pre-hospital.
Give single high-dose oral benzylpenicillin (Option D) is incorrect. Benzylpenicillin is
ineffective when administered orally.
Give single high-dose oral cephalosporin (Option E) is incorrect. Oral antibiotics would be
inappropriate owing to slower onset and lower bioavailability.
46735

Next Question

Previous Question

Tag Question

Feedback

End Review

Difficulty: Average
Peer Responses

Session Progress
Responses Correct:

Responses Incorrect:

207

Responses Total:

207

Responses - % Correct:

0%

Blog (https://www.pastest.com/blog) About Pastest (https://www.pastest.com/about-us)


Contact Us (https://www.pastest.com/contact-us) Help (https://www.pastest.com/help)
Pastest 2016

https://mypastest.pastest.com/Secure/TestMe/Browser/429893#Top

2/2

8/11/2016

MyPastest

Back to Filters (/Secure/TestMe/Filter/429893/QA)

Question 17 of 207

You are asked to add a new medication to the medication chart of a patient receiving
warfarin and you are concerned about a possible drug interaction.
Which one of the following drugs is least likely to induce cytochrome P450?
A

Carbamazepine

Ethanol

Paracetamol

Phenytoin

Rifampicin

Explanation

The answer is Paracetamol


Paracetamol is partly metabolised by the P450 system but it has no significant effect on
enzyme activity.

Carbamazepine (Option A) is incorrect. Carbamazepine is a powerful inducer of hepatic


enzyme activity.
Ethanol (Option B) is incorrect. Chronic alcohol intake causes upregulation of hepatic
enzyme activity; acute intake may inhibit enzyme activity.
Phenytoin (Option D) is incorrect. Phenytoin is a hepatic enzyme inducer.
Rifampicin (Option E) is incorrect. Rifampicin is a powerful inducer of the P450 enzyme
system.
46957

Next Question

https://mypastest.pastest.com/Secure/TestMe/Browser/429893#Top

1/2

8/11/2016

MyPastest

Previous Question

Tag Question

Feedback

End Review

Difficulty: Average
Peer Responses

Session Progress
Responses Correct:

Responses Incorrect:

207

Responses Total:

207

Responses - % Correct:

0%

Blog (https://www.pastest.com/blog) About Pastest (https://www.pastest.com/about-us)


Contact Us (https://www.pastest.com/contact-us) Help (https://www.pastest.com/help)
Pastest 2016

https://mypastest.pastest.com/Secure/TestMe/Browser/429893#Top

2/2

8/11/2016

MyPastest

Back to Filters (/Secure/TestMe/Filter/429893/QA)

Question 18 of 207

A 56-year-old man is entering his sixth year of combination antiretroviral treatment for HIV
infection. Over the past few years he has noticed increasing gynaecomastia and abdominal
fat, and his partner has complained that he appears to be acquiring a buffalo hump. Routine
clinical examination reveals a blood pressure of 160/85 mmHg, and a glucose concentration
of 16.1 mmol/l (36).
What diagnosis best fits with this clinical picture?
A

Antiretroviral insulin-resistance syndrome

Cushings disease

Impaired glucose tolerance

Metabolic syndrome

Type-2 diabetes

Explanation

The answer is Antiretroviral insulin-resistance syndrome


Long-term use of combination antiretroviral therapy including protease inhibitor
regimens is associated with a redistribution of body fat in some patients. HIV
lipodystrophy follows an insulin-resistance pattern, with a loss of fat on the face,
increasing abdominal fat and deposition of subcutaneous fat on the back. Other features
of the insulin-resistance (metabolic syndrome) also occur, including low high-density
lipoprotein (HDL) cholesterol and high triglyceride levels, hypertension and impaired
glucose tolerance, often preceding the development of type-2 diabetes. There is some
evidence to suggest that insulin sensitisers (glitazones) may be an effective treatment
for HIV lipodystrophy.

Cushings disease (Option B) is incorrect. The clinical features are strongly suggestive of
Cushing syndrome but the history makes antiretroviral insulin-resistance syndrome a better
answer. Cushings disease refers to functional adrenal adenoma.

https://mypastest.pastest.com/Secure/TestMe/Browser/429893#Top

1/2

8/11/2016

MyPastest

Impaired glucose tolerance (Option C) is incorrect. The high blood glucose is more likely to
indicate underlying type-2 diabetes than impaired glucose tolerance.
Metabolic syndrome (Option D) is incorrect. Although the patients features are consistent
with metabolic syndrome, this diagnosis alone would not explain the changes to breast and
fat tissue.
Type-2 diabetes (Option E) is incorrect. Although the high blood glucose concentration
establishes a diagnosis of type-2 diabetes, this only explains part of the clinical features and
offers a less complete answer than antiretroviral insulin-resistance syndrome.
46884

Next Question

Previous Question

Tag Question

Feedback

End Review

Difficulty: Average
Peer Responses

Session Progress
Responses Correct:

Responses Incorrect:

207

Responses Total:

207

Responses - % Correct:

0%

Blog (https://www.pastest.com/blog) About Pastest (https://www.pastest.com/about-us)


Contact Us (https://www.pastest.com/contact-us) Help (https://www.pastest.com/help)
Pastest 2016

https://mypastest.pastest.com/Secure/TestMe/Browser/429893#Top

2/2

8/11/2016

MyPastest

Back to Filters (/Secure/TestMe/Filter/429893/QA)

Question 19 of 207

A 61-year-old man comes to the clinic for a check-up some 6 weeks after his inferior
myocardial infarction. Current medication includes ramipril, bisoprolol, aspirin and
simvastatin. He asks for advice about when the best time is to take his simvastatin.
At what time would simvastatin therapy be expected to give the greatest cholesterollowering effect?
A

After breakfast

After evening meal

First thing in the morning

Last thing at night

Timing doesnt matter

Explanation

The answer is Last thing at night


Simvastatin inhibits 3-hydroxy-3-methyl-glutaryl coA reductase (HMG-coA reductase).
HMG-coA reductase activity is greatest at night in the fasting state and when liver blood
flow increases owing to resting in the supine position. Simvastatin works by inhibiting
production of low-density lipoprotein (LDL) cholesterol by the liver.

After breakfast (Option A) is incorrect. Simvastatin may still inhibit cholesterol synthesis, but
synthetic activity is generally lower in the daytime in any case, suppressed by dietary
cholesterol intake. Simvastatin is metabolised by CYP3A4, which is competitively inhibited by
a number of substances, including grapefruit juice.
After evening meal (Option B) is incorrect. Simvastatin may still inhibit cholesterol synthesis,
but synthetic activity is generally suppressed by dietary cholesterol intake.
First thing in the morning (Option C) is incorrect. Simvastatin may still inhibit cholesterol
synthesis, but synthetic activity is generally lower in the daytime in any case, suppressed by
dietary cholesterol intake.
https://mypastest.pastest.com/Secure/TestMe/Browser/429893#Top

1/2

8/11/2016

MyPastest

Timing doesnt matter (Option E) is incorrect. Timing at bedtime is likely to give a better
overall reduction in circulating cholesterol concentrations; night-time simvastatin dosing was
used in the clinical trials that demonstrated efficacy in reducing cardiovascular events.
46995

Next Question

Previous Question

Tag Question

Feedback

End Review

Difficulty: Average
Peer Responses

Session Progress
Responses Correct:

Responses Incorrect:

207

Responses Total:

207

Responses - % Correct:

0%

Blog (https://www.pastest.com/blog) About Pastest (https://www.pastest.com/about-us)


Contact Us (https://www.pastest.com/contact-us) Help (https://www.pastest.com/help)
Pastest 2016

https://mypastest.pastest.com/Secure/TestMe/Browser/429893#Top

2/2

8/11/2016

MyPastest

Back to Filters (/Secure/TestMe/Filter/429893/QA)

Question 20 of 207

A 67-year-old man is admitted to the Acute Medical Unit with breathlessness due to
pulmonary oedema. He has a background history of metastatic cancer and has previously
been treated with several cycles of chemotherapy.
Which one of the following cytotoxic agents is most frequently associated with the
development of cardiotoxicity?
A

Bleomycin

Cisplatin

Cyclophosphamide

Doxorubicin

Vincristine

Explanation

The answer is Doxorubicin


Anthracyclines (doxorubicin, daunorubicin) are particularly known to produce
cumulative, dose-dependent cardiotoxicity, which manifests as impaired left ventricular
function and congestive cardiac failure.

Bleomycin (Option A) is incorrect. Bleomycin characteristically causes progressive interstitial


pulmonary fibrosis with bibasal pulmonary infiltrate seen on chest radiography.
Cisplatin (Option B) is incorrect. cis-Platinum (cisplatin) is a heavy metal compound often
used for testicular and ovarian tumours. It causes nephrotoxicity in 30% of patients due to
damage to distal tubules and collecting ducts, which may lead to persistent magnesium
wasting.
Cyclophosphamide (Option C) is incorrect. Cyclophosphamide is a prodrug of an alkylating
agent acrolein; it is metabolised in the liver to acrolein, which causes haemorrhagic cystitis.
Vincristine (Option E) is incorrect. Vincristine is a vinca alkaloid which, when used on a
weekly basis, carries a risk of progressive peripheral neuropathy. After treatment
https://mypastest.pastest.com/Secure/TestMe/Browser/429893#Top

1/2

8/11/2016

MyPastest

discontinuation, there may be some degree of recovery, especially improvement of sensory


changes rather than motor neuropathy.
46632

Next Question

Previous Question

Tag Question

Feedback

End Review

Difficulty: Average
Peer Responses

Session Progress
Responses Correct:

Responses Incorrect:

207

Responses Total:

207

Responses - % Correct:

0%

Blog (https://www.pastest.com/blog) About Pastest (https://www.pastest.com/about-us)


Contact Us (https://www.pastest.com/contact-us) Help (https://www.pastest.com/help)
Pastest 2016

https://mypastest.pastest.com/Secure/TestMe/Browser/429893#Top

2/2

8/11/2016

MyPastest

Back to Filters (/Secure/TestMe/Filter/429893/QA)

Question 21 of 207

A 17-year-old male is admitted to the Emergency Department. He volunteers that he has


taken some 40 paracetamol tablets around 5 h earlier, after splitting up from his girlfriend.
His paracetamol levels are in the range for N-acetylcysteine treatment and you commence
this.
Which one of the following is the main mechanism by which N-acetylcysteine prevents liver
damage?
A

It depletes cysteine

It depletes glutathione

It replenishes cysteine

It replenishes glutathione

It replenishes lysine

Explanation

The answer is It replenishes glutathione


The major mode of action of N-acetylcysteine is thought to be as a precursor for
glutathione, which acts as a sulfhydryl donor to allow conjugation with N-acetyl-pbenzoquinone imine (NAPQI), the toxic metabolite of paracetamol. Patients with protein
malnutrition who have fewer sulfhydryl groups available for conjugation or those on
hepatic enzyme inducers such as alcohol are at higher risk from paracetamol toxicity.

It depletes cysteine (Option A) is incorrect. It increases intracellular cysteine.


It depletes glutathione (Option B) is incorrect. It increases intracellular stores.
It replenishes cysteine (Option C) is incorrect. Acetylcysteine enters hepatocytes and is
converted to cysteine, so that treatment replenishes cysteine concentrations and allows
restoration of glutathione concentrations. Replenishment of glutathione is the preferred
answer because it is the glutathione depletion that permits liver injury and the therapeutic
effect is from glutathione replenishment.
https://mypastest.pastest.com/Secure/TestMe/Browser/429893#Top

1/2

8/11/2016

MyPastest

It replenishes lysine (Option E) is incorrect. There is no direct effect on lysine concentrations.


46974

Next Question

Previous Question

Tag Question

Feedback

End Review

Difficulty: Average
Peer Responses

Session Progress
Responses Correct:

Responses Incorrect:

207

Responses Total:

207

Responses - % Correct:

0%

Blog (https://www.pastest.com/blog) About Pastest (https://www.pastest.com/about-us)


Contact Us (https://www.pastest.com/contact-us) Help (https://www.pastest.com/help)
Pastest 2016

https://mypastest.pastest.com/Secure/TestMe/Browser/429893#Top

2/2

8/11/2016

MyPastest

Back to Filters (/Secure/TestMe/Filter/429893/QA)

Question 22 of 207

You are treating a 56-year-old man with diabetes and hypertension and trying to optimise his
medication list.
Which of the following drugs is most likely to produce large clinical effects for a relatively
small change in dose?
A

Bendrofluazide

Furosemide

Hydrochlorothiazide

Losartan

Pioglitazone

Explanation

The answer is Furosemide


The doseresponse curve for loop diuretics such as furosemide is progressive over a
broad range of dosages, indicating that changes in dose are expected to alter clinical
drug effects. This is in contrast to other medications where a flat doseresponse
relationship exists, and modest changes in drug dose have negligible effects of drug
response, eg thiazide diuretics, ACE inhibitors.

Bendrofluazide (Option A) is incorrect. The doseresponse relationship for thiazides and


blood pressure lowering reaches a plateau after the use of relatively low doses. In contrast,
there is a linear relationship between thiazide dose and adverse effects, including
hypokalaemia. Therefore, increasing the dose of bendroflumethiazide beyond 2.5 mg daily
offers little therapeutic benefit but will increase the risk of adverse effects.
Hydrochlorothiazide (Option C) is incorrect. The doseresponse relationship for thiazides and
blood pressure lowering reaches a plateau after the use of relatively low doses. In contrast,
there is a linear relationship between thiazide dose and adverse effects, including
hypokalaemia. Therefore, increasing the drug dose offers little therapeutic benefit but will
increase the risk of adverse effects.
https://mypastest.pastest.com/Secure/TestMe/Browser/429893#Top

1/2

8/11/2016

MyPastest

Losartan (Option D) is incorrect. ACE inhibitors and angiotensin receptor blockers are known
to observe a comparatively flat doseresponse relationship, and doses higher than the
therapeutic range offer little additional blood pressure lowering effect but increase the risk
of nephrotoxicity.
Pioglitazone (Option E) is incorrect. The doseresponse curve reaches a plateau for
pioglitazone at above 45 mg; there would be negligible additional blood glucose control
achieved by administered doses higher than the licensed dose range of 3045 mg.
46853

Next Question

Previous Question

Tag Question

Feedback

End Review

Difficulty: Average
Peer Responses

Session Progress
Responses Correct:

Responses Incorrect:

207

Responses Total:

207

Responses - % Correct:

0%

Blog (https://www.pastest.com/blog) About Pastest (https://www.pastest.com/about-us)


Contact Us (https://www.pastest.com/contact-us) Help (https://www.pastest.com/help)
Pastest 2016

https://mypastest.pastest.com/Secure/TestMe/Browser/429893#Top

2/2

8/11/2016

MyPastest

Back to Filters (/Secure/TestMe/Filter/429893/QA)

Question 23 of 207

A 53-year-old man is admitted in an intoxicated state having drunk a large quantity of


methanol. His investigations show that the osmolal gap is raised, and there is a metabolic
acidosis. You decide to administer fomepizole antidote.
When treating a methanol overdose with fomepizole, which of the following describes the
pharmacological action of fomepizole?
A

Competitive agonism

Competitive inhibition

Non-competitive agonism

Non-competitive inhibition

Partial agonism

Explanation

The answer is Competitive inhibition


Methanol is metabolised by alcohol dehydrogenase to formaldehyde and thence to
formic acid, which leads to profound acidosis, renal failure and blindness. Fomepizole (4methylpyrazole) is a competitive inhibitor of alcohol dehydrogenase and so is used to
treat methanol and ethylene glycol poisoning. Ethanol also competes for metabolism by
alcohol dehydrogenase and can lessen the extent of metabolism of methanol or ethylene
glycol; however, fomepizole is more effective and less toxic.

Competitive agonism (Option A) is incorrect. Fomepizole is a competitive inhibitor.


Non-competitive agonism (Option C) is incorrect. Fomepizole is a competitive inhibitor.
Non-competitive inhibition (Option D) is incorrect. Fomepizole is a competitive inhibitor.
Partial agonism (Option E) is incorrect. Fomepizole is a competitive inhibitor.
46988

Next Question
https://mypastest.pastest.com/Secure/TestMe/Browser/429893#Top

1/2

8/11/2016

MyPastest

Previous Question

Tag Question

Feedback

End Review

Difficulty: Average
Peer Responses

Session Progress
Responses Correct:

Responses Incorrect:

207

Responses Total:

207

Responses - % Correct:

0%

Blog (https://www.pastest.com/blog) About Pastest (https://www.pastest.com/about-us)


Contact Us (https://www.pastest.com/contact-us) Help (https://www.pastest.com/help)
Pastest 2016

https://mypastest.pastest.com/Secure/TestMe/Browser/429893#Top

2/2

8/11/2016

MyPastest

Back to Filters (/Secure/TestMe/Filter/429893/QA)

Question 24 of 207

A child is born with an open spina bifida despite adequate folate intake by the mother during
pregnancy. The mother is a known epileptic and had elected to continue her routine
antiepileptic medications during pregnancy, after the pros and cons had been discussed.
Which one of the following drugs would be most likely to cause this teratogenic effect?
A

Carbamazepine

Phenobarbital

Phenytoin

Primidone

Sodium valproate

Explanation

The answer is Sodium valproate


Administration of sodium valproate during pregnancy may predispose to the
development of neural tube defects in the fetus, including spina bifida.

Carbamazepine (Option A) is incorrect. Carbamazepine may also cause spina bifida as a


teratogenic effect, although less commonly than sodium valproate, hence option E is the
preferred option.
Phenobarbital (Option B) is incorrect. Phenobarbital causes withdrawal symptoms in the
newborn; spina bifida is not a recognised adverse outcome.
Phenytoin (Option C) is incorrect. Phenytoin causes the fetal hydantoin syndrome with facial
dysmorphism.
Primidone (Option D) is incorrect. Primidone causes withdrawal symptoms in the newborn;
spina bifida is not a recognised adverse outcome.
46753

Next Question
https://mypastest.pastest.com/Secure/TestMe/Browser/429893#Top

1/2

8/11/2016

MyPastest

Previous Question

Tag Question

Feedback

End Review

Difficulty: Average
Peer Responses

Session Progress
Responses Correct:

Responses Incorrect:

207

Responses Total:

207

Responses - % Correct:

0%

Blog (https://www.pastest.com/blog) About Pastest (https://www.pastest.com/about-us)


Contact Us (https://www.pastest.com/contact-us) Help (https://www.pastest.com/help)
Pastest 2016

https://mypastest.pastest.com/Secure/TestMe/Browser/429893#Top

2/2

8/11/2016

MyPastest

Back to Filters (/Secure/TestMe/Filter/429893/QA)

Question 25 of 207

An 18-year-old boy is brought by ambulance to the Emergency Department. He has had a row
with his father who found him in his bedroom some 2 h later in an unrousable state. It is
known that his father takes tablets for blood pressure. On examination the patient has a pulse
of 42 bpm and a blood pressure of 74/40 mmHg.
Which of the following is the most appropriate treatment for this patient?
A

External pacing

Intravenous glucagon

Isoprenaline infusion

Repeated small doses of adrenaline

Temporary pacing wire

Explanation

The answer is Intravenous glucagon


The prominent features in the scenario are bradycardia and hypotension, suggesting a blocker overdose. Where there is profound hypotension, as in this case, treatment is with
intravenous glucagon, bolus 10 mg, followed by intravenous infusion. Glucagon exerts an
inotropic effect independent of -receptor activation and raises myocardial cAMP levels.
Bradycardia may be corrected by glucagon; other treatments include atropine,
isoprenaline infusion and insulin. Hypoglycaemia may also occur in association with blocker overdose bolus doses of intravenous 50% dextrose followed by 10% dextrose
infusion is the management of choice.

External pacing (Option A) is incorrect. External pacing would only be considered if the
patient fails to respond to glucagon and other medical therapies.
Isoprenaline infusion (Option C) is incorrect. Intravenous isoprenaline infusion may be
considered as a means of correcting bradycardia, normally only when patients have failed to
respond to glucagon.

https://mypastest.pastest.com/Secure/TestMe/Browser/429893#Top

1/2

8/11/2016

MyPastest

Repeated small doses of adrenaline (Option D) is incorrect. Small doses of adrenaline are less
likely to be successful than glucagon, given that the -receptors often become irreversibly
blocked in the setting of -blocker overdose.
Temporary pacing wire (Option E) is incorrect. A temporary pacing wire may be needed
when glucagon and other medical treatments fail.
46951

Next Question

Previous Question

Tag Question

Feedback

End Review

Difficulty: Average
Peer Responses

Session Progress
Responses Correct:

Responses Incorrect:

207

Responses Total:

207

Responses - % Correct:

0%

Blog (https://www.pastest.com/blog) About Pastest (https://www.pastest.com/about-us)


Contact Us (https://www.pastest.com/contact-us) Help (https://www.pastest.com/help)
Pastest 2016

https://mypastest.pastest.com/Secure/TestMe/Browser/429893#Top

2/2

8/11/2016

MyPastest

Back to Filters (/Secure/TestMe/Filter/429893/QA)

Question 26 of 207

A 60-year-old lady presents with a 5-day history of malaise, mild jaundice and abdominal
discomfort. Transaminases are markedly raised and you diagnose a possible drug-induced
hepatitis.
Which of the following drugs would be most likely to be responsible?
A

Amiodarone

Chlorpromazine

Ethinylestradiol

Isoprenaline

Tetracyline

Explanation

The answer is Amiodarone


Drug-induced hepatitis may occur after high drug doses, eg azathioprine, isoniazid and
pyrazinamide (note that it is the metabolite of isoniazid that causes hepatitis, and it is
more likely to occur in patients that have a slow acetylator genotype). In some cases the
occurrence of hepatitis is less clearly related to dose, eg amiodarone or nifedipine.
Methyldopa may cause an autoimmune hepatitis.

Chlorpromazine (Option B) is incorrect. Chlorpromazine causes cholestatic jaundice rather


than drug-induced hepatitis.
Ethinylestradiol (Option C) is incorrect. Oestrogens may interfere with bilirubin excretion
giving rise to jaundice without hepatitis.
Isoprenaline (Option D) is incorrect. Isoprenaline is not known to cause hepatitis.
Tetracyline (Option E) is incorrect. Tetracyclines may cause dose-dependent fatty change in
the liver.
46907

https://mypastest.pastest.com/Secure/TestMe/Browser/429893#Top

1/2

8/11/2016

MyPastest

Next Question

Previous Question

Tag Question

Feedback

End Review

Difficulty: Average
Peer Responses

Session Progress
Responses Correct:

Responses Incorrect:

207

Responses Total:

207

Responses - % Correct:

0%

Blog (https://www.pastest.com/blog) About Pastest (https://www.pastest.com/about-us)


Contact Us (https://www.pastest.com/contact-us) Help (https://www.pastest.com/help)
Pastest 2016

https://mypastest.pastest.com/Secure/TestMe/Browser/429893#Top

2/2

8/11/2016

MyPastest

Back to Filters (/Secure/TestMe/Filter/429893/QA)

Question 27 of 207

The blood test results of one of your clinic patients show serum potassium 3.4 mmol/l and
serum bicarbonate 15 mmol/l.
Which one of the following diuretics is most likely to account for these biochemical findings?
A

Acetazolamide

Amiloride

Bumetanide

Furosemide

Metolazone

Explanation

The answer is Acetazolamide


The prominent finding is a metabolic acidosis, and very borderline hypokalaemia.
Acetazolamide (carbonic anhydrase inhibitor) inhibits proximal tubule bicarbonate
reabsorption in a similar fashion to type-2 renal tubular acidosis (RTA). In contrast, loop
diuretics and thiazide diuretics are associated with significant renal potassium and
hydrogen ion loss, associated with a metabolic alkalosis.

Amiloride (Option B) is incorrect. Amiloride may also cause a metabolic acidosis by


inhibiting the sodium channel in the collecting duct, which inhibits renal acid secretion or
bicarbonate reabsorption; however, the metabolic acidosis is usually less prominent than
acetazolamide, which is why acetazolamide is the preferred answer.
Bumetanide (Option C) is incorrect. Loop diuretics promote sodium chloride loss and cause
volume depletion, and are characteristically associated with metabolic alkalosis.
Furosemide (Option D) is incorrect. Loop diuretics promote sodium chloride loss and cause
volume depletion, and are characteristically associated with metabolic alkalosis.
Metolazone (Option E) is incorrect. Metolazone produces significant hypokalaemia, and tends
to cause metabolic acidosis.
46626

https://mypastest.pastest.com/Secure/TestMe/Browser/429893#Top

1/2

8/11/2016

MyPastest

46626

Next Question

Previous Question

Tag Question

Feedback

End Review

Difficulty: Average
Peer Responses

Session Progress
Responses Correct:

Responses Incorrect:

207

Responses Total:

207

Responses - % Correct:

0%

Blog (https://www.pastest.com/blog) About Pastest (https://www.pastest.com/about-us)


Contact Us (https://www.pastest.com/contact-us) Help (https://www.pastest.com/help)
Pastest 2016

https://mypastest.pastest.com/Secure/TestMe/Browser/429893#Top

2/2

8/11/2016

MyPastest

Back to Filters (/Secure/TestMe/Filter/429893/QA)

Question 28 of 207

You see a 53-year-old woman in the endocrinology clinic who is receiving treatment for
Graves disease. She is concerned about the possible adverse effects of her medications.
Which one of the following is most likely to have been caused by carbimazole?
A

Acne

Alopecia

Aplastic anaemia

Hypertrichosis

Tremor

Explanation

The answer is Alopecia


Carbimazole is used to treat hyperthyroidism. Its side-effects include nausea, rash,
pruritus, arthralgia, alopecia, agranulocytosis and jaundice. Patients taking carbimazole
must be warned of the danger of neutropaenia and are advised to seek medical attention
quickly if they develop a sore throat or other symptoms of infection.

Acne (Option A) is incorrect. Acne is a recognised complication of phenytoin and certain


hormone modulators.
Aplastic anaemia (Option C) is incorrect. Agranulocytosis may occur, but aplastic anaemia is
not a characteristic adverse effect.
Hypertrichosis (Option D) is incorrect. Hypertrichosis is a recognised adverse effect of
minoxidil and ciclosporin.
Tremor (Option E) is incorrect. Tremor is a characteristic adverse effect of excess thyroid
hormone, 2-adrenoceptor agonists (e.g. salbutamol), theophylline and lithium.
46970

Next Question
https://mypastest.pastest.com/Secure/TestMe/Browser/429893#Top

1/2

8/11/2016

MyPastest

Previous Question

Tag Question

Feedback

End Review

Difficulty: Average
Peer Responses

Session Progress
Responses Correct:

Responses Incorrect:

207

Responses Total:

207

Responses - % Correct:

0%

Blog (https://www.pastest.com/blog) About Pastest (https://www.pastest.com/about-us)


Contact Us (https://www.pastest.com/contact-us) Help (https://www.pastest.com/help)
Pastest 2016

https://mypastest.pastest.com/Secure/TestMe/Browser/429893#Top

2/2

8/11/2016

MyPastest

Back to Filters (/Secure/TestMe/Filter/429893/QA)

Question 29 of 207

When administering narcotic analgesics to patients with renal failure, which one of the
following is true?
A

Opiates are safe and no dosage adjustment is necessary

Opiate metabolites are excreted in the same way as in a patient with normal renal
function

Prolonged elimination half-lives of both parent compound and metabolites


contribute to an increase in the risk of adverse effects

Therapeutic monitoring of morphine levels is commonly performed

Use of opiates is absolutely contraindicated

Explanation

The answer is Prolonged elimination half-lives of both parent compound and metabolites
contribute to an increase in the risk of adverse effects
Opiates and their metabolites have prolonged elimination half-lives and accumulate in
renal failure causing adverse effects. They should be used with caution, and the dose
reduced and/or dosing interval increased.

Opiates are safe and no dosage adjustment is necessary (Option A) is incorrect. Adjustment
of dose and/or frequency is often needed.
Opiate metabolites are excreted in the same way as in a patient with normal renal function
(Option B) is incorrect. Opioid metabolites, eg morphine-6-sulfate, may accumulate in
patients with renal impairment.
Therapeutic monitoring of morphine levels is commonly performed (Option D) is incorrect.
Blood concentrations of drug and metabolites correlate poorly with clinical features of
opioid toxicity, so that therapeutic drug monitoring is inappropriate.
Use of opiates is absolutely contraindicated (Option E) is incorrect. Opioids may be used, but
with increased caution.
46956

https://mypastest.pastest.com/Secure/TestMe/Browser/429893#Top

1/2

8/11/2016

MyPastest

46956

Next Question

Previous Question

Tag Question

Feedback

End Review

Difficulty: Average
Peer Responses

Session Progress
Responses Correct:

Responses Incorrect:

207

Responses Total:

207

Responses - % Correct:

0%

Blog (https://www.pastest.com/blog) About Pastest (https://www.pastest.com/about-us)


Contact Us (https://www.pastest.com/contact-us) Help (https://www.pastest.com/help)
Pastest 2016

https://mypastest.pastest.com/Secure/TestMe/Browser/429893#Top

2/2

8/11/2016

MyPastest

Back to Filters (/Secure/TestMe/Filter/429893/QA)

Question 30 of 207

You are asked to review a 59-year-old man who has been referred to the hypertension clinic
for advice. He had attended the Emergency Department 2 weeks earlier due to a sudden
attack of flushing, swelling of his tongue and airway, and hypotension. He has a history of
hypertension for which he takes amlodipine and enalapril.
What would be the best long-term course of action?
A

Add regular treatment with an H1 blocker

Add regular treatment with an H2 blocker

Stop his amlodipine tablets and substitute another antihypertensive agent

Stop his enalapril tablets and substitute another antihypertensive agent

Train him how to use an EpiPen

Explanation

The answer is Stop his enalapril tablets and substitute another antihypertensive agent Enalapril and other angiotensin converting enzyme (ACE) inhibitors, and angiotensin-IIreceptor blockers are associated with the occurrence of angioedema, characterised by
sweating, flushing, hypotension, facial oedema, airway oedema and respiratory distress.
Treatment involves discontinuation of ACE inhibitor therapy.

Add regular treatment with an H1 blocker (Option A) is incorrect. The causative agent must
be stopped, which takes priority over other treatments, including antihistamines for ongoing
symptoms.
Add regular treatment with an H2 blocker (Option B) is incorrect. The causative agent must
be stopped, which takes priority over other treatments, including antihistamines for ongoing
symptoms.
Stop his amlodipine tablets and substitute another antihypertensive agent (Option C) is
incorrect. Although angioedema may also be seen with a range of other drugs, including in
case reports with calcium antagonists, the frequency is much higher with agents working via
the ACE pathway.
https://mypastest.pastest.com/Secure/TestMe/Browser/429893#Top

1/2

8/11/2016

MyPastest

Train him how to use an EpiPen (Option E) is incorrect. ACE-inhibitor-induced angioedema is


unlikely to recur provided that the patient avoids any further exposure to ACE inhibitors or
angiotensin receptor blockers.
46838

Next Question

Previous Question

Tag Question

Feedback

End Review

Difficulty: Average
Peer Responses

Session Progress
Responses Correct:

Responses Incorrect:

207

Responses Total:

207

Responses - % Correct:

0%

Blog (https://www.pastest.com/blog) About Pastest (https://www.pastest.com/about-us)


Contact Us (https://www.pastest.com/contact-us) Help (https://www.pastest.com/help)
Pastest 2016

https://mypastest.pastest.com/Secure/TestMe/Browser/429893#Top

2/2

8/11/2016

MyPastest

Back to Filters (/Secure/TestMe/Filter/429893/QA)

Question 31 of 207

You are asked to see a 78-year-old man, a nursing-home resident who has recently moved
into the care home due to progressive Alzheimers disease. He has had several subacute
confusional episodes since his arrival, for which the duty GP has been called twice in the past
month, and he has been prescribed an antipsychotic to reduce his agitation. Past history of
note includes previous alcoholism and an episode of biliary colic many years ago. He reports
no abdominal pain. On examination he is deeply jaundiced. On blood testing, his alkaline
phosphatase activity and bilirubin concentration are markedly raised.
What diagnosis fits best with this clinical picture?
A

Acute pancreatitis

Cholecystitis

Pancreatic carcinoma

Phenothiazine-related hepatotoxicity

Renewed excess alcohol consumption

Explanation

The answer is Phenothiazine-related hepatotoxicity


Adverse drug effects Phenothiazines such as chlorpromazine can produce
hepatocanalicular cholestasis (cholestatic jaundice) owing to a hypersensitivity reaction.
It usually occurs within 1 month of starting the drug, in around 1% of patients. Other
drugs that cause similar pathophysiology include erythromycin, cimetidine, ciclosporin,
nifedipine, nitrofurantoin, imipramine, azathioprine and dextropropoxyphene. Hepatic
adenomas, or even hepatocellular carcinoma, may occur in cases of prolonged use of
danazol or high-hormone-content combined oral contraceptive medication.

Acute pancreatitis (Option A) is incorrect. The absence of abdominal pain or fever makes this
diagnosis unlikely.
Cholecystitis (Option B) is incorrect. The absence of abdominal pain or fever makes this
diagnosis unlikely.
https://mypastest.pastest.com/Secure/TestMe/Browser/429893#Top

1/2

8/11/2016

MyPastest

Pancreatic carcinoma (Option C) is incorrect. Pancreatic cancer may cause obstructive


jaundice, which is typically painless. Although the diagnosis is possible, it is uncommon and
less likely than phenothiazine-related hepatotoxicity.
Renewed excess alcohol consumption (Option E) is incorrect. Care home residents may
continue to drink alcohol, but alcoholic hepatitis would be less likely given the absence of
any other signs of liver disease.
46878

Next Question

Previous Question

Tag Question

Feedback

End Review

Difficulty: Average
Peer Responses

Session Progress
Responses Correct:

Responses Incorrect:

207

Responses Total:

207

Responses - % Correct:

0%

Blog (https://www.pastest.com/blog) About Pastest (https://www.pastest.com/about-us)


Contact Us (https://www.pastest.com/contact-us) Help (https://www.pastest.com/help)
Pastest 2016

https://mypastest.pastest.com/Secure/TestMe/Browser/429893#Top

2/2

8/11/2016

MyPastest

Back to Filters (/Secure/TestMe/Filter/429893/QA)

Question 32 of 207

You review a 68-year-old man in the Outpatient Neurology Clinic who has been referred with
a diagnosis of suspected Parkinsons disease. After discussion with the patient and his family,
you decide to initiate levodopa therapy.
Which one of the following statements best describes the pharmacological effects of
levodopa?
A

Can be given subcutaneously

Causes postural hypotension due to direct vascular effect

Co-administration of benserazide increases the central nervous system levels of


dopamine by inhibiting dopa-decarboxylase in the substantia nigra

It is an enzyme inducer

Undergoes metabolism in the central but not the peripheral nervous system

Explanation

The answer is Causes postural hypotension due to direct vascular effect Levodopa is a pro-drug that crosses the bloodbrain barrier and is then converted to
dopamine. The direct effects of circulating levodopa and dopamine include nausea,
arrhythmias and postural hypotension.

Can be given subcutaneously (Option A) is incorrect. Levodopa is available only in oral


formulations. Apomorphine is administered subcutaneously in severe Parkinsons disease.
Co-administration of benserazide increases the central nervous system (CNS) levels of
dopamine by inhibiting dopa-decarboxylase in the substantia nigra (Option C) is incorrect.
Benserazide is a peripheral dopa-decarboxylase inhibitor that permits higher quantities of
levodopa to cross the bloodbrain barrier; benserazide does not inhibit dopa-decarboxylase
activity within the CNS.
It is an enzyme inducer (Option D) is incorrect. Levodopa does not have a direct effect on
hepatic enzyme activity.
https://mypastest.pastest.com/Secure/TestMe/Browser/429893#Top

1/2

8/11/2016

MyPastest

Undergoes metabolism in the central but not the peripheral nervous system (Option E) is
incorrect. Levodopa is metabolised to dopa in the central and peripheral nervous system; the
intended therapeutic effect requires metabolism in the central nervous system. Peripheral
nervous system effects mediate adverse effects.
46958

Next Question

Previous Question

Tag Question

Feedback

End Review

Difficulty: Average
Peer Responses

Session Progress
Responses Correct:

Responses Incorrect:

207

Responses Total:

207

Responses - % Correct:

0%

Blog (https://www.pastest.com/blog) About Pastest (https://www.pastest.com/about-us)


Contact Us (https://www.pastest.com/contact-us) Help (https://www.pastest.com/help)
Pastest 2016

https://mypastest.pastest.com/Secure/TestMe/Browser/429893#Top

2/2

8/11/2016

MyPastest

Back to Filters (/Secure/TestMe/Filter/429893/QA)

Question 33 of 207

A 42-year-old patient who received a renal transplant some 4 months earlier comes to the
Emergency Department complaining of nausea, anorexia and lethargy, which have increased
over the past 2 weeks since a new medication was started for control of palpitations and
hypertension. She is taking a Ciclosporin based immunosuppressive regimen. On examination
her BP is 123/82 mmHg; pulse is 80/min and regular. Her BMI is 22. There is an abdominal scar
covering the recent transplant. Blood work reveals a 30% rise in creatinine over the past 2
weeks, and Ciclosporin above the upper limit of the normal range.
Which of the following is the most likely cause?
A

Amlodipine

Bisoprolol

Ramipril

Valsartan

Verapamil

Explanation
The answer is Verapamil It is important to be cautious with respect to introducing inhibitors of CYP 3A4 in conjunction
with Ciclosporin. Inhibitors of 3A4 where significant caution is advised include Verapamil,
Diltiazem, Imidazole anti-fungals, and macrolide antibiotics. Amlodipine is a substrate of 3A4,
rather than being a potent inhibitor or inducer. Bisoprolol undergoes dual P450 metabolism
via 2D6 and 3A4. Ramipril is metabolised to an active metabolite, ramiliprat, by the liver, but
does not lead to increased Ciclosporin levels. CYP 2C9 is the P450 enzyme most involved in
the metabolism of Valsartan.
target="_blank">http://www.medicines.org.uk/EMC/medicine/1307/SPC/Neoral+Soft+Gelatin+Capsules,+Neoral+Oral+Solution/
(http://www.medicines.org.uk/EMC/medicine/1307/SPC/Neoral+Soft+Gelatin+Capsules,+Neoral+Oral+Solution/#INTERACTION
40140

Next Question

Previous Question

Tag Question

Feedback

End Review

Difficulty: Difficult
Peer Responses

Session Progress

https://mypastest.pastest.com/Secure/TestMe/Browser/429893#Top

1/2

8/11/2016

MyPastest

Responses Correct:

Responses Incorrect:

207

Responses Total:

207

Responses - % Correct:

0%

Blog (https://www.pastest.com/blog) About Pastest (https://www.pastest.com/about-us)


Contact Us (https://www.pastest.com/contact-us) Help (https://www.pastest.com/help)
Pastest 2016

https://mypastest.pastest.com/Secure/TestMe/Browser/429893#Top

2/2

8/11/2016

MyPastest

Back to Filters (/Secure/TestMe/Filter/429893/QA)

Question 34 of 207

A patient with a history of angina is being investigated for dyspnoea. Blood tests confirm
haemolytic anaemia and a peripheral smear shows the presence of Heinz bodies and
methaemoglobinaemia.
Which of the following medications is most likely responsible for this complication?
A

Amlodipine

Aspirin

Isosorbide mononitrate

Metoprolol

Verapamil

Explanation

The answer is Isosorbine mononitrate


Methaemoglobinaemia results from the oxidation of ferrous iron in the haemoglobin to
the ferric form; it is associated with precipitation of ferric iron as Heinz bodies, and
eventually leads to haemolytic anaemia. Nitrates may cause this reaction, including amyl
nitrate.

Amlodipine (Option A) is incorrect. Amlodipine is not associated with


methaemoglobinaemia.
Aspirin (Option B) is incorrect. Aspirin is not associated with methaemoglobinaemia.
Metoprolol (Option D) is incorrect. Metoprolol is not associated with methaemoglobinaemia.
Verapamil (Option E) is incorrect. Verapamil is not associated with methaemoglobinaemia.
46867

Next Question

https://mypastest.pastest.com/Secure/TestMe/Browser/429893#Top

1/2

8/11/2016

MyPastest

Previous Question

Tag Question

Feedback

End Review

Difficulty: Average
Peer Responses

Session Progress
Responses Correct:

Responses Incorrect:

207

Responses Total:

207

Responses - % Correct:

0%

Blog (https://www.pastest.com/blog) About Pastest (https://www.pastest.com/about-us)


Contact Us (https://www.pastest.com/contact-us) Help (https://www.pastest.com/help)
Pastest 2016

https://mypastest.pastest.com/Secure/TestMe/Browser/429893#Top

2/2

8/11/2016

MyPastest

Back to Filters (/Secure/TestMe/Filter/429893/QA)

Question 35 of 207

An elderly, normotensive man with poor left ventricular function presents to the Emergency
Department with dizziness and palpitations. An ECG shows a broad-complex tachycardia.
Which of the following drugs would be the best choice of treatment?
A

Amiodarone

Flecainide

Lidocaine

Sotalol

Verapamil

Explanation

The answer is Amiodarone


This patient, a normotensive man with poor left ventricular function and a broadcomplex tachycardia, has ventricular tachycardia (VT). Amiodarone may be used to
cardiovert VT to sinus rhythm and to prevent ventricular fibrillation. Amiodarone may be
administered by repeated IV bolus doses, or continuous IV infusion.

Flecainide (Option B) is incorrect. Flecainide may cause ventricular fibrillation in patients


with ventricular tachycardia, particularly in patients with underlying heart failure and
ischaemic heart disease.
Lidocaine (Option C) is incorrect. In the presence of poor left ventricular function, lidocaine
and -blockers should not be used.
Sotalol (Option D) is incorrect. In the presence of poor left ventricular function, lidocaine and
-blockers should not be used.
Verapamil (Option E) is incorrect. Verapamil may precipitate a circulatory collapse in VT and
is therefore contraindicated in VT.
46741

https://mypastest.pastest.com/Secure/TestMe/Browser/429893#Top

1/2

8/11/2016

MyPastest

Next Question

Previous Question

Tag Question

Feedback

End Review

Difficulty: Average
Peer Responses

Session Progress
Responses Correct:

Responses Incorrect:

207

Responses Total:

207

Responses - % Correct:

0%

Blog (https://www.pastest.com/blog) About Pastest (https://www.pastest.com/about-us)


Contact Us (https://www.pastest.com/contact-us) Help (https://www.pastest.com/help)
Pastest 2016

https://mypastest.pastest.com/Secure/TestMe/Browser/429893#Top

2/2

8/11/2016

MyPastest

Back to Filters (/Secure/TestMe/Filter/429893/QA)

Question 36 of 207

A 51-year-old woman with a history of type-2 diabetes and bipolar disorder is admitted for
review because of low sodium (118 mmol/l). On examination her blood pressure is 139/72
mmHg, her pulse is 70 beats per minute, regular, and she is not in cardiac failure.
Investigations:
Hb

12.4 g/dl

White cell count 5.3 109/l


Platelets

190 109/l

Na+

118 mmol/l

K+

3.8 mmol/l

Creatinine

92 mol/l

HbA1c

57.38 mmol/mol (7.4%)

Fasting glucose

6.4 mmol/l

Which one of the following drugs is most likely to be responsible?


A

Moclobemide

Carbamazepine

Lithium

Gliclazide

Pioglitazone

Explanation
Drug-induced hyponatraemia

https://mypastest.pastest.com/Secure/TestMe/Browser/429893#Top

1/2

8/11/2016

MyPastest

As well as being used for the management of epilepsy, carbamazepine is used in the
management of bipolar disorder
While moclobemide may rarely be associated with hyponatraemia, carbamazepine
possesses effects similar to those of antidiuretic hormone and hence it is commonly
associated with hyponatraemia
In this case other options for treatment of bipolar disorder could be considered
It is unlikely that chronic fluid restriction would resolve the degree of hyponatraemia
seen here
20939

Next Question

Previous Question

Tag Question

Feedback

End Review

Difficulty: Difficult
Peer Responses

Session Progress
Responses Correct:

Responses Incorrect:

207

Responses Total:

207

Responses - % Correct:

0%

Blog (https://www.pastest.com/blog) About Pastest (https://www.pastest.com/about-us)


Contact Us (https://www.pastest.com/contact-us) Help (https://www.pastest.com/help)
Pastest 2016

https://mypastest.pastest.com/Secure/TestMe/Browser/429893#Top

2/2

8/11/2016

MyPastest

Back to Filters (/Secure/TestMe/Filter/429893/QA)

Question 37 of 207

You are asked to arrange some blood tests to monitor a patient who is receiving
amphotericin therapy, and you are considering which tests to arrange.
Which of the following metabolic disturbances is most likely to be associated with
amphotericin treatment?
A

Metabolic alkalosis

Hypermagnesaemia

Hypocalcaemia

Hypokalaemia

Hyponatraemia

Explanation

The answer is Hypokalaemia


Nephrotoxicity due to amphotericin is due to a tubular toxicity, and causes wasting of
potassium, and to a lesser extent sodium and magnesium. Metabolic acidosis may occur
due to type 1 (distal) renal tubular acidosis, and polyuria may occur due to nephrogenic
diabetes insipidus. Renal adverse effects generally occur in patients that have received
daily doses of more than 3 g, and are usually reversible on decreasing the dosage or
stopping treatment.

Metabolic alkalosis (Option A) is incorrect. Metabolic acidosis may occur (distal renal tubular
acidosis).
Hypermagnesaemia (Option B) is incorrect. Hypomagnesaemia is a recognised complication.
Hypocalcaemia (Option C) is incorrect. Hypocalcaemia is not a recognised feature.
Hyponatraemia (Option E) is incorrect. Hyponatraemia may occur, but is usually less
prominent than hypokalaemia, and therefore represents a less suitable answer in this case.
46864

https://mypastest.pastest.com/Secure/TestMe/Browser/429893#Top

1/2

8/11/2016

MyPastest

Next Question

Previous Question

Tag Question

Feedback

End Review

Difficulty: Average
Peer Responses

Session Progress
Responses Correct:

Responses Incorrect:

207

Responses Total:

207

Responses - % Correct:

0%

Blog (https://www.pastest.com/blog) About Pastest (https://www.pastest.com/about-us)


Contact Us (https://www.pastest.com/contact-us) Help (https://www.pastest.com/help)
Pastest 2016

https://mypastest.pastest.com/Secure/TestMe/Browser/429893#Top

2/2

8/11/2016

MyPastest

Back to Filters (/Secure/TestMe/Filter/429893/QA)

Question 38 of 207

A 62-year-old man who takes multiple medications for cardiovascular disease and diabetes
comes to the clinic for review. He has been complaining of a blue discolouration to his vision
over the past 5 weeks since his last medication review. Examination does not reveal any
significant change in acuity over the past 6 months and no abnormality is seen on
fundoscopy.
Which of the following medications he is taking is the most likely cause?
A

Amlodipine

Digoxin

Ramipril

Sildenafil

Sitagliptin

Explanation
The answer is Sildenafil Sildenafil is a PDE-5 inhibitor, but at high doses it also inhibits PDE-6, which leads to blue
discoloration of vision. This can often be managed by reducing the dose of Sildenafil. It is
seen across the class of PDE-5 inhibitors when they are used at high dose. Digoxin is
recognised at potentially toxic levels to lead to xanthopsia, or yellowing of vision. Reports of
hypoglycaemia and dizziness were made during the Sitagliptin clinical trial program, but
excess of visual system AEs or SAEs was reported. Diplopia is rarely reported in conjunction
with Amlodipine therapy, and conjunctivitis with Ramipril therapy.
<a
href="https://www.medicines.org.uk/emc/medicine/1474/SPC/Viagra+25mg,+50mg,+100mg/"
">https://www.medicines.org.uk/emc/medicine/1474/SPC/Viagra+25mg,+50mg,+100mg/
37798

Next Question

https://mypastest.pastest.com/Secure/TestMe/Browser/429893#Top

1/2

8/11/2016

MyPastest

Previous Question

Tag Question

Feedback

End Review

Difficulty: Average
Peer Responses

Session Progress
Responses Correct:

Responses Incorrect:

207

Responses Total:

207

Responses - % Correct:

0%

Blog (https://www.pastest.com/blog) About Pastest (https://www.pastest.com/about-us)


Contact Us (https://www.pastest.com/contact-us) Help (https://www.pastest.com/help)
Pastest 2016

https://mypastest.pastest.com/Secure/TestMe/Browser/429893#Top

2/2

8/11/2016

MyPastest

Back to Filters (/Secure/TestMe/Filter/429893/QA)

Question 39 of 207

You review a 28-year-old man who has been admitted in a state of collapse from a night club.
His friends admit that because of pressure at work he has been using increasing amounts of
cocaine recently.
Which of the following features is most commonly associated with cocaine?
A

Bradycardia

Hypotension

Hypothermia

Metabolic acidosis

Metabolic alkalosis

Explanation

The answer is Metabolic acidosis


Cocaine blocks the re-uptake of biogenic amines, and inhibition of dopamine re-uptake is
the cause of the psychomotor agitation that commonly accompanies cocaine use.
Inhibtion of noradrenaline (norepinephrine) reuptake causes excess sympathomimetic
activity; the euphoric effect is caused by inhibition of serotonin reuptake. Adverse effects
include agitation, tachycardia, hypertension, sweating, hallucinations and seizures.
Metabolic acidosis, hyperthermia, rhabdomyolysis, ventricular arrhythmias and
myocardial infarction are recognised. Regular use may be associated with premature
coronary artery disease, dilated cardiomyopathy and increased risk of cerebral
haemorrhage.

Bradycardia (Option A) is incorrect. Tachycardia is a more typical finding.


Hypotension (Option B) is incorrect. High blood pressure rather than hypotension is typical.
Hypothermia (Option C) is incorrect. Hyperthermia is a recognised feature, not hypothermia.
Metabolic alkalosis (Option E) is incorrect. It is metabolic acidosis, rather than than alkalosis.
46918

https://mypastest.pastest.com/Secure/TestMe/Browser/429893#Top

1/2

8/11/2016

MyPastest

Next Question

Previous Question

Tag Question

Feedback

End Review

Difficulty: Average
Peer Responses

Session Progress
Responses Correct:

Responses Incorrect:

207

Responses Total:

207

Responses - % Correct:

0%

Blog (https://www.pastest.com/blog) About Pastest (https://www.pastest.com/about-us)


Contact Us (https://www.pastest.com/contact-us) Help (https://www.pastest.com/help)
Pastest 2016

https://mypastest.pastest.com/Secure/TestMe/Browser/429893#Top

2/2

8/11/2016

MyPastest

Back to Filters (/Secure/TestMe/Filter/429893/QA)

Question 40 of 207

A 32-year-old patient was admitted with an acute overdose of lithium. She was immediately
started on an infusion of normal saline. On admission the lithium level was 2.4 mmol/l; after 10
h the lithium level was found to be 1.8 mmol/l.
Based upon the lithium concentrations, what interval might be most likely before lithium
concentrations are approaching an undetectable level?
A

4h

8h

12 h

24 h

72 h

Explanation

The answer is 72 h
The half-life of lithium is around 20 h, although this time period may be prolonged in the
elderly or in chronic lithium users. In this case, after 10 h the lithium level has fallen by
around 25%, consistent with a half-life of around 20 h. This is not, of course, a reliable
method because the fall in lithium concentrations is due to elimination plus distribution
from circulation to the tissues. Nonetheless, based on a crude estimate of half-life of 20
h, then lithium concentrations would be 1.2 mmol/l after 20 h, 0.6 mmol/l after 40 h, 0.3
mmol/l after 60 h and 0.15 mmol/l after 80 h.

4 h (Option A) is incorrect. This is too short, based upon a half-life estimate of 20 h.


8 h (Option B) is incorrect. This is too short, based upon a half-life estimate of 20 h.
12 h (Option C) is incorrect. This is too short, based upon a half-life estimate of 20 h.
24 h (Option D) is incorrect. This is too short, based upon a half-life estimate of 20 h.
46982

https://mypastest.pastest.com/Secure/TestMe/Browser/429893#Top

1/2

8/11/2016

MyPastest

Next Question

Previous Question

Tag Question

Feedback

End Review

Difficulty: Average
Peer Responses

Session Progress
Responses Correct:

Responses Incorrect:

207

Responses Total:

207

Responses - % Correct:

0%

Blog (https://www.pastest.com/blog) About Pastest (https://www.pastest.com/about-us)


Contact Us (https://www.pastest.com/contact-us) Help (https://www.pastest.com/help)
Pastest 2016

https://mypastest.pastest.com/Secure/TestMe/Browser/429893#Top

2/2

8/11/2016

MyPastest

Back to Filters (/Secure/TestMe/Filter/429893/QA)

Question 41 of 207

You are involved in a meeting with your pharmacy and biochemistry department colleagues
to discuss the local hospital policy on therapeutic drug monitoring.
For which of the following drugs would therapeutic drug monitoring be most appropriate?
A

Beta-interferon

Cyclophosphamide

Propranolol

Vancomycin

Vigabatrin

Explanation

The answer is Vancomycin


Therapeutic drug monitoring is only useful for drugs that have a narrow therapeutic
index, and where there is a good correlation between serum concentration and
therapeutic effect. It should be remembered, however, that particularly with
anticonvulsants and digoxin, it is important to rely on clinical judgement in interpreting
the drug concentrations, including response to treatment and occurrence of adverse
effects. Intravenous vancomycin has a narrow therapeutic window, and monitoring is
needed to minimise the risk of kidney injury.

Beta-interferon (Option A) is incorrect. There is no obvious relationship between interferon


drug concentrations and clinical effectiveness or occurrence of adverse effects.
Cyclophosphamide (Option B) is incorrect. There is no obvious relationship between
cyclophosphamide drug concentrations and clinical effectiveness or occurrence of adverse
effects.
Propranolol (Option C) is incorrect. The dose of propranolol and other -blockers may be
titrated according to patient symptoms using heart rate as a guide to effectiveness.

https://mypastest.pastest.com/Secure/TestMe/Browser/429893#Top

1/2

8/11/2016

MyPastest

Vigabatrin (Option E) is incorrect. It is possible to measure vigabatrin concentrations to


confirm drug compliance, but there is a much poorer correlation between drug
concentrations and clinical response.
46898

Next Question

Previous Question

Tag Question

Feedback

End Review

Difficulty: Average
Peer Responses

Session Progress
Responses Correct:

Responses Incorrect:

207

Responses Total:

207

Responses - % Correct:

0%

Blog (https://www.pastest.com/blog) About Pastest (https://www.pastest.com/about-us)


Contact Us (https://www.pastest.com/contact-us) Help (https://www.pastest.com/help)
Pastest 2016

https://mypastest.pastest.com/Secure/TestMe/Browser/429893#Top

2/2

8/11/2016

MyPastest

Back to Filters (/Secure/TestMe/Filter/429893/QA)

Question 42 of 207

You review a 64-year-old man who is on warfarin therapy for recurrent atrial fibrillation. He
presents to the Emergency Department with extensive unprovoked bruising and an
international normalised ratio (INR) check reveals that his INR is raised at 6.5.
Which of the following drugs when co-administered with warfarin may result in increased
INR?
A

Carbamazepine

Ciprofloxacin

Phenobarbital

Primidone

Rifampicin

Explanation
The answer is Ciprofloxacin
Chloramphenicol, ciprofloxacin, clarithromycin, erythromycin, metronidazole and
omeprazole are all capable of inhibiting hepatic enzyme activity so that the effects of
warfarin are that INR and bleeding risk increase.

Carbamazepine (Option A) is incorrect. Carbamazepine is a powerful inducer of hepatic


enzyme activity such that over a period of time warfarin concentrations and INR decrease.
Phenobarbital (Option C) is incorrect. Phenobarbital is a powerful inducer of hepatic enzyme
activity such that over a period of time warfarin concentrations and INR decrease.
Primidone (Option D) is incorrect. Primodone is a prodrug metabolised to phenobarbital, a
powerful inducer of hepatic enzyme activity such that over a period of time warfarin
concentrations and INR decrease.
Rifampicin (Option E) is incorrect. Rifampicin is an inducer of hepatic enzyme activity such
that over a period of time warfarin concentrations and INR decrease.
46919

https://mypastest.pastest.com/Secure/TestMe/Browser/429893#Top

1/2

8/11/2016

MyPastest

Next Question

Previous Question

Tag Question

Feedback

End Review

Difficulty: Average
Peer Responses

Session Progress
Responses Correct:

Responses Incorrect:

207

Responses Total:

207

Responses - % Correct:

0%

Blog (https://www.pastest.com/blog) About Pastest (https://www.pastest.com/about-us)


Contact Us (https://www.pastest.com/contact-us) Help (https://www.pastest.com/help)
Pastest 2016

https://mypastest.pastest.com/Secure/TestMe/Browser/429893#Top

2/2

8/11/2016

MyPastest

Back to Filters (/Secure/TestMe/Filter/429893/QA)

Question 43 of 207

You are reviewing a patient in the medical admissions unit who has been referred from the
Emergency Department after a suspected overdose. The patient appears anxious and tearful
but is refusing to disclose what he may have taken. On examination of his pupils, you note
that they are dilated.
Which of the following is most likely to account for the dilated pupils?
A

Chlorpromazine

Cocaine

Fentanyl

Mirtazapine

Sodium valproate

Explanation

The answer is Cocaine


Dilated pupils (mydriasis) may be caused by drugs with sympathomimetic activity (eg
cocaine, amphetamines and pseudoephedrine) or anticholinergic drugs (eg
antihistamines, atropine and tricyclic antidepressants).

Chlorpromazine (Option A) is incorrect. Phenothiazines may cause miosis.


Fentanyl (Option C) is incorrect. Opioids cause miosis, which may be severe enough to cause
pin-point pupils.
Mirtazapine (Option D) is incorrect. This may cause miosis. In mirtazapine overdose, there
may be pin-point pupils that mimic the effects of opioids.
Sodium valproate (Option E) is incorrect. This may cause miosis.
45803

Next Question

https://mypastest.pastest.com/Secure/TestMe/Browser/429893#Top

1/2

8/11/2016

MyPastest

Previous Question

Tag Question

Feedback

End Review

Difficulty: Average
Peer Responses

Session Progress
Responses Correct:

Responses Incorrect:

207

Responses Total:

207

Responses - % Correct:

0%

Blog (https://www.pastest.com/blog) About Pastest (https://www.pastest.com/about-us)


Contact Us (https://www.pastest.com/contact-us) Help (https://www.pastest.com/help)
Pastest 2016

https://mypastest.pastest.com/Secure/TestMe/Browser/429893#Top

2/2

8/11/2016

MyPastest

Back to Filters (/Secure/TestMe/Filter/429893/QA)

Question 44 of 207

A 56-year-old woman has been referred to the endocrinology clinic for review of
thyrotoxicosis and consideration of possible radioactive iodine treatment.
Which one of the following statements best describes the role of radioactive iodine (131I ) in
the treatment of thyrotoxicosis?
A

Associated with increased incidence of late leukaemia

Hypoparathyroidism secondary to -emissions and ablation of the parathyroid gland


occurs in 30% of cases

Intravenous administration is used to avoid gastrointestinal toxicity

May worsen Graves exophthalmos in the first 3 months

Three doses 1 month apart is the optiomal regimen

Explanation

The answer is May worsen Graves exophthalmos in the first 3 months


131

I is administered as an oral solution or capsule of sodium 131I, which is rapidly


concentrated in thyroid tissue. Beta-emissions result in ablation of the gland over the
following 618 weeks. Beta-emissions have very low tissue penetration capability. It is
generally accepted that there is increased risk of worsening Graves exophthalmos during
radioactive iodine therapy

Associated with increased incidence of late leukaemia (Option A) is incorrect. Radioactive


iodine has been studied in very large numbers of patients, including a prospective study of
36 000 patients that found no increased risk of cancer or leukaemia.
Hypoparathyroidism secondary to -emissions and ablation of the parathyroid gland occurs
in 30% of cases (Option B) is incorrect. The parathyroid gland is not affected because the
radiation is selectively taken up by thyroid tissue, and is capable of penetrating only 0.5 mm
of tissue.
Intravenous administration is used to avoid gastrointestinal toxicity (Option C) is incorrect.
Radioactive iodine is administered orally.
https://mypastest.pastest.com/Secure/TestMe/Browser/429893#Top

1/2

8/11/2016

MyPastest

Three doses 1 month apart is the optiomal regimen (Option E) is incorrect. Most patients
require only one treatment, although 1015% experience treatment failure and require a
second or subsequent doses.
46627

Next Question

Previous Question

Tag Question

Feedback

End Review

Difficulty: Average
Peer Responses

Session Progress
Responses Correct:

Responses Incorrect:

207

Responses Total:

207

Responses - % Correct:

0%

Blog (https://www.pastest.com/blog) About Pastest (https://www.pastest.com/about-us)


Contact Us (https://www.pastest.com/contact-us) Help (https://www.pastest.com/help)
Pastest 2016

https://mypastest.pastest.com/Secure/TestMe/Browser/429893#Top

2/2

8/11/2016

MyPastest

Back to Filters (/Secure/TestMe/Filter/429893/QA)

Question 45 of 207

A 45-year-old woman who smokes 30 cigarettes per day is admitted to the Emergency
Department resuscitation room after suffering an out-of-hospital cardiac arrest. Her husband
says that she has recently suffered a chest infection treated by her GP with erythromycin. She
has a past history of penicillin allergy, and has been taking some antifungal tablets for chronic
fungal vaginal infection.
Given her medication history, which of the following causes of cardiac arrest is more
probable?
A

Anaphylaxis to erythromycin

Myocardial infarction

Supraventricular tachycardia

Torsades de pointes ventricular tachycardia

Ventricular tachycardia

Explanation

The answer is Torsades de pointes ventricular tachycardia


This patient is likely to have suffered torsades de pointes ventricular tachycardia as a
result of an interaction between erythromycin (which is a cause of long QT) and her
antifungal tablets, most likely ketoconazole, used for recurrent episodes of vaginal
candidiasis. Similar potential for drug-induced QT prolongation exists for a number of
drugs, including the interaction between erythromycin and moxifloxacin. Cisapride, a
prokinetic agent used for the treatment of gastro-oesophageal reflux disease, was
withdrawn due to the occurrence of QT prolongation, particularly when used in
combination with erythromycin. When prescribing erythromycin, it is important to
consult the Interactions section in the British National Formulary in order to rule out
possible interactions of this type.

Anaphylaxis to erythromycin (Option A) is incorrect. Although possible, it is uncommon for


patients with multidrug allergy to have allergy to macrolide antibiotics.
https://mypastest.pastest.com/Secure/TestMe/Browser/429893#Top

1/2

8/11/2016

MyPastest

Myocardial infarction (Option B) is incorrect. There are no obvious risk factors for myocardial
infarction, and no known association between her medications and atherosclerosis.
Supraventricular tachycardia (Option C) is incorrect. This is unrelated to any of the clinical
details provided.
Ventricular tachycardia (Option E) is incorrect. Although ventricular tachycardia is possibly
correct, torsades de pointes offers a better explanation in view of her history of erythromycin
and antifungal treatments.
46883

Next Question

Previous Question

Tag Question

Feedback

End Review

Difficulty: Average
Peer Responses

Session Progress
Responses Correct:

Responses Incorrect:

207

Responses Total:

207

Responses - % Correct:

0%

Blog (https://www.pastest.com/blog) About Pastest (https://www.pastest.com/about-us)


Contact Us (https://www.pastest.com/contact-us) Help (https://www.pastest.com/help)
Pastest 2016

https://mypastest.pastest.com/Secure/TestMe/Browser/429893#Top

2/2

8/11/2016

MyPastest

Back to Filters (/Secure/TestMe/Filter/429893/QA)

Question 46 of 207

An elderly woman is taking furosemide and ramipril for heart failure. She visited her GP
complaining of pain in her left knee and was prescribed rofecoxib for analgesia. Two weeks
later, she was admitted to the A&E department complaining of breathlessness and pedal
oedema.
What is the most likely cause of her having developed these symptoms?
A

Anaemia due to gastrointestinal bleeding

Drug interaction leading to decreased furosemide effect

Drug interaction leading to increased ramipril levels

Fluid retention due to rofecoxib

Rofecoxib induced deterioration in creatinine

Explanation

The answer is Fluid retention due to rofecoxib


Rofecoxib is a cyclo-oxygenase-2 (Cox-2) specific inhibitor and has less of an adverse
effect on the gastrointestinal tract; thus gastric irritation and bleeding is much less
common than with other NSAIDs such as ibuprofen or mefenamic acid. However,
rofecoxib can cause fluid retention and worsen pre-existing cardiac failure. One trial
versus celecoxib demonstrated oedema rates of around 7.7% in patients treated with
rofecoxib 25 mg, over a 6-week period.

Anaemia due to gastrointestinal bleeding (Option A) is incorrect. Coxibs can provoke


gastrointestinal irritation and bleeding, albeit much less commonly than after NSAIDs.
Drug interaction leading to decreased furosemide effect (Option B) is incorrect. Rofecoxib is
not known to impair response to furosemide.
Drug interaction leading to increased ramipril levels (Option C) is incorrect. Increased
ramipril concentrations are unlikely to occur, and would not account for symptoms of
breathlessness or oedema.
https://mypastest.pastest.com/Secure/TestMe/Browser/429893#Top

1/2

8/11/2016

MyPastest

Rofecoxib induced deterioration in creatinine (Option E) is incorrect. Coxibs may cause renal
impairment, but this is much less commonly encountered than after non-selective NSAIDs.
46743

Next Question

Previous Question

Tag Question

Feedback

End Review

Difficulty: Average
Peer Responses

Session Progress
Responses Correct:

Responses Incorrect:

207

Responses Total:

207

Responses - % Correct:

0%

Blog (https://www.pastest.com/blog) About Pastest (https://www.pastest.com/about-us)


Contact Us (https://www.pastest.com/contact-us) Help (https://www.pastest.com/help)
Pastest 2016

https://mypastest.pastest.com/Secure/TestMe/Browser/429893#Top

2/2

8/11/2016

MyPastest

Back to Filters (/Secure/TestMe/Filter/429893/QA)

Question 47 of 207

An 18-year-old student presents to the Emergency Department with a short history of fever,
vomiting and diarrhoea. She has recently returned to the UK after a holiday in Turkey during
which she had been commenced on an antibiotic for a suspected urinary tract infection. On
examination she has a widespread macular rash associated with ring-like lesions, with some
ulceration in her oral cavity.
Which drug is most likely to account for these findings?
A

Amoxicillin

Ampicillin

Ciprofloxacin

Co-trimoxazole

Erythromycin

Explanation

The answer is Co-trimoxazole


The widespread macular rash and target lesions are characteristic of erythema
multiforme, and the features are consistent with StevensJohnson syndrome. This is a
particularly serious and potentially fatal type of skin and mucous membrane eruption. It
can occur after a number of antibiotics but is most strongly associated with
sulfonamides including sulfamethoxazole (this is combined with trimethoprim in cotrimoxazole). Therefore, sulfonamides and co-trimoxazole are used less commonly, and
reserved for severe infections.

Amoxicillin (Option A) is incorrect. Amoxicillin may cause widespread maculopapular rash,


urticaria and fixed drug eruptions. It can be effective for empirical treatment of urinary tract
infections (UTIs) but in many countries and regions there is significant resistance among
various strains of Escherichia coli.
Ampicillin (Option B) is incorrect. Ampicillin may cause widespread maculopapular rash,
urticaria and fixed drug eruptions. It can be effective for empirical treatment of urinary tract
https://mypastest.pastest.com/Secure/TestMe/Browser/429893#Top

1/2

8/11/2016

MyPastest

infections (UTIs) but in many countries and regions there is significant resistance among
various strains of Escherichia coli.
Ciprofloxacin (Option C) is incorrect. Ciprofloxacin is effective in treating UTIs, even when
they are caused by multidrug-resistant bacteria, eg Pseudomonas spp. They are normally
reserved for second-line treatment for UTIs to minimise emergence of resistant strains.
StevensJohnson syndrome is not associated with ciprofloxacin.
Erythromycin (Option E) is incorrect. Erythromycin is generally regarded as much less
effective versus gram-negative bacteria and unlikely to be effective treatment for UTI.
46781

Next Question

Previous Question

Tag Question

Feedback

End Review

Difficulty: Average
Peer Responses

Session Progress
Responses Correct:

Responses Incorrect:

207

Responses Total:

207

Responses - % Correct:

0%

Blog (https://www.pastest.com/blog) About Pastest (https://www.pastest.com/about-us)


Contact Us (https://www.pastest.com/contact-us) Help (https://www.pastest.com/help)
Pastest 2016

https://mypastest.pastest.com/Secure/TestMe/Browser/429893#Top

2/2

8/11/2016

MyPastest

Back to Filters (/Secure/TestMe/Filter/429893/QA)

Question 48 of 207

You are asked to review a 63-year-old man as the medical registrar on duty. He is currently an
in-patient on the psychiatric ward and receiving treatment for severe depression. The
psychiatry team has considered a potential diagnosis of serotonin syndrome.
Which of the following features is most strongly suggestive of serotonin syndrome?
A

Flaccid paresis of both legs

Heart rate 92 bpm

Myoclonus at both ankles

Profuse sweating

Temperature 37.6 C

Explanation

The answer is Myoclonus at both ankles


The serotonin syndrome occurs primarily because of interactions between drugs that
promote serotonin release within the central nervous system. This includes monoamineoxidase inhibitors (MAOIs), selective serotonin-reuptake inhibitors (SSRIs), tricyclic
antidepressants, antipsychotics and tramadol. Typical features include hypertonia,
hyperreflexia, hyperthermia, sweating, tremor, myoclonus and tachycardia. Treatment
consists of stopping the causative drugs, and supportive measures to reduce agitation
including benzodiazepines. Cyproheptadine possesses serotonin-antagonist activity and
may be helpful, and active cooling may be required in some patients.

Flaccid paresis of both legs (Option A) is incorrect. Tone and reflexes are normally increased.
Heart rate 92 bpm (Option B) is incorrect. Tachycardia is a recognised feature, but a heart
rate of 92 is non-specific, and may be caused by a broad range of disorders.
Profuse sweating (Option D) is incorrect. Profuse sweating may be a feature, but is nonspecific, and may be caused by a broad range of disorders.

https://mypastest.pastest.com/Secure/TestMe/Browser/429893#Top

1/2

8/11/2016

MyPastest

Temperature 37.6 C (Option E) is incorrect. Hyperthermia is a key feature, but a temperature


of 37.6 C is non-specific, and may be caused by a broad range of disorders.
46490

Next Question

Previous Question

Tag Question

Feedback

End Review

Difficulty: Average
Peer Responses

Session Progress
Responses Correct:

Responses Incorrect:

207

Responses Total:

207

Responses - % Correct:

0%

Blog (https://www.pastest.com/blog) About Pastest (https://www.pastest.com/about-us)


Contact Us (https://www.pastest.com/contact-us) Help (https://www.pastest.com/help)
Pastest 2016

https://mypastest.pastest.com/Secure/TestMe/Browser/429893#Top

2/2

8/11/2016

MyPastest

Back to Filters (/Secure/TestMe/Filter/429893/QA)

Question 49 of 207

A 59-year-old woman, who is being treated with trastuzumab for breast cancer, comes to the
clinic complaining of increasing shortness of breath and ankle swelling. On examination her
blood pressure is 115/80 mmHg and her pulse is 95 beats per minute and regular. She has
biventricular failure with bilateral ankle swelling and inspiratory crackles in both lower zones
on auscultation.
Investigations:
Hb

10.9 g/dl

White cell count 9.1 109/l


Platelets

165 109/l

Na+

141 mmol/l

K+

4.2 mmol/l

Creatinine

129 mol/l

ECHO

suggestive of dilated cardiomyopathy

Which one of the following is the causative mechanism of cardiac failure in patients on
trastuzumab?
A

Calcium channel blockade

Erb-b2 inhibition

Oestrogen receptor inhibition

Platelet-derived growth factor inhibition

Erb-b1 inhibition

Explanation
Trastuzumab
https://mypastest.pastest.com/Secure/TestMe/Browser/429893#Top

1/2

8/11/2016

MyPastest

Studies have shown that activation of Erb-b2 (also known as HER-2), the receptor
blocked by trastuzumab (Herceptin), is important in preventing the development of
cardiomyopathy
A knockout mouse model with deletion of the erb-b2 receptor showed a predisposition
for the development of cardiomyopathy, which was actually worsened by anthracycline
therapy
Reference

Crone, S. A., Zhao, Y.-Y., Fan, L. et al. 2002. ErbBe2 is essential in the prevention of dilated
cardiomyopathy. Nature Medicine, 8, 459465.
22449

Next Question

Previous Question

Tag Question

Feedback

End Review

Difficulty: Average
Peer Responses

Session Progress
Responses Correct:

Responses Incorrect:

207

Responses Total:

207

Responses - % Correct:

0%

Blog (https://www.pastest.com/blog) About Pastest (https://www.pastest.com/about-us)


Contact Us (https://www.pastest.com/contact-us) Help (https://www.pastest.com/help)
Pastest 2016

https://mypastest.pastest.com/Secure/TestMe/Browser/429893#Top

2/2

8/11/2016

MyPastest

Back to Filters (/Secure/TestMe/Filter/429893/QA)

Question 50 of 207

A 35-year-old chronic alcoholic was admitted to hospital with severe tremor due to
unplanned alcohol withdrawal, and has undergone a period of observed detoxification over
the past week. She is now mobilising independently and ready for discharge. She is keen to
remain abstinent after discharge. She has already begun attending counselling meetings.
Other past history of note includes irritable bowel syndrome (IBS).
Which of the following drugs might be most useful in maintaining abstinence from alcohol in
this patient?
A

Acamprosate

Chlordiazepoxide

Diazepam

Disulfiram

Naltrexone

Explanation

The answer is Acamprosate


Acamprosate is a taurine derivative that increases -aminobutyric acid (GABA) activity
within the brain, an inhibitory CNS neurotransmitter. It has relatively few side-effects but
may reduce alcohol craving and improves abstinence rates. It is contraindicated in
pregnancy, and severe liver or kidney failure.

Chlordiazepoxide (Option B) is incorrect. Benzodiazepines such as chlordiazepoxide are


useful for managing the initial withdrawal phase by increasing seizure threshold and reducing
acute withdrawal symptoms including delirium tremens.
Diazepam (Option C) is incorrect. Benzodiazepines such as diazepam are useful for
managing the initial withdrawal phase by increasing seizure threshold and reducing acute
withdrawal symptoms including delirium tremens.
Disulfiram (Option D) is incorrect. Disulfiram may provoke an unpleasant reaction if taken
with alcohol, and may promote alcohol avoidance. It is not used in acute management of
https://mypastest.pastest.com/Secure/TestMe/Browser/429893#Top

1/2

8/11/2016

MyPastest

alcohol withdrawal.
Naltrexone (Option E) is incorrect. Naltrexone reduces the pleasure that alcohol brings and
craving when it is withdrawn, and can reduce relapse rates by up to half. However, it is
associated with a number of adverse effects, including nausea, vomiting, anxiety,
nervousness, insomnia, lethargy, arthralgia, increased sweating and lacrimation, diarrhoea or
constipation, increased thirst, and liver and kidney dysfunction. The adverse gastrointestinal
effects of naltrexone may discourage use in a patient with IBS.
46762

Next Question

Previous Question

Tag Question

Feedback

End Review

Difficulty: Average
Peer Responses

Session Progress
Responses Correct:

Responses Incorrect:

207

Responses Total:

207

Responses - % Correct:

0%

Blog (https://www.pastest.com/blog) About Pastest (https://www.pastest.com/about-us)


Contact Us (https://www.pastest.com/contact-us) Help (https://www.pastest.com/help)
Pastest 2016

https://mypastest.pastest.com/Secure/TestMe/Browser/429893#Top

2/2

8/11/2016

MyPastest

Back to Filters (/Secure/TestMe/Filter/429893/QA)

Question 51 of 207

A 58-year-old man presents to the clinic with easy bruising and bleeding from his gums. He
has a history of type-2 diabetes and has recently been prescribed a medication for
neuropathy. In addition he has an infection but cannot remember which antibiotic his doctor
has prescribed for it. Chronic medication of note includes warfarin, which is prescribed for
atrial fibrillation after a failed cardioversion. His INR is normally stable at 2.1; it has recently
increased to 5.0.
Which one of the following medications is most likely to be responsible for his recent
disturbance in coagulation control?
A

Ciprofloxacin

Azithromycin

Lansoprazole

Rifampicin

Carbamazepine

Explanation
Drug interactions
Rifampicin and carbamazepine are inducers of CYP-450 and hence are associated with
decreased INR
Lansoprazole and other proton pump inhibitors, while they affect stomach pH, are not
usually associated with significant disturbance in INR
While a caution is advised for use of macrolides with warfarin, azithromycin appears to
be least likely to lead to an interaction
This leaves ciprofloxacin, for which a strong caution is advised because of potentiation
of warfarins effect
22480

Next Question
https://mypastest.pastest.com/Secure/TestMe/Browser/429893#Top

1/2

8/11/2016

MyPastest

Previous Question

Tag Question

Feedback

End Review

Difficulty: Average
Peer Responses

Session Progress
Responses Correct:

Responses Incorrect:

207

Responses Total:

207

Responses - % Correct:

0%

Blog (https://www.pastest.com/blog) About Pastest (https://www.pastest.com/about-us)


Contact Us (https://www.pastest.com/contact-us) Help (https://www.pastest.com/help)
Pastest 2016

https://mypastest.pastest.com/Secure/TestMe/Browser/429893#Top

2/2

8/11/2016

MyPastest

Back to Filters (/Secure/TestMe/Filter/429893/QA)

Question 52 of 207

A 60-year-old lady, who has been an inpatient in the medical unit for several weeks, is noted
to have a haemoglobin concentration of 9.9 g/dl (1316). The blood film shows red cell
fragments and you suspect haemolysis. The haematology specialist registrar has asked for a
list of all her medications because she suspects a drug cause.
Which of the following drugs is most likely to cause haemolysis and anaemia?
A

Atenolol

Captopril

Erythromycin

Penicillin

Verapamil

Explanation

The answer is Penicillin


Drug-induced haemolysis may be related to underlying genetic causes, for example
glucose-6-phophate dehydrogenase (G6PD) deficiency predisposes to haemolysis when
patients receive dapsone, primaquine, aspirin or quinolones. Acquired autoimmune
haemolysis is a recognised complication of methyldopa, penicillin, quinine and quinidine.
Direct (non-immune) red cell toxicity may occur after lead exposure.

Atenolol (Option A) is incorrect. Atenolol may cause bradyarrhythmia and bronchospasm but
anaemia is not a recognised complication.
Captopril (Option B) is incorrect. Captopril may cause renal failure.
Erythromycin (Option C) is incorrect. Erythromycin is a powerful liver enzyme inhibitor;
certain preparations may cause cholestasis.
Verapamil (Option E) is incorrect. Verapamil may cause bradycardia and heart block, and
gastrointestinal upset, but it is not a recognised cause of haemolysis.
46909

https://mypastest.pastest.com/Secure/TestMe/Browser/429893#Top

1/2

8/11/2016

MyPastest

Next Question

Previous Question

Tag Question

Feedback

End Review

Difficulty: Average
Peer Responses

Session Progress
Responses Correct:

Responses Incorrect:

207

Responses Total:

207

Responses - % Correct:

0%

Blog (https://www.pastest.com/blog) About Pastest (https://www.pastest.com/about-us)


Contact Us (https://www.pastest.com/contact-us) Help (https://www.pastest.com/help)
Pastest 2016

https://mypastest.pastest.com/Secure/TestMe/Browser/429893#Top

2/2

8/11/2016

MyPastest

Back to Filters (/Secure/TestMe/Filter/429893/QA)

Question 53 of 207

A patient with metastatic carcinoma of the breast is admitted for chemotherapy. Her history
reveals that she has had several deep vein thromboses in the past, as well as a pulmonary
embolism when she was younger.
Which of the following chemotherapeutic agents should be particularly avoided in her case?
A

Anastrozole

Buserelin

Goserelin

Letrozole

Tamoxifen

Explanation

The answer is Tamoxifen


Tamoxifen is a partial oestrogen agonist and can increase the risk of thromboembolism,
particularly during and immediately after major surgery or periods of immobility.

Anastrozole (Option A) is incorrect. Anastrazole reduces sex hormone concentrations and


does not increase the risk of venous thromboembolism.
Buserelin (Option B) is incorrect. Buserelin reduces sex hormone concentrations and does
not increase the risk of venous thromboembolism.
Goserelin (Option C) is incorrect. Goserelin reduces sex hormone concentrations and does
not increase the risk of venous thromboembolism.
Letrozole (Option D) is incorrect. Letrozole reduces oestrogen concentrations and does not
increase the risk of venous thromboembolism.
46813

Next Question

https://mypastest.pastest.com/Secure/TestMe/Browser/429893#Top

1/2

8/11/2016

MyPastest

Previous Question

Tag Question

Feedback

End Review

Difficulty: Average
Peer Responses

Session Progress
Responses Correct:

Responses Incorrect:

207

Responses Total:

207

Responses - % Correct:

0%

Blog (https://www.pastest.com/blog) About Pastest (https://www.pastest.com/about-us)


Contact Us (https://www.pastest.com/contact-us) Help (https://www.pastest.com/help)
Pastest 2016

https://mypastest.pastest.com/Secure/TestMe/Browser/429893#Top

2/2

8/11/2016

MyPastest

Back to Filters (/Secure/TestMe/Filter/429893/QA)

Question 54 of 207

Which of the following is the main reaction involved in the normal metabolism of
paracetamol?
A

Acetylation

Conjugation to glucuronic acid

Conjugation to glutathione

Cytochrome P450-dependent oxidation

Hydrolysis

Explanation

The answer is Conjugation to glucoronic acid


Paracetamol is a commonly used painkiller, and when taken in overdose may cause
severe and fatal liver damage. The key pathways responsible for metabolism are
glucuronidation and sulfation. In acute overdose or when the maximum daily dose is
exceeded over a prolonged period, these pathways become saturated so that an
increasing quantity is metabolised via non-specific P450 oxidative metabolism. Excess
paracetamol is then oxidatively metabolised in the liver via the mixed function oxidase
P450 system to a toxic metabolite, N-acetyl-p-benzoquinone imine (NAPQI). NAPQI has
an extremely short half-life and is rapidly conjugated wtih glutathione, a sulfhydryl donor,
and is renally excreted. Under conditions of excessive NAPQI formation or reduced
glutathione stores, NAPQI covalently binds to vital proteins and the lipid bilayer of
hepatocyte membranes. This results in hepatocellular death and centrilobular liver
necrosis.

Acetylation (Option A) is incorrect. Acetylation does not contribute to paracetamol


metabolism.
Conjugation to glutathione (Option C) is incorrect. Glutathione forms conjugates with the
NAPQI metabolite rather than paracetamol itself.

https://mypastest.pastest.com/Secure/TestMe/Browser/429893#Top

1/2

8/11/2016

MyPastest

Cytochrome P450-dependent oxidation (Option D) is incorrect. Oxidative metabolism


accounts for only a small proportion of paracetamol metabolism after therapeutic doses.
Hydrolysis (Option E) is incorrect. This is not relevant to paracetamol metabolism.
46936

Next Question

Previous Question

Tag Question

Feedback

End Review

Difficulty: Average
Peer Responses

Session Progress
Responses Correct:

Responses Incorrect:

207

Responses Total:

207

Responses - % Correct:

0%

Blog (https://www.pastest.com/blog) About Pastest (https://www.pastest.com/about-us)


Contact Us (https://www.pastest.com/contact-us) Help (https://www.pastest.com/help)
Pastest 2016

https://mypastest.pastest.com/Secure/TestMe/Browser/429893#Top

2/2

8/11/2016

MyPastest

Back to Filters (/Secure/TestMe/Filter/429893/QA)

Question 55 of 207

A 72-year-old woman is admitted with ureteric colic. There is a past history of epilepsy but
she has been fit-free for nearly 15 years. This patient has suffered multiple renal stones and
has chronic renal impairment with a serum creatinine of 210 mol/l. You prescribe pethidine
for pain relief and she initially settles. However, you are asked to see her urgently on call the
following morning as she has suffered a generalised seizure.
What is the likely cause of the generalised seizure?
A

Accumulation of morphine 6-glucuronide after pethidine administration

Accumulation of norpethidine leading to toxicity

Accumulation of pethidine leading to toxicity

An epileptic fit in keeping with her history of previous epilepsy

Pseudoseizure

Explanation

The answer is Accumulation of norpethidine leading to toxicity


Pethidine is metabolised to norpethidine, but in patients with renal impairment
norpethidine accumulates rather than being excreted through the kidneys. Norpethidine
is toxic and is associated with a risk of seizures.

Accumulation of morphine 6-glucuronide after pethidine administration (Option A) is


incorrect. Morphine 6-glucuronide accumulates in renal failure when patients are treated with
morphine, increasing the likelihood of opiod toxicity, but this is irrelevant to administration of
pethidine.
Accumulation of pethidine leading to toxicity (Option C) is incorrect. It is the metabolite of
pethidine rather than pethidine itself that causes seizures.
An epileptic fit in keeping with her history of previous epilepsy (Option D) is incorrect.
Seizures happen in patients with epilepsy, but this is a less satisfactory explanation in this
situation, as norpethedine will have provoked seizure.
https://mypastest.pastest.com/Secure/TestMe/Browser/429893#Top

1/2

8/11/2016

MyPastest

Pseudoseizure (Option E) is incorrect. There is no association between pethidine and


pseudoseizures.
46833

Next Question

Previous Question

Tag Question

Feedback

End Review

Difficulty: Average
Peer Responses

Session Progress
Responses Correct:

Responses Incorrect:

207

Responses Total:

207

Responses - % Correct:

0%

Blog (https://www.pastest.com/blog) About Pastest (https://www.pastest.com/about-us)


Contact Us (https://www.pastest.com/contact-us) Help (https://www.pastest.com/help)
Pastest 2016

https://mypastest.pastest.com/Secure/TestMe/Browser/429893#Top

2/2

8/11/2016

MyPastest

Back to Filters (/Secure/TestMe/Filter/429893/QA)

Question 56 of 207

A 39-year-old lady is noted to have a low serum sodium concentration of 127 mmol/l (137
144), as well as a high urine osmolality. A diagnosis of syndrome of inappropriate antidiuretic
hormone secretion (SIADH) is made, and a drug-related cause is suspected.
Which of the following drugs is most likely to be responsible?
A

Carbamazepine

Chlorpropamide

Demeclocycline

Lithium

Rifampicin

Explanation

The answer is Carbamazepine


Antidiuretic hormone (ADH) secretion from the posterior pituitary can be stimulated by
some drugs, notably carbamazepine. The responsiveness of the collecting ducts to ADH
can be sensitised by rifampicin and sulfonylureas, especially chlorpropamide.

Chlorpropramide (Option B) is incorrect. Chlorpropamide may cause SIADH by increasing


the sensitivity of collecting duct ADH receptors, but is a less noticeable adverse effect than
that of carbamazepine.
Demeclocycline (Option C) is incorrect. Demeclocycline interferes with renal responsiveness
to ADH and is used as a treatment of SIADH, although it may cause diabetes insipidus.
Lithium (Option D) is incorrect. Lithium typically causes nephrogenic diabetes insipidus,
especially after long-term use. SIADH is a very rare but recognised complication of lithium
due to increased secretion of ADH by the posterior pituitary.
Rifampicin (Option E) is incorrect. Rifampicin is a recognised cause of SIADH, but occurs
much less commonly than after carbamazepine, hence option A is the preferred answer.
46902

https://mypastest.pastest.com/Secure/TestMe/Browser/429893#Top

1/2

8/11/2016

MyPastest

Next Question

Previous Question

Tag Question

Feedback

End Review

Difficulty: Average
Peer Responses

Session Progress
Responses Correct:

Responses Incorrect:

207

Responses Total:

207

Responses - % Correct:

0%

Blog (https://www.pastest.com/blog) About Pastest (https://www.pastest.com/about-us)


Contact Us (https://www.pastest.com/contact-us) Help (https://www.pastest.com/help)
Pastest 2016

https://mypastest.pastest.com/Secure/TestMe/Browser/429893#Top

2/2

8/11/2016

MyPastest

Back to Filters (/Secure/TestMe/Filter/429893/QA)

Question 57 of 207

A 60-year-old man, who has been taking warfarin for 3 years for atrial fibrillation, with a
previously stable INR, presents for review. He noticed some bruising on his arms from
working in the garden. INR is now markedly elevated at 7.0
Which of the following interactions with warfarin is the most likely cause of this clinical
picture?
A

Carrot juice

Cranberry juice

Orange juice

St Johns wort

Tomato juice

Explanation

The answer is Cranberry juice


A warning was given by the Committee on Safety of Medicines in 2003 about the
possible interaction between cranberry juice and warfarin levels. The cause is thought to
be bioflavonoids contained in the cranberry juice, which block cytochrome-P450-related
warfarin metabolism (CYP2C9). Of note, grapefruit juice interferes with CYP3A4
metabolism, which affects levels of a number of drugs, including ciclosporin and
simvastatin.

Carrot juice (Option A) is incorrect. There is no significant interaction between warfarin and
carrot juice.
Orange juice (Option C) is incorrect. There is no significant interaction between warfarin and
orange juice.
St Johns wort (Option D) is incorrect. St Johns Wort is an enzyme inducer and regular
treatment will lead to reduced levels of warfarin and INR.

https://mypastest.pastest.com/Secure/TestMe/Browser/429893#Top

1/2

8/11/2016

MyPastest

Tomato juice (Option E) is incorrect. There is no significant interaction between warfarin and
tomato juice.
46971

Next Question

Previous Question

Tag Question

Feedback

End Review

Difficulty: Average
Peer Responses

Session Progress
Responses Correct:

Responses Incorrect:

207

Responses Total:

207

Responses - % Correct:

0%

Blog (https://www.pastest.com/blog) About Pastest (https://www.pastest.com/about-us)


Contact Us (https://www.pastest.com/contact-us) Help (https://www.pastest.com/help)
Pastest 2016

https://mypastest.pastest.com/Secure/TestMe/Browser/429893#Top

2/2

8/11/2016

MyPastest

Back to Filters (/Secure/TestMe/Filter/429893/QA)

Question 58 of 207

You are asked by the hospital formulary to review a pharmaceutical companys application for
a new drug to be added to the list of approved medications. The drug is a combination
product made up of two long-standing drugs and the firm claims bioequivalence.
What is the best definition of bioequivalence in this context?
A

The two drugs compared contain the same active components

The two drugs compared contain the same ingredients and have the same
pharmacodynamics

The two drugs compared contain the same ingredients and have the same
pharmacokinetics

The two drugs compared have similar bioavailability

The two drugs compared have the same pharmacokinetic and pharmacodynamic
effects

Explanation

The answer is The two drugs compared have the same pharmacokinetic and
pharmacodynamic effects
Bioequivalence is all about demonstrating similar biological effect, including both
pharmacodynamic and pharmacokinetic properties. Considering only pharmacokinetic or
pharmacodynamic characteristics in isolation provides less comparative data. There may
be differences between products in excipients or delivery vehicle, which means that the
biological effects are different when the products are compared. Combination products
bring different issues.

The two drugs compared contain the same active components (Option A) is incorrect.
Containing the same active chemicals is not sufficient to ensure bioequivalence owing to
differences in excipients or formulation that might affect pharmacokinetic or
pharmacodynamic properties.

https://mypastest.pastest.com/Secure/TestMe/Browser/429893#Top

1/2

8/11/2016

MyPastest

The two drugs compared contain the same ingredients and have the same
pharmacodynamics (Option B) is incorrect. Pharmacodynamic equivalence alone is
insufficient to ensure bioequivalence.
The two drugs compared contain the same ingredients and have the same pharmacokinetics
(Option C) is incorrect. Pharmacokinetic equivalence alone is insufficient to ensure
bioequivalence.
The two drugs compared have similar bioavailability (Option D) is incorrect. Bioavailability,
one aspect of pharmacokinetic characteristics, does not ensure bioequivalence.
46843

Next Question

Previous Question

Tag Question

Feedback

End Review

Difficulty: Average
Peer Responses

Session Progress
Responses Correct:

Responses Incorrect:

207

Responses Total:

207

Responses - % Correct:

0%

Blog (https://www.pastest.com/blog) About Pastest (https://www.pastest.com/about-us)


Contact Us (https://www.pastest.com/contact-us) Help (https://www.pastest.com/help)
Pastest 2016

https://mypastest.pastest.com/Secure/TestMe/Browser/429893#Top

2/2

8/11/2016

MyPastest

Back to Filters (/Secure/TestMe/Filter/429893/QA)

Question 59 of 207

You are the medical registrar on-call and asked to review a 46-year-old patient who is
receiving medications for a severe respiratory tract infection. The nurse has noted that his
urine has become discoloured orangered and is worried that this might be related to his
treatment.
Which of the following drugs would be most likely to cause this?
A

B complex vitamins

Erythromycin

Nelfinavir

Phenolphthalein

Rifampicin

Explanation

The answer is Rifampicin


Patients on rifampicin should be warned that their urine, tears and other secretions will
develop a bright orangered colour. This is dose-dependent and fully reverses once
treatment ceases.

B complex vitamins (Option A) is incorrect. B-complex vitamins may darken the urine, but do
not produce widespread changes in other body secretions.
Erythromycin (Option B) is incorrect. Erythromycin does not cause a change in urine colour.
Nelfinavir (Option C) is incorrect. Nelfinavir, a protease inhibitor, may be associated with
haematuria (dark or bright red discoloration) and crystalluria owing to renal stone formation.
There is no effect on other bodily secretions. The occurrence of discoloration is much less
common than with rifampicin, hence option E is the preferred answer.
Phenolphthalein (Option D) is incorrect. Phenolphthalein stains alkaline urine pink.
46769

https://mypastest.pastest.com/Secure/TestMe/Browser/429893#Top

1/2

8/11/2016

MyPastest

Next Question

Previous Question

Tag Question

Feedback

End Review

Difficulty: Average
Peer Responses

Session Progress
Responses Correct:

Responses Incorrect:

207

Responses Total:

207

Responses - % Correct:

0%

Blog (https://www.pastest.com/blog) About Pastest (https://www.pastest.com/about-us)


Contact Us (https://www.pastest.com/contact-us) Help (https://www.pastest.com/help)
Pastest 2016

https://mypastest.pastest.com/Secure/TestMe/Browser/429893#Top

2/2

8/11/2016

MyPastest

Back to Filters (/Secure/TestMe/Filter/429893/QA)

Question 60 of 207

A 35-year-old Asian man is admitted to hospital with fever and rigors. Blood films are
reported as showing the presence of Plasmodium vivax and you diagnose acute malaria.
Which of the following antimalarial treatments is most likely to be a slow-acting
schizonticide?
A

Artemesinin

Mefloquine

Mepacrine

Pyrimethamine

Quinine

Explanation

The answer is Pyrimethamine


Antimalarials that act on the erythrocytic phase of schizogony are called schizonticides.
Antimalarials may be considered as fast-acting, high-efficacy blood schizonticides that
may be effective as monotherapy, or slow-acting, low-efficacy schizonticides that
normally need to be administered in combination. Pyrimethamine is used in the
treatment of uncomplicated malaria, particularly for chloroquine-resistant P. falciparum.
It acts on both the erythrocytic and hepatic phases of infection. It inhibits dihydrofolate
reductase in the parasite thus preventing the biosynthesis of purines and pyrimidines,
and thereby halting the processes of DNA replication, cell division and reproduction. It is
normally used alongside a sulfonamide. Doxycycline is also a very slow-acting
antimalarial.

Artemesinin (Option A) is incorrect. Artemisinin is a fast-acting schizonticides.


Mefloquine (Option B) is incorrect. Mefloquine is an intermediate-acting schizonticide.
Mepacrine (Option C) is incorrect. Mepacrine is a fast-acting schizonticide.
Quinine (Option E) is incorrect. Quinine is an intermediate-acting schizonticide.
46858

https://mypastest.pastest.com/Secure/TestMe/Browser/429893#Top

1/2

8/11/2016

MyPastest

46858

Next Question

Previous Question

Tag Question

Feedback

End Review

Difficulty: Average
Peer Responses

Session Progress
Responses Correct:

Responses Incorrect:

207

Responses Total:

207

Responses - % Correct:

0%

Blog (https://www.pastest.com/blog) About Pastest (https://www.pastest.com/about-us)


Contact Us (https://www.pastest.com/contact-us) Help (https://www.pastest.com/help)
Pastest 2016

https://mypastest.pastest.com/Secure/TestMe/Browser/429893#Top

2/2

8/11/2016

MyPastest

Back to Filters (/Secure/TestMe/Filter/429893/QA)

Question 61 of 207

A 25-year-old woman is diagnosed as having Graves disease with hyperthyroidism. She is


prescribed carbimazole treatment, and she has a number of questions concerning the
possible adverse effects.
Which of the following serious adverse effects are most likely to occur after carbimazole?
A

Cholestatic jaundice

Hepatitis

Hypoprothrombinaemia

Immunosuppression

Reversible agranulocytosis

Explanation

The answer is Reversible agranulocytosis


The active metabolite of carbimazole, methimazole, reduces the synthesis of new thyroid
hormones by inhibiting the iodination of tyrosine and blocks the coupling of
iodotyrosine. Methimazole also has a minor immunosuppressive action, leading to a
reduction in serum thyroid-stimulating-hormone-receptor antibody (TRAb)
concentrations. Symptoms improve within 1014 days, and euthyroid levels achieved after
34 weeks. Adverse effects are usually seen within 728 days of starting treatment.
Agranulocytosis cannot be predicted by routine white cell counts, but is fortunately
reversible when the drug is discontinued; patients are warned to stop the drug
immediately and contact their doctor should they develop a sore throat or fever.

Cholestatic jaundice (Option A) is incorrect. Cholestatic jaundice is a recognised feature but


this is very uncommon, and much less clinically significant than agranulocytosis.
Hepatitis (Option B) is incorrect. Hepatitis is a very rare complication of carbimazole
treatment.
Hypoprothrombinaemia (Option C) is incorrect. Hypoprothombinaemia may occur as thyroid
function falls to normal, but the clinical impact is very small, hence reversible
https://mypastest.pastest.com/Secure/TestMe/Browser/429893#Top

1/2

8/11/2016

MyPastest

agranulocystosis is the preferred answer.


Immunosuppression (Option D) is incorrect. Although an immunosuppressive effect is noted
concerning TSH receptor antibodies, the clinical significance is uncertain. Agranulocytosis is
a better characterised adverse effect and hence is the preferred answer.
46739

Next Question

Previous Question

Tag Question

Feedback

End Review

Difficulty: Average
Peer Responses

Session Progress
Responses Correct:

Responses Incorrect:

207

Responses Total:

207

Responses - % Correct:

0%

Blog (https://www.pastest.com/blog) About Pastest (https://www.pastest.com/about-us)


Contact Us (https://www.pastest.com/contact-us) Help (https://www.pastest.com/help)
Pastest 2016

https://mypastest.pastest.com/Secure/TestMe/Browser/429893#Top

2/2

8/11/2016

MyPastest

Back to Filters (/Secure/TestMe/Filter/429893/QA)

Question 62 of 207

A 51-year-old man with a history of alcoholism comes to the Cardiology Clinic for review. He
has failed a cardioversion for atrial fibrillation, and despite trying Bisoprolol and Flecainide,
remains in an irregular rhythm. On examination in the clinic his BP is 108/72 mmHg, pulse is
70/min (irregular). His chest is clear, there is minor bilateral pitting oedema affecting both
ankles. He has some spider naevi on examination of the torso.
Investigations:
Hb

10.9 g/dl

WCC

9.2 x109/l

PLT

211 x109/l

Na+

137 mmol/l

K+

4.3 mmol/l

Creatinine 88 micromol/l
CXR

reveals cardiomegaly

You are considering anti-coagulating him.


Which of the following is the strongest predictor of bleeding in this situation?
A

AST/ALT ratio

History of depression

Serum albumin < 35 g/l

Signs of fluid overload

Units alcohol per week

Explanation
The answer is AST/ALT ratio -

https://mypastest.pastest.com/Secure/TestMe/Browser/429893#Top

1/2

8/11/2016

MyPastest

One study, the Veterans AffaiRs Study to Improve Anticoagulation (VARIA) database of
103,897 patients receiving warfarin across 100 sites, provides us with useful data in this
respect. Among alcohol abusers, elevated AST: ALT >2.0 corresponded to more than three
times the number of haemorrhages (HR 3.02, p?<?0.001 compared to nonusers). It is also
likely that low albumin (i.e.<30), an indicator of impaired synthetic function is also associated
with risk of bleeding, although AST/ALT is the strongest predictor.
http://www.ncbi.nlm.nih.gov/pubmed/23620189
(http://www.ncbi.nlm.nih.gov/pubmed/23620189)
40174

Next Question

Previous Question

Tag Question

Feedback

End Review

Difficulty: Difficult
Peer Responses

Session Progress
Responses Correct:

Responses Incorrect:

207

Responses Total:

207

Responses - % Correct:

0%

Blog (https://www.pastest.com/blog) About Pastest (https://www.pastest.com/about-us)


Contact Us (https://www.pastest.com/contact-us) Help (https://www.pastest.com/help)
Pastest 2016

https://mypastest.pastest.com/Secure/TestMe/Browser/429893#Top

2/2

8/11/2016

MyPastest

Back to Filters (/Secure/TestMe/Filter/429893/QA)

Question 63 of 207

An 18-year-old young woman presents with a 4-day history of cough, headache, fever and
joint pains. Blood tests show the presence of raised antibody titres and the presence of cold
agglutinins. A diagnosis of Mycoplasma pneumoniae infection is made.
Which drug would be the most appropriate first-line treatment for this patient?
A

Cefuroxime

Clarithromycin

Co-trimoxazole

Penicillin

Rifampicin

Explanation

The answer is Clarithromycin


Clarithromycin or tetracyclines are preferred for Mycoplasma infections; tetracyclines are
also used to treat a Coxiella burnetii infection and psittacosis.

Cefuroxime (Option A) is incorrect. Cephalosporins are inactive against Mycoplasma


infection.
Co-trimoxazole (Option C) is incorrect. Co-trimoxazole is given in Pneumocystis jirovecii
(previously known as Pneumocystis carinii) infection.
Penicillin (Option D) is incorrect. Penicillin is used commonly in pneumococcal infection but
is ineffective against Mycoplasma pneumoniae.
Rifampicin (Option E) is incorrect. Rifampicin therapy may be effective in severe cases of
Mycoplasma pneumoniae but would rarely be considered as first line owing to a greater rate
of occurrence of adverse effects.
46744

Next Question
https://mypastest.pastest.com/Secure/TestMe/Browser/429893#Top

1/2

8/11/2016

MyPastest

Previous Question

Tag Question

Feedback

End Review

Difficulty: Average
Peer Responses

Session Progress
Responses Correct:

Responses Incorrect:

207

Responses Total:

207

Responses - % Correct:

0%

Blog (https://www.pastest.com/blog) About Pastest (https://www.pastest.com/about-us)


Contact Us (https://www.pastest.com/contact-us) Help (https://www.pastest.com/help)
Pastest 2016

https://mypastest.pastest.com/Secure/TestMe/Browser/429893#Top

2/2

8/11/2016

MyPastest

Back to Filters (/Secure/TestMe/Filter/429893/QA)

Question 64 of 207

A 60-year-old patient has been under regular blood pressure monitoring by the practice
nurse, and a decision is made to commence antihypertensive therapy. Two days later he
presents to the Emergency Department with a severe dizziness on standing, fast palpitations
and lightheadedness.
Which one of the following drugs is most likely to account for these symptoms?
A

Amlodipine

Atenolol

Doxazosin

Hydrochlorothiazide

Verapamil

Explanation

The answer is Doxazosin


Alpha-selective antagonists in particular can cause significant vasodilatation, orthostatic
hypotension and postural hypotension. Patients require careful dose titration and should
be warned of the possibility of postural dizziness. The same side-effects may also be
observed with ACE inhibitors, although these tend to lessen with continued therapy (first
dose hypotension).

Amlodipine (Option A) is incorrect. Dihydropyridine-type (eg nifedipine) calcium-channel


blockers may cause vasodilatation and postural dizziness, but to a lesser extent than
recognised after -blockers or ACE inhibitors.
Atenolol (Option B) is incorrect. Atenolol lowers blood pressure by significantly reducing
cardiac output and heart rate; it tends to increase peripheral vascular resistance, which is
counterproductive as monotherapy but makes the addition of a vasodilating drug a logical
choice.
Hydrochlorothiazide (Option D) is incorrect. Hydrochlorothiazide is expected to cause some
volume depletion, but its blood pressure lowering effects are of gradual onset over several
https://mypastest.pastest.com/Secure/TestMe/Browser/429893#Top

1/2

8/11/2016

MyPastest

weeks, and less likely to cause a dramatic blood pressure drop over several days.
Verapamil (Option E) is incorrect. Verapamil predominantly blocks cardiac calcium channels
in therapeutic doses, and is unlikely to cause significant vasodilatation and would suppress
heart rate.
46785

Next Question

Previous Question

Tag Question

Feedback

End Review

Difficulty: Average
Peer Responses

Session Progress
Responses Correct:

Responses Incorrect:

207

Responses Total:

207

Responses - % Correct:

0%

Blog (https://www.pastest.com/blog) About Pastest (https://www.pastest.com/about-us)


Contact Us (https://www.pastest.com/contact-us) Help (https://www.pastest.com/help)
Pastest 2016

https://mypastest.pastest.com/Secure/TestMe/Browser/429893#Top

2/2

8/11/2016

MyPastest

Back to Filters (/Secure/TestMe/Filter/429893/QA)

Question 65 of 207

You review a 58-year-old man with type-2 diabetes treated with metformin, and hypertension
that is being treated with nifedipine. His weight has increased and his glycaemic control is
poor, so you wish to escalate his medications. You consider adding a glitazone to his therapy.
Which metabolic pathway is mainly responsible for the metabolism of pioglitazone?
A

CYP2C8

CYP2C9

CYP2D6

CYP3A2

CYP3A4

Explanation

The answer is CYP2C8


The main cytochrome P450 enzyme pathway responsible for pioglitazone metabolism is
CYP2C8. Gemfibrozil and trimethoprim inhibit activity of the CYP2C8 isoenzyme and,
therefore, increase pioglitazone concentrations and increase the risk of adverse effects.
Pioglitazone lowers HbA1c by around 11.3% and raises HDL cholesterol. Its adverse
effects include weight gain, fluid retention, heart failure and reduced bone mineral
density.

CYP2C9 (Option B) is incorrect. CYP2C9 is responsible for metabolism of many non-steroidal


anti-inflammatory drugs (NSAIDs) and warfarin; it may be inhibited by fluconazole.
CYP2D6 (Option C) is incorrect. CYP2D6 is responsible for metabolism of a number of
antipsychotics, and converts codeine to morphine; it may be inhibited by fluoxetine and
paroxetine.
CYP3A2 (Option D) is incorrect. CYP3A2 catalyses conversion of testosterone to 6-hydroxytestosterone; it may be inhibited by macrolide antibiotics.

https://mypastest.pastest.com/Secure/TestMe/Browser/429893#Top

1/2

8/11/2016

MyPastest

CYP3A4 (Option E) is incorrect. CYP3A4 is responsible for metabolism of a very large


number of different drugs, including simvastatin; CYP3A4 may be inhibited by protease
inhibitors and macrolide antibiotics.
46881

Next Question

Previous Question

Tag Question

Feedback

End Review

Difficulty: Average
Peer Responses

Session Progress
Responses Correct:

Responses Incorrect:

207

Responses Total:

207

Responses - % Correct:

0%

Blog (https://www.pastest.com/blog) About Pastest (https://www.pastest.com/about-us)


Contact Us (https://www.pastest.com/contact-us) Help (https://www.pastest.com/help)
Pastest 2016

https://mypastest.pastest.com/Secure/TestMe/Browser/429893#Top

2/2

8/11/2016

MyPastest

Back to Filters (/Secure/TestMe/Filter/429893/QA)

Question 66 of 207

A 42-year-old man with long-standing epilepsy presents to the clinic for review. He complains
of increasing ataxia over the past few months and is particularly distressed by bilateral
Dupuytrens contractures that he is developing. On examination he has bilateral poor coordination, nystagmus and ataxia on walking; there is axillary and inguinal lymphadenopathy.
Sensory testing reveals decreased sensation in both feet.
Which one of the following medications is most likely to be responsible for these findings?
A

Carbamazepine

Lamotrigine

Phenytoin

Sodium valproate

Topiramate

Explanation

The answer is Phenytoin


Phenytoin has a range of adverse effects, which include peripheral neuropathy,
Dupuytrens contractures, benign lymphadenopathy, acne and gum hyperplasia.

Carbamazepine (Option A) is incorrect. Although carbamazepine may cause ataxia, it is


much less likely to cause peripheral neuropathy than phenytoin.
Lamotrigine (Option B) is incorrect. Lamotrigine is rarely associated with peripheral
neuropathy, and there is no association with Dupuytrens contracture.
Sodium valproate (Option D) is incorrect. Sodium valproate may be a suitable alternative
medication to substitute in this patient.
Topiramate (Option E) is incorrect. Neuropathy is rarely associated with topiramate.
46998

Next Question
https://mypastest.pastest.com/Secure/TestMe/Browser/429893#Top

1/2

8/11/2016

MyPastest

Previous Question

Tag Question

Feedback

End Review

Difficulty: Average
Peer Responses

Session Progress
Responses Correct:

Responses Incorrect:

207

Responses Total:

207

Responses - % Correct:

0%

Blog (https://www.pastest.com/blog) About Pastest (https://www.pastest.com/about-us)


Contact Us (https://www.pastest.com/contact-us) Help (https://www.pastest.com/help)
Pastest 2016

https://mypastest.pastest.com/Secure/TestMe/Browser/429893#Top

2/2

8/11/2016

MyPastest

Back to Filters (/Secure/TestMe/Filter/429893/QA)

Question 67 of 207

You review a 72-year-old woman who is complaining of severe nausea and lethargy. She has
chronic atrial fibrillation for which she takes digoxin 125 g/day. Her GP has recently added a
thiazide diuretic to her antihypertensive regime. Serum potassium level is 3.0 mmol/l (3.5
4.9). Her pulse is 42 bpm, with a blood pressure of 122/70 mmHg.
What is the best course of action in this case?
A

Administer FAB fragment antidigoxin antibodies

Introduce a small dose of spironolactone

Permanently stop her digoxin therapy

Start potassium supplements but continue diuretic therapy

Stop her thiazide diuretic, temporarily stop the digoxin, and substitute another
antihypertensive agent

Explanation

The answer is Stop her thiazide diuretic, temporarily stop the digoxin, and substitute
another antihypertensive agent
It is likely that, in this patient, the recent addition of the thiazide has precipitated a fall in
serum potassium concentration. The symptoms of digoxin toxicity include anorexia and
nausea, often with altered vision. Arrhythmias may occur, including ventricular premature
beats, bigeminy, ventricular tachycardia and atrioventricular (AV) block. Digoxin toxicity
is potentially severe when concentrations exceed 2.5 nmol/l. Management includes
cautious restoration of serum potassium levels, and symptomatic management of
arrhythmias. In severe cases of toxicity, digoxin may be permanently stopped and
another antiarrhythmic substituted if needed.

Administer FAB fragment antidigoxin antibodies (Option A) is incorrect. In this case there is
no circulatory compromise, and so a temporary reduction/cessation in digoxin therapy with
correction of serum potassium is the best course of action.

https://mypastest.pastest.com/Secure/TestMe/Browser/429893#Top

1/2

8/11/2016

MyPastest

Introduce a small dose of spironolactone (Option B) is incorrect. Spironolactone may be a


suitable alternative to bendroflumethiazide, but digoxin must be stopped, at least
temporarily, which is why option B is a less good answer.
Permanently stop her digoxin therapy (Option C) is incorrect. Digoxin can normally be
reintroduced, perhaps at a lower dose or after cessation of diuretic treatment.
Start potassium supplements but continue diuretic therapy (Option D) is incorrect. Shortterm potassium supplements may be considered, but it would be illogical to continue diuretic
treatment at this stage.
46875

Next Question

Previous Question

Tag Question

Feedback

End Review

Difficulty: Average
Peer Responses

Session Progress
Responses Correct:

Responses Incorrect:

207

Responses Total:

207

Responses - % Correct:

0%

Blog (https://www.pastest.com/blog) About Pastest (https://www.pastest.com/about-us)


Contact Us (https://www.pastest.com/contact-us) Help (https://www.pastest.com/help)
Pastest 2016

https://mypastest.pastest.com/Secure/TestMe/Browser/429893#Top

2/2

8/11/2016

MyPastest

Back to Filters (/Secure/TestMe/Filter/429893/QA)

Question 68 of 207

A total of 630 mg of a new investigational intravenous anaesthetic agent is injected into a 28year-old man as part of a phase 2 study. The half-life of the agent is 30 min.
How much time will it take before the total body drug content falls below 20 mg?
A

90 min

2h

150 min

4h

8h

Explanation

The answer is 150 min


Half-life is the time taken for the drug concentration to fall by 50%. 20 mg is
approximately 1/32nd of a 630-mg dose given IV originally; that is to say 1/32nd = 1/2
1/2 1/2 1/2 1/2, or five sequential half-lives; 5 30 min = 150 min.

90 min (Option A) is incorrect. It will take five half-lives, namely 150 min.
2 h (Option B) is incorrect. It will take five half-lives, namely 150 min.
4 h (Option D) is incorrect. It will take five half-lives, namely 150 min.
8 h (Option E) is incorrect. It will take five half-lives, namely 150 min.
46984

Next Question

Previous Question

Tag Question

https://mypastest.pastest.com/Secure/TestMe/Browser/429893#Top

Feedback

End Review

1/2

8/11/2016

MyPastest

Difficulty: Average
Peer Responses

Session Progress
Responses Correct:

Responses Incorrect:

207

Responses Total:

207

Responses - % Correct:

0%

Blog (https://www.pastest.com/blog) About Pastest (https://www.pastest.com/about-us)


Contact Us (https://www.pastest.com/contact-us) Help (https://www.pastest.com/help)
Pastest 2016

https://mypastest.pastest.com/Secure/TestMe/Browser/429893#Top

2/2

8/11/2016

MyPastest

Back to Filters (/Secure/TestMe/Filter/429893/QA)

Question 69 of 207

A 35-year-old woman, who is receiving fortnightly intramuscular injections for rheumatoid


arthritis, presents to her GP with sore throat, cough, and tingling and numbness in her hands
and feet. Full blood count shows haemoglobin 78 g/l, white cell count 1.8 109/l and platelets
34 109/l.
Which of the following medications is most likely to account for these findings?
A

Gold

Hydroxychloroquine

Indometacin

Methotrexate

Sulfasalazine

Explanation

The answer is Gold


Gold (sodium aurothiomalate) is given by deep intramuscular injection, initially weekly,
then fortnightly, and maintained monthly thereafter. A number of serious adverse effects
are recognised, including peripheral neuropathy, skin reactions, nephrotic syndrome,
pulmonary fibrosis and aplastic pancytopenia.

Hydroxychloroquine (Option B) is incorrect. Hydroxychloroquine may cause bone marrow


suppression and pancytopenia. Gold is the preferred answer because hydroxychloroquine
would be less likely to cause neuropathy and would not be given by monthly injection (oral
administration).
Indometacin (Option C) is incorrect. Indometacin and other NSAIDs may promote
gastrointestinal blood loss and iron deficiency anaemia; these are normally administered
orally.
Methotrexate (Option D) is incorrect. Methotrexate may cause hepatic and pulmonary
fibrosis; it is normally given weekly by oral, subcutaneous or intramuscular routes.
https://mypastest.pastest.com/Secure/TestMe/Browser/429893#Top

1/2

8/11/2016

MyPastest

Sulfasalazine (Option E) is incorrect. Sulfasalazine may cause bone marrow suppression and
pancytopenia. Gold is the preferred answer because sulfasalazine would be less likely to
cause neuropathy and would not be given by monthly injection (oral administration).
46615

Next Question

Previous Question

Tag Question

Feedback

End Review

Difficulty: Average
Peer Responses

Session Progress
Responses Correct:

Responses Incorrect:

207

Responses Total:

207

Responses - % Correct:

0%

Blog (https://www.pastest.com/blog) About Pastest (https://www.pastest.com/about-us)


Contact Us (https://www.pastest.com/contact-us) Help (https://www.pastest.com/help)
Pastest 2016

https://mypastest.pastest.com/Secure/TestMe/Browser/429893#Top

2/2

8/11/2016

MyPastest

Back to Filters (/Secure/TestMe/Filter/429893/QA)

Question 70 of 207

A 40-year-old man has undergone bone marrow transplant treatment for leukaemia.
Unfortunately, his recovery has been complicated by the occurrence of systemic fungal
infection, and he is prescribed amphotericin.
Which of the following statements best describes the pharmacological characteristic of this
drug?
A

Adverse effects are idiosyncratic and unpredictable

Adverse effects are uncommon after intravenous use

It has a low affinity for ergosterol present in the fungal cell wall

It may be administered intramuscularly

Lipid-bound preparations cause fewer adverse effects

Explanation

The answer is Lipid-bound preparations cause fewer adverse effects


Amphotericin is a polyene antifungal agent that binds to ergosterol and thereby forms
micropores. These increase fungal cell wall permeability and cause lysis. Amphotericin
may exert either fungicidal or fungistatic activity, depending on its concentration at the
site of infection and sensitivity of the organism. It is administered intravenously and
adverse effects are common, the most important being dose-related nephrotoxicity and
hypokalaemia. Acute infusion-related adverse effects include fever and rigors,
hypotension, nausea and vomiting, headache, and thrombophlebitis. Adverse effects
associated with long-term therapy include anaemia, and renal tubular loss of potassium,
sodium and magnesium. Lipisomal preparations are associated with less toxicity.

Adverse effects are idiosyncratic and unpredictable (Option A) is incorrect. Adverse effects
are dose and concentration dependent, and therefore predictable.
Adverse effects are uncommon after intravenous use (Option B) is incorrect. Adverse effects
occur commonly.

https://mypastest.pastest.com/Secure/TestMe/Browser/429893#Top

1/2

8/11/2016

MyPastest

It has a low affinity for ergosterol present in the fungal cell wall (Option C) is incorrect.
Ergosterol is a major therapeutic target for amphotericin.
It may be administered intramuscularly (Option D) is incorrect. Amphotericin may only be
administered by intravenous route.
46863

Next Question

Previous Question

Tag Question

Feedback

End Review

Difficulty: Average
Peer Responses

Session Progress
Responses Correct:

Responses Incorrect:

207

Responses Total:

207

Responses - % Correct:

0%

Blog (https://www.pastest.com/blog) About Pastest (https://www.pastest.com/about-us)


Contact Us (https://www.pastest.com/contact-us) Help (https://www.pastest.com/help)
Pastest 2016

https://mypastest.pastest.com/Secure/TestMe/Browser/429893#Top

2/2

8/11/2016

MyPastest

Back to Filters (/Secure/TestMe/Filter/429893/QA)

Question 71 of 207

A 49-year-old man has returned to the hypertension clinic for review. He is complaining of an
irritating dry cough that has developed after he commenced treatment with lisinopril.
Which of the following mechanisms is most likely to account for development of cough?
A

Bronchoconstriction

Excess bradykinin within lung parenchyma

Hypersalivation

Increased bronchial mucous secretion

Vasodilation causing pulmonary congestion

Explanation

The answer is Excess bradykinin within lung parenchyma


ACE inhibitors prevent the breakdown of bradykinin within the lungs, and coughing is
thought to be triggered by excess bradykinin that accumulates. Cough due to ACE
inhibitors is reported in 10% of men and 20% of women, although the reason for a gender
difference is unclear. Other adverse effecs of ACE inhibitors include dysgeusia (an
unpleasant metallic taste), postural hypotension, first dose hypotension and renal failure.

Bronchoconstriction (Option A) is incorrect. This is not a recognised feature.


Hypersalivation (Option C) is incorrect. This is not a recognised feature.
Increased bronchial mucous secretion (Option D) is incorrect. This is not a recognised
feature.
Vasodilation causing pulmonary congestion (Option E) is incorrect. ACE inhibitors are known
to cause systemic vasodilation and possibly also within the pulmonary vasculature, but this is
not thought to be relevant to the development of cough.
46601

Next Question
https://mypastest.pastest.com/Secure/TestMe/Browser/429893#Top

1/2

8/11/2016

MyPastest

Previous Question

Tag Question

Feedback

End Review

Difficulty: Average
Peer Responses

Session Progress
Responses Correct:

Responses Incorrect:

207

Responses Total:

207

Responses - % Correct:

0%

Blog (https://www.pastest.com/blog) About Pastest (https://www.pastest.com/about-us)


Contact Us (https://www.pastest.com/contact-us) Help (https://www.pastest.com/help)
Pastest 2016

https://mypastest.pastest.com/Secure/TestMe/Browser/429893#Top

2/2

8/11/2016

MyPastest

Back to Filters (/Secure/TestMe/Filter/429893/QA)

Question 72 of 207

A 17-year-old boy presents to the endocrinology outpatient clinic for investigation of


anosmia. His parents mention that he is childlike and has not developed secondary sexual
characteristics like his peers. On examination he is overweight but euthyroid, and you note
the absence of secondary hair growth.
What treatment would you consider most appropriate in this condition?
A

Buserelin

Cyclic oestrogen and progestogen

Human chorionic gonadotrophin

Nafarelin acetate

Octreotide

Explanation

The answer is Human chorionic gonadotrophin


This 17-year-old boy, presenting with obesity, anosmia and failure to develop secondary
sexual characteristics, most probably has Kallmann syndrome. This syndrome is a
combination of anosmia, obesity and hypogonadotrophic hypogonadism, and is an Xlinked recessive disorder that causes an isolated deficiency of gonadotrophin-releasing
hormone (GnRH). Long-term treatment of males is with human chorionic gonadotrophin
or testosterone, which allows restoration of pubertal development and secondary sex
characteristics.

Buserelin (Option A) is incorrect. Buserelin is a GnRH analogue that initially stimulates and
later inhibits release of FSH and LH, and is used to suppress the growth of some hormonesensitive tumours, eg prostate cancer. Pulsed GnRH analogue therapy may, however, be used
in an attempt to maintain secondary sexual characteristics and fertility in males who require
it.
Cyclic oestrogen and progestogen (Option B) is incorrect. Cyclic oestrogen and progestogen
are indicated only for female patients.
https://mypastest.pastest.com/Secure/TestMe/Browser/429893#Top

1/2

8/11/2016

MyPastest

Nafarelin acetate (Option D) is incorrect. Nafarelin acetate is a GnRH analogue that inhibits
pituitary release of FSH and LH, and is used to suppress hormone-sensitive disorders
including endometriosis and uterine fibroids.
Octreotide (Option E) is incorrect. Octreotide is a long-acting analogue of somatostatin and
is used in the treatment of acromegaly. It also suppresses the luteinising hormone response
to GnRH.
46750

Next Question

Previous Question

Tag Question

Feedback

End Review

Difficulty: Average
Peer Responses

Session Progress
Responses Correct:

Responses Incorrect:

207

Responses Total:

207

Responses - % Correct:

0%

Blog (https://www.pastest.com/blog) About Pastest (https://www.pastest.com/about-us)


Contact Us (https://www.pastest.com/contact-us) Help (https://www.pastest.com/help)
Pastest 2016

https://mypastest.pastest.com/Secure/TestMe/Browser/429893#Top

2/2

8/11/2016

MyPastest

Back to Filters (/Secure/TestMe/Filter/429893/QA)

Question 73 of 207

You review a 45-year-old woman in the respiratory clinic for review of her asthma symptoms.
After discussion with the consultant, you decide to prescribe aminophylline. The patient asks
you about the possible side effects of the medication.
Which of the following adverse effects is most likely to occur?
A

Agitation and insomnia

Arrhythmias

Diarrhoea

Hyperkalaemia

Hypotension

Explanation

The answer is Agitation and insomnia


Theophylline is the pharmacologically active molecule but is sparingly soluble in water;
aminophylline is a combination of theophylline and ethylenediamine that is water soluble
and increases theophylline absorption around 20-fold. Aminophylline is normally
reserved for patients with severe asthma because there is a high rate of occurrence of
adverse effects, including effets on the central nervous system: headache, insomnia,
agitation, seizures, severe nausea and vomiting. Gastrointestinal irritation may occur
causing vomiting and peptic ulceration. Metabolic disturbances include hypokalaemia
and metabolic acidosis. In the setting of theophylline toxicity or overdose, hypotension,
profound hypokalaemia, and ventricular and supraventricular arrhythmias may occur,
especially when drug concentrations are > 25 g/ml.

Arrhythmias (Option B) is incorrect. Arrhythmias are anticipated only in the setting of


theophylline toxicity, and therapeutic drug monitoring is required to minimise the risk of
toxicity.
Diarrhoea (Option C) is incorrect. Diarrhoea is a recognised feature but is much less common
than nausea, vomiting, headache, agitation and insomnia.
https://mypastest.pastest.com/Secure/TestMe/Browser/429893#Top

1/2

8/11/2016

MyPastest

Hyperkalaemia (Option D) is incorrect. Theophylline is associated with hypokalaemia,


particularly in the context of theophylline toxicity.
Hypotension (Option E) is incorrect. Hypotension is a recognised feature of theophylline
toxicity, but does not normally occur after therapeutic doses.
46600

Next Question

Previous Question

Tag Question

Feedback

End Review

Difficulty: Average
Peer Responses

Session Progress
Responses Correct:

Responses Incorrect:

207

Responses Total:

207

Responses - % Correct:

0%

Blog (https://www.pastest.com/blog) About Pastest (https://www.pastest.com/about-us)


Contact Us (https://www.pastest.com/contact-us) Help (https://www.pastest.com/help)
Pastest 2016

https://mypastest.pastest.com/Secure/TestMe/Browser/429893#Top

2/2

8/11/2016

MyPastest

Back to Filters (/Secure/TestMe/Filter/429893/QA)

Question 74 of 207

You are asked to review a patient attending the endocrine clinic for investigation of
galactorrhoea.
Which of the following drugs is most likely to cause this adverse effect?
A

Atenolol

Furosemide

Metoclopramide

Rifampicin

Verapamil

Explanation

The answer is Metoclopramide


Metoclopramide causes extrapyramidal effects (especially in children and young adults),
hyperprolactinaemia and occasionally tardive dyskinesia on prolonged administration.
Hyperprolactinaemia may give rise to galactorrhoea. Other adverse effects of
metoclopramide include drowsiness, restlessness, diarrhoea, depression, neuroleptic
malignant syndrome, rashes, pruritus and oedema. Rare adverse effects include cardiac
conduction abnormalities, including methaemoglobinaemia (more severe in glucose-6phosphate dehydrogenase deficiency). Other drug causes include chlorpromazine,
haloperidol, metoclopramide, -methyldopa, opioids, fluoxetine and ranitidine.
Recreational agents including cocaine and marijuana are also recognised causes.

Atenolol (Option A) is incorrect. Atenolol is not a recognised cause.


Furosemide (Option B) is incorrect. Furosemide is not a recognised cause.
Rifampicin (Option D) is incorrect. Rifampicin is not a recognised cause.
Verapamil (Option E) is incorrect. Verapamil is a recognised cause, but is less commonly
implicated than metoclopramide, which is why metoclopramide is the preferred answer.
46945

https://mypastest.pastest.com/Secure/TestMe/Browser/429893#Top

1/2

8/11/2016

MyPastest

Next Question

Previous Question

Tag Question

Feedback

End Review

Difficulty: Average
Peer Responses

Session Progress
Responses Correct:

Responses Incorrect:

207

Responses Total:

207

Responses - % Correct:

0%

Blog (https://www.pastest.com/blog) About Pastest (https://www.pastest.com/about-us)


Contact Us (https://www.pastest.com/contact-us) Help (https://www.pastest.com/help)
Pastest 2016

https://mypastest.pastest.com/Secure/TestMe/Browser/429893#Top

2/2

8/11/2016

MyPastest

Back to Filters (/Secure/TestMe/Filter/429893/QA)

Question 75 of 207

A 39-year-old woman who is planning to travel overseas to a malaria-endemic area later this
month presents to the Emergency Department after an intentional overdose of her
chloroquine medications.
Which of the following features is most likely attributable to chloroquine toxicity?
A

Acute blindness

Hyperglycaemia

Hyperkalaemia

Increased PR interval on the ECG

QRS prolongation on the ECG

Explanation

The answer is QRS prolongation on the ECG


Symptoms of chloroquine toxicity includes nausea, headache, visual disturbance, cardiac
arrhythmia, convulsions and coma. Ingestion of more than 2 g in an adult may be fatal.
Hypoglycaemia is a recognised adverse effect. Treatment includes oral activated
charcoal, which should be given to patients who present within 1 hour. Chloroquine
interferes with cardiac sodium channel conductance, which results in prolongation of the
QRS duration.

Acute blindness (Option A) is incorrect. Quinine toxicity is similar to chloroquine, and in


addition quinine may cause blindness, normally 1620 hours after ingestion.
Hyperglycaemia (Option B) is incorrect. Hypoglycaemia is a recognised adverse effect but is
a less specific indicator of severe toxicity than QRS prolongation.
Hyperkalaemia (Option C) is incorrect. Hypokalaemia is a recognised feature, although this
appears to confer some cardioprotective effects, and potassium replacement should be
cautious, ideally avoided within the first 8 hours after ingestion, if possible.

https://mypastest.pastest.com/Secure/TestMe/Browser/429893#Top

1/2

8/11/2016

MyPastest

Increased PR interval on the ECG (Option D) is incorrect. Chloroquine may interfere with
sodium channel conductance, giving rise to QRS prolongation.
45800

Next Question

Previous Question

Tag Question

Feedback

End Review

Difficulty: Average
Peer Responses

Session Progress
Responses Correct:

Responses Incorrect:

207

Responses Total:

207

Responses - % Correct:

0%

Blog (https://www.pastest.com/blog) About Pastest (https://www.pastest.com/about-us)


Contact Us (https://www.pastest.com/contact-us) Help (https://www.pastest.com/help)
Pastest 2016

https://mypastest.pastest.com/Secure/TestMe/Browser/429893#Top

2/2

8/11/2016

MyPastest

Back to Filters (/Secure/TestMe/Filter/429893/QA)

Question 76 of 207

A 42-year-old woman has recently undergone a total abdominal hysterectomy and bilateral
salpingo-oophorectomy for pelvic inflammatory disease. She asks you about the risks and
benefits of oestrogen replacement therapy.
Which of the following statements is most accurate regarding long-term oestrogen therapy?
A

A combination of oestrogen and progesterone is more effective than oestrogen


alone

Loss of trabecular and not cortical bone is prevented by oestrogen therapy

Oestrogen therapy may fully prevent bone loss

Oestrogen therapy reduces the occurrence of osteoporotic fractures in old age

The benefits of oestrogen are greatest when treatment is maintained more than 10
years

Explanation

The answer is Oestrogen therapy reduces the occurrence of osteoporitic fractures in old
age
Osteoporosis occurs in all races. Reduction in total-body bone mass begins in women in
their late 20s and progresses through adult life. The rate of bone loss is accelerated at
the time of menopause. Oestrogen minimises the rate of trabecular bone loss in
vertebrae and cortical bone loss at the radius.

A combination of oestrogen and progesterone is more effective than oestrogen alone


(Option A) is incorrect. As the uterus has been removed in this patient, there is no need for
additional progesterone therapy.
Loss of trabecular and not cortical bone is prevented by oestrogen therapy (Option B) is
incorrect. Oestrogen replacement is capable of preventing trabecular and cortical bone loss.
Oestrogen therapy may fully prevent bone loss (Option C) is incorrect. Signficant bone loss
will have already occurred by age 42, and oestrogen replacement will slow the rate of
ongoing bone loss but not prevent it fully.
https://mypastest.pastest.com/Secure/TestMe/Browser/429893#Top

1/2

8/11/2016

MyPastest

The benefits of oestrogen are greatest when treatment is maintained more than 10 years
(Option E) is incorrect. The benefits of oestrogen therapy for preventing bone loss do not
extend beyond 10 years.
46594

Next Question

Previous Question

Tag Question

Feedback

End Review

Difficulty: Average
Peer Responses

Session Progress
Responses Correct:

Responses Incorrect:

207

Responses Total:

207

Responses - % Correct:

0%

Blog (https://www.pastest.com/blog) About Pastest (https://www.pastest.com/about-us)


Contact Us (https://www.pastest.com/contact-us) Help (https://www.pastest.com/help)
Pastest 2016

https://mypastest.pastest.com/Secure/TestMe/Browser/429893#Top

2/2

8/11/2016

MyPastest

Back to Filters (/Secure/TestMe/Filter/429893/QA)

Question 77 of 207

A 40-year-old bank clerk has been receiving phenelzine for severe depression under the care
of the psychiatry team. She is brought to the Emergency Department in an agitated state
with high fever, tremor and agitation. Her partner says that 1 week earlier, an out-of-hours GP
had prescribed an additional medication to address her worsening low mood.
Which of the following is most likely to have provoked the adverse reaction?
A

Fluoxetine

Isocarboxazid

Lithium

St Johns Wort

Tranylcypromide

Explanation

The answer is Fluoxetine


A dangerous pharmacodynamic interaction can occur when fluoxetine or one of the
newer selective serotonin-reuptake inhibitors (SSRIs) is used in the presence of a
monoamine oxidase inhibitor (MAOI). The combination of increased stores of the
monoamine plus inhibition of reuptake after release is thought to result in marked
increases of serotonin in the synapses, leading to a serotonin syndrome. This rare but
serious disorder is characterised by agitation, hyperthermia, dysarthria, tremor, muscle
rigidity, myoclonus and autonomic instability. A similar interaction may occur with other
drugs capable of increasing serotonergic activity, including tricyclic antidepressants,
antipsychotics and tramadol.

Isocarboxazid (Option B) is incorrect. Isocarboxazid is an MAOI that may also provoke


serotonergic syndrome in combination with other agents, but because it shares the same
basic mechanism as phenylzine it is less likely to provoke serotonergic syndrome than the
combination of MAOI and SSRI.

https://mypastest.pastest.com/Secure/TestMe/Browser/429893#Top

1/2

8/11/2016

MyPastest

Lithium (Option C) is incorrect. Lithium is not a significant risk factor for serotonergic
syndrome.
St Johns Wort (Option D) is incorrect. St Johns Wort may be used to treat mild depression,
but it is not known to significantly increase serotonin release within the central nervous
system.
Tranylcypromide (Option E) is incorrect. Tranylcypromine is an MAOI that may also provoke
serotonergic syndrome in combination with other agents, but because it shares the same
basic mechanism as phenylzine it is less likely to provoke serotonergic syndrome than the
combination of MAOI and SSRI.
46782

Next Question

Previous Question

Tag Question

Feedback

End Review

Difficulty: Average
Peer Responses

Session Progress
Responses Correct:

Responses Incorrect:

207

Responses Total:

207

Responses - % Correct:

0%

Blog (https://www.pastest.com/blog) About Pastest (https://www.pastest.com/about-us)


Contact Us (https://www.pastest.com/contact-us) Help (https://www.pastest.com/help)
Pastest 2016

https://mypastest.pastest.com/Secure/TestMe/Browser/429893#Top

2/2

8/11/2016

MyPastest

Back to Filters (/Secure/TestMe/Filter/429893/QA)

Question 78 of 207

A 56-year-old patient is currently being treated for rheumatoid arthritis, depression and
epilepsy. He presents with bilateral central visual field defects.
Which of the following drugs is most likely to be responsible for this adverse effect?
A

Amitriptyline

Carbamazepine

Hydroxychloroquine

Prednisolone

Vigabatrin

Explanation

The answer is Vigabatrin


Vigabatrin is associated with visual field defects. The onset of symptoms varies from 1
month to several years after starting therapy. In most cases, visual field defects persist
despite discontinuation of the drug. Visual field assessment should be carried out before
treatment and at 6-month intervals, and patients should be warned to report any new
visual symptoms that develop so they can be referred for an urgent ophthalmological
opinion; gradual withdrawal of vigabatrin should be considered.

Amitriptyline (Option A) is incorrect. Amitriptyline may cause mydriasis owing to its


anticholinergic properties.
Carbamazepine (Option B) is incorrect. Carbamazepine may cause nystagmus, especially in
high doses.
Hydroxychloroquine (Option C) is incorrect. Hydroxychloroquine may cause retinopathy, but
this is a very rare adverse effect, which is why vigabatrin is the preferred answer.
Prednisolone (Option D) is incorrect. Prednisolone may predispose to development of
cataracts.
46941

https://mypastest.pastest.com/Secure/TestMe/Browser/429893#Top

1/2

8/11/2016

MyPastest

Next Question

Previous Question

Tag Question

Feedback

End Review

Difficulty: Average
Peer Responses

Session Progress
Responses Correct:

Responses Incorrect:

207

Responses Total:

207

Responses - % Correct:

0%

Blog (https://www.pastest.com/blog) About Pastest (https://www.pastest.com/about-us)


Contact Us (https://www.pastest.com/contact-us) Help (https://www.pastest.com/help)
Pastest 2016

https://mypastest.pastest.com/Secure/TestMe/Browser/429893#Top

2/2

8/11/2016

MyPastest

Back to Filters (/Secure/TestMe/Filter/429893/QA)

Question 79 of 207

A 60-year-old man presents with polyuria and is noted to have a serum sodium level of 159
mmol/l (137144). A diagnosis of nephrogenic diabetes insipidus is considered.
Which of the following medications would be most likely to have caused this?
A

Aspirin

Demeclocycline

Methotrexate

Propranolol

Thiazide diuretics

Explanation

The answer is Demeclocycline


Nephrogenic diabetes insipidus is caused by acquired resistance of the renal tubules to
vasopressin (antidiuretic hormone, ADH). Drugs capable of causing nephrogenic
diabetes insipidus include: lithium, demeclocycline, amphotericin and glibenclamide. This
property of demeclocycline is occasionally exploited in the treatment of syndrome of
inappropriate ADH secretion.

Aspirin (Option A) is incorrect. Low-dose aspirin interferes with renal tubular secretion of
urate, so that in time total body urate accumulates and increases the risk of precipitating
acute gout.
Methotrexate (Option C) is incorrect. Methotrexate may cause retroperitoneal fibrosis and
obstructive uropathy.
Propranolol (Option D) is incorrect. Beta-blockers and ACE inhibitors are recognised causes
of type-4 renal tubular acidosis.
Thiazide diuretics (Option E) is incorrect. Thiazide diuretics cause sodium and potassium
depletion.
46903

https://mypastest.pastest.com/Secure/TestMe/Browser/429893#Top

1/2

8/11/2016

MyPastest

Next Question

Previous Question

Tag Question

Feedback

End Review

Difficulty: Average
Peer Responses

Session Progress
Responses Correct:

Responses Incorrect:

207

Responses Total:

207

Responses - % Correct:

0%

Blog (https://www.pastest.com/blog) About Pastest (https://www.pastest.com/about-us)


Contact Us (https://www.pastest.com/contact-us) Help (https://www.pastest.com/help)
Pastest 2016

https://mypastest.pastest.com/Secure/TestMe/Browser/429893#Top

2/2

8/11/2016

MyPastest

Back to Filters (/Secure/TestMe/Filter/429893/QA)

Question 80 of 207

A 72-year-old man presents with acute-onset lumbar spine pain. This occurred while he was
digging the vegetable patch at home; he tried paracetamol and ibuprofen at home with little
effect. His daughter brought him to the Emergency Department as he was unable to mobilise
to the toilet at home because of pain. There is no significant neurology on examination. He
has a history of chronic obstructive pulmonary disease (COPD) and rheumatoid arthritis. He
takes high-dose seretide for his COPD and low-dose prednisolone (5 mg) for his rheumatoid
arthritis. X-ray reveals an osteoporotic fracture of L4.
Which of the following would be the most appropriate short-term pain relief in this case?
A

Naproxen 1.5 g bd

Paracetamol 1 g po qds

Prednisolone 40 mg od

Tramadol 50100 mg qds

Tramadol 50100 mg qds and paracetamol 1 g qds

Explanation

The answer is Tramadol 50100 mg qds and paracetamol 1 g qds


This man (a 72-year-old with a history of COPD) has an acute osteoporotic fracture of
the lumbar spine, likely caused by chronic use of corticosteroids. Any single class of
analgesic is unlikely to provide adequate pain relief and hence the combination of
tramadol and paracetamol may be most effective. Suitable longer-term treatments for
his osteoporosis would include either the use of bisphosphonate therapy or calcium and
vitamin D supplementation.

Naproxen 1.5 g bd (Option A) is incorrect. A non-steroidal drug may be an effective


analgesic, but this high dose of naproxen would be hazardous in view of the risk of peptic
ulceration, particularly in the context of corticosteroid use. Moderate non-steroidal antiinflammatory drug (NSAID) doses could be considered, and gastroprotection with a protein
pump inhibitor may be needed.
https://mypastest.pastest.com/Secure/TestMe/Browser/429893#Top

1/2

8/11/2016

MyPastest

Paracetamol 1 g po qds (Option B) is incorrect. Although regular paracetamol may be an


important component, it is unlikely to offer sufficient analgesia by itself.
Prednisolone 40 mg od (Option C) is incorrect. Increased prednisolone doses may offer antiinflammatory effects, but would not provide effective analgesia in the setting of vertebral
fracture.
Tramadol 50100 mg qds (Option D) is incorrect. Tramadol may offer effective analgesia but
option E is preferred since the combination of tramadol with paracetamol is likely to be more
effective, and avoid the need for very large doses of tramadol as monotherapy.
46949

Next Question

Previous Question

Tag Question

Feedback

End Review

Difficulty: Average
Peer Responses

Session Progress
Responses Correct:

Responses Incorrect:

207

Responses Total:

207

Responses - % Correct:

0%

Blog (https://www.pastest.com/blog) About Pastest (https://www.pastest.com/about-us)


Contact Us (https://www.pastest.com/contact-us) Help (https://www.pastest.com/help)
Pastest 2016

https://mypastest.pastest.com/Secure/TestMe/Browser/429893#Top

2/2

8/11/2016

MyPastest

Back to Filters (/Secure/TestMe/Filter/429893/QA)

Question 81 of 207

You are responsible for treating a 56-year-old woman with an acute attack of gout affecting
her left knee. She had previously been receiving allopurinol treatment.
Which of the following statements most accurately depicts the effects of allopurinol in the
setting of acute gout?
A

Complete remission of symptoms within a few days of treatment

Exacerbation and prolongation of the attack

May cause renal impairment with doses > 300 mg/day

Reduces swelling and erythema of the affected joint

Reduction of uric acid levels to normal within a week of commencing treatment

Explanation

The answer is Exacerbation and prolongation of the attack


Non-steroidal anti-inflammatory drugs (NSAIDs) in high doses are the drugs of choice in
acute gout; colchicine is a suitable alternative but may cause diarrhoea. Allopurinol is
used when the attacks are frequent and severe, or there is associated renal impairment.
Initiation of allopurinol treatment during an attack can exacerbate and prolong the
episode, hence it is prudent to wait until the attack settles. Allopurinol starting dose is
normally 100 mg/day building up to 300 mg/day or more if needed to prevent acute
gout episodes.

Complete remission of symptoms within a few days of treatment (Option A) is incorrect.


Allopurinol will not be expected to allow prompt resolution of acute gout. High doses of
NSAIDs may allow remission of acute gout, typically within 510 days.
May cause renal impairment with doses > 300 mg/day (Option C) is incorrect. Renal
impairment impairs urate clearance, leading to accumulation of urate within the body and
increased risk of gout.
Reduces swelling and erythema of the affected joint (Option D) is incorrect. NSAIDs and
colchicine will reduce acute inflammation of the affected joint, but allopurinol would not be
https://mypastest.pastest.com/Secure/TestMe/Browser/429893#Top

1/2

8/11/2016

MyPastest

expected to have any significant effect.


Reduction of uric acid levels to normal within a week of commencing treatment (Option E) is
incorrect. Allopurinol may lower serum urate by around 2030%; with prolonged therapy, the
aim of using allopurinol is to lower total body urate content and reduce the risk of acute gout
episodes.
46604

Next Question

Previous Question

Tag Question

Feedback

End Review

Difficulty: Average
Peer Responses

Session Progress
Responses Correct:

Responses Incorrect:

207

Responses Total:

207

Responses - % Correct:

0%

Blog (https://www.pastest.com/blog) About Pastest (https://www.pastest.com/about-us)


Contact Us (https://www.pastest.com/contact-us) Help (https://www.pastest.com/help)
Pastest 2016

https://mypastest.pastest.com/Secure/TestMe/Browser/429893#Top

2/2

8/11/2016

MyPastest

Back to Filters (/Secure/TestMe/Filter/429893/QA)

Question 82 of 207

A patient with AIDS has been prescribed a non-nucleoside reverse transcriptase inhibitor.
Which of the following drugs acts predominantly by this mechanism?
A

Abacavir

Lopinavir R

Nelfinavir

Nevirapine

Stavudine

Explanation

The answer is Nevirapine


Nevirapine is the only non-nucleoside reverse transcriptase inhibitor in this list.

Abacavir (Option A) is incorrect. Abacavir is a nucleoside analogue.


Lopinavir R (Option B) is incorrect. Lopinavir R is a protease inhibitor.
Nelfinavir (Option C) is incorrect. Nelfinavir is a protease inhibitor.
Stavudine (Option E) is incorrect. Stavudine is a nucleoside analogue.
46870

Next Question

Previous Question

Tag Question

Feedback

End Review

Difficulty: Average
Peer Responses
https://mypastest.pastest.com/Secure/TestMe/Browser/429893#Top

1/2

8/11/2016

MyPastest

Session Progress
Responses Correct:

Responses Incorrect:

207

Responses Total:

207

Responses - % Correct:

0%

Blog (https://www.pastest.com/blog) About Pastest (https://www.pastest.com/about-us)


Contact Us (https://www.pastest.com/contact-us) Help (https://www.pastest.com/help)
Pastest 2016

https://mypastest.pastest.com/Secure/TestMe/Browser/429893#Top

2/2

8/11/2016

MyPastest

Back to Filters (/Secure/TestMe/Filter/429893/QA)

Question 83 of 207

You review a 69-year-old man in the cardiology clinic who is attending for routine follow-up
of his angina. He is receiving clopidogrel, simvastatin, bisoprolol, amlodipine and ramipril. He
tells you that he has been experiencing occasional angina on exertion, and you consider that
a trial of nicorandil therapy is appropriate.
Which of the following statements best describes the pharmacological effects of nicorandil?
A

Headache occurs in 24% of patients

It increases ventricular filling pressures

It inhibits ATP-dependent potassium channels

It reverses the hypotensive effect of sildenafil

Oral ulceration is a recognised adverse effect

Explanation

The answer is Oral ulceration is a recognised adverse effect


Nicorandil is an activator of ATP-dependent potassium channels. Its pharmacological
actions are relaxation of smooth muscle in veins and increased venous capacitance,
which leads to reduced ventricular filling pressures and dilatation of the coronary
arterioles. Headache is the most common unwanted effect (~35% of patients), which
appears to be dose-dependent and lessens with continued treatment. Other adverse
effects include oral ulceration, flushing and gastrointestinal disturbance.

Headache occurs in 24% of patients (Option A) is incorrect. Headache occurs in up to 35%


of patients initiated on nicorandil.
It increases ventricular filling pressures (Option B) is incorrect. It reduces venous return to
the heart and filling pressures.
It inhibits ATP-dependent potassium channels (Option C) is incorrect. Nicorandil activates
ATP-dependent potassium channels.

https://mypastest.pastest.com/Secure/TestMe/Browser/429893#Top

1/2

8/11/2016

MyPastest

It reverses the hypotensive effect of sildenafil (Option D) is incorrect. Nicorandil can


potentiate the hypotensive effect of phosphodiesterase inhibitors such as sildenafil, and coprescribing is contraindicated.
46378

Next Question

Previous Question

Tag Question

Feedback

End Review

Difficulty: Average
Peer Responses

Session Progress
Responses Correct:

Responses Incorrect:

207

Responses Total:

207

Responses - % Correct:

0%

Blog (https://www.pastest.com/blog) About Pastest (https://www.pastest.com/about-us)


Contact Us (https://www.pastest.com/contact-us) Help (https://www.pastest.com/help)
Pastest 2016

https://mypastest.pastest.com/Secure/TestMe/Browser/429893#Top

2/2

8/11/2016

MyPastest

Back to Filters (/Secure/TestMe/Filter/429893/QA)

Question 84 of 207

What is the main reason for the difference between the dose of sublingual glyceryl trinitrate
(GTN) and oral isosorbide mononitrate needed to exert the same therapeutic effect?
A

Absorption

First-pass metabolism

Lipid solubility

More rapid renal clearance

Phase II metabolism

Explanation

The answer is First-pass metabolism


Among the various routes of drug delivery, the oral route is preferred for convenience.
However, a key disadvantage for some drugs is first-pass metabolism and enzymatic
degradation within the gastrointestinal tract. This renders oral administration of certain
classes of drugs infeasible, especially peptides and proteins, eg insulin. Where there is
significant first-pass metabolism, other transmucosal routes of drug delivery (ie the
mucosal linings of the nasal, rectal, vaginal, ocular and oral cavities) offer distinct
advantages over peroral administration for systemic drug delivery.

Absorption (Option A) is incorrect. Drug absorption is normally effective; it is metabolism of


the absorbed drug that substantially lowers drug bioavailability.
Lipid solubility (Option C) is incorrect. Both drugs have similar lipid solubility.
More rapid renal clearance (Option D) is incorrect. Neither drug is significantly dependent
upon renal elimination for clearance.
Phase II metabolism (Option E) is incorrect. Phase II metabolism refers to conjugation. It is
phase I, oxidative metabolism of nitrates, that diminishes oral bioavailabilty.
46934

https://mypastest.pastest.com/Secure/TestMe/Browser/429893#Top

1/2

8/11/2016

MyPastest

Next Question

Previous Question

Tag Question

Feedback

End Review

Difficulty: Average
Peer Responses

Session Progress
Responses Correct:

Responses Incorrect:

207

Responses Total:

207

Responses - % Correct:

0%

Blog (https://www.pastest.com/blog) About Pastest (https://www.pastest.com/about-us)


Contact Us (https://www.pastest.com/contact-us) Help (https://www.pastest.com/help)
Pastest 2016

https://mypastest.pastest.com/Secure/TestMe/Browser/429893#Top

2/2

8/11/2016

MyPastest

Back to Filters (/Secure/TestMe/Filter/429893/QA)

Question 85 of 207

A 45-year-old, petrol-station attendant complains of tingling and numbness in his hands and
feet, breathlessness, lethargy, weight gain and fatigue. He has a past medical history of
paroxysmal atrial fibrillation and depression and is receiving a number of different
medications. He attends the dermatology outpatient clinic because he has developed a
greyish-blue discoloration affecting his face.
Which of the following factors is most likely to account for his symptoms?
A

Amiodarone

Lithium

Occupational lead exposure

Occupational mercury exposure

Prednisolone

Explanation

The answer is Amiodarone


Amiodarone may cause skin deposits, photodermatitis, a greyish-blue discoloration.
Other adverse effects include tingling and numbness in the hands and feet,
breathlessness, lethargy, weight gain, fatigue and slowing of peripheral reflexes.
Pulmonary toxicity is one of the most important adverse effects and requires immediate
drug cessation. Other effects are inhibition of peripheral conversion of thyroxine (T4) to
triiodothyronine (T3); it also contains large amounts of inorganic iodine; treatment may
therefore cause hypo- or hyperthyroidism.

Lithium (Option B) is incorrect. Lithium blocks the endocytosis of monoiodotyrosine (MIT)


and diiodotyrosine (DIT) by the follicular cells of the thyroid and thereby interferes with T3
and T4 synthesis. The interaction between lithium and thyroid function is complex because it
also impairs TSH receptor sensitivity. Even at therapeutic doses, lithium may cause
hypothyroidism.

https://mypastest.pastest.com/Secure/TestMe/Browser/429893#Top

1/2

8/11/2016

MyPastest

Occupational lead exposure (Option C) is incorrect. Chronic lead poisoning may occur due to
exposure to leaded petrol, but this is very uncommon now. Lead poisoning does not cause
skin discoloration but may give rise to blue lead lines in the gingival margins.
Occupational mercury exposure (Option D) is incorrect. Mercury poisoning is extremely rare.
Chronic mercury poisoning by mercury vapour results in a classic triad of tremor,
neuropsychiatric disturbance and gingivostomatitis. Elemental mercury is less toxic than
mercury salts that may be encountered in the manufacture of electrical equipment and other
industrial uses. Methylmercury intoxication mainly affects the central nervous system and
results in paraesthesias, ataxia, deafness, dysarthria and progressive constriction of visual
fields.
Prednisolone (Option E) is incorrect. Prednisolone is associated with salt and water retention,
weight gain and a Cushingoid appearance when used at high dose for a prolonged period.
46780

Next Question

Previous Question

Tag Question

Feedback

End Review

Difficulty: Average
Peer Responses

Session Progress
Responses Correct:

Responses Incorrect:

207

Responses Total:

207

Responses - % Correct:

0%

Blog (https://www.pastest.com/blog) About Pastest (https://www.pastest.com/about-us)


Contact Us (https://www.pastest.com/contact-us) Help (https://www.pastest.com/help)
Pastest 2016

https://mypastest.pastest.com/Secure/TestMe/Browser/429893#Top

2/2

8/11/2016

MyPastest

Back to Filters (/Secure/TestMe/Filter/429893/QA)

Question 86 of 207

A patient in a general surgical ward has received aminoglycosides and cephalosporins


intravenously for 10 days. Shortly after completion of this treatment he developed watery
diarrhoea that was associated with abdominal pain. You are asked to review him because he
has been complaining of generalised abdominal pain, fever and frequent bloody diarrhoea.
Which one of the following oral therapies would be most appropriate?
A

Amphotericin B

Erythromycin

Ispaghula husk

Prednisolone

Vancomycin

Explanation

The answer is Vancomycin


The clinical features are consistent with pseudomembranous colitis, an acute exudative
infection of the colon caused by Clostridium difficile. The name pseudomembranous
colitis derives from plaques of necrotic membrane that adhere to the mucosal surface in
the clinically most severe form of the disease. The single most pertinent detail of this
mans medical history is his previous extensive antimicrobial treatment.
Pseudomembranous colitis has been reported to follow the use of every broad-spectrum
antimicrobial in common medical practice, including ciprofloxacin, clindamycin,
ampicillin, amoxicillin and cephalosporins. Treatment involves stopping the causative
antimicrobial if possible. Oral vancomycin has minimal gastrointestinal absorption so that
the effect of oral administration is confined mainly to the gastrointestinal tract. Oral or
intravenous metronidazole is an effective alternative treatment.

Amphotericin B (Option A) is incorrect. Amphotercin is an antifungal that is ineffective


against C. difficile.
Erythromycin (Option B) is incorrect. Erythromycin is ineffective against C. difficile.
https://mypastest.pastest.com/Secure/TestMe/Browser/429893#Top

1/2

8/11/2016

MyPastest

Ispaghula husk (Option C) is incorrect. A high-fibre diet is unlikely to confer any benefit.
Probiotic yoghurts may be of some value in treatment of C. difficile.
Prednisolone (Option D) is incorrect. Prednisolone may be effective in the setting of noninfective inflammatory bowel disorders, but the clinical context makes C. difficile infection
much more likely here.
46787

Next Question

Previous Question

Tag Question

Feedback

End Review

Difficulty: Average
Peer Responses

Session Progress
Responses Correct:

Responses Incorrect:

207

Responses Total:

207

Responses - % Correct:

0%

Blog (https://www.pastest.com/blog) About Pastest (https://www.pastest.com/about-us)


Contact Us (https://www.pastest.com/contact-us) Help (https://www.pastest.com/help)
Pastest 2016

https://mypastest.pastest.com/Secure/TestMe/Browser/429893#Top

2/2

8/11/2016

MyPastest

Back to Filters (/Secure/TestMe/Filter/429893/QA)

Question 87 of 207

You have seen a patient with bronchiectasis and wish to prescribe a theophylline preparation
as a bronchodilator.
Which of the following best describes a main feature of this drugs activity?
A

50% of the drug is plasma-protein bound

It blocks the adenosine receptor

It enhances diaphragm contractility

Most of the drug is excreted unchanged in urine

Side-effects increase in a linear way versus dose

Explanation

The answer is It blocks the adenosine receptor


Adenosine acts as a local mediator for many organ systems. Blockade of the receptors
by theophylline causes relaxation of bronchial smooth muscle, constriction of cerebral
blood vessels, stimulation of the cardiac pacemaker and stimulation of gastric secretions.
In addition to blocking adenosine, theophylline inhibits phosphodiesterase, resulting in
calcium ion release from the sarcoplasmic reticulum in skeletal and cardiac muscle, thus
enhancing their contractility. Theophylline is extensively metabolised by the liver; it has a
narrow therapeutic window, so drugs that inhibit liver metabolism may cause
theophylline toxicity.

50% of the drug is plasma-protein bound (Option A) is incorrect. Theophylline is highly


protein bound.
It enhances diaphragm contractility (Option C) is incorrect. Theophylline may enhance
diaphragm contractility, which may assist respiration but this is not the major
pharmacological mechanism of action.
Most of the drug is excreted unchanged in urine (Option D) is incorrect. Only 10% of the drug
is excreted unchanged in the urine as most of it is extensively metabolised in the liver.
https://mypastest.pastest.com/Secure/TestMe/Browser/429893#Top

1/2

8/11/2016

MyPastest

Side-effects increase in a linear way versus dose (Option D) is incorrect. Theophylline is


subject to saturable liver metabolism pathways, and a small increase in dose may provoke
toxicity.
46869

Next Question

Previous Question

Tag Question

Feedback

End Review

Difficulty: Average
Peer Responses

Session Progress
Responses Correct:

Responses Incorrect:

207

Responses Total:

207

Responses - % Correct:

0%

Blog (https://www.pastest.com/blog) About Pastest (https://www.pastest.com/about-us)


Contact Us (https://www.pastest.com/contact-us) Help (https://www.pastest.com/help)
Pastest 2016

https://mypastest.pastest.com/Secure/TestMe/Browser/429893#Top

2/2

8/11/2016

MyPastest

Back to Filters (/Secure/TestMe/Filter/429893/QA)

Question 88 of 207

A 43-year-old woman is admitted to the Intensive Care Unit after an intentional overdose
involving a mixture of different medications. You notice that the patient has been
commenced on haemodialysis.
Which one of the following drug toxicities is LEAST likely to be improved by haemodialysis?
A

Alcohol

Barbiturates

Ecstasy

Lithium

Salicylate

Explanation

The answer is Ecstasy


There is no antidote for ecstasy overdose, and haemodialysis has not been shown to
offer any benefit in treatment of acute toxicity. Haemodialysis, peritoneal dialysis,
charcoal or resin haemoperfusion, haemofiltration, plasmapheresis and exchange
transfusion are used to enhance clearance of drug from the body; greatest clearance is
seen with drugs that have high water solubility, low plasma protein binding, small
volumes of distribution and a long half-life.

Alcohol (Option A) is incorrect. Ethanol, methanol and ethylene glycol clearance can be
significantly enhanced by dialysis.
Barbiturates (Option B) is incorrect. Although now an uncommon means of overdose,
haemodialysis is particularly effective in removal of barbiturates.
Lithium (Option D) is incorrect. Lithium toxicity may be severe and result in cognitive and
neurological impairment. Haemodialysis is highly effective in removing lithium from the
circulating compartment, but prolonged dialysis (18 h) is required to achieve an effective
removal of total body lithium.
https://mypastest.pastest.com/Secure/TestMe/Browser/429893#Top

1/2

8/11/2016

MyPastest

Salicylate (Option E) is incorrect. Severe salicylate poisoning may cause metabolic acidosis
that is difficult to correct by bicarbonate administration alone; haemodialysis is highly
effective in removing salicylates and restoring acidbase balance.
46968

Next Question

Previous Question

Tag Question

Feedback

End Review

Difficulty: Average
Peer Responses

Session Progress
Responses Correct:

Responses Incorrect:

207

Responses Total:

207

Responses - % Correct:

0%

Blog (https://www.pastest.com/blog) About Pastest (https://www.pastest.com/about-us)


Contact Us (https://www.pastest.com/contact-us) Help (https://www.pastest.com/help)
Pastest 2016

https://mypastest.pastest.com/Secure/TestMe/Browser/429893#Top

2/2

8/11/2016

MyPastest

Back to Filters (/Secure/TestMe/Filter/429893/QA)

Question 89 of 207

A 57-year-old woman with cardiac failure is being managed in the High Dependency Unit. Her
heart rate and systolic blood pressure are low, and after discussion with the cardiology team,
you decide that she should receive inotropic support.
Of the following drugs, which is most likely to cause significant tachycardia?
A

Adrenaline

Dobutamine

Dopamine

Noradrenaline

Phenylephrine

Explanation

The answer is Adrenaline


Adrenaline exerts agonist effects on both - and -adrenoceptors and less intensive
vasoconstriction than norepinephrine. Adrenaline stimulates both 1- and 2-receptors
with approximately equal potency; 1 effects may cause tachycardia, particularly at
higher doses, whereas 2 effects include vasodilatation.

Dobutamine (Option B) is incorrect. Dobutamine is relatively selective for peripheral 1receptors and may cause tachycardia, with less effect on peripheral vasculature.
Dopamine (Option C) is incorrect. Dopamine causes peripheral vasoconstriction, and has
equivocal effects on heart rate (in some cases there may be a baroreflex mediated
bradycardia).
Noradrenaline (Option D) is incorrect. Norepinephrine stimulates mainly -adrenoceptors in
the periphery, resulting in intense vasoconstriction; the increase in afterload may worsen
heart failure.
Phenylephrine (Option E) is incorrect. Phenylephrine primarily results in a rise in BP, it may
result in reflex bradycardia when given IV because of carotid body stimulation.
46894

https://mypastest.pastest.com/Secure/TestMe/Browser/429893#Top

1/2

8/11/2016

MyPastest

46894

Next Question

Previous Question

Tag Question

Feedback

End Review

Difficulty: Average
Peer Responses

Session Progress
Responses Correct:

Responses Incorrect:

207

Responses Total:

207

Responses - % Correct:

0%

Blog (https://www.pastest.com/blog) About Pastest (https://www.pastest.com/about-us)


Contact Us (https://www.pastest.com/contact-us) Help (https://www.pastest.com/help)
Pastest 2016

https://mypastest.pastest.com/Secure/TestMe/Browser/429893#Top

2/2

8/11/2016

MyPastest

Back to Filters (/Secure/TestMe/Filter/429893/QA)

Question 90 of 207

A 55-year-old man diagnosed with hypertension who was not responding to recommended
lifestyle changes was commenced on drug treatment 1 month ago. There is a past history of
benign prostatic hypertropy and the GP has tailored therapy to try and treat both conditions.
He complains of dizziness, and severe postural hypotension is found.
What is the most likely aetiological agent?
A

Bendrofluazide

Bisoprolol

Doxazosin

Lisinopril

Losartan

Explanation

The answer is Doxazosin


Doxazosin, like other -adrenergic blocking agents, can cause marked hypotension,
especially postural hypotension, with syncope and other postural symptoms such as
dizziness. In this case, doxazosin may be used to control the symptoms of benign
prostatic hypertropy (BPH) as an additional potential benefit. Orthostatic hypotension is
most common after the first dose, but can also occur when there is a dosage increase, or
if the dose is interrupted for more than a few days and restarted. To decrease the
likelihood of excessive hypotension and syncope, it is essential that treatment is initiated
with the low dose and then gradually increased.

Bendrofluazide (Option A) is incorrect. Thiazides exert a modest diuretic effect soon after
initiation; however, the blood pressure lowering effects are more gradual in onset over
several days to weeks.
Bisoprolol (Option B) is incorrect. Beta-blockers are less likely to provoke postural
hypotension than -blockers, ACE inhibitors, or angiotensin receptor blockers; this may be

https://mypastest.pastest.com/Secure/TestMe/Browser/429893#Top

1/2

8/11/2016

MyPastest

due to their propensity to cause vasoconstriction owing to the unopposed effect of


catecholamines causing vasocontriction mediated by -adrenoceptors.
Lisinopril (Option D) is incorrect. ACE inhibitors commonly cause first-dose hypotension and
postural hypotension, but the co-existent BPH makes doxazosin a better option.
Losartan (Option E) is incorrect. Angiotension receptor blockers commonly cause first-dose
hypotension and postural hypotension, but the co-existent BPH makes doxazosin a better
option.
46939

Next Question

Previous Question

Tag Question

Feedback

End Review

Difficulty: Average
Peer Responses

Session Progress
Responses Correct:

Responses Incorrect:

207

Responses Total:

207

Responses - % Correct:

0%

Blog (https://www.pastest.com/blog) About Pastest (https://www.pastest.com/about-us)


Contact Us (https://www.pastest.com/contact-us) Help (https://www.pastest.com/help)
Pastest 2016

https://mypastest.pastest.com/Secure/TestMe/Browser/429893#Top

2/2

8/11/2016

MyPastest

Back to Filters (/Secure/TestMe/Filter/429893/QA)

Question 91 of 207

A 17-year-old girl is admitted from her birthday party. She does not normally drink alcohol,
but you understand she was given some by a friend, along with a pill to loosen up and have
fun. On examination she has GCS 14, blood pressure is 155/95 mmHg and pulse is 125/min.
Her respiratory rate is 24/min. Her pupils are dilated.
Which one of the following offers the best explanation for her clinical features?
A

Diclofenac

Ecstasy

Morphine

Thyroxine

Verapamil

Explanation

The answer is Ecstasy


Ecstasy or MDMA is a common drug of abuse, closely related to methamphetamine, and
causes release of both serotonin and catecholamines, leading to feelings of euphoria. It
causes hyperthermia and is often associated with rhabdomyolysis and hyponatraemia.
Pupils are dilated.

Diclofenac (Option A) is incorrect. Diclofenac ingestion may cause gastrointestinal irritation


but would not explain the reduced conscious level and tachycardia.
Morphine (Option C) is incorrect. Morphine would suppress conscious level but would be
expected to suppress respiration and cause small pupils.
Thyroxine (Option D) is incorrect. Thyroxine is unlikely to alter conscious level or respiratory
rate; the clinical effects, eg increased heart rate, may take several hours to days to manifest.
Verapamil (Option E) is incorrect. Verapamil would be expected to cause bradycardia.
46978

https://mypastest.pastest.com/Secure/TestMe/Browser/429893#Top

1/2

8/11/2016

MyPastest

Next Question

Previous Question

Tag Question

Feedback

End Review

Difficulty: Average
Peer Responses

Session Progress
Responses Correct:

Responses Incorrect:

207

Responses Total:

207

Responses - % Correct:

0%

Blog (https://www.pastest.com/blog) About Pastest (https://www.pastest.com/about-us)


Contact Us (https://www.pastest.com/contact-us) Help (https://www.pastest.com/help)
Pastest 2016

https://mypastest.pastest.com/Secure/TestMe/Browser/429893#Top

2/2

8/11/2016

MyPastest

Back to Filters (/Secure/TestMe/Filter/429893/QA)

Question 92 of 207

You see a 76-year-old man in the Coronary Care Unit 3 days after sustaining a non-ST
elevation myocardial infarction (NSTEMI). He has been started on a number of new
cardioactive medications.
Which one of the following antiplatelet agents increases cellular concentration of cyclic
adenosine monophosphate (cAMP) by inhibiting phosphodiesterase?
A

Abciximab

Aspirin

Clopidogrel

Dipyridamole

Ticlopidine

Explanation

The answer is Dipyridamole


Dipyridamole is a weak antiplatelet agent that acts by increasing the cellular
concentration of cyclic adenosine monophosphate (cAMP). It inhibits the
phosphodiesterase enzyme, which is responsible for convertin cAMP to inactive 5' AMP,
thereby increasing cAMP concentrations. Raised cAMP and cyclic guanosine
monophosphate (cGMP) inhibit platelet activation and aggregation.

Abciximab (Option A) is incorrect. Abciximab is a humanised monoclonal antibody that


selectively blocks the GPIIbIIIa receptor on the platelet surface.
Aspirin (Option B) is incorrect. Aspirin is a potent inhibitor of platelet cyclo-oxygenase, an
enzyme that converts arachidonic acid to thromboxane A2 (TxA2), a strong promoter of
platelet aggregation. The platelet has no protein synthetic apparatus; therefore, the effects
of aspirin are irreversible and last for the life of the platelet (810 days), even though aspirin
has only a short plasma half-life.
Clopidogrel (Option C) is incorrect. Clopidogrel, like ticlopidine, antagonises platelet ADP
receptor, resulting in inhibition of platelet activation. Antagonism is non-competitive,
https://mypastest.pastest.com/Secure/TestMe/Browser/429893#Top

1/2

8/11/2016

MyPastest

irreversible and results in 5070% inhibition of fibrinogen binding.


Ticlopidine (Option E) is incorrect. Ticlopidine is an adenosine diphosphate (ADP) receptor
antagonist.
46628

Next Question

Previous Question

Tag Question

Feedback

End Review

Difficulty: Average
Peer Responses

Session Progress
Responses Correct:

Responses Incorrect:

207

Responses Total:

207

Responses - % Correct:

0%

Blog (https://www.pastest.com/blog) About Pastest (https://www.pastest.com/about-us)


Contact Us (https://www.pastest.com/contact-us) Help (https://www.pastest.com/help)
Pastest 2016

https://mypastest.pastest.com/Secure/TestMe/Browser/429893#Top

2/2

8/11/2016

MyPastest

Back to Filters (/Secure/TestMe/Filter/429893/QA)

Question 93 of 207

A cancer patient is admitted for investigation of severe abdominal pain, diarrhoea and fever
after receiving radiation therapy treatment for underlying cancer. On questioning, it is found
that the patient had been receiving chemotherapy.
Which one of the following chemotherapy agents is most likely to have increased the risk of
radiation toxicity?
A

6-Mercaptopurine

Amifostine

Cyclophosphamide

Dactinomycin

Vincristine

Explanation

The answer is Dactinomycin


Dactinomycin is a radiosensitiser agent that increases cellular response to radiation
treatment. A number of other chemotherapy agents are capable of promoting the effects
of radiation, including: metronidazole, 5-fluorouracil, gemcitabine, cisplatin, hydroxyurea,
paclitaxel, mitomycin C and topotecan.

6-Mercaptopurine (Option A) is incorrect. 6-Mercaptopurine is not thought to act as a


radiosensitiser.
Amifostine (Option B) is incorrect. In contrast to the effects of radiosensitisers, amifostine
serves to protect against radiation toxicity.
Cyclophosphamide (Option C) is incorrect. Cyclophosphamide is not thought to act as a
radiosensitiser.
Vincristine (Option E) is incorrect. Vincristine is not thought to act as a radiosensitiser.
46865

https://mypastest.pastest.com/Secure/TestMe/Browser/429893#Top

1/2

8/11/2016

MyPastest

Next Question

Previous Question

Tag Question

Feedback

End Review

Difficulty: Average
Peer Responses

Session Progress
Responses Correct:

Responses Incorrect:

207

Responses Total:

207

Responses - % Correct:

0%

Blog (https://www.pastest.com/blog) About Pastest (https://www.pastest.com/about-us)


Contact Us (https://www.pastest.com/contact-us) Help (https://www.pastest.com/help)
Pastest 2016

https://mypastest.pastest.com/Secure/TestMe/Browser/429893#Top

2/2

8/11/2016

MyPastest

Back to Filters (/Secure/TestMe/Filter/429893/QA)

Question 94 of 207

A 58-year-old obese patient presents with swelling of the first metatarsophalangeal joint. On
examination it is acutely inflamed, tender, warm to the touch, and the overlying skin is red,
shiny and itchy.
What is the most appropriate therapy?
A

Allopurinol

Aspirin

Indometacin

Paracetamol

Prednisolone

Explanation

The answer is Indometacin


The clinical scenario is typical of acute gout. Full doses of any of the non-steroidal antiinflammatory drugs (NSAIDs) are effective in terminating attacks of acute gout. Oral
corticosteroids may be considered, but are less effective in reducing pain and swelling.

Allopurinol (Option A) is incorrect. Allopurinol is a xanthine oxidase inhibitor that causes a


modest reduction in serum urate concentration and, over a sufficient period of time, will
reduce total body burden of urate thereby making gout episodes less frequent. It may
provoke acute gout, and it is normally introduced only after the acute episode has resolved.
Aspirin (Option B) is incorrect. Aspirin inhibits tubular secretion of urate so that serum urate
concentrations increase. High-dose aspirin may be as effective as NSAIDs in the acute
setting, but is less commonly used.
Paracetamol (Option D) is incorrect. Paracetamol is less effective than NSAIDs for reducing
acute inflammatory joint disease.
Prednisolone (Option E) is incorrect. Corticosteroids are a second-line therapy that may be
considered when NSAIDs are not tolerated or contraindicated.
46812

https://mypastest.pastest.com/Secure/TestMe/Browser/429893#Top

1/2

8/11/2016

MyPastest

46812

Next Question

Previous Question

Tag Question

Feedback

End Review

Difficulty: Average
Peer Responses

Session Progress
Responses Correct:

Responses Incorrect:

207

Responses Total:

207

Responses - % Correct:

0%

Blog (https://www.pastest.com/blog) About Pastest (https://www.pastest.com/about-us)


Contact Us (https://www.pastest.com/contact-us) Help (https://www.pastest.com/help)
Pastest 2016

https://mypastest.pastest.com/Secure/TestMe/Browser/429893#Top

2/2

8/11/2016

MyPastest

Back to Filters (/Secure/TestMe/Filter/429893/QA)

Question 95 of 207

A 32-year-old man presents to the Emergency Department following an intentional overdose


of an unknown quantity of ferrous sulfate. He is complaining of abdominal pain and nausea.
Which of the following features is most likely to indicate severe iron poisoning?
A

Blood iron concentration of 120 mol/l at 6 hours post-ingestion

Heart rate 110 per minute

Hypocalcaemia on the initial blood tests

Presence of radio-opaque tablets on plain abdominal X-ray

Unable to administer oral activated charcoal due to vomiting

Explanation

The answer is Blood iron concentration of 120 mol/l at 6 hours post-ingestion


Patients with serum iron concentrations higher than 90 mol/l may have moderate to
severe toxicity, and normally require chelation therapy with intravenous desferrioxamine.
Undissolved iron tablets are radiopaque, but the presence on a plain X-ray does not
correlate well with toxicity. Early features of iron overdose are due to the direct irritant
effects of iron and include vomiting, diarrhoea and abdominal pain. Severe symptoms
may occur after a latent period of up to 20 hours, including metabolic acidosis, liver
toxicity and widespread organ failure. Initial hyperglycaemia can occur following
significant ingestion of iron, but hypoglycaemia can be seen later in cases of severe
poisoning with associated hepatic failure.

Heart rate 110 per minute (Option B) is incorrect. Tachycardia may occur, but is a secondary
feature, and not specific to iron poisoning.
Hypocalcaemia on the initial blood tests (Option C) is incorrect. Iron poisoning is not usually
associated with any significant effect on serum calcium concentration.
Presence of radio-opaque tablets on plain abdominal X-ray (Option D) is incorrect. Although
undissolved iron tablets are radio-opaque, their presence on a plain X-ray does not correlate
well with toxicity.
https://mypastest.pastest.com/Secure/TestMe/Browser/429893#Top

1/2

8/11/2016

MyPastest

Unable to administer oral activated charcoal due to vomiting (Option E) is incorrect. Severe
vomiting is a recognised feature of iron poisoning but is non-specific. Iron is not adsorbed by
activated charcoal (neither is lithium) and this treatment is not expected to be helpful.
45792

Next Question

Previous Question

Tag Question

Feedback

End Review

Difficulty: Average
Peer Responses

Session Progress
Responses Correct:

Responses Incorrect:

207

Responses Total:

207

Responses - % Correct:

0%

Blog (https://www.pastest.com/blog) About Pastest (https://www.pastest.com/about-us)


Contact Us (https://www.pastest.com/contact-us) Help (https://www.pastest.com/help)
Pastest 2016

https://mypastest.pastest.com/Secure/TestMe/Browser/429893#Top

2/2

8/11/2016

MyPastest

Back to Filters (/Secure/TestMe/Filter/429893/QA)

Question 96 of 207

A 50-year-old, mildly hypertensive businessman, on low-dose aspirin, is a regular at parties


where he scouts for more business. He has type-2 diabetes and is taking metformin and
gliclazide. Recently, he was prescribed erythromycin and paracetamol for an ear infection. He
presents to the Emergency Department with abdominal pain and nausea, and investigations
show serum bicarbonate 14 mmol/l and lactic 4.7 mmol/l.
Which of the following drugs is most likely to have interacted with metformin to cause this
complication?
A

Alcohol

Aspirin

Erythromycin

Gliclazide

Paracetamol

Explanation

The answer is Alcohol


Excess alcohol intake may significantly increase the risk of lactic acidosis in patients
taking metformin therapy. Metformin is not subject to metabolism and is normally
subject to renal excretion; it may accumulate and cause lactic acidosis in the setting of
acute kidney injury. Drugs capable of provoking kidney injury may increase the risk of
metformin toxicity, including contrast media, cyclosporin and aminoglycosides.
Metformin should therefore be stopped before, and for 48 h after, contrast radiography.
Metformin is excreted by the renal tubules and this process can be inhibited by
cimetidine, but not the other H2 receptor antagonists.

Aspirin (Option B) is incorrect. Aspirin may cause metabolic acidosis, particularly in the
context of an aspirin overdose; however, aspirin does not cause a lactic acidosis.
Erythromycin (Option C) is incorrect. Metformin is not metabolised, and therefore not
subject to the effects of erythromycin on hepatic metabolism.
https://mypastest.pastest.com/Secure/TestMe/Browser/429893#Top

1/2

8/11/2016

MyPastest

Gliclazide (Option D) is incorrect. Gliclazide may be administered safely alongside


metformin.
Paracetamol (Option E) is incorrect. Paracetamol does not interact with metformin.
46767

Next Question

Previous Question

Tag Question

Feedback

End Review

Difficulty: Average
Peer Responses

Session Progress
Responses Correct:

Responses Incorrect:

207

Responses Total:

207

Responses - % Correct:

0%

Blog (https://www.pastest.com/blog) About Pastest (https://www.pastest.com/about-us)


Contact Us (https://www.pastest.com/contact-us) Help (https://www.pastest.com/help)
Pastest 2016

https://mypastest.pastest.com/Secure/TestMe/Browser/429893#Top

2/2

8/11/2016

MyPastest

Back to Filters (/Secure/TestMe/Filter/429893/QA)

Question 97 of 207

You are involved in developing a study protocol to test a new drug for heart failure, and wish
to conduct a 6-month-long clinical trial. During this time you are hoping to detect adverse
events that might be expected to occur with an incidence of 1 in 2000 patients or more
frequently.
How many patients would need to be recruited for treatment with the new drug to have a
95% chance of detecting the occurrence of one adverse event?
A

1000

2000

6000

12 000

24 000

Explanation

The answer is 6000


Roughly speaking, for a study to have 95% power to detect a certain adverse event in a
clinical trial you would need to enrol around three times as many patients as the
expected event frequency, so in this case 6000 patients would be needed. If the
frequency expected was 1 in 10 000, then you would need to recruit around 30 000
patients and so on. The practicalities and costs of conducting very large-scale trials
mean that most new drugs are registered after trials in only a few thousand patients at
most; therefore, adverse events occurring with a frequency of 1 in 10 000 or less are
often only detected after the launch of a drug. For this reason, new drugs are afforded
Black triangle status in the UK to signify the need for intensive monitoring and reporting
of all adverse effects to the regulatory authorities via the Yellow-card system.

1000 (Option A) is incorrect. This would reliably allow detection of adverse effects that occur
in 1 in 300 patients or more frequently.

https://mypastest.pastest.com/Secure/TestMe/Browser/429893#Top

1/2

8/11/2016

MyPastest

2000 (Option B) is incorrect. This would reliably allow detection of adverse effects that
occur in 1 in 650 patients or more frequently.
12 000 (Option D) is incorrect. This would reliably allow detection of adverse effects that
occur in 1 in 4000 patients or more frequently.
24 000 (Option E) is incorrect. Not this many patients would require recruitment; this would
reliably allow detection of adverse effects that occur as rarely as 1 in 8000 patients.
46837

Next Question

Previous Question

Tag Question

Feedback

End Review

Difficulty: Average
Peer Responses

Session Progress
Responses Correct:

Responses Incorrect:

207

Responses Total:

207

Responses - % Correct:

0%

Blog (https://www.pastest.com/blog) About Pastest (https://www.pastest.com/about-us)


Contact Us (https://www.pastest.com/contact-us) Help (https://www.pastest.com/help)
Pastest 2016

https://mypastest.pastest.com/Secure/TestMe/Browser/429893#Top

2/2

8/11/2016

MyPastest

Back to Filters (/Secure/TestMe/Filter/429893/QA)

Question 98 of 207

You are reviewing a 72-year-old woman in the cardiology outpatient clinic. She has a 3-year
history of congestive cardiac failure and is receiving a number of different medications.
Which of the following drugs is most likely to prolong survival in patients with heart failure?
A

Aspirin

Digoxin

Enalapril

Furosemide

Isosorbide mononitrate

Explanation

The answer is Enalapril


Treatment with ACE inhibitors and angiotensin receptor antagonists has been shown to
prolong survival rates in patients with congestive heart failure. These agents exert a
number of beneficial actions on the cardiovascular system, including vasodilatation and
minimisation of cardiac dilatation.

Aspirin (Option A) is incorrect. Aspirin prolongs survival in patients with coronary occlusion
or myocardial infarction.
Digoxin (Option B) is incorrect. Digoxin reduces heart failure deaths but has no effect on
overall cardiovascular mortality.
Furosemide (Option D) is incorrect. Furosemide is effective in reducing heart failure
symptoms and may reduce hospitalisations, but does not alter mortality.
Isosorbide mononitrate (Option E) is incorrect. Nitrates improve cardiovascular
haemodynamics and may reduce symptoms. Nitrates combined with hydralazine (an arterial
vasodilator) reduce heart failure mortality, but there is no evidence that nitrates alone do
this.
46597

https://mypastest.pastest.com/Secure/TestMe/Browser/429893#Top

1/2

8/11/2016

MyPastest

Next Question

Previous Question

Tag Question

Feedback

End Review

Difficulty: Average
Peer Responses

Session Progress
Responses Correct:

Responses Incorrect:

207

Responses Total:

207

Responses - % Correct:

0%

Blog (https://www.pastest.com/blog) About Pastest (https://www.pastest.com/about-us)


Contact Us (https://www.pastest.com/contact-us) Help (https://www.pastest.com/help)
Pastest 2016

https://mypastest.pastest.com/Secure/TestMe/Browser/429893#Top

2/2

8/11/2016

MyPastest

Back to Filters (/Secure/TestMe/Filter/429893/QA)

Question 99 of 207

You are asked to review a 50-year-old man in the Intensive Care Unit who has apparently had
a single epileptic seizure, several days after admission. In the absence of other obvious
causes, you consider the possibility that this might have been an adverse drug effect.
Which of the following drugs might be most likely to have caused this?
A

Amiodarone

Chlordiazepoxide

Clomethiazole

Diazepam

Intravenous benzylpenicillin

Explanation

The answer is Intravenous benzylpenicillin


High doses of intravenous penicillin can provoke seizures; other antibiotics are also a
recognised cause of seizures, namely quinolones such as ciprofloxacin. Overdose
involving a number of different drugs may provoke seizures, including tricyclic
antidepressants, mefenamic acid, anticonvulsants and class I antiarrhythmics.

Amiodarone (Option A) is incorrect. Amiodarone has rarely been reported as a possible


cause of seizures.
Chlordiazepoxide (Option B) is incorrect. Chlordiazepoxide raises seizure threshold and may
be used to treat seizures. Sudden cessation may provoke seizures.
Clomethiazole (Option C) is incorrect. Clomethiazole (chlormethiazole) raises seizure
threshold and may be used to prevent or treat seizures.
Diazepam (Option D) is incorrect. Diazepam raises seizure threshold and may be used to
treat seizures. Sudden cessation may provoke seizures.
46900

https://mypastest.pastest.com/Secure/TestMe/Browser/429893#Top

1/2

8/11/2016

MyPastest

Next Question

Previous Question

Tag Question

Feedback

End Review

Difficulty: Average
Peer Responses

Session Progress
Responses Correct:

Responses Incorrect:

207

Responses Total:

207

Responses - % Correct:

0%

Blog (https://www.pastest.com/blog) About Pastest (https://www.pastest.com/about-us)


Contact Us (https://www.pastest.com/contact-us) Help (https://www.pastest.com/help)
Pastest 2016

https://mypastest.pastest.com/Secure/TestMe/Browser/429893#Top

2/2

8/11/2016

MyPastest

Back to Filters (/Secure/TestMe/Filter/429893/QA)

Question 100 of 207

A 57-year-old woman attends the medical clinic for review. She was commenced on a new
medicine 6 months earlier, and has noticed that her weight has progressively increased.
Which of the following drugs is most likely to cause weight gain?
A

Carbamazepine

Diazepam

Phenytoin

Pregabalin

Sodium valproate

Explanation

The answer is Sodium valproate


Sodium valproate causes the following side-effects: gastric irritation, nausea, ataxia and
tremor, hyperammonaemia, increased appetite and weight gain.

Carbamazepine (Option A) is incorrect. Carbamazepine may cause minor weight gain, and is
a less preferred answer than sodium valproate in this instance.
Diazepam (Option B) is incorrect. Diazepam may cause drowsiness, but weight gain is not a
significant adverse effect.
Phenytoin (Option C) is incorrect. Phenytoin may cause weight gain, but this is normally
minor, hence sodium valproate is the preferred answer here.
Pregabalin (Option D) is incorrect. Pregabalin may cause drowsiness, but weight gain is not a
significant adverse effect.
46938

Next Question

https://mypastest.pastest.com/Secure/TestMe/Browser/429893#Top

1/2

8/11/2016

MyPastest

Previous Question

Tag Question

Feedback

End Review

Difficulty: Average
Peer Responses

Session Progress
Responses Correct:

Responses Incorrect:

207

Responses Total:

207

Responses - % Correct:

0%

Blog (https://www.pastest.com/blog) About Pastest (https://www.pastest.com/about-us)


Contact Us (https://www.pastest.com/contact-us) Help (https://www.pastest.com/help)
Pastest 2016

https://mypastest.pastest.com/Secure/TestMe/Browser/429893#Top

2/2

8/11/2016

MyPastest

Back to Filters (/Secure/TestMe/Filter/429893/QA)

Question 101 of 207

A 74-year-old woman with a history of ischaemic heart disease and chronic atrial fibrillation is
treated with Digoxin, Ramipril, Furosemide, Bisoprolol and Clopidogrel. She has suffered from
diarrhoea and vomiting over the past few days and you are concerned that she may have prerenal failure. Examination reveals a BP of 95/60 mmHg, her pulse is 42/min, AF. Her chest is
clear and her abdomen is generally tender as a result of the gastroenteritis, although it is soft.
Investigations;
Hb

12.8 g/dl

WCC

7.9 x109/l

PLT

221 x109/l

Na+

137 mmol/l

K+

5.6 mmol/l

Creatinine

255 micromol/l (up from 135 micromol/l at last clinic


visit)

Urea

21.2 mmol/l

Which of the following features, related to her medication, may be seen in this situation?
A

Blue discoloration of vision

Blurred vision

Loss of colour vision

Loss of night vision

Yellowing of vision

Explanation
The answer is Yellowing of vision -

https://mypastest.pastest.com/Secure/TestMe/Browser/429893#Top

1/2

8/11/2016

MyPastest

Digoxin toxicity is known to be associated with yellow discoloration of vision, such that when
patients are proactively questioned about their eyesight they often mention that everything
looks jaundiced. Blue discoloration of vision is seen in patients who use high dose PDE-5
inhibitors for the treatment of erectile dysfunction because of crossover inhibition of the
PDE-6 enzyme. Loss of night vision is seen with congenital conditions such as retinitis
pigmentosa, while loss of contrast and colour vision is seen in optic neuritis.
38084

Next Question

Previous Question

Tag Question

Feedback

End Review

Difficulty: Easy
Peer Responses

Session Progress
Responses Correct:

Responses Incorrect:

207

Responses Total:

207

Responses - % Correct:

0%

Blog (https://www.pastest.com/blog) About Pastest (https://www.pastest.com/about-us)


Contact Us (https://www.pastest.com/contact-us) Help (https://www.pastest.com/help)
Pastest 2016

https://mypastest.pastest.com/Secure/TestMe/Browser/429893#Top

2/2

8/11/2016

MyPastest

Back to Filters (/Secure/TestMe/Filter/429893/QA)

Question 102 of 207

A patient with type-2 diabetes is found to have consistently high blood pressure in clinic, with
his latest recording 156/90 mmHg. Resting ECG shows normal sinus rhythm, and urinalysis
shows the presence of microalbuminuria.
Which of the following would be the most appropriate class of antihypertensive drugs to
consider for this patient?
A

ACE inhibitor

Alpha-blocker

Beta-blocker

Calcium-channel blocker

Diuretic

Explanation

The answer is ACE inhibitor


Antihypertensive treatment retards the progression of diabetic nephropathy from its
earliest stages, slowing the progression of microalbuminuria to overt proteinuria and the
decline of glomerular filtration rate in established diabetic nephropathy.
Strongest evidence of benefit is from clinical trials involving angiotensin converting
enzyme (ACE) inhibitors and angiotensin receptor blockers (ARBs), suggesting that
there may be specific mechanisms related to the reninangiotensinaldosterone system
in addition to blood pressure lowering. The blood pressure target is the lowest that can
be achieved without adverse effects, ideally less than 130/80 mmHg, or less than 125/75
mmHg when there is proteinuria.

Alpha-blocker (Option B) is incorrect. Alpha-blocker therapy is associated with a greater risk


of developing heart failure, and is used only where blood pressure cannot be controlled
adequately by other antihypertensive agents.
Beta-blocker (Option C) is incorrect. Beta-blockers show less effective prevention of stroke
and new-onset atrial fibrillation compared to agents that block the reninangiotensin
https://mypastest.pastest.com/Secure/TestMe/Browser/429893#Top

1/2

8/11/2016

MyPastest

aldosterone pathway, even when capable of achieving the same blood pressure control.
Therefore, they are normally reserved for second-line therapy.
Calcium-channel blocker (Option D) is incorrect. Calcium-channel blockers are useful when
added to an ACE inhibitor in order to achieve target blood pressure, but there is less
supportive data for calcium-channel blockers in delaying progression of diabetic
nephropathy.
Diuretic (Option E) is incorrect. Thiazide diuretics are effective in lowering blood pressure,
but may aggravate hyperglycaemia and dyslipidaemia.
46795

Next Question

Previous Question

Tag Question

Feedback

End Review

Difficulty: Average
Peer Responses

Session Progress
Responses Correct:

Responses Incorrect:

207

Responses Total:

207

Responses - % Correct:

0%

Blog (https://www.pastest.com/blog) About Pastest (https://www.pastest.com/about-us)


Contact Us (https://www.pastest.com/contact-us) Help (https://www.pastest.com/help)
Pastest 2016

https://mypastest.pastest.com/Secure/TestMe/Browser/429893#Top

2/2

8/11/2016

MyPastest

Back to Filters (/Secure/TestMe/Filter/429893/QA)

Question 103 of 207

A 32-year-old man presents with epigastric tenderness and fever. He has a past history of
treated epilepsy. On examination he has a blood pressure of 100/60 mmHg, pulse of 110 bpm
and regular and severe pain on palpation of the epigastrium. Blood tests reveal
hypocalcaemia, metabolic acidosis and a markedly elevated serum amylase. He cannot
remember what he takes for his epilepsy.
Which of the following antiepileptic agents is most likely to have caused this acute illness?
A

Carbamazepine

Lamotrigine

Phenytoin

Topiramate

Valproate

Explanation

The answer is Valproate


The clinical scenario is strongly suggestive of acute pancreatitis. The commonest factors
causing this are biliary disease and alcohol, but in up to 10% of cases a drug cause may
be identified:
Thiazides, furosemide diuretics
Corticosteroids
Tetracyclines, erythromycin, nitrofurantoin, metronidazole
Oestrogens
Valproate
Azathioprine
Methyldopa
Procainamide
Angiotensin converting enzyme (ACE) inhibitors
https://mypastest.pastest.com/Secure/TestMe/Browser/429893#Top

1/2

8/11/2016

MyPastest

Ranitidine

Carbamazepine (Option A) is incorrect. Acute pancreatitis is not a characteristic adverse


effect of carbamazepine; acute hepatitis may occur.
Lamotrigine (Option B) is incorrect. Acute pancreatitis is not a characteristic adverse effect
of lamotrigine.
Phenytoin (Option C) is incorrect. Acute pancreatitis is not a characteristic adverse effect of
phenytoin; acute hepatitis may occur.
Topiramate (Option D) is incorrect. Acute pancreatitis is not a characteristic adverse effect of
topiramate.
46952

Next Question

Previous Question

Tag Question

Feedback

End Review

Difficulty: Average
Peer Responses

Session Progress
Responses Correct:

Responses Incorrect:

207

Responses Total:

207

Responses - % Correct:

0%

Blog (https://www.pastest.com/blog) About Pastest (https://www.pastest.com/about-us)


Contact Us (https://www.pastest.com/contact-us) Help (https://www.pastest.com/help)
Pastest 2016

https://mypastest.pastest.com/Secure/TestMe/Browser/429893#Top

2/2

8/11/2016

MyPastest

Back to Filters (/Secure/TestMe/Filter/429893/QA)

Question 104 of 207

A 40-year-old woman with type-2 diabetes is receiving regular insulin therapy.


Which of the following enzymes is most likely to be inhibited by insulin?
A

Acetyl-CoA carboxylase

ATP citrate lyase

Glucose 6-phosphate dehydrogenase

Glycogen synthetase

Pyruvate carboxylase

Explanation

The answer is Pyruvate carboxylase


Insulin inhibits enzymes involved in gluconeogenesis, including pyruvate carboxylase.
Additional effects are to stimulate glycogenesis by stimulating glycogen synthetase in
liver and muscle. Other effects include activation of the hexose monophosphate (HMP)
shunt by inducing the synthesis of glucose 6-phosphate dehydrogenase, and activation
of acetyl-CoA carboxylase and ATP citrate lyase to increase the synthesis of fatty acids.

Acetyl-CoA carboxylase (Option A) is incorrect. Insulin stimulates acetyl-CoA carboxylase


and ATP citrate lyase, which are central elements of the Krebs cycle and result in formation
of free fatty acids.
ATP citrate lyase (Option B) is incorrect. Insulin stimulates acetyl-CoA carboxylase and ATP
citrate lyase, which are central elements of the Krebs cycle and result in formation of free
fatty acids.
Glucose 6-phosphate dehydrogenase (Option C) is incorrect. Insulin stimulates activity of
glucose 6-phosphate dehydrogenase activity.
Glycogen synthetase (Option D) is incorrect. Insulin stimulates glycogen storage, in part by
activating glycogen synthetase.
46854

https://mypastest.pastest.com/Secure/TestMe/Browser/429893#Top

1/2

8/11/2016

MyPastest

Next Question

Previous Question

Tag Question

Feedback

End Review

Difficulty: Average
Peer Responses

Session Progress
Responses Correct:

Responses Incorrect:

207

Responses Total:

207

Responses - % Correct:

0%

Blog (https://www.pastest.com/blog) About Pastest (https://www.pastest.com/about-us)


Contact Us (https://www.pastest.com/contact-us) Help (https://www.pastest.com/help)
Pastest 2016

https://mypastest.pastest.com/Secure/TestMe/Browser/429893#Top

2/2

8/11/2016

MyPastest

Back to Filters (/Secure/TestMe/Filter/429893/QA)

Question 105 of 207

A 64-year-old woman with long-standing osteoarthritis presents to hospital with a 3-day


history of melaena. She has been receiving naproxen 500 mg twice daily for the past 4
months.
Which of the following statements best pertains to gastrointestinal bleeding due to NSAID
therapy?
A

It is due to decreased platelet adhesiveness

It is due to depletion of mucosal prostaglandin E levels

Occurs when the patient complains of severe dyspepsia

Only occurs in patients who have pre-existing gastric and/or duodenal ulcers

Only occurs with high-risk NSAIDs such as piroxicam

Explanation

The answer is It is due to depletion of mucosal prostaglandin E (PGE) levels


Patients who have pre-existing gastric or duodenal ulcers are more prone to
gastrointestinal (GI) bleeds with non-steroidal anti-inflammatory drug (NSAID)
treatment. However, GI bleeds are also common in those with no such previous history
and may occur in patients without any significant history of previous dyspepsia. The risk
of causing GI bleeds differs with different agents; ibuprofen confers a relatively low risk
whereas piroxicam and azapropazone confer the highest risk. Endoscopic evidence of
peptic ulceration is found in 20% of NSAID users even in the absence of symptoms.
NSAIDs cause direct damage to the gastric mucosal barrier by depleting mucosal PGE
levels; NSAIDs decrease the gastroduodenal defence mechanisms and cytoprotective
effect of PGE, resulting in mucosal injury, erosions and ulceration.

It is due to decreased platelet adhesiveness (Option A) is incorrect. Disruption of the


mucosal integrity causes bleeding, not decreased platelet adhesiveness.
Occurs when the patient complains of severe dyspepsia (Option C) is incorrect. There is a
poor correlation between symptoms and severity of ulceration and bleeding risk.
https://mypastest.pastest.com/Secure/TestMe/Browser/429893#Top

1/2

8/11/2016

MyPastest

Only occurs in patients who have pre-existing gastric and/or duodenal ulcers (Option D) is
incorrect. NSAIDs may provoke peptic ulceration even in the absence of prior ulceration.
Only occurs with high-risk NSAIDs such as piroxicam (Option E) is incorrect. Ulceration may
occur after exposure to any NSAID.
46612

Next Question

Previous Question

Tag Question

Feedback

End Review

Difficulty: Average
Peer Responses

Session Progress
Responses Correct:

Responses Incorrect:

207

Responses Total:

207

Responses - % Correct:

0%

Blog (https://www.pastest.com/blog) About Pastest (https://www.pastest.com/about-us)


Contact Us (https://www.pastest.com/contact-us) Help (https://www.pastest.com/help)
Pastest 2016

https://mypastest.pastest.com/Secure/TestMe/Browser/429893#Top

2/2

8/11/2016

MyPastest

Back to Filters (/Secure/TestMe/Filter/429893/QA)

Question 106 of 207

A 24-year-old woman is undergoing chemotherapy for acute leukaemia. She is admitted to


the Acute Medical Unit due to severe vomiting and you decide to administer intravenous
hydration and antiemetic treatment.
Which of the following would be the most appropriate antiemetic choice?
A

Metoclopramide IM

Ondansetron IV

Oral domperidone

Oral ondansetron

Prochlorperazine IM

Explanation

The answer is Ondansetron IV


Ondansetron is a potent 5-hydroxytryptamine-3 (5-HT3) receptor antagonist that is well
tolerated. The main adverse effect is headache. Given that vomiting is likely to continue,
intravenous would be preferred rather than oral route of administration. Studies have
shown that 5-HT3 antagonists are generally more effective in treating postchemotherapy nausea and vomiting than other antiemetic agents.

Metoclopramide IM (Option A) is incorrect. Metoclopramide is less likely to be effective than


ondansetron. Antidopaminergic drugs, such as metoclopramide, should be avoided owing to
increased risk of side-effects in young women in particular.
Oral domperidone (Option C) is incorrect. The oral route would be inappropriate;
domperidone would be less likely to be effective than ondansetron, but could be considered
as a second-line agent if ondansetron is ineffective or not tolerated.
Oral ondansetron (Option D) is incorrect. The oral route would be inappropriate in the setting
of active vomiting.

https://mypastest.pastest.com/Secure/TestMe/Browser/429893#Top

1/2

8/11/2016

MyPastest

Prochlorperazine IM (Option E) is incorrect. Prochlorperazine is less likely to be effective


than ondansetron, and the onset of action would be less predictable after intramuscular
rather than intravenous administration.
46954

Next Question

Previous Question

Tag Question

Feedback

End Review

Difficulty: Average
Peer Responses

Session Progress
Responses Correct:

Responses Incorrect:

207

Responses Total:

207

Responses - % Correct:

0%

Blog (https://www.pastest.com/blog) About Pastest (https://www.pastest.com/about-us)


Contact Us (https://www.pastest.com/contact-us) Help (https://www.pastest.com/help)
Pastest 2016

https://mypastest.pastest.com/Secure/TestMe/Browser/429893#Top

2/2

8/11/2016

MyPastest

Back to Filters (/Secure/TestMe/Filter/429893/QA)

Question 107 of 207

You are called by a local GP who asks for advice about the best antibiotic to use to treat an
acute skin infection in a woman who is breast-feeding her 2-week-old baby.
Which of the following antibiotics would be the best option in this patient?
A

Ciprofloxacin

Clarithromycin

Flucloxacillin

Fluconazole

Ofloxacin

Explanation

The answer is Flucloxacillin


The penicillins are considered to be relatively safe in breast-feeding.

Ciprofloxacin (Option A) is incorrect. Ciprofloxacin is subject to widespread tissue


distribution, including secretion in significant quantities in breast milk. Quinolones are
thought to increase the risk of tendon abnormalities.
Clarithromycin (Option B) is incorrect. Clarithromycin is not recommended by the
manufacturer unless the benefits of treatment outweigh the risk, although macrolides are
relatively well used in breast-feeding women with no apparent adverse events.
Fluconazole (Option D) is incorrect. Fluconazole is also not recommended for use by the
manufacturer in patients who are breast-feeding. It is a lipophilic agent and levels
accumulated in the infant via breast-feeding are greater than those recommended for use in
children. It is important to note that significant changes in cytochrome P450 metabolism
take place during the first few months of life, so that safety in a parent or adult population is
no guarantee of safety in the neonate.
Ofloxacin (Option E) is incorrect. Ofloxacin is subject to widespread tissue distribution,
including secretion in significant quantities in breast milk. Quinolones are thought to increase
https://mypastest.pastest.com/Secure/TestMe/Browser/429893#Top

1/2

8/11/2016

MyPastest

the risk of tendon abnormalities.


46841

Next Question

Previous Question

Tag Question

Feedback

End Review

Difficulty: Average
Peer Responses

Session Progress
Responses Correct:

Responses Incorrect:

207

Responses Total:

207

Responses - % Correct:

0%

Blog (https://www.pastest.com/blog) About Pastest (https://www.pastest.com/about-us)


Contact Us (https://www.pastest.com/contact-us) Help (https://www.pastest.com/help)
Pastest 2016

https://mypastest.pastest.com/Secure/TestMe/Browser/429893#Top

2/2

8/11/2016

MyPastest

Back to Filters (/Secure/TestMe/Filter/429893/QA)

Question 108 of 207

Probenecid may be used to reduce the renal clearance of certain drugs, thereby prolonging
their half-life and duration of action.
For which one of the following drugs is the co-administration of probenecid most likely to
provide improved clinical effectiveness?
A

Aspirin

Bendroflumethiazide

Furosemide

Penicillin

Quinine

Explanation

The answer is Penicillin


Drugs may be secreted into the proximal convoluted tubule of the nephron by cation
transporters (eg quinine, pethidine, morphine) or anion transporters (eg penicillins,
bendroflumethiazide, furosemide, cephalosporins). Probenecid interferes with anion
transport and therby reduces the renal elimination of these drugs, and prolongs their
duration of action. Penicillin has a short duration of action and requires multiple daily
dose administration for efficacy; co-administration of probenecid allows maintenance of
penicillin blood concentrations higher than the minimum inhibitory concentrations with
less frequent dose administration.

Aspirin (Option A) is incorrect. Aspirin has a short duration of action of 12 hours, but this is
sufficient to inhibit platelet function for much longer because platelets are unable to
regenerate cyclo-oxygenase. Probenecid may prolong the duration of aspirin action, but this
would not alter its pharmacodynamic actions.
Bendroflumethiazide (Option B) is incorrect. Probenecid is capable of interfering with
bendroflumethiazide elimination, but this will not produce any meaningful clinical benefit
because the blood-pressure-lowering effects of bendroflumethiazide persist for several
https://mypastest.pastest.com/Secure/TestMe/Browser/429893#Top

1/2

8/11/2016

MyPastest

weeks.
Furosemide (Option C) is incorrect. Probenecid may enhance the actions of furosemide, but
this is unlikely to offer any meaningful clinical benefit.
Quinine (Option E) is incorrect. Probenecid does not interfere with quinine elimination.
45806

Next Question

Previous Question

Tag Question

Feedback

End Review

Difficulty: Average
Peer Responses

Session Progress
Responses Correct:

Responses Incorrect:

207

Responses Total:

207

Responses - % Correct:

0%

Blog (https://www.pastest.com/blog) About Pastest (https://www.pastest.com/about-us)


Contact Us (https://www.pastest.com/contact-us) Help (https://www.pastest.com/help)
Pastest 2016

https://mypastest.pastest.com/Secure/TestMe/Browser/429893#Top

2/2

8/11/2016

MyPastest

Back to Filters (/Secure/TestMe/Filter/429893/QA)

Question 109 of 207

A 25-year-old patient, who has a past history of cluster headaches, presents to the Acute
Medical Unit with an acute attack of cluster headache.
Which one of the following drugs is most likely to be an effective treatment?
A

Aspirin

Atenolol

Levocabastine

Paracetamol

Sumatriptan

Explanation

The answer is Sumatriptan


Sumatriptan is effective, and offers rapid relief of symptoms. Alternatively, administration
of 100% oxygen at a rate of 712 l/min is useful in aborting an attack. Prophylaxis is
considered if cluster headaches are frequent or last more than 3 weeks, normally with
verapamil or lithium. Ergotamine may be used intermittently, but should be used with
caution in view of adverse effects related to vasospasm.

Aspirin (Option A) is incorrect. Aspirin is not a recognised treatment for cluster headaches.
Atenolol (Option B) is incorrect. Atenolol is ineffective. Non-cardioselective -blockers
including propranolol may be effective in preventing migraine.
Levocabastine (Option C) is incorrect. Levocabastine is an anti-histamine used for the
treatment of allergic conjunctivitis.
Paracetamol (Option D) is incorrect. Paracetamol is often ineffective for cluster headache.
46821

Next Question

https://mypastest.pastest.com/Secure/TestMe/Browser/429893#Top

1/2

8/11/2016

MyPastest

Previous Question

Tag Question

Feedback

End Review

Difficulty: Average
Peer Responses

Session Progress
Responses Correct:

Responses Incorrect:

207

Responses Total:

207

Responses - % Correct:

0%

Blog (https://www.pastest.com/blog) About Pastest (https://www.pastest.com/about-us)


Contact Us (https://www.pastest.com/contact-us) Help (https://www.pastest.com/help)
Pastest 2016

https://mypastest.pastest.com/Secure/TestMe/Browser/429893#Top

2/2

8/11/2016

MyPastest

Back to Filters (/Secure/TestMe/Filter/429893/QA)

Question 110 of 207

A patient is admitted to the ward for observation after sustaining a head injury while running
away from the police. After 36 h on the ward he is suffering from hallucinations,
hypersalivation, irritability and diarrhoea. You suspect he is withdrawing from the effects of
previous recreational drug abuse.
Withdrawal of which of the following would be most likely to account for these features?
A

Alcohol

Amphetamine

Cocaine

Ecstasy

Heroin

Explanation

The answer is Heroin


The following are all signs of heroin withdrawal: sweating, diarrhoea, nausea and
vomiting, irritability and restlessness, and muscle ache. Management of long-term heroin
withdrawal involves methadone or buprenorphine maintenance programmes to avoid the
hazards of heroin injection.

Alcohol (Option A) is incorrect. Ethanol withdrawal may cause similar symptoms, but is less
likely to cause excess salivation or diarrhoea.
Amphetamine (Option B) is incorrect. Amphetamines are predominantly short-acting
stimulants, and withdrawal is uncommon.
Cocaine (Option C) is incorrect. Cocaine is a short-acting potent stimulant.
Ecstasy (Option D) is incorrect. Ecstasy is an intermediate-acting stimulant that enhances
serotonin activity within the central nervous system. Withdrawal may cause dysphoria.
46993

https://mypastest.pastest.com/Secure/TestMe/Browser/429893#Top

1/2

8/11/2016

MyPastest

Next Question

Previous Question

Tag Question

Feedback

End Review

Difficulty: Average
Peer Responses

Session Progress
Responses Correct:

Responses Incorrect:

207

Responses Total:

207

Responses - % Correct:

0%

Blog (https://www.pastest.com/blog) About Pastest (https://www.pastest.com/about-us)


Contact Us (https://www.pastest.com/contact-us) Help (https://www.pastest.com/help)
Pastest 2016

https://mypastest.pastest.com/Secure/TestMe/Browser/429893#Top

2/2

8/11/2016

MyPastest

Back to Filters (/Secure/TestMe/Filter/429893/QA)

Question 111 of 207

A 74-year-old woman with a history of depression presents to the Emergency Department


with a history of agitation, slurred speech, tremor and reduced conscious level. Serum lithium
concentration is reported as 3.3 mmol/l.
Which one of the following treatments would be most effective in treating lithium toxicity?
A

Activated charcoal

Forced diuresis with sodium chloride

Haemodialysis

Methionine

Methylprednisolone

Explanation

The answer is Haemodialysis


Toxicity is usually associated with levels > 1.5 mmol/l, and clinical features include tremor,
thirst, polyuria, diarrhoea, hypertonia, seizures and impaired consciousness.
Haemodialysis is the treatment of choice for severe lithium toxicity. In milder cases,
symptomatic treatment is usually all that is needed. All patients with lithium poisoning
should have measurement of lithium levels and plasma urea, electrolytes and osmolality.

Activated charcoal (Option A) is incorrect. Activated charcoal adsorbs drugs in the gut, and
increases removal of drugs from the body by interfering with entero-hepatic recirculation.
Forced diuresis with sodium chloride (Option B) is incorrect. Forced diuresis is not effective
in minimising toxicity and may be hazardous; sufficient IV fluids should be administered to
maintain hydration, with correction of electrolytes.
Methionine (Option D) is incorrect. Methionine is used as an oral antidote for paracetamol
poisoning in those who cannot tolerate acetylcysteine or where acetylcysteine is not
available.

https://mypastest.pastest.com/Secure/TestMe/Browser/429893#Top

1/2

8/11/2016

MyPastest

Methylprednisolone (Option E) is incorrect. Methylprednisolone may enhance renal clearance


to a very limited extent, but would be too slow and ineffective to have a role in lithium
toxicity.
46733

Next Question

Previous Question

Tag Question

Feedback

End Review

Difficulty: Average
Peer Responses

Session Progress
Responses Correct:

Responses Incorrect:

207

Responses Total:

207

Responses - % Correct:

0%

Blog (https://www.pastest.com/blog) About Pastest (https://www.pastest.com/about-us)


Contact Us (https://www.pastest.com/contact-us) Help (https://www.pastest.com/help)
Pastest 2016

https://mypastest.pastest.com/Secure/TestMe/Browser/429893#Top

2/2

8/11/2016

MyPastest

Back to Filters (/Secure/TestMe/Filter/429893/QA)

Question 112 of 207

A 72-year-old man comes to the Elderly Care clinic with his wife for the results of tests to
determine the underlying cause of dementia, diagnosed some 2 months earlier because of
progressively increasing confusion and memory loss over the past year. Whilst he is still able
to wash and dress himself, his wife is finding it increasingly difficult to cope with him
wandering and trying to get out of the house at night. He takes no regular medication. On
examination his BP is 132/72 mmHg; pulse is 79/min and regular. He looks slightly unkempt.
His BMI is 22 kg/m2. MMSE is 12/30.
They are given a diagnosis of Alzheimers disease.
Which of the following is the most appropriate initial therapy?
A

Amitriptyline

Donepezil

Lorazepam

Memantine

Risperidone

Explanation
The answer is Donepezil Acetylcholinesterase inhibitor treatment (Donepezil, Galantamine or Rivastigmine) should be
considered in patients with mild or moderate Alzheimer's disease although should ideally be
initiated by a specialist. In patients where these first line therapies are not tolerated, or are
contra-indicated, NICE recommends use of Memantine. Tricyclic antidepressants such as
Amitriptyline are not recommended because they may worsen underlying confusion. Antipsychotics should be avoided if possible, although if needed for significant delusions and
aggressive or confused behaviour, Risperidone would be considered first line. Lorazepam is a
potential option, particularly for IM use in acute confusion and aggression.
38677

Next Question

https://mypastest.pastest.com/Secure/TestMe/Browser/429893#Top

1/2

8/11/2016

MyPastest

Previous Question

Tag Question

Feedback

End Review

Difficulty: Average
Peer Responses

Session Progress
Responses Correct:

Responses Incorrect:

207

Responses Total:

207

Responses - % Correct:

0%

Blog (https://www.pastest.com/blog) About Pastest (https://www.pastest.com/about-us)


Contact Us (https://www.pastest.com/contact-us) Help (https://www.pastest.com/help)
Pastest 2016

https://mypastest.pastest.com/Secure/TestMe/Browser/429893#Top

2/2

8/11/2016

MyPastest

Back to Filters (/Secure/TestMe/Filter/429893/QA)

Question 113 of 207

A 22-year-old female presents to the Emergency Department complaining of rapid, regular


palpitations, which started suddenly while she was studying for exams. On examination she is
tachycardic at 155 bpm, normotensive and the rest of the examination is unremarkable.
Electrocardiogram (ECG) shows a narrow, complex tachycardia, and this is unaltered by vagal
manoeuvres.
Which of the following subsequent measures would be the most appropriate next step?
A

24-h urine collection for catecholamines

Direct current (DC) cardioversion

Intravenous adenosine

Intravenous amiodarone

Intravenous digoxin

Explanation

The answer is Intravenous adenosine The narrow, complex tachycardia indicates a supraventricular arrhythmia, possibly due to
atrial flutter, nodal tachycardia or atrial fibrillation. Such arrhythmias may be precipitated
by ingestion of large quantities of caffeine or other stimulant drugs. Adenosine may be
effective in terminating supraventricular re-entrant tachycardias, and can help to
distinguish underlying atrial rhythm.

24-h urine collection for catecholamines (Option A) is incorrect. This may be considered at a
later stage, depending on the clinical history.
Direct current (DC) cardioversion (Option B) is incorrect. DC shock would be inappropriate
as a first measure before contemplating adenosine, but may be considered if the rhythm
disturbance persists.
Intravenous amiodarone (Option D) is incorrect. Intravenous amiodarone is not normally
indicated in the first instance because the rhythm is often benign and may self-terminate or
terminate promptly after adenosine.
https://mypastest.pastest.com/Secure/TestMe/Browser/429893#Top

1/2

8/11/2016

MyPastest

Intravenous digoxin (Option E) is incorrect. Intravenous digoxin could render subsequent DC


shock (if needed) more hazardous in provoking a more sinister arrhythmia.
46916

Next Question

Previous Question

Tag Question

Feedback

End Review

Difficulty: Average
Peer Responses

Session Progress
Responses Correct:

Responses Incorrect:

207

Responses Total:

207

Responses - % Correct:

0%

Blog (https://www.pastest.com/blog) About Pastest (https://www.pastest.com/about-us)


Contact Us (https://www.pastest.com/contact-us) Help (https://www.pastest.com/help)
Pastest 2016

https://mypastest.pastest.com/Secure/TestMe/Browser/429893#Top

2/2

8/11/2016

MyPastest

Back to Filters (/Secure/TestMe/Filter/429893/QA)

Question 114 of 207

You review a 54-year-old woman in clinic who has recently started on ezetimibe for treatment
of hypercholesterolaemia.
Which of the following statements best describes the pharmacological actions of ezetimibe?
A

It inhibits the cytochrome P450 enzyme system

Its mechanism of action is to reduce cholesterol synthesis

May lower serum digoxin concentrations

Reduces intestinal cholesterol absorption

Significantly reduces absorption of fat-soluble vitamins

Explanation

The answer is Reduces intestinal cholesterol absorption


Ezetimibe prevents cholesterol absorption from the gut by directly inhibiting cholesterol
receptors on enterocytes. In contrast to anion-exchange resins (e.g. cholestyramine),
ezetimibe does not significantly alter absorption of fat-soluble drugs (e.g. digoxin,
thyroxine) or fat-soluble vitamins (A, D, E and K).

It inhibits the cytochrome P450 enzyme system (Option A) is incorrect. Ezetimibe has no
effect on the cytochrome P450 enzyme system.
Its mechanism of action is to reduce cholesterol synthesis (Option B) is incorrect. This is the
mechanism of statins, which inhibit HMG-CoA reductase, the rate-limiting enzyme in
cholesterol synthesis.
May lower serum digoxin concentrations (Option C) is incorrect. Despite digoxin being a fatsoluble drug, ezetimibe has no significant effect on its absorption (this is unlike the effect of
cholestyramine that significantly reduces digoxin absorption).
Significantly reduces absorption of fat-soluble vitamins (Option E) is incorrect. Ezetimibe
does not significantly alter absorption of fat-soluble vitamins.
46381

https://mypastest.pastest.com/Secure/TestMe/Browser/429893#Top

1/2

8/11/2016

MyPastest

Next Question

Previous Question

Tag Question

Feedback

End Review

Difficulty: Average
Peer Responses

Session Progress
Responses Correct:

Responses Incorrect:

207

Responses Total:

207

Responses - % Correct:

0%

Blog (https://www.pastest.com/blog) About Pastest (https://www.pastest.com/about-us)


Contact Us (https://www.pastest.com/contact-us) Help (https://www.pastest.com/help)
Pastest 2016

https://mypastest.pastest.com/Secure/TestMe/Browser/429893#Top

2/2

8/11/2016

MyPastest

Back to Filters (/Secure/TestMe/Filter/429893/QA)

Question 115 of 207

A 56-year-old man presents to the Emergency Department with jaundice three weeks after
discharge following failed cardioversion for atrial fibrillation. He is a smoker of 10 cigarettes
per day and was started on a number of new medications during his hospital stay.
Examination reveals a BP of 125/72 mmHg, pulse of 80/min, atrial fibrillation. He has
jaundiced sclerae and mild tenderness in the right upper quadrant of the abdomen on
palpation. He does not appear to be in cardiac failure.
Investigations;
Hb

13.1 g/dl

WCC

8.7 x109/l

PLT

197 x109/l

Na+

138 mmol/l

K+

4.3 mmol/l

Creatinine

100 micromol/l

Bilirubin

82 micromol/l

AST

1420 U/l

ALP

395 U/l

PT

38.1 s

Echocardiogram

Structurally normal heart with no evidence of


valve disease

Which of the following is the most likely cause?


A

Amiodarone

Amlodipine

Furosemide

Ramipril

https://mypastest.pastest.com/Secure/TestMe/Browser/429893#Top

1/2

8/11/2016

MyPastest

Warfarin

Explanation
The answer is Amiodarone Amiodarone leads to abnormal liver function in 15-50% of patients for whom it is prescribed.
This can range from an isolated modest range in transaminases, to fulminant hepatitis. Whilst
onset is rare within the first month of therapy, it is well described. In this situation it should
be discontinued as soon as possible. One potential alternative in this situation given the
structurally normal heart on echocardiography is flecanide. Hepatitis is very rarely described
in conjunction with Amlodipine therapy. Cholestatic jaundice, rather than a hepatitic picture,
is more characteristically described with Ramipril therapy.
38083

Next Question

Previous Question

Tag Question

Feedback

End Review

Difficulty: Easy
Peer Responses

Session Progress
Responses Correct:

Responses Incorrect:

207

Responses Total:

207

Responses - % Correct:

0%

Blog (https://www.pastest.com/blog) About Pastest (https://www.pastest.com/about-us)


Contact Us (https://www.pastest.com/contact-us) Help (https://www.pastest.com/help)
Pastest 2016

https://mypastest.pastest.com/Secure/TestMe/Browser/429893#Top

2/2

8/11/2016

MyPastest

Back to Filters (/Secure/TestMe/Filter/429893/QA)

Question 116 of 207

A 17-year-old woman is brought into the Emergency Department by friends after she had
taken an overdose of an unknown quantity of a non-steroidal anti-inflammatory drug
(NSAID), thought to be mefenamic acid.
Which of the following statements is true concerning NSAID overdoses?
A

Convulsions occur in 50% of patients following NSAID overdose

Gastrointestinal symptoms are due to the stimulation of cyclo-oxygenase

Mefenamic acid is the NSAID most likely to cause convulsions

Metabolic alkalosis usually occurs following large ingestions of NSAIDs

Multi-dose activated charcoal is normally used for NSAID overdoses

Explanation

The answer is Mefenamic acid is the NSAID most likely to cause convulsions
Following overdoses of most NSAIDs, the main effects are mild gastrointestinal upset
with epigastric tenderness, nausea, vomiting and diarrhea. These effects are mainly due
to the inhibition of cyclo-oxygenase. Up to 10% of patients will have seizures after NSAID
overdose, particularly after mefenamic acid. Large ingestions may cause acidosis, renal
impairment, gastrointestinal haemorrhage and CNS effects (drowsiness, coma, cerebellar
signs). Treatment is with activated charcoal in patients presenting within 1 h, and
supportive care. Proton-pump inhibitors may reduce the symptoms of gastrointestinal
toxicity.

Convulsions occur in 50% of patients following NSAID overdose (Option A) is incorrect.


Seizures may occur, but the rate of occurrence is around 10%.
Gastrointestinal symptoms are due to the stimulation of cyclo-oxygenase (Option B) is
incorrect. It is inhibition rather than stimulation of cyclo-oxygenase that occurs.
Metabolic alkalosis usually occurs following large ingestions of NSAIDs (Option D) is
incorrect. NSAID ingestion may cause metabolic acidosis.
https://mypastest.pastest.com/Secure/TestMe/Browser/429893#Top

1/2

8/11/2016

MyPastest

Multi-dose activated charcoal is normally used for NSAID overdoses (Option E) is incorrect.
This is normally indicated after aspirin overdose, to enhance drug clearance, but is not
normally used for NSAIDs.
46493

Next Question

Previous Question

Tag Question

Feedback

End Review

Difficulty: Average
Peer Responses

Session Progress
Responses Correct:

Responses Incorrect:

207

Responses Total:

207

Responses - % Correct:

0%

Blog (https://www.pastest.com/blog) About Pastest (https://www.pastest.com/about-us)


Contact Us (https://www.pastest.com/contact-us) Help (https://www.pastest.com/help)
Pastest 2016

https://mypastest.pastest.com/Secure/TestMe/Browser/429893#Top

2/2

8/11/2016

MyPastest

Back to Filters (/Secure/TestMe/Filter/429893/QA)

Question 117 of 207

A 54-year-old woman with a long history of rheumatoid arthritis presents with a dry cough
and increasing dyspnoea. Investigation shows SaO2 85% on air. On examination she displays
bilateral crackles. An infection screen is negative and you suspect drug-induced fibrosis.
Which one of the following is most likely to have been the causative drug?
A

Azathioprine

Ciclosporin

Methotrexate

Prednisolone

Sulfasalazine

Explanation

The answer is Methotrexate


Methotrexate can also cause pneumonitis, hepatic fibrosis and folate deficiency.

Azathioprine (Option A) is incorrect. Azathioprine causes dose-related myelosuppression,


hair loss, hypotension, cholestatic jaundice, arrhythmia and hypersensitivity reactions.
Ciclosporin (Option B) is incorrect. Ciclosporin is nephrotoxic and can cause cholestasis,
hypertrichosis, hypertension, gingival hyperplasia, tremor, nausea and vomiting.
Prednisolone (Option D) is incorrect. Prednisolone has numerous side-effects affecting many
organ systems but does not cause pulmonary fibrosis; it may be used as a treatment for
fibrosing alveolitis.
Sulfasalazine (Option E) is incorrect. Both sulfasalazine and methotrexate can cause
pulmonary fibrosis, but it is more commonly encountered with methotrexate therapy.
Sulfasalazine can cause myelosuppression, decreased sperm count, hepatitis, oral ulcers, rash
and haemolytic anaemia.
46969

https://mypastest.pastest.com/Secure/TestMe/Browser/429893#Top

1/2

8/11/2016

MyPastest

Next Question

Previous Question

Tag Question

Feedback

End Review

Difficulty: Average
Peer Responses

Session Progress
Responses Correct:

Responses Incorrect:

207

Responses Total:

207

Responses - % Correct:

0%

Blog (https://www.pastest.com/blog) About Pastest (https://www.pastest.com/about-us)


Contact Us (https://www.pastest.com/contact-us) Help (https://www.pastest.com/help)
Pastest 2016

https://mypastest.pastest.com/Secure/TestMe/Browser/429893#Top

2/2

8/11/2016

MyPastest

Back to Filters (/Secure/TestMe/Filter/429893/QA)

Question 118 of 207

A 23-year-old man was brought in to the Emergency Department from a nightclub after he
was found unconscious. His heart rate is 134 bpm, blood pressure was measured at 165/90
mmHg on admission. He had the following investigation results: K+ 2.5 mmol/l, Na+ 138
mmol/l, urea 5.5 mmol/l and creatinine 85 mol/l.
Which of the following is the most likely cause?
A

Amphetamine

Atenolol

Cannabis

Ethylene glycol

Heroin

Explanation

The answer is Amphetamine


Amphetamine use may be associated with mydriasis, hypertension, tachycardia, skin
pallor, hyperexcitability and, in the initial stages, agitation and increased talkativeness
and euphoria. Hypokalaemia is seen as a result of sympathetic stimulation. Severe
toxicity may be associated with hyperpyrexia, rhabdomyolysis, acute renal failure and
acute liver failure. High blood pressure may require -blockade, and agitated patients
may be sedated with benzodiazepines.

Atenolol (Option B) is incorrect. Atenolol may cause significant bradycardia and heart block.
Cannabis (Option C) is incorrect. Cannabis rarely causes such significant increased heart rate
and blood pressure.
Ethylene glycol (Option D) is incorrect. Ethylene glycol toxicity may cause profound
hypocalcaemia and renal failure; hypokalaemia is uncommon.
Heroin (Option E) is incorrect. Opioids are associated with hypotension and bradycardia.
46929

https://mypastest.pastest.com/Secure/TestMe/Browser/429893#Top

1/2

8/11/2016

MyPastest

Next Question

Previous Question

Tag Question

Feedback

End Review

Difficulty: Average
Peer Responses

Session Progress
Responses Correct:

Responses Incorrect:

207

Responses Total:

207

Responses - % Correct:

0%

Blog (https://www.pastest.com/blog) About Pastest (https://www.pastest.com/about-us)


Contact Us (https://www.pastest.com/contact-us) Help (https://www.pastest.com/help)
Pastest 2016

https://mypastest.pastest.com/Secure/TestMe/Browser/429893#Top

2/2

8/11/2016

MyPastest

Back to Filters (/Secure/TestMe/Filter/429893/QA)

Question 119 of 207

A 54-year-old gardener with long-standing depression presents to the Emergency


Department after taking a deliberate overdose of pesticide product. The package label states
that the produce contains Paraquat.
What is the main mechanism of Paraquat toxicity?
A

Cell injury and death mediated by oxygen free radical toxicity

Early death may be caused by pulmonary fibrosis

Liver injury

Oesophageal irritation and obstruction

Tissue hypoxia

Explanation

The correct answer is Cell injury and death is mediated by oxygen free radical toxicity
Paraquat is highly toxic, and as little as 2 g in an adult may be fatal (10 ml of a
concentrated 20% solution). Paraquat is rapidly absorbed and is sequestered in the
lungs. Its mechanism of toxicity is reaction with oxygen to produce hydrogen peroxide
and superoxide, which cause oxidative tissue injury, inflammation and an acute alveolitis.
Death tends to occur due to respiratory failure within hours to days in patients that
ingest more than 6 g of Paraquat. Pulmonary fibrosis is a later complication that can
develop up to 6 weeks after ingestion. Early features may be caused by corrosive effects
on the gastrointestinal tract and oropharynx.

Early death may be caused by pulmonary fibrosis (Option B) is incorrect. Pulmonary fibrosis
is a later complication that can develop 6 weeks or more after ingestion.
Liver injury (Option C) is incorrect. This is a less important mechanism than pulmonary and
gastrointestinal toxicity.
Oesophageal irritation and obstruction (Option D) is incorrect. Although these are
recognised features, the main mechanism of death is pulmonary toxicity.
https://mypastest.pastest.com/Secure/TestMe/Browser/429893#Top

1/2

8/11/2016

MyPastest

Tissue hypoxia (Option E) is incorrect. Although severe tissue hypoxia is undesirable, minor
hypoxia can be tolerated. Oxygen supplementation can increase free radical formation and
worsen pulmonary toxicity; supplementary administration should be avoided if possible.
45796

Next Question

Previous Question

Tag Question

Feedback

End Review

Difficulty: Average
Peer Responses

Session Progress
Responses Correct:

Responses Incorrect:

207

Responses Total:

207

Responses - % Correct:

0%

Blog (https://www.pastest.com/blog) About Pastest (https://www.pastest.com/about-us)


Contact Us (https://www.pastest.com/contact-us) Help (https://www.pastest.com/help)
Pastest 2016

https://mypastest.pastest.com/Secure/TestMe/Browser/429893#Top

2/2

8/11/2016

MyPastest

Back to Filters (/Secure/TestMe/Filter/429893/QA)

Question 120 of 207

The husband of a woman admitted to hospital with meningococcal meningitis asks you about
the risks that he too may develop meningitis, and asks you about the possible role of
prophylactic antibiotic treatment.
What is the best choice of antibiotic prophylaxis for close contacts of patients with
meningococcal meningitis?
A

Ceftriaxone

Ciprofloxacin

Co-trimoxazole

Rifampicin

Vancomycin

Explanation

The answer is Ciprofloxacin


Household and other close contacts of patients with pyogenic meningitis should be
given oral ciprofloxacin.

Ceftriaxone (Option A) is incorrect. Ceftriaxone is the drug of choice for treatment of


meningitis without a blanching rash, aged 1850 years.
Co-trimoxazole (Option C) is incorrect. Co-trimoxazole is useful for adults over 50 years of
age with bacterial meningitis, where Listeria monocytogenes infection is suspected.
Rifampicin (Option D) is incorrect. Rifampicin is effective against meningococcal meningitis
but a recent Cochrane review has supported ciprofloxacin ahead of it as prophylaxis of
choice owing to greater tolerability.
Vancomycin (Option E) is incorrect. Vancomycin is given in penicillin-resistant pneumococcal
infection and to those with a history of anaphylaxis to -lactam antibiotics.
46736

https://mypastest.pastest.com/Secure/TestMe/Browser/429893#Top

1/2

8/11/2016

MyPastest

Next Question

Previous Question

Tag Question

Feedback

End Review

Difficulty: Average
Peer Responses

Session Progress
Responses Correct:

Responses Incorrect:

207

Responses Total:

207

Responses - % Correct:

0%

Blog (https://www.pastest.com/blog) About Pastest (https://www.pastest.com/about-us)


Contact Us (https://www.pastest.com/contact-us) Help (https://www.pastest.com/help)
Pastest 2016

https://mypastest.pastest.com/Secure/TestMe/Browser/429893#Top

2/2

8/11/2016

MyPastest

Back to Filters (/Secure/TestMe/Filter/429893/QA)

Question 121 of 207

You are reviewing a 73-year-old man with multiple cardiac problems. He is currently taking
ramipril, metoprolol, furosemide, amiodarone and aspirin. Over the past few months he has
noted problems with night glare and his optician has diagnosed corneal microdeposits.
Which of the following drugs in his regime is most likely to be responsible?
A

Amiodarone

Aspirin

Furosemide

Metoprolol

Ramipril

Explanation

The answer is Amiodarone


Amiodarone has a number of long-term side-effects, one of which is the development of
corneal microdeposits; this may manifest in night-time visual glare. Amiodarone contains
iodine and may result in the development of hypo- or hyperthyroidism, both of which
may be complex to manage. Rare but recognised respiratory adverse effects include
pneumonitis and pulmonary fibrosis, and should be considered in patients that present
with increasing shortness of breath and cough. Hepatotoxicity may occur, and patients
on amiodarone should be monitored for liver function test abnormalities.

Aspirin (Option B) is incorrect. Aspirin is not expected to cause visual disturbance.


Furosemide (Option C) is incorrect. Furosemide is not expected to cause visual disturbance.
Metoprolol (Option D) is incorrect. Beta-blockers are not expected to cause visual
impairment; they may be used to treat glaucoma.
Ramipril (Option E) is incorrect. ACE inhibitors are not expected to cause visual impairment.
46836

https://mypastest.pastest.com/Secure/TestMe/Browser/429893#Top

1/2

8/11/2016

MyPastest

Next Question

Previous Question

Tag Question

Feedback

End Review

Difficulty: Average
Peer Responses

Session Progress
Responses Correct:

Responses Incorrect:

207

Responses Total:

207

Responses - % Correct:

0%

Blog (https://www.pastest.com/blog) About Pastest (https://www.pastest.com/about-us)


Contact Us (https://www.pastest.com/contact-us) Help (https://www.pastest.com/help)
Pastest 2016

https://mypastest.pastest.com/Secure/TestMe/Browser/429893#Top

2/2

8/11/2016

MyPastest

Back to Filters (/Secure/TestMe/Filter/429893/QA)

Question 122 of 207

A 25-year-old woman is admitted to hospital with severe diarrhoea following treatment with
amoxicillin for a bad throat infection. Stool culture yields Clostridium difficile. She becomes
dehydrated requiring IV fluids and is unable to tolerate oral medication owing to her sore
throat.
What is the best IV treatment?
A

Erythromycin

Metronidazole

Neomycin

Tobramycin

Vancomycin

Explanation

The answer is Metronidazole


If parenteral treatment is required, IV metronidazole is effective, as excretion of
metronidazole into bile occurs, and exudation from the inflamed colon results in
bactericidal levels in faeces.

Erythromycin (Option A) is incorrect. Erythromycin may itself cause an overgrowth of C.


difficile and is a prokinetic agent that may worsen diarrhoea symptoms.
Neomycin (Option C) is incorrect. Drugs of the neomycin group are not significantly
absorbed from the gut and oral formulations are effective against bacterial gastroenteritis.
Tobramycin (Option D) is incorrect. Tobramycin is an aminoglycoside active against Grampositive and Gram-negative bacteria; it is not useful in the treatment of C. difficile infection.
Vancomycin (Option E) is incorrect. Both vancomycin and metronidazole are effective after
oral administration. Intravenous vancomycin is less effective than metronidazole as it has
lower availability within the gut after intravenous administration.
46775

https://mypastest.pastest.com/Secure/TestMe/Browser/429893#Top

1/2

8/11/2016

MyPastest

Next Question

Previous Question

Tag Question

Feedback

End Review

Difficulty: Average
Peer Responses

Session Progress
Responses Correct:

Responses Incorrect:

207

Responses Total:

207

Responses - % Correct:

0%

Blog (https://www.pastest.com/blog) About Pastest (https://www.pastest.com/about-us)


Contact Us (https://www.pastest.com/contact-us) Help (https://www.pastest.com/help)
Pastest 2016

https://mypastest.pastest.com/Secure/TestMe/Browser/429893#Top

2/2

8/11/2016

MyPastest

Back to Filters (/Secure/TestMe/Filter/429893/QA)

Question 123 of 207

Therapeutic drug monitoring involving direct measurement of circulating drug concentrations


may be of help in some clinical situations.
Which of the following drugs is most likely to benefit from therapeutic drug monitoring?
A

Carbimazole

Cyclophosphamide

Rifampicin

Theophylline

Warfarin

Explanation

The answer is Theophylline Therapeutic drug monitoring may be helpful where (1) there is a close relationship
between drug concentrations and drug effect or toxicity, and (2) there is a narrow
therapeutic index, ie small difference between drug concentrations required for efficacy
versus those that might cause toxicity. Theophylline has a narrow therapeutic window
and needs close monitoring of its serum level to avoid toxicity; many drugs that interfere
with P450 enzyme activity may influence theophylline concentrations.

Carbimazole (Option A) is incorrect. The dose of carbimazole required to treat


hyperthyroidism is best determined by correlation with clinical features and measuring
thyroxine and thyroid-stimulating hormone levels.
Cyclophosphamide (Option B) is incorrect. Bone marrow and bladder toxicity related to
cyclophosphamide may be monitored by regular full blood count measurement and
urinalysis.
Rifampicin (Option C) is incorrect. Rifampicin has a wide therapeutic window, therefore
monitoring serum concentrations is not indicated.

https://mypastest.pastest.com/Secure/TestMe/Browser/429893#Top

1/2

8/11/2016

MyPastest

Warfarin (Option E) is incorrect. Measurements of warfarin concentrations are of limited


clinical value. The INR (International Normalised Ratio) is used to monitor the effect of
warfarin, which has greater practical significance in terms of efficacy and bleeding risk.
46607

Next Question

Previous Question

Tag Question

Feedback

End Review

Difficulty: Average
Peer Responses

Session Progress
Responses Correct:

Responses Incorrect:

207

Responses Total:

207

Responses - % Correct:

0%

Blog (https://www.pastest.com/blog) About Pastest (https://www.pastest.com/about-us)


Contact Us (https://www.pastest.com/contact-us) Help (https://www.pastest.com/help)
Pastest 2016

https://mypastest.pastest.com/Secure/TestMe/Browser/429893#Top

2/2

8/11/2016

MyPastest

Back to Filters (/Secure/TestMe/Filter/429893/QA)

Question 124 of 207

You are asked to review a 55-year-old woman who is currently taking aciclovir for an episode
of severe shingles. She is also taking digoxin for atrial fibrillation and diazepam for anxiety,
fluoxetine for depression and temazepam at night.
Which of these drugs would be expected to have greatest water solubility?
A

Aciclovir

Diazepam

Digoxin

Fluoxetine

Temazepam

Explanation

The answer is Digoxin


Digoxin is highly soluble at physiological pH and is readily reconstituted for intravenous
administration. Drugs with high water solubility may be more readily absorbed in the
fasted state, whereas drugs with high lipid solubility may be more readily absorbed in the
presence of dietary fat intake.

Aciclovir (Option A) is incorrect. Aciclovir is water soluble at 25C, but has poor solubility at
physiological pH and body temperature.
Diazepam (Option B) is incorrect. Diazepam is highly lipid soluble, and readily crosses the
bloodbrain barrier. Intravenous formulations are prepared as a soya emulsion to overcome
the lack of water solubility.
Fluoxetine (Option D) is incorrect. Fluoxetine is lipid soluble, which may be beneficial for
crossing the bloodbrain barrier.
Temazepam (Option E) is incorrect. Temazpam is highly lipid soluble, and readily crosses the
blood brainbarrier.
46888

https://mypastest.pastest.com/Secure/TestMe/Browser/429893#Top

1/2

8/11/2016

MyPastest

Next Question

Previous Question

Tag Question

Feedback

End Review

Difficulty: Average
Peer Responses

Session Progress
Responses Correct:

Responses Incorrect:

207

Responses Total:

207

Responses - % Correct:

0%

Blog (https://www.pastest.com/blog) About Pastest (https://www.pastest.com/about-us)


Contact Us (https://www.pastest.com/contact-us) Help (https://www.pastest.com/help)
Pastest 2016

https://mypastest.pastest.com/Secure/TestMe/Browser/429893#Top

2/2

8/11/2016

MyPastest

Back to Filters (/Secure/TestMe/Filter/429893/QA)

Question 125 of 207

You are at the scene of a cardiac arrest within the hospital, when one of the nurses
accidentally injects the distal part of her thumb with adrenaline from an auto injector. The
finger has become white and intensely painful.
Which one of the following is the most appropriate way to manage the problem?
A

Administer 60 mg of nifedipine orally

Administer 50 mg of oral atenolol

Give sublingual GTN

Give local phentolamine

Administer IV labetalol

Explanation
Accidental administration of adrenaline
Where there is significant digital ischaemia, emergency medicine protocols (Velissariou
et al., 2004) suggest that local infiltration of phentolamine (an -blocker) is the most
effective treatment
An alternative possibility is locally applied GTN paste
Oral or IV options are much less effective because when phentolamine is infiltrated
locally, much higher doses can be achieved
Reference
Velissariou, I., Cottrell, S., Berry, K., Wilson, B. 2004. Management of adrenaline (epinephrine)
induced digital ischaemia in children after accidental injection from an EpiPen.
Emergency Medicine Journal 21, 387388.
20944

Next Question

https://mypastest.pastest.com/Secure/TestMe/Browser/429893#Top

1/2

8/11/2016

MyPastest

Previous Question

Tag Question

Feedback

End Review

Difficulty: Average
Peer Responses

Session Progress
Responses Correct:

Responses Incorrect:

207

Responses Total:

207

Responses - % Correct:

0%

Blog (https://www.pastest.com/blog) About Pastest (https://www.pastest.com/about-us)


Contact Us (https://www.pastest.com/contact-us) Help (https://www.pastest.com/help)
Pastest 2016

https://mypastest.pastest.com/Secure/TestMe/Browser/429893#Top

2/2

8/11/2016

MyPastest

Back to Filters (/Secure/TestMe/Filter/429893/QA)

Question 126 of 207

You see a 67-year-old man in the medical outpatient department for investigation of low back
pain. He has a past medical history of prostate cancer and is currently receiving buserelin.
Which of the following best explains the pharmacological mechanism of action of buserelin?
A

Blockade of androgen receptors

Decreased androgen production

Increased androgen metabolism

Inhibition of 5-reductase activity

Prevention of the progression of bone lesions

Explanation

The answer is Decreased androgen production


Buserelin is a gonadotrophin releasing hormone agonist that exerts its actions at the
level of the pituitary gland. Initially treatment causes increased gonadotrophin release;
however, after a few weeks of continued therapy, gonadotrophin production is inhibited,
and testosterone levels fall. The initial increase in testosterone levels may be
accompanied by a flare in disease symptoms in some patients.

Blockade of androgen receptors (Option A) is incorrect. Flutamide, nilutamide, bicalutamide


and enzalutamide are androgen receptor blockers.
Increased androgen metabolism (Option C) is incorrect. This mechanism is not relevant to
buserelin.
Inhibition of 5-reductase activity (Option D) is incorrect. 5-reductase inhibitors (e.g.
finasteride, dutasteride and alfatradiol) prevent conversion of testosterone to its more potent
metabolite dihydrotestosterone (DHT).
Prevention of the progression of bone lesions (Option E) is incorrect. Bisphosphonates may
be helpful.
46959

https://mypastest.pastest.com/Secure/TestMe/Browser/429893#Top

1/2

8/11/2016

MyPastest

Next Question

Previous Question

Tag Question

Feedback

End Review

Difficulty: Average
Peer Responses

Session Progress
Responses Correct:

Responses Incorrect:

207

Responses Total:

207

Responses - % Correct:

0%

Blog (https://www.pastest.com/blog) About Pastest (https://www.pastest.com/about-us)


Contact Us (https://www.pastest.com/contact-us) Help (https://www.pastest.com/help)
Pastest 2016

https://mypastest.pastest.com/Secure/TestMe/Browser/429893#Top

2/2

8/11/2016

MyPastest

Back to Filters (/Secure/TestMe/Filter/429893/QA)

Question 127 of 207

A 66-year-old man with a history of chronic obstructive airways disease asks you about the
benefits of acetylcysteine therapy.
Which of the following best describes the mode of action of N-acetylcysteine?
A

Dopamine agonism

Histamine antagonism

Phase I induction

Reduction of circulating oxidative stress

Reduction of the formation of nitric oxide

Explanation

The answer is Reduction of circulating oxidative stress


N-acetylcysteine (NAC) exhibits direct and indirect antioxidant properties. NACs free
thiol group is capable of directly interacting with electrophilic groups. This leads to
intermediate formation of NAC thiol, with NAC disulfide as a major end-product. NAC
exerts an indirect antioxidant effect related to its role as a glutathione (GSH) precursor;
GSH is a tripeptide made up of glutamic acid, cysteine and glycine, and is a major factor
that protects against intracellular oxidative stress. It is the sulfhydryl group of
acetylcysteine and GSH that confers the antioxidant effects. GSH synthesis takes place
mainly in the liver (which acts as a reservoir) and the lungs.

Dopamine agonism (Option A) is incorrect. Acetylcysteine has no significant effect on


dopaminergic pathways.
Histamine antagonism (Option B) is incorrect. Acetylcysteine provokes histamine release, a
major mechanism that underlies the occurrence of anaphylactoid reactions.
Phase I induction (Option C) is incorrect. Acetylcysteine has no significant effects on hepatic
enzyme activity.

https://mypastest.pastest.com/Secure/TestMe/Browser/429893#Top

1/2

8/11/2016

MyPastest

Reduction of the formation of nitric oxide (Option E) is incorrect. Acetylcysteine promotes


formation of nitric oxide, one of the factors that promotes flushing and vasodilatation.
46944

Next Question

Previous Question

Tag Question

Feedback

End Review

Difficulty: Average
Peer Responses

Session Progress
Responses Correct:

Responses Incorrect:

207

Responses Total:

207

Responses - % Correct:

0%

Blog (https://www.pastest.com/blog) About Pastest (https://www.pastest.com/about-us)


Contact Us (https://www.pastest.com/contact-us) Help (https://www.pastest.com/help)
Pastest 2016

https://mypastest.pastest.com/Secure/TestMe/Browser/429893#Top

2/2

8/11/2016

MyPastest

Back to Filters (/Secure/TestMe/Filter/429893/QA)

Question 128 of 207

You are asked to review a 46-year-old man who is being treated for suspected occupational
poisoning by heavy metals, and is receiving treatment with d-penicillamine.
For which of the following heavy metals is d-penicillamine most likely to be effective?
A

Arsenic

Cadmium

Copper

Mercury

Thallium

Explanation

The answer is Copper


d-Penicillamine may be used to reduce the body burden of copper after occupational
exposure or in patients with Wilsons disease. It acts as a chelating agent, and is also
effective as a means of reducing total body lead burden.

Arsenic (Option A) is incorrect. The mainstay of treatment is supportive care, but in severe
cases then DMPS or DMSA may be considered as chelating agents to reduce arsenic toxicity.
Cadmium (Option B) is incorrect. Cadmium may be chelated by sodium calcium edetate in
severe cases.
Mercury (Option D) is incorrect. Mercury metal is inert, but various mercurial salts are
associated with toxic effects. Historically, dimercaprol has been used to chelate mercury, but
it is associated with adverse effects; DMPS or DMSA are preferred chelating agents in severe
mercury poisoning.
Thallium (Option E) is incorrect. Thallium metal is poorly absorbed and of limited toxicity;
thallium salts are more readily absorbed and toxic. The most appropriate antidote in thallium
poisoning is Prussian Blue.
46624

https://mypastest.pastest.com/Secure/TestMe/Browser/429893#Top

1/2

8/11/2016

MyPastest

Next Question

Previous Question

Tag Question

Feedback

End Review

Difficulty: Average
Peer Responses

Session Progress
Responses Correct:

Responses Incorrect:

207

Responses Total:

207

Responses - % Correct:

0%

Blog (https://www.pastest.com/blog) About Pastest (https://www.pastest.com/about-us)


Contact Us (https://www.pastest.com/contact-us) Help (https://www.pastest.com/help)
Pastest 2016

https://mypastest.pastest.com/Secure/TestMe/Browser/429893#Top

2/2

8/11/2016

MyPastest

Back to Filters (/Secure/TestMe/Filter/429893/QA)

Question 129 of 207

A 39-year-old woman, on oral medication for severe rheumatoid arthritis, attends the clinic
with a history of nausea, lethargy and drowsiness. Investigations reveal a pancytopaenia, urea
30 mmol/l and creatinine 600 mol/l.
Which one of the following drugs is most likely to cause these adverse effects?
A

d-Penicillamine

Gold

Hydroxychloroquine

Infliximab

Methotrexate

Explanation

The answer is d-penicillamine


d-Penicillamine is associated with development of pancytopaenia and acute
tubulointerstitial nephritis.

Gold (Option B) is incorrect. Gold may present with marrow suppression and nephropathy as
well; however, gold is administered by deep intramuscular injection.
Hydroxychloroquine (Option C) is incorrect. Hydroxychloroquine is usually prescribed for
mild to moderate rheumatoid arthritis and not in severe cases, as it is a relatively weak
antirheumatic drug; its main side-effects include corneal deposits, decrease in peripheral
vision and retinopathy.
Infliximab (Option D) is incorrect. Although major side-effects with infliximab are rare,
suppression of the immune response and reactivation of latent tuberculosis or the
development of immunosuppression-related malignancy may occur.
Methotrexate (Option E) is incorrect. Methotrexate may rarely cause myelosuppression;
hepatotoxicity and hepatic fibrosis are much more common adverse reactions.
46773

https://mypastest.pastest.com/Secure/TestMe/Browser/429893#Top

1/2

8/11/2016

MyPastest

Next Question

Previous Question

Tag Question

Feedback

End Review

Difficulty: Average
Peer Responses

Session Progress
Responses Correct:

Responses Incorrect:

207

Responses Total:

207

Responses - % Correct:

0%

Blog (https://www.pastest.com/blog) About Pastest (https://www.pastest.com/about-us)


Contact Us (https://www.pastest.com/contact-us) Help (https://www.pastest.com/help)
Pastest 2016

https://mypastest.pastest.com/Secure/TestMe/Browser/429893#Top

2/2

8/11/2016

MyPastest

Back to Filters (/Secure/TestMe/Filter/429893/QA)

Question 130 of 207

You review a 36-year-old man with a family history of premature cardiovascular disease.
Fasting cholesterol is 8.2 mmol/l and high-density lipoprotein (HDL) is 1.4 mmol/l. You elect
to commence him on atorvastatin 20 mg daily.
Which of the following is best recognised as a mechanism of action of statins?
A

They increase lipoprotein lipase

They lower very-low-density lipoprotein (VLDL) synthesis

They stimulate fatty acid synthetase

They stimulate 3-hydroxy-3-methylglutaryl coenzyme A (HMG-CoA) reductase

They stimulate sodiumpotassium ATPase activity

Explanation

The answer is They lower very-low-density lipoprotein (VLDL) synthesis


Statins inhibit 3-hydroxy-3-methylglutaryl coenzyme A (HMG-CoA) reductase, which
catalyses the rate-limiting step in cholesterol synthesis. Statins lead to increased hepatic
low-density lipoprotein (LDL) receptors and reduced hepatic very-low-density
lipoprotein (VLDL) synthesis, coupled with increased VLDL clearance. Statins may cause
myositis and abnormal liver function, although the risk of these adverse effects is
normally outweighed by the potential benefits of reduction of cardiovascular disease in
high-risk patients. Incidence of myositis is increased with concomitant use of fibric acid
derivatives, and statins are known to interact with both ciclosporin and nicotinic acid.
Macrolide antibiotics and digoxin also interact with some statins.

They increase lipoprotein lipase (Option A) is incorrect. Some statins are recognised to
increase lipoprotein lipase activity, thereby reducing triglyceride concentrations; the effect is
modest, and data concerning atorvastatin are controversial.
They stimulate fatty acid synthetase (Option C) is incorrect. Statins have a modest effect to
reduce the synthesis of triglycerides.

https://mypastest.pastest.com/Secure/TestMe/Browser/429893#Top

1/2

8/11/2016

MyPastest

They stimulate 3-hydroxy-3-methylglutaryl coenzyme A (HMG-CoA) reductase (Option D) is


incorrect. Statins inhibit the activity of this enzyme.
They stimulate sodiumpotassium ATPase activity (Option E) is incorrect. Statins have no
significant effect on sodiumpotassium ATPase activity.
46948

Next Question

Previous Question

Tag Question

Feedback

End Review

Difficulty: Average
Peer Responses

Session Progress
Responses Correct:

Responses Incorrect:

207

Responses Total:

207

Responses - % Correct:

0%

Blog (https://www.pastest.com/blog) About Pastest (https://www.pastest.com/about-us)


Contact Us (https://www.pastest.com/contact-us) Help (https://www.pastest.com/help)
Pastest 2016

https://mypastest.pastest.com/Secure/TestMe/Browser/429893#Top

2/2

8/11/2016

MyPastest

Back to Filters (/Secure/TestMe/Filter/429893/QA)

Question 131 of 207

A 56-year-old recently bereaved man is admitted with bradycardia and profound


hypotension. He has a history of treatment for hypertension, and you are concerned that his
cardiovascular findings are caused by atenolol toxicity.
Which of the following pharmaceutical agents is most suitable in initial treatment of
suspected -blocker overdose?
A

Adrenaline

Glucagon

Insulin

Neostigmine

Pyridostigmine

Explanation

The answer is Glucagon


Overdose The initial treatment of choice for a -blocker overdose is atropine. However,
the preferred treatment for patients with profound bradycardia and hypotension is an
intravenous bolus of a glucagon followed by an intravenous glucagon infusion. Care
should be taken to protect the airway because vomiting is a characteristic adverse effect
of glucagon administration.

Adrenaline (Option A) is incorrect. Adrenaline and isoprenaline can be given as an alternative


if glucagon is unavailable. In severe -blocker poisoning there is irreversible binding of blocker to the -adrenoceptor, so that isoprenaline is of limited efficacy. In cases of severe
bradycardia unresponsive to medical therapy, cardiac pacing may be necessary, but this may
fail to capture and cardiopulmonary bypass may be required until the -blocker is cleared
from the body.
Insulin (Option C) is incorrect. Insulin therapy plays an important role as an antidote in
calcium-channel blocker poisoining. Insulin administration may be indicated in -blocker
poisoning that fails to respond to inotropes and glucagon administration.
https://mypastest.pastest.com/Secure/TestMe/Browser/429893#Top

1/2

8/11/2016

MyPastest

Neostigmine (Option D) is incorrect. Neostigmine and pyridostigmine are used for the
reversal of muscle relaxants.
Pyridostigmine (Option E) is incorrect. Neostigmine and pyridostigmine are used for the
reversal of muscle relaxants.
46830

Next Question

Previous Question

Tag Question

Feedback

End Review

Difficulty: Average
Peer Responses

Session Progress
Responses Correct:

Responses Incorrect:

207

Responses Total:

207

Responses - % Correct:

0%

Blog (https://www.pastest.com/blog) About Pastest (https://www.pastest.com/about-us)


Contact Us (https://www.pastest.com/contact-us) Help (https://www.pastest.com/help)
Pastest 2016

https://mypastest.pastest.com/Secure/TestMe/Browser/429893#Top

2/2

8/11/2016

MyPastest

Back to Filters (/Secure/TestMe/Filter/429893/QA)

Question 132 of 207

A 20-year-old woman who suffers from intermittent torticollis has severe vomiting and
dehydration after a food-poisoning incident. You decide to give her an antiemetic as she is
unable to tolerate oral intake.
Which medication would be most likely to cause adverse effects and should be avoided?
A

Cetirizine

Domperidone

Ondansetron

Prochlorperazine

Promethazine

Explanation

The answer is Prochlorperazine


Torticollis or acute dystonia may occur in between 0.5% and 1% of patients given either
prochlorperazine or metoclopramide, and the rate of occurrence is higher in young
women. Benzatropine and procyclidine may be used to manage acute drug-induced
dystonia.

Cetirizine (Option A) is incorrect. Cetirizine may worsen dry mouth sensation, but this poses
less of a hazard than prochlorperazine.
Domperidone (Option B) is incorrect. Domperidone would be effective in this situation.
Ondansetron (Option C) is incorrect. Ondansetron would be effective in this situation.
Promethazine (Option E) is incorrect. Promethazine may cause drowsiness.
46990

Next Question

https://mypastest.pastest.com/Secure/TestMe/Browser/429893#Top

1/2

8/11/2016

MyPastest

Previous Question

Tag Question

Feedback

End Review

Difficulty: Average
Peer Responses

Session Progress
Responses Correct:

Responses Incorrect:

207

Responses Total:

207

Responses - % Correct:

0%

Blog (https://www.pastest.com/blog) About Pastest (https://www.pastest.com/about-us)


Contact Us (https://www.pastest.com/contact-us) Help (https://www.pastest.com/help)
Pastest 2016

https://mypastest.pastest.com/Secure/TestMe/Browser/429893#Top

2/2

8/11/2016

MyPastest

Back to Filters (/Secure/TestMe/Filter/429893/QA)

Question 133 of 207

A 22-year-old man was admitted to the Acute Medical Unit after partial seizures, and has
been discharged home with a supply of new medications. You are reviewing him 3 weeks
later in the outpatient department and he complains of double vision and unsteady gait,
which have progressively worsened over the past 2 weeks.
Which one of the following drugs is most likely to have caused these symptoms?
A

Carbamazepine

Gabapentin

Tiagabine

Topiramate

Vigabatrin

Explanation

The answer is Carbamazepine


The most common dose-related adverse effects of carbamazepine are related to
cerebellar toxicity, including diplopia and ataxia. The effects are dose-dependent, and
are fully reversible on stopping therapy. Phenytoin may also cause dose-dependent
cerebellar toxicity.

Gabapentin (Option B) is incorrect. Gabapentin causes drowsiness, headache and tremor.


Tiagabine (Option C) is incorrect. Tiagabine may cause dizziness, confusion, somnolence and
ataxia.
Topiramate (Option D) is incorrect. Topirimate typically causes drowsiness, altered mood,
acute myopia, glaucoma and urolithiasis.
Vigabatrin (Option E) is incorrect. Vigabatrin can cause drowsiness, dizziness and weight
gain. Long-term therapy has been associated with visual field defects in up to one-third of
patients, which may be irreversible.
46779

https://mypastest.pastest.com/Secure/TestMe/Browser/429893#Top

1/2

8/11/2016

MyPastest

Next Question

Previous Question

Tag Question

Feedback

End Review

Difficulty: Average
Peer Responses

Session Progress
Responses Correct:

Responses Incorrect:

207

Responses Total:

207

Responses - % Correct:

0%

Blog (https://www.pastest.com/blog) About Pastest (https://www.pastest.com/about-us)


Contact Us (https://www.pastest.com/contact-us) Help (https://www.pastest.com/help)
Pastest 2016

https://mypastest.pastest.com/Secure/TestMe/Browser/429893#Top

2/2

8/11/2016

MyPastest

Back to Filters (/Secure/TestMe/Filter/429893/QA)

Question 134 of 207

A patient on regular treatment for anxiety symptoms stops his medication abruptly on the
advice of one of his friends. Several days later, he develops acute anxiety, insomnia, irritability,
rage, feelings of unreality and depersonalisation, diplopia, paraesthesias, palpitations, flushing
and hyperventilation.
Cessation of which drug is most likely to have caused these withdrawal symptoms?
A

Amitriptyline

Buspirone

Lorazepam

Paroxetine

Phenelzine

Explanation

The answer is Lorazepam


Benzodiazepines (lorazepam, diazepam, chlordiazepoxide) withdrawal symptoms may be
even worse than the condition for which the drug was originally prescribed. Symptoms
appear sooner with the rapidly eliminated benzodiazepines such as lorazepam.

Amitriptyline (Option A) is incorrect. Withdrawal symptoms due to tricyclic antidepressants


(eg amitriptyline) are rare and include cholinergic effects such as abdominal cramps,
diarrhoea, vomiting and dehydration, extrapyramidal symptoms, anxiety, psychosis, delirium
and mania.
Buspirone (Option B) is incorrect. Buspirone is claimed not to have withdrawal effects; nor
does it have sedative or muscle relaxant properties. It is an azapirone, and is therefore
distinct from other anxiolytics such as benzodiazepines.
Paroxetine (Option D) is incorrect. Withdrawal of paroxetine or other SSRI antidepressants
can lead to deterioration in mood and cognition and orofacial dystonias.

https://mypastest.pastest.com/Secure/TestMe/Browser/429893#Top

1/2

8/11/2016

MyPastest

Phenelzine (Option E) is incorrect. Abrupt withdrawal of phenelzine leads to panic, shaking,


sweats and nausea.
46740

Next Question

Previous Question

Tag Question

Feedback

End Review

Difficulty: Average
Peer Responses

Session Progress
Responses Correct:

Responses Incorrect:

207

Responses Total:

207

Responses - % Correct:

0%

Blog (https://www.pastest.com/blog) About Pastest (https://www.pastest.com/about-us)


Contact Us (https://www.pastest.com/contact-us) Help (https://www.pastest.com/help)
Pastest 2016

https://mypastest.pastest.com/Secure/TestMe/Browser/429893#Top

2/2

8/11/2016

MyPastest

Back to Filters (/Secure/TestMe/Filter/429893/QA)

Question 135 of 207

You are drawing up guidelines for the use of insulin glargine in basal bolus regimes ahead of
isophane insulin.
When considering your guidelines, in which of the following situations does insulin glargine
have the clearest advantage over isophane?
A

In patients with type-1 diabetes who have significant daytime hypoglycaemia on


isophane

In patients with type-2 diabetes who are taking more than 15 U of isophane per day

In patients with type-2 diabetes who have episodes of daytime hypoglycaemia on


isophane

In patients with type-1 diabetes who have significant nocturnal hypoglycaemia on


isophane

In patients with type-2 diabetes who have weight gain on isophane

Explanation
Insulin
The greatest benefit with respect to reduction in the absolute number of nocturnal
hypoglycaemic events for glargine versus NPH is in Type 1 diabetes patients
Day time events are not significantly impacted
HbA1c results are similar between treated groups
In paediatric patients no significant improvement/impact of glargine on hypos has been
seen in patients with Type 1 diabetes
NICE only recommends use of insulin glargine in patients with type-2 diabetes who
have significant hypoglycaemia on isophane insulin
22481

Next Question

https://mypastest.pastest.com/Secure/TestMe/Browser/429893#Top

1/2

8/11/2016

MyPastest

Previous Question

Tag Question

Feedback

End Review

Difficulty: Average
Peer Responses

Session Progress
Responses Correct:

Responses Incorrect:

207

Responses Total:

207

Responses - % Correct:

0%

Blog (https://www.pastest.com/blog) About Pastest (https://www.pastest.com/about-us)


Contact Us (https://www.pastest.com/contact-us) Help (https://www.pastest.com/help)
Pastest 2016

https://mypastest.pastest.com/Secure/TestMe/Browser/429893#Top

2/2

8/11/2016

MyPastest

Back to Filters (/Secure/TestMe/Filter/429893/QA)

Question 136 of 207

A 72-year-old man is reviewed in the Emergency Department. He has been feeling tired and
unwell for several weeks. There is a past history of glaucoma, chronic obstructive pulmonary
disease, congestive heart failure and type-2 diabetes. His GP has recently been investigating
for anaemia. Blood tests show a metabolic acidosis with normal anion gap.
Which of the following drugs is most likely to be responsible for the acidbase disturbance?
A

Acetazolamide

Aspirin

Iron sulfate

Metformin

Ramipril

Explanation

The answer is Acetazolamide


Normal anion gap acidosis may be caused by gastrointestinal (GI) bicarbonate loss, eg
diarrhoea, pancreatic fistula, renal bicarbonate loss, renal failure and hypoaldosteronism.
Acetazolamide is a carbonic anhydrase inhibitor that causes renal sodium and
bicarbonate loss.

Aspirin (Option B) is incorrect. Aspirin provides a source of unmeasured anion so that


metabolic acidosis is associated with a raised anion gap.
Iron sulfate (Option C) is incorrect. Iron overdose may be associated with the development of
lactic acidosis.
Metformin (Option D) is incorrect. Metformin is a lactate dehydrogenase inhibitor that causes
metabolic acidosis due to accumulation of lactate; lactic acidosis causes a high anion gap.
Ramipril (Option E) is incorrect. Ramipril may uncommonly cause type-4 renal tubular
acidosis.
46950

https://mypastest.pastest.com/Secure/TestMe/Browser/429893#Top

1/2

8/11/2016

MyPastest

Next Question

Previous Question

Tag Question

Feedback

End Review

Difficulty: Average
Peer Responses

Session Progress
Responses Correct:

Responses Incorrect:

207

Responses Total:

207

Responses - % Correct:

0%

Blog (https://www.pastest.com/blog) About Pastest (https://www.pastest.com/about-us)


Contact Us (https://www.pastest.com/contact-us) Help (https://www.pastest.com/help)
Pastest 2016

https://mypastest.pastest.com/Secure/TestMe/Browser/429893#Top

2/2

8/11/2016

MyPastest

Back to Filters (/Secure/TestMe/Filter/429893/QA)

Question 137 of 207

A 27-year-old farmer presents with fever, cough and dyspnoea. On auscultation, expiratory
crackles are heard at both apices. A chest radiograph shows a round lesion in the right apex
with an air halo above it. A full blood count reveals a high eosinophil count.
Which of the following drugs would be most effective in this case?
A

Amphotericin

Fluconazole

Itraconazole

Nystatin

Terbinafine

Explanation

The answer is Amphotericin


This patient, a farmer presenting with fever, cough and dyspnoea, most probably has an
aspergilloma. If asymptomatic, patients may simply require follow-up, and 10% resolve
spontaneously. Where patients are symptomatic, particularly if there is haemoptysis, then
aspergilloma is often managed surgically by lobectomy. Alternative treatments include
bronchial arterial embolisation and local instillation of amphotericin. Pre- and postoperative systemic antifungal treatment is routinely used to prevent recurrence.
Treatment options include amphotericin with flucytosine, itraconazole and voraconazole,
but the benefit of long-term antifungal treatment is uncertain.

Fluconazole (Option B) is incorrect. Fluconazole is useful in candidiasis and central nervous


system infections with Cryptococcus neoformans, and is usually commenced after initial
treatment with amphotericin and flucytosine.
Itraconazole (Option C) is incorrect. Itraconazole is the agent of choice for non-lifethreatening blastomycosis and histoplasmosis. It is moderately effective against invasive
aspergillosis but there is less evidence to support its use than amphotericin, which is why it is
a less good answer.
https://mypastest.pastest.com/Secure/TestMe/Browser/429893#Top

1/2

8/11/2016

MyPastest

Nystatin (Option D) is incorrect. Local instillation of nystatin may be effective, but is often
given with amphotericin. Nystatin is poorly absorbed through mucous membranes and is
thus useful in oral, vaginal and enteric candidiasis.
Terbinafine (Option E) is incorrect. Terbinafine is used to treat superficial mycoses such as
dermatophyte infections.
46862

Next Question

Previous Question

Tag Question

Feedback

End Review

Difficulty: Average
Peer Responses

Session Progress
Responses Correct:

Responses Incorrect:

207

Responses Total:

207

Responses - % Correct:

0%

Blog (https://www.pastest.com/blog) About Pastest (https://www.pastest.com/about-us)


Contact Us (https://www.pastest.com/contact-us) Help (https://www.pastest.com/help)
Pastest 2016

https://mypastest.pastest.com/Secure/TestMe/Browser/429893#Top

2/2

8/11/2016

MyPastest

Back to Filters (/Secure/TestMe/Filter/429893/QA)

Question 138 of 207

You review a 42-year-old woman with type-1 diabetes who has undergone a renal transplant.
She is taking azathioprine and tacrolimus for long-term immunosuppression.
Which of the following most accurately characterises the pharmacological effects of posttransplant immunosuppressant agents?
A

Azathioprine has a half-life of 2.5 days

Azathioprine inhibits pyrimidine synthesis

Sirolimus is a calcineurin inhibitor

Tacrolimus has a half-life of around 1 h

Tacrolimus is a calcineurin inhibitor

Explanation

The answer is Tacrolimus is a calcineurin inhibitor


This is the primary mechanism by which tacrolimus prevents cytokine formation within Tlymphocytes, including reduced expression of interleukin-2.

Azathioprine has a half-life of 2.5 days (Option A) is incorrect. Azathioprine has a half-life of
around 5 h, but the duration of effect may be significantly longer.
Azathioprine inhibits pyrimidine synthesis (Option B) is incorrect. Azathioprine inhibits
purine synthesis, an essential step in the proliferation of leukocytes.
Sirolimus is a calcineurin inhibitor (Option C) is incorrect. Sirolimus is not a calcineurin
inhibitor, despite having a similar name to tacrolimus. Sirolimus inhibits the cellular response
to interleukin-2.
Tacrolimus has a half-life of around 1 h (Option D) is incorrect. Tacrolimus has a half-life of
around 11 h (similar to sirolimus).
46953

Next Question
https://mypastest.pastest.com/Secure/TestMe/Browser/429893#Top

1/2

8/11/2016

MyPastest

Previous Question

Tag Question

Feedback

End Review

Difficulty: Average
Peer Responses

Session Progress
Responses Correct:

Responses Incorrect:

207

Responses Total:

207

Responses - % Correct:

0%

Blog (https://www.pastest.com/blog) About Pastest (https://www.pastest.com/about-us)


Contact Us (https://www.pastest.com/contact-us) Help (https://www.pastest.com/help)
Pastest 2016

https://mypastest.pastest.com/Secure/TestMe/Browser/429893#Top

2/2

8/11/2016

MyPastest

Back to Filters (/Secure/TestMe/Filter/429893/QA)

Question 139 of 207

A 72-year-old woman presents for review in the fracture clinic. There is evidence of
osteoporosis and you decide to start bisphosphonate therapy. Nausea is listed as a very
common side-effect of the drug, and the patient asks for clarification of this.
What is the expected rate of occurrence for side-effects listed as very common?
A

110% of patients

2550% of patients

5075% of patients

More than 10% of patients

More than 75% of patients

Explanation

The answer is More than 10% of patients


Oesophagitis is also recognised after regular bisphosphonate use, but much less
common than simple nausea.
Very common side-effects are said to occur with a frequency of greater than 1 in 10
patients (10%).
Very common side-effects More than 1 in 10

(>10%)

Common side-effects

1 in 100 to 1 in 10

(1 - 10%)

Uncommon side-effects

1 in 1000 to 1 in 100

(0.1 - 1%)

Rare side-effects

1 in 10 000 to 1 in 1000 (0.01 - 0.1%)

Very rare side-effects

Less than 1 in 10 000

https://mypastest.pastest.com/Secure/TestMe/Browser/429893#Top

(<0.01%)

1/2

8/11/2016

MyPastest

110% of patients (Option A) is incorrect. 110% would be common rather than very
common.
2550% of patients (Option B) is incorrect. This would be extremely common. Very common
is defined as anything that occurs with a frequency greater than 10%.
5075% of patients (Option C) is incorrect. This would be extremely common. Very common
is defined as anything that occurs with a frequency greater than 10%.
More than 75% of patients (Option E) is incorrect. This would be extremely common! Very
common is defined as anything that occurs with a frequency greater than 10%.
46885

Next Question

Previous Question

Tag Question

Feedback

End Review

Difficulty: Average
Peer Responses

Session Progress
Responses Correct:

Responses Incorrect:

207

Responses Total:

207

Responses - % Correct:

0%

Blog (https://www.pastest.com/blog) About Pastest (https://www.pastest.com/about-us)


Contact Us (https://www.pastest.com/contact-us) Help (https://www.pastest.com/help)
Pastest 2016

https://mypastest.pastest.com/Secure/TestMe/Browser/429893#Top

2/2

8/11/2016

MyPastest

Back to Filters (/Secure/TestMe/Filter/429893/QA)

Question 140 of 207

You are called to see a 16-year-old girl who told her parents that she partied all night and
took five Ecstasy tablets. The most recent tablets were ingested 6 h ago. On examination she
is restless, dehydrated, her blood pressure is 100/60 mmHg and her pulse is 100/min.
What is the next step in her management?
A

Gastric lavage

Intravenous NaCl infusion

Intubation

Provocation of vomiting

Sedation

Explanation

The answer is Intravenous NaCl infusion


Ecstasy (3,4-methylenedioxymethamfetamine, MDMA) stimulates serotonergic activity
within the central nervous system. Gastric lavage may be considered if a substantial
overdose has been ingested in the preceding 1 h. Treatment involves supportive care, IV
fluids to maintain hydration status, correction of biochemical disturbance, diazepam to
reduce agitation, and active cooling if needed. Nitrates and calcium antagonists may be
needed to control blood pressure.

Gastric lavage (Option A) is incorrect. Gastric lavage is unlikely to be of benefit at 6 h postingestion.


Intubation (Option C) is incorrect. Intubation is not indicated at present, but this may be
indicated if hyperthermia fails to settle after adequate sedation.
Provocation of vomiting (Option D) is incorrect. Induced vomiting is rarely, if ever, effective in
reducing drug toxicity, and may be complicated by oesophageal rupture and gastrointestinal
bleeding.

https://mypastest.pastest.com/Secure/TestMe/Browser/429893#Top

1/2

8/11/2016

MyPastest

Sedation (Option E) is incorrect. Sedation is an important aspect of treatment but would


normally follow measures to address fluid and electrolyte balance.
46817

Next Question

Previous Question

Tag Question

Feedback

End Review

Difficulty: Average
Peer Responses

Session Progress
Responses Correct:

Responses Incorrect:

207

Responses Total:

207

Responses - % Correct:

0%

Blog (https://www.pastest.com/blog) About Pastest (https://www.pastest.com/about-us)


Contact Us (https://www.pastest.com/contact-us) Help (https://www.pastest.com/help)
Pastest 2016

https://mypastest.pastest.com/Secure/TestMe/Browser/429893#Top

2/2

8/11/2016

MyPastest

Back to Filters (/Secure/TestMe/Filter/429893/QA)

Question 141 of 207

A 19-year-old man attends the early medical review clinic 2 days after presenting to the
Emergency Department because of witnessed tonicclonic seizures. A CT head scan is
normal, and neurological examination findings are normal. He suffered a similar episode 18
months ago, and you decide to commence antiepileptic medications.
Which of the following medications would be the most appropriate first-line therapy for this
patient?
A

Lamotrigine

Phenobarbital

Phenytoin

Sodium valproate

Topiramate

Explanation

The answer is Sodium valproate


The most common drugs used to control generalised seizures in young adults are sodium
valproate and carbamazepine. Carbamazepine is a powerful enzyme inducer that may
interact with other medications; toxicity is associated with ataxia. Sodium valproate is
recognised to cause weight gain and associated with a polycystic ovary (PCOS)-like
syndrome in young women.

Lamotrigine (Option A) is incorrect. Lamotrigine is commonly used as an adjunctive drug, or


as an alternative to valproate; it has a relatively benign side-effect profile versus the other
options.
Phenobarbital (Option B) is incorrect. Phenobarbital can cause folate deficiency,
osteomalacia, neuropathy and agitation in children.
Phenytoin (Option C) is incorrect. Phenytoin may also be effective but is known to cause
serious adverse effects, including gingival hypertrophy, hirsutism, cerebellar signs, rickets
and lymphadenopathy.
https://mypastest.pastest.com/Secure/TestMe/Browser/429893#Top

1/2

8/11/2016

MyPastest

Topiramate (Option E) is incorrect. Topiramate may be used for intractable seizures or added
when seizures are difficult to control with other drugs.
46613

Next Question

Previous Question

Tag Question

Feedback

End Review

Difficulty: Average
Peer Responses

Session Progress
Responses Correct:

Responses Incorrect:

207

Responses Total:

207

Responses - % Correct:

0%

Blog (https://www.pastest.com/blog) About Pastest (https://www.pastest.com/about-us)


Contact Us (https://www.pastest.com/contact-us) Help (https://www.pastest.com/help)
Pastest 2016

https://mypastest.pastest.com/Secure/TestMe/Browser/429893#Top

2/2

8/11/2016

MyPastest

Back to Filters (/Secure/TestMe/Filter/429893/QA)

Question 142 of 207

A 25-year-old woman is prescribed an antimicrobial agent to treat a tooth infection. She


presents to the Acute Medical Unit with severe headache, blurred vision and a convergent
squint. On examination, there is diplopia, maximal on looking to the left side. The patient is
unable to abduct her left eye beyond the midline.
Which of the following drugs is most likely to be responsible for her present condition?
A

Amoxicillin

Ceftriaxone

Erythromycin

Gentamicin

Tetracycline

Explanation

The answer is Tetracycline


This patient has clinical features indicating a left VIth cranial nerve palsy, a finding in
intracranial hypertension. A number of drugs are associated with intracranial
hypertension, including tetracyclines, amiodarone, oral contraceptive pills and withdrawal
of corticosteroid therapy. The underlying mechanism is unclear.

Amoxicillin (Option A) is incorrect. Amoxicillin is not known to cause raised intracranial


pressure.
Ceftriaxone (Option B) is incorrect. Ceftriaxone is not known to cause raised intracranial
pressure.
Erythromycin (Option C) is incorrect. Erythromycin is less likely to cause raised intracranial
pressure than tetracyclines.
Gentamicin (Option D) is incorrect. Gentamicin is not known to cause raised intracranial
pressure.
46859

https://mypastest.pastest.com/Secure/TestMe/Browser/429893#Top

1/2

8/11/2016

MyPastest

Next Question

Previous Question

Tag Question

Feedback

End Review

Difficulty: Average
Peer Responses

Session Progress
Responses Correct:

Responses Incorrect:

207

Responses Total:

207

Responses - % Correct:

0%

Blog (https://www.pastest.com/blog) About Pastest (https://www.pastest.com/about-us)


Contact Us (https://www.pastest.com/contact-us) Help (https://www.pastest.com/help)
Pastest 2016

https://mypastest.pastest.com/Secure/TestMe/Browser/429893#Top

2/2

8/11/2016

MyPastest

Back to Filters (/Secure/TestMe/Filter/429893/QA)

Question 143 of 207

A patient in status asthmaticus is not responding to nebulised salbutamol and terbutaline.


The consultant respiratory physician has advised an intravenous infusion of aminophylline.
Which of the following enzymes is this drug most likely to inhibit in order to relieve the
symptoms?
A

Adenyl cyclase

Alcohol dehydrogenase

Guanyl cyclase

Monoamine oxidase

Phosphodiesterase

Explanation

The answer is Phosphodiesterase


Phosphodiesterase catalyses the conversion of cAMP to 5'-adenosine monophosphate
(5'-AMP). This reaction is blocked by aminophylline or theophylline, which causes the
accumulation of cAMP intracellularly and this results in bronchodilatation, cardiac
stimulation and vasodilatation.

Adenyl cyclase (Option A) is incorrect. Adenyl cyclase catalyses the conversion of ATP to
cyclic adenosine monophosphate (cAMP).
Alcohol dehydrogenase (Option B) is incorrect. Alcohol dehydrogenase is a key enzyme
involved in metabolism of ethanol, methanol and ethylene glycol; fomepizole is a competitive
inhibitor of alcohol dehydrogenase.
Guanyl cyclase (Option C) is incorrect. Guanyl cyclase catalyses the formation of guanosine
monophosphate, an important intracellular messenger.
Monoamine oxidase (Option D) is incorrect. Monamine oxidase inhibition is reserved for
depression that fails to respond to other treatments. Peripheral effects are mediated by
potentiation of catecholamines and include tachycardia and hypertension.
46868

https://mypastest.pastest.com/Secure/TestMe/Browser/429893#Top

1/2

8/11/2016

MyPastest

46868

Next Question

Previous Question

Tag Question

Feedback

End Review

Difficulty: Average
Peer Responses

Session Progress
Responses Correct:

Responses Incorrect:

207

Responses Total:

207

Responses - % Correct:

0%

Blog (https://www.pastest.com/blog) About Pastest (https://www.pastest.com/about-us)


Contact Us (https://www.pastest.com/contact-us) Help (https://www.pastest.com/help)
Pastest 2016

https://mypastest.pastest.com/Secure/TestMe/Browser/429893#Top

2/2

8/11/2016

MyPastest

Back to Filters (/Secure/TestMe/Filter/429893/QA)

Question 144 of 207

A 34-year-old patient has been treated for rosacea for the last 3 months. He develops bluegrey skin pigmentation.
Which of the following drugs is most likely to be responsible for this?
A

Amiodarone

Ciprofloxacin

Doxycycline

Erythromycin

Minocycline

Explanation

The answer is Minocycline


Cutaneous pigmentation due to minocycline causes discoloration of the skin, which can
be slate grey in appearance and is reversible after drug cessation. Other adverse effects
include anorexia, dizziness, tinnitus and vertigo (more common in women), acute renal
failure, discoloration of conjunctiva, tears and sweat, as well as systemic lupus
erythematosus.

Amiodarone (Option A) is incorrect. Amiodarone may cause slate grey discoloration of the
skin, but this would fit less well with the clinical scenario, hence option E is the preferred
answer.
Ciprofloxacin (Option B) is incorrect. Ciprofloxacin does not cause altered skin pigmentation
or discoloration.
Doxycycline (Option C) is incorrect. Doxycycline does not cause altered skin pigmentation or
discoloration.
Erythromycin (Option D) is incorrect. Erythromycin does not cause altered skin pigmentation
or discoloration.
46935

https://mypastest.pastest.com/Secure/TestMe/Browser/429893#Top

1/2

8/11/2016

MyPastest

Next Question

Previous Question

Tag Question

Feedback

End Review

Difficulty: Average
Peer Responses

Session Progress
Responses Correct:

Responses Incorrect:

207

Responses Total:

207

Responses - % Correct:

0%

Blog (https://www.pastest.com/blog) About Pastest (https://www.pastest.com/about-us)


Contact Us (https://www.pastest.com/contact-us) Help (https://www.pastest.com/help)
Pastest 2016

https://mypastest.pastest.com/Secure/TestMe/Browser/429893#Top

2/2

8/11/2016

MyPastest

Back to Filters (/Secure/TestMe/Filter/429893/QA)

Question 145 of 207

A patient who has mild benign prostatic hyperplasia has been advised to take finasteride.
Production of which of the following androgens is most likely to be inhibited as a result of
intake of this drug?
A

Androstenedione

Androsterone

Dehydroepiandrosterone

Dihydrotestosterone

Testosterone

Explanation

The answer is Dihydrotestosterone


Finasteride inhibits 5-reductase, which converts testosterone to dihydrotestosterone
(DHT). DHT is much more active than testosterone and binds more avidly to cytoplasmic
receptors to stimulate prostate growth. DTH may promote development of benign
prostatic hyperplasia in the elderly.

Androstenedione (Option A) is incorrect. Androstenedione concentrations are not diminished


significantly.
Androsterone (Option B) is incorrect. Androsterone is a very weak androgen that is
metabolised to DHT by hydroxytestosterone dehydrogenase; this reaction is not blocked by
finasteride.
Dehydroepiandrosterone (Option C) is incorrect. Dihydroepiandrosterone is produced by the
adrenals, gonads and other tissues, and is a precursor in synthesis of androgen.
Testosterone (Option E) is incorrect. Testosterone concentrations are not diminished
significantly.
46871

https://mypastest.pastest.com/Secure/TestMe/Browser/429893#Top

1/2

8/11/2016

MyPastest

Next Question

Previous Question

Tag Question

Feedback

End Review

Difficulty: Average
Peer Responses

Session Progress
Responses Correct:

Responses Incorrect:

207

Responses Total:

207

Responses - % Correct:

0%

Blog (https://www.pastest.com/blog) About Pastest (https://www.pastest.com/about-us)


Contact Us (https://www.pastest.com/contact-us) Help (https://www.pastest.com/help)
Pastest 2016

https://mypastest.pastest.com/Secure/TestMe/Browser/429893#Top

2/2

8/11/2016

MyPastest

Back to Filters (/Secure/TestMe/Filter/429893/QA)

Question 146 of 207

A 50-year-old patient takes 3.5 g aspirin daily together with omeprazole for her rheumatoid
arthritis. One morning her pain is worse and she takes double the dose of aspirin before
breakfast (7 g). Shortly after, she vomits and complains of tinnitus, sweating, dizziness and
hyperventilation.
What is the most likely immediate effect of excess aspirin on acidbase status?
A

Metabolic acidosis

Metabolic alkalosis

Normal pH

Respiratory acidosis

Respiratory alkalosis

Explanation

The answer is Respiratory alkalosis


When ingested in overdose, salicylates directly stimulate the respiratory centre to
produce an increase in both the depth and rate of breathing, thereby causing a
respiratory alkalosis. This occurs even before metabolic acidosis, and is not solely a
compensatory response.

Metabolic acidosis (Option A) is incorrect. Metabolic acidosis is not an immediate effect of


aspirin ingestion, but is a characteristic feature as toxicity progresses and may be very
severe, possibly requiring administration of intravenous sodium bicarbonate or haemodialysis
to restore normal acidbase status.
Metabolic alkalosis (Option B) is incorrect. Metabolic alkalosis is not a recognised feature.
Normal pH (Option C) is incorrect. pH may be normal after minor aspirin overdose, but in
aspirin overdose with severe symptoms this is unlikely to remain normal.
Respiratory acidosis (Option D) is incorrect. Respiratory acidosis is most likely to occur after
overdose involving sedative agents, eg opioids or benzodiazepines.
46814

https://mypastest.pastest.com/Secure/TestMe/Browser/429893#Top

1/2

8/11/2016

MyPastest

46814

Next Question

Previous Question

Tag Question

Feedback

End Review

Difficulty: Average
Peer Responses

Session Progress
Responses Correct:

Responses Incorrect:

207

Responses Total:

207

Responses - % Correct:

0%

Blog (https://www.pastest.com/blog) About Pastest (https://www.pastest.com/about-us)


Contact Us (https://www.pastest.com/contact-us) Help (https://www.pastest.com/help)
Pastest 2016

https://mypastest.pastest.com/Secure/TestMe/Browser/429893#Top

2/2

8/11/2016

MyPastest

Back to Filters (/Secure/TestMe/Filter/429893/QA)

Question 147 of 207

A 49-year-old woman suffers with migraine, and smokes 30 cigarettes per day. She has
noticed that taking paracetamol 1 g as advised by her GP has no effect on pain relief.
Which one of the following factors is the most likely to account for the lack of response to
paracetamol?
A

Altered volume of distribution

Delayed gastric emptying in migraine

First-pass metabolism

Hepatic enzyme induction

Reduced gut blood flow

Explanation

The answer is Hepatic enzyme induction


The contents of smoke inhaled from cigarettes can induce hepatic microsomal enzymes
of the cytochrome P450 system, one of the pathways involved in metabolism of
paracetamol. Common drugs that induce hepatic enzymes are phenytoin,
carbamazepine, barbiturates, rifampicin, chronic alcohol excess and sulfonylureas.

Altered volume of distribution (Option A) is incorrect. The volume of distribution is


unaltered.
Delayed gastric emptying in migraine (Option B) is incorrect. There may be delayed gastric
emptying in migraine, but this has a limited effect on paracetamol absorption.
First-pass metabolism (Option C) is incorrect. Although chronic smokers will have higher
levels of hepatic enzyme activity, the effect of this is greater on paracetamol metabolism and
clearance than on initial drug metabolism after absorption.
Reduced gut blood flow (Option E) is incorrect. Blood flow is unlikely to be altered
significantly.
46960

https://mypastest.pastest.com/Secure/TestMe/Browser/429893#Top

1/2

8/11/2016

MyPastest

Next Question

Previous Question

Tag Question

Feedback

End Review

Difficulty: Average
Peer Responses

Session Progress
Responses Correct:

Responses Incorrect:

207

Responses Total:

207

Responses - % Correct:

0%

Blog (https://www.pastest.com/blog) About Pastest (https://www.pastest.com/about-us)


Contact Us (https://www.pastest.com/contact-us) Help (https://www.pastest.com/help)
Pastest 2016

https://mypastest.pastest.com/Secure/TestMe/Browser/429893#Top

2/2

8/11/2016

MyPastest

Back to Filters (/Secure/TestMe/Filter/429893/QA)

Question 148 of 207

A 29-year-old woman is undergoing treatment for secondary infertility after resection of a


pituitary adenoma. She is reviewed in the endocrinology clinic and tells you that she has been
suffering from headache, dizziness on standing, and symptoms suggestive of Raynauds
phenomenon since starting a new medication prescribed at her previous clinic appointment.
Which one of the following drugs is most likely to explain these adverse effects?
A

Bromocriptine

Buserelin

Clomifene citrate

Human chorionic gonadotrophin (hCG)

Human menopausal gonadotrophin (hMG)

Explanation

The answer is Bromocriptine


Bromocriptine is a dopamine agonist used to treat Parkinsons disease. It inhibits
prolactin release from the anterior pituitary and is used in treatment of
hyperprolactinaemia and acromegaly. Adverse effects include nausea, headache and
light-headedness due to orthostatic hypotension. High doses are associated with coldinduced peripheral digital vasospasm. It should be used with caution in patients with a
history of coronary artery disease and Raynauds disease.

Buserelin (Option B) is incorrect. Buserelin is a gonadotrophin-releasing hormone (GnRH)


agonist and may cause hot flushes, decreased libido and vaginal dryness.
Clomifene citrate (Option C) is incorrect. Clomifene blocks oestrogen receptors in the
hypothalamus and stimulates pituitary release of gonadotrophin-releasing hormone (GnRH),
which in turn stimulates ovarian function in women suffering from anovulation or
oligovulation. Adverse effects characteristically include ovarian hyperstimulation (gross
ovarian enlargement, ascites, hydrothorax, hypovolaemia and shock), hot flushes, headache,
weight gain and depression.
https://mypastest.pastest.com/Secure/TestMe/Browser/429893#Top

1/2

8/11/2016

MyPastest

Human chorionic gonadotrophin (hCG) (Option D) is incorrect. Human chorionic


gonadotrophin treatment may cause headache, depression and peripheral oedema.
Human menopausal gonadotrophin (hMG) (Option E) is incorrect. Human menopausal
gonadotrophin can cause ovarian hyperstimulation, and reversible ovarian enlargement in
20% of cases.
46778

Next Question

Previous Question

Tag Question

Feedback

End Review

Difficulty: Average
Peer Responses

Session Progress
Responses Correct:

Responses Incorrect:

207

Responses Total:

207

Responses - % Correct:

0%

Blog (https://www.pastest.com/blog) About Pastest (https://www.pastest.com/about-us)


Contact Us (https://www.pastest.com/contact-us) Help (https://www.pastest.com/help)
Pastest 2016

https://mypastest.pastest.com/Secure/TestMe/Browser/429893#Top

2/2

8/11/2016

MyPastest

Back to Filters (/Secure/TestMe/Filter/429893/QA)

Question 149 of 207

A 22-year-old man is diagnosed with pulmonary tuberculosis and is started on treatment. A


month later, he presents with anorexia, malaise and fever. Laboratory investigations reveal: Hb
12.8 g/dl; WCC 12.0 109/l; urea 27 mmol/l; creatinine 480 mol/l. Urinalysis shows the
presence of numerous pus cells but no growth on culture.
What is the most likely cause for his symptoms and investigations?
A

Acute interstitial nephritis caused by rifampicin

Adverse effects of ethambutol

Isoniazid toxicity

Renal tuberculosis

Superimposed urinary tract infection

Explanation

The answer is Acute interstitial nephritis caused by rifampicin


Renal failure in this patient, who presents with anorexia, malaise, fever and pus cells on
urinalysis 1 month after starting treatment for primary tuberculosis, is likely to be due to
acute interstitial nephritis caused by rifampicin toxicity.

Adverse effects of ethambutol (Option B) is incorrect. Ethambutol is associated with


retrobulbar neuritis and arthralgia. Uncommonly it may be associated with hyperuricaemia,
and interstitial nephritis (less common than anticipated after rifampicin).
Isoniazid toxicity (Option C) is incorrect. Isoniazid therapy may give rise to peripheral
neuropathy, hepatitis and rash.
Renal tuberculosis (Option D) is incorrect. As the patient is already on treatment, spread of
tuberculosis to the kidneys is unlikely to develop.
Superimposed urinary tract infection (Option E) is incorrect. A superimposed urinary tract
infection, by itself, would not be expected to cause renal failure.
46747

https://mypastest.pastest.com/Secure/TestMe/Browser/429893#Top

1/2

8/11/2016

MyPastest

Next Question

Previous Question

Tag Question

Feedback

End Review

Difficulty: Average
Peer Responses

Session Progress
Responses Correct:

Responses Incorrect:

207

Responses Total:

207

Responses - % Correct:

0%

Blog (https://www.pastest.com/blog) About Pastest (https://www.pastest.com/about-us)


Contact Us (https://www.pastest.com/contact-us) Help (https://www.pastest.com/help)
Pastest 2016

https://mypastest.pastest.com/Secure/TestMe/Browser/429893#Top

2/2

8/11/2016

MyPastest

Back to Filters (/Secure/TestMe/Filter/429893/QA)

Question 150 of 207

You are considering the use of a new antihypertensive medication in an 81-year-old woman,
but have some concerns about age-related differences in metabolism in the elderly.
Which of the following factors is most likely to account for differences in drug metabolism
between the elderly and younger age groups?
A

Improved hepatic metabolism in the elderly

Increased cardiac output in the elderly

Increased GFR in the elderly

Less likelihood of concomitant medication in the elderly

Reduced cardiac output in the elderly

Explanation

The answer is Reduced cardiac output in the elderly


All of the following may account for differences in drug metabolism in the elderly:
Diminished renal function
Altered proportions of body fat and water
Reduced cardiac output
Some degree of altered hepatic metabolism
Disease
Concomitant medication use
For these reasons, clinical studies in drug development are rarely able to detect
problems that may occur with the use of new drugs in older adults. Many problems
associated with the use of new drugs in older adults may only be discovered through
adverse event reporting during the post-marketing period.

https://mypastest.pastest.com/Secure/TestMe/Browser/429893#Top

1/2

8/11/2016

MyPastest

Improved hepatic metabolism in the elderly (Option A) is incorrect. Hepatic metabolism is


often slightly lower in older adults, so that a dose reduction should be considered for drugs
extensively cleared by liver metabolism.
Increased cardiac output in the elderly (Option B) is incorrect. Cardiac output often declines
in adulthood, although the impact on drug handling is often minimal.
Increased GFR in the elderly (Option C) is incorrect. eGFR is often decreased in older
patients so that drugs cleared by renal elimination may accumulate.
Less likelihood of concomitant medication in the elderly (Option D) is incorrect. On the
contrary, older adults are susceptible to polypharmacy as a result of multiple long-term
health disorders accumulating.
46852

Next Question

Previous Question

Tag Question

Feedback

End Review

Difficulty: Average
Peer Responses

Session Progress
Responses Correct:

Responses Incorrect:

207

Responses Total:

207

Responses - % Correct:

0%

Blog (https://www.pastest.com/blog) About Pastest (https://www.pastest.com/about-us)


Contact Us (https://www.pastest.com/contact-us) Help (https://www.pastest.com/help)
Pastest 2016

https://mypastest.pastest.com/Secure/TestMe/Browser/429893#Top

2/2

8/11/2016

MyPastest

Back to Filters (/Secure/TestMe/Filter/429893/QA)

Question 151 of 207

A 63-year-old patient has a history of chronic renal failure and atrial fibrillation for which he
receives warfarin. He presented with an acutely painful right big toe. Investigation: uric acid
390 micromol/l, creatinine 200.
What is the most appropriate treatment?
A

Allopurinol

Colchicine

Ibuprofen

Indometacin

Paracetamol

Explanation

The answer is Colchicine


Colchicine may be given to patients with acute gout, even if there is renal failure. Most
common side-effects of colchicine include nausea, diarrhoea and abdominal pain. Up to
6 mg may be used in divided doses to gain control of symptoms after an acute attack,
and maintenance dose is around 1.5 mg given, once again, in divided doses.

Allopurinol (Option A) is incorrect. Allopurinol has a long half-life and may accumulate in
patients with chronic renal failure and paradoxically worsen gout symptoms when used in the
initial stages without the protection of non-steroidal anti-inflammatory drugs (NSAIDs) or
colchicine.
Ibuprofen (Option C) is incorrect. NSAIDs such as ibuprofen and indometacin are effective in
alleviating the pain and inflammation of acute gout, but they interact with warfarin therapy
to increase the risk of bleeding and may worsen renal impairment.
Indometacin (Option D) is incorrect. NSAIDs such as ibuprofen and indometacin are effective
in alleviating the pain and inflammation of acute gout, but they interact with warfarin therapy
to increase the risk of bleeding and may worsen renal impairment.
https://mypastest.pastest.com/Secure/TestMe/Browser/429893#Top

1/2

8/11/2016

MyPastest

Paracetamol (Option E) is incorrect. Paracetamol is effective for mild to moderate pain but
possesses little anti-inflammatory action, so it has limited therapeutic benefit in acute gout.
46932

Next Question

Previous Question

Tag Question

Feedback

End Review

Difficulty: Average
Peer Responses

Session Progress
Responses Correct:

Responses Incorrect:

207

Responses Total:

207

Responses - % Correct:

0%

Blog (https://www.pastest.com/blog) About Pastest (https://www.pastest.com/about-us)


Contact Us (https://www.pastest.com/contact-us) Help (https://www.pastest.com/help)
Pastest 2016

https://mypastest.pastest.com/Secure/TestMe/Browser/429893#Top

2/2

8/11/2016

MyPastest

Back to Filters (/Secure/TestMe/Filter/429893/QA)

Question 152 of 207

A 58-year-old man has a history of asthma, obesity, gastro-oesophageal reflux disease, low
back pain and ischaemic heart disease (IHD). He presents with large, itchy wheals over the
trunk and limbs and a sensation of tightness in his throat.
Which one of following drugs is likely to have triggered this skin eruption?
A

Aspirin

Glyceryl trinitrate spray

Omeprazole

Paracetamol

Simvastatin

Explanation

The answer is Aspirin


This is the typical picture of aspirin sensitivity with flushing, bronchospasm and urticaria
being common features. Other drugs that may cause this type of picture on first
exposure include intravenous morphine and N-acetylcysteine used to treat paracetamol
overdose. An alternative to aspirin use in this patient is clopidogrel, an alternative
platelet inhibitor. Aspirin desensitisation may be used to overcome this problem.

Glyceryl trinitrate (GTN) spray (Option B) is incorrect. Hypersensitivity or allergic reactions


to GTN are very uncommon. Common adverse effects include flushing, hypotension, and
headache.
Omeprazole (Option C) is incorrect. Omeprazole may cause acute interstitial nephritis.
Paracetamol (Option D) is incorrect. Paracetamol is not associated with hypersensitivity or
allergic reactions.
Simvastatin (Option E) is incorrect. Simvastatin may be associated with allergic reactions, but
much less commonly than aspirin, hence aspirin is the preferred answer. Other adverse
effects include hepatitis and myopathy.
46923

https://mypastest.pastest.com/Secure/TestMe/Browser/429893#Top

1/2

8/11/2016

MyPastest

46923

Next Question

Previous Question

Tag Question

Feedback

End Review

Difficulty: Average
Peer Responses

Session Progress
Responses Correct:

Responses Incorrect:

207

Responses Total:

207

Responses - % Correct:

0%

Blog (https://www.pastest.com/blog) About Pastest (https://www.pastest.com/about-us)


Contact Us (https://www.pastest.com/contact-us) Help (https://www.pastest.com/help)
Pastest 2016

https://mypastest.pastest.com/Secure/TestMe/Browser/429893#Top

2/2

8/11/2016

MyPastest

Back to Filters (/Secure/TestMe/Filter/429893/QA)

Question 153 of 207

You see a 57-year-old business man in the hypertension clinic for review. He has been recently
started on losartan by his GP and he is referred for further advice and investigation.
Which mechanism of action best accounts for the blood pressure lowering action of
losartan?
A

Angiotension-II receptor blockade

Calcium-channel blockade

Fall in plasma renin concentrations

Increased plasma aldosterone concentrations

Inhibition of angiotensin converting enzyme

Explanation

The answer is Angiotensin-II receptor blockade


Angiotensin II is a powerful vasoconstrictor. Angiotensin-II subtype-1 receptor
antagonists, also known as angiotensin receptor blockers (ARBs), include losartan,
candesartan and valsartan. The major actions of angiotensin receptor antagonists reflect
inhibition of the classic effects of the reninangiotensin system mediated by the AT1
receptor on the blood vessels, heart, adrenal gland, kidneys, brain and sympathetic
nervous system. Like ACE inhibitors, ARBs may cause hyperkalaemia and renal failure,
especially in sodium-depleted patients and in those with a critical reduction of renal
blood flow. They are less likely than ACE inhibitors to cause cough (1% versus 15%).

Calcium-channel blockade (Option B) is incorrect. Calcium-channel blockers may be


relatively selective for cardiac channels (diltiazem, verapamil) or peripheral vascular channels
(eg nifedipine, amlodipine). This relative selectivity is lost in overdose when effects are
exerted at both cardiac and peripheral channels.
Fall in plasma renin concentrations (Option C) is incorrect. Neither ARBs nor ACE inhibitors
have a significant effect on renin concentrations; both act at downstream parts of the renin
angiotensinaldosterone system.
https://mypastest.pastest.com/Secure/TestMe/Browser/429893#Top

1/2

8/11/2016

MyPastest

Increased plasma aldosterone concentrations (Option D) is incorrect. Both ARBs and ACE
inhibitors are capable of lowering aldosterone concentrations.
Inhibition of angiotensin converting enzyme (Option E) is incorrect. ACE inhibitors (eg
ramipril, lisinopril) block a different step in the reninangiotensin system, the enzyme that
converts angiotensin I to angiotensin II (the same enzyme breaks down bradykinin to kinins,
so that ACE inhibitors cause accumulation of bradykinin in the lungs and cough).
46788

Next Question

Previous Question

Tag Question

Feedback

End Review

Difficulty: Average
Peer Responses

Session Progress
Responses Correct:

Responses Incorrect:

207

Responses Total:

207

Responses - % Correct:

0%

Blog (https://www.pastest.com/blog) About Pastest (https://www.pastest.com/about-us)


Contact Us (https://www.pastest.com/contact-us) Help (https://www.pastest.com/help)
Pastest 2016

https://mypastest.pastest.com/Secure/TestMe/Browser/429893#Top

2/2

8/11/2016

MyPastest

Back to Filters (/Secure/TestMe/Filter/429893/QA)

Question 154 of 207

A 34-year-old man is brought to the Emergency Department by the police having been found
in the street in an agitated state. He was apparently shouting and aggressively begging
outside a local burger bar when he was noted to collapse, suffering a short (1 minute) tonic
clonic seizure. Previous attendances at the Emergency Department with opiate,
benzodiazepine and alcohol abuse are noted. Most recently he was enrolled in an addiction
treatment program and had been prescribed an SSRI. He tells you he can feel insects crawling
all over his skin and hears voices telling him that he is worthless. On examination his BP is
145/85 mmHg; pulse is 92/min and regular. He is covered in sweat.
Which of the following is he most likely to be withdrawing from?
A

Alcohol

Diazepam

Diamorphine

GHB

Sertraline

Explanation
The answer is Alcohol The withdrawal seizure, signs of autonomic hyperactivity and tactile hallucinations fit well
with alcohol withdrawal. Sedative withdrawal is associated with hyperactivity and potentially
seizures, although tactile hallucinations are usually not a feature. Generally opiate withdrawal
is not associated with seizures, but is associated with a flu-like illness and features such as
rhinorrhoea, sneezing, yawning, lacrimation, abdominal cramping, leg cramping, piloerection,
nausea and vomiting. GHB withdrawal symptoms are similar to those of benzodiazepine
withdrawal. Sertraline withdrawal is associated with anxiety.
40147

Next Question

Previous Question

Tag Question

https://mypastest.pastest.com/Secure/TestMe/Browser/429893#Top

Feedback

End Review
1/2

8/11/2016

MyPastest

Difficulty: Average
Peer Responses

Session Progress
Responses Correct:

Responses Incorrect:

207

Responses Total:

207

Responses - % Correct:

0%

Blog (https://www.pastest.com/blog) About Pastest (https://www.pastest.com/about-us)


Contact Us (https://www.pastest.com/contact-us) Help (https://www.pastest.com/help)
Pastest 2016

https://mypastest.pastest.com/Secure/TestMe/Browser/429893#Top

2/2

8/11/2016

MyPastest

Back to Filters (/Secure/TestMe/Filter/429893/QA)

Question 155 of 207

The use of performance-enhancing anabolic steroids by professional athletes is generally


decreasing owing to random drug testing. However, the use among members of the public
attending gyms may be increasing. A 19-year-old man asks you some questions about the
possible effects.
Which of the following statements best describes the possible cardiovascular and metabolic
effects of anabolic steroid use?
A

Blood concentrations of HDL-cholesterol are increased

Blood concentrations of LDL-cholesterol are decreased

Blood pressure decreases during prolonged use

Erythrocyte sedimentation rate increases

Haematocrit increases during prolonged use

Explanation

The answer is Haematocrit increases during prolonged use


Anabolic steroids can be taken orally (eg stanozolol) but are often injected owing to
extensive first-pass metabolism (eg testosterone enantate). Among their many unwanted
effects, they increase the risk of cardiovascular disease, particularly with prolonged use.
These effects include blood pressure elevation, increased LDL-cholesterol and decreased
HDL-cholesterol. With prolonged use haematocrit increases, which may cause a
prothrombotic tendency; plasma fibrinogen concentrations decrease with prolonged use.

Blood concentrations of HDL-cholesterol are increased (Option A) is incorrect. HDLcholesterol concentrations tend to decrease.
Blood concentrations of LDL-cholesterol are decreased (Option B) is incorrect. LDLcholesterol concentrations may increase.
Blood pressure decreases during prolonged use (Option C) is incorrect. Blood pressure tends
to increase, increasing stroke risk.
https://mypastest.pastest.com/Secure/TestMe/Browser/429893#Top

1/2

8/11/2016

MyPastest

Erythrocyte sedimentation rate (ESR) increases (Option D) is incorrect. There is no


significant effect on ESR.
46379

Next Question

Previous Question

Tag Question

Feedback

End Review

Difficulty: Average
Peer Responses

Session Progress
Responses Correct:

Responses Incorrect:

207

Responses Total:

207

Responses - % Correct:

0%

Blog (https://www.pastest.com/blog) About Pastest (https://www.pastest.com/about-us)


Contact Us (https://www.pastest.com/contact-us) Help (https://www.pastest.com/help)
Pastest 2016

https://mypastest.pastest.com/Secure/TestMe/Browser/429893#Top

2/2

8/11/2016

MyPastest

Back to Filters (/Secure/TestMe/Filter/429893/QA)

Question 156 of 207

A 45-year-old woman has been diagnosed with polycystic ovarian syndrome. She has been
receiving anticonvulsant treatment for a number of years.
Which one of the following anticonvulsants is most likely to be associated with the
development of polycystic ovarian syndrome (PCOS)?
A

Phenobarbitone

Phenytoin

Sodium valproate

Topiramate

Vigabatrin

Explanation

The answer is Sodium valproate


Sodium valproate has been associated with the development of polycystic ovarian
syndrome (PCOS), a condition associated with central obesity, hirsutism, irregular
periods or secondary amenorrhoea, infertility and insulin resistance. The underlying
pathogenesis of PCOS is not yet established, but there appears to be defective ovarian
production of oestradiol, with overproduction of precursor molecules, which are
converted into androgens in extra-glandular tissue. Characteristic features of PCOS
include:
A low follicle-stimulating hormone level
High luteinising hormone level
High oestradiol level
High androgen levels
Occasionally virilism may occur

https://mypastest.pastest.com/Secure/TestMe/Browser/429893#Top

1/2

8/11/2016

MyPastest

Phenobarbitone (Option A) is incorrect. PCOS is not a recognised adverse effect of


phenobarbitone.
Phenytoin (Option B) is incorrect. PCOS is not a recognised adverse effect of phenytoin.
Topiramate (Option D) is incorrect. PCOS is not a recognised adverse effect of Topiramate.
Vigabatrin (Option E) is incorrect. PCOS is not a recognised adverse effect of vigabatrin.
46729

Next Question

Previous Question

Tag Question

Feedback

End Review

Difficulty: Average
Peer Responses

Session Progress
Responses Correct:

Responses Incorrect:

207

Responses Total:

207

Responses - % Correct:

0%

Blog (https://www.pastest.com/blog) About Pastest (https://www.pastest.com/about-us)


Contact Us (https://www.pastest.com/contact-us) Help (https://www.pastest.com/help)
Pastest 2016

https://mypastest.pastest.com/Secure/TestMe/Browser/429893#Top

2/2

8/11/2016

MyPastest

Back to Filters (/Secure/TestMe/Filter/429893/QA)

Question 157 of 207

You are discussing with an obstetric FY2 some of the potential risks of prescribing to a
patient in early pregnancy.
Which of the drugs listed below would be considered safe in the first trimester of pregnancy?
A

Acetylcysteine

Carbamazepine

Lithium

Sodium valproate

Warfarin

Explanation

The answer is Acetylcysteine


A number of drugs are known to have teratogenic effects, and should normally be
avoided. These are listed in British National Formulary, including androgens causing
cardiac deformities, ethanol causing foetal alcohol syndrome, diethylstilbestrol causing
vaginal carcinoma, phenobarbital causing cleft palate, and thalidomide causing
phocomelia. For many drugs, insufficient information exists to inform outcomes after use
in pregnancy. Data collected by the UK Teratology Information Service indicate that
acetylcysteine, as used in the setting of paracetamol overdose, does not appear to
increase risks.

Carbamazepine (Option B) is incorrect. This is a recognised cause of microcephaly.


Lithium (Option C) is incorrect. This is a recognised cause of neonatal goitre and
hypothyroidism.
Sodium valproate (Option D) is incorrect. This is a recognised cause of neural tube defects.
Warfarin (Option E) is incorrect. This is a recognised cause of chondrodysplasia punctate.
45812

https://mypastest.pastest.com/Secure/TestMe/Browser/429893#Top

1/2

8/11/2016

MyPastest

Next Question

Previous Question

Tag Question

Feedback

End Review

Difficulty: Average
Peer Responses

Session Progress
Responses Correct:

Responses Incorrect:

207

Responses Total:

207

Responses - % Correct:

0%

Blog (https://www.pastest.com/blog) About Pastest (https://www.pastest.com/about-us)


Contact Us (https://www.pastest.com/contact-us) Help (https://www.pastest.com/help)
Pastest 2016

https://mypastest.pastest.com/Secure/TestMe/Browser/429893#Top

2/2

8/11/2016

MyPastest

Back to Filters (/Secure/TestMe/Filter/429893/QA)

Question 158 of 207

A 59-year-old man is admitted with unstable angina. He has a history of type-2 diabetes and
a previous inferior myocardial infarction. His ECG shows anterior ST depression and he has
ongoing chest pain despite nitrates. He goes for angioplasty and is treated with abciximab.
Which one of the following correctly describes the mode of action of abciximab?
A

Cyclooxygenase inhibitor

Prostaglandin E inhibitor

Glycoprotein 2b 3a inhibitor

Phosphodiesterase inhibitor

Thromboxane A2 inhibitor

Explanation
Abciximab
Abciximab is an inhibitor of the glycoprotein 2b 3a receptor on the platelet membrane
The receptor mediates platelet aggregation
Inhibition by abciximab, a chimeric/human monoclonal antibody, leads to decreased
thrombus formation
Abciximab is indicated in both the management of unstable angina and the prevention
of ischaemic complications in patients who have undergone percutaneous coronary
intervention
Other notes

Aspirin inhibits cyclooxygenase


Clopidogrel inhibits the platelet ADP receptor
21360

Next Question
https://mypastest.pastest.com/Secure/TestMe/Browser/429893#Top

1/2

8/11/2016

MyPastest

Previous Question

Tag Question

Feedback

End Review

Difficulty: Easy
Peer Responses

Session Progress
Responses Correct:

Responses Incorrect:

207

Responses Total:

207

Responses - % Correct:

0%

Blog (https://www.pastest.com/blog) About Pastest (https://www.pastest.com/about-us)


Contact Us (https://www.pastest.com/contact-us) Help (https://www.pastest.com/help)
Pastest 2016

https://mypastest.pastest.com/Secure/TestMe/Browser/429893#Top

2/2

8/11/2016

MyPastest

Back to Filters (/Secure/TestMe/Filter/429893/QA)

Question 159 of 207

A 22-year-old woman is brought to the Emergency Department having ingested at least 20


tablets of paracetamol 8 h earlier in front of her boyfriend. She had initially refused to let him
call an ambulance but then relented. She weighs 61 kg.
What is the most appropriate immediate management?
A

Inform the local liver unit for management of acute liver failure

Intravenous N-acetylcysteine

Liver function tests, prothrombin time and INR estimations then decide treatment

Oral activated charcoal

Plasma paracetamol concentration estimation then decide treatment

Explanation

The answer is Intravenous N-acetylcysteine


Activated charcoal is useful if given within 1 h of the paracetamol overdose, and may
sufficiently reduce paracetamol absorption so that acetylcysteine antidote is not
required. At 8 h post-ingestion, charcoal would be ineffective. It would be helpful to
establish baseline liver function tests, electrolytes, prothrombin time and paracetamol
concentrations, but acetylcysteine treatment should not be delayed while these results
are awaited. Acetylcysteine is normally advised in cases where the paracetamol dose
exceeds 75 mg/kg body weight, and where paracetamol concentrations are above the
nomogram if patients have taken an overdose at a single timepoint. If patients present
sufficiently early after overdose, then it may be reasonable to await paracetamol
concentrations before treating. However, treatment should not be delayed for more than
8 h after ingestion; therefore acetylcysteine should be commenced while the
paracetamol concentrations are awaited.

Inform the local liver unit for management of acute liver failure (Option A) is incorrect. It
would be too early for fulminant liver failure to have occurred.

https://mypastest.pastest.com/Secure/TestMe/Browser/429893#Top

1/2

8/11/2016

MyPastest

Liver function tests, prothrombin time and INR estimations then decide treatment (Option C)
is incorrect. Although it will be important to check these blood tests, treatment should not
be delayed while the results are awaited.
Oral activated charcoal (Option D) is incorrect. At 8 h after ingestion, oral activated charcoal
will be ineffective in reducing paracetamol exposure.
Plasma paracetamol concentration estimation then decide treatment (Option E) is incorrect.
A paracetamol concentration should be sent, but antidote treatment should not be delayed
while the results are awaited.
46602

Next Question

Previous Question

Tag Question

Feedback

End Review

Difficulty: Average
Peer Responses

Session Progress
Responses Correct:

Responses Incorrect:

207

Responses Total:

207

Responses - % Correct:

0%

Blog (https://www.pastest.com/blog) About Pastest (https://www.pastest.com/about-us)


Contact Us (https://www.pastest.com/contact-us) Help (https://www.pastest.com/help)
Pastest 2016

https://mypastest.pastest.com/Secure/TestMe/Browser/429893#Top

2/2

8/11/2016

MyPastest

Back to Filters (/Secure/TestMe/Filter/429893/QA)

Question 160 of 207

A patient assessed in the Dementia Clinic is found to meet the diagnostic criteria for
moderately severe Alzheimers disease with a MMSE of 12/30.
Which of the following drugs is most likely to slow progression of the disease?
A

Atropine

Diphenhydramine

Donepezil

Levodopa

Promethazine

Explanation

The answer is Donepezil


Pharmacological treatments can be divided into symptom-oriented treatments
(depression, agitation and psychotic symptoms) and disease-orientated treatments that
include cholinesterase inhibitors (eg donepezil, rivastigmine) and memantine. Therapy
can achieve modest improvements in cognition in 2550% of patients.

Atropine (Option A) is incorrect. Atropine confers anticholinergic effects but has no value in
treatment of dementia.
Diphenhydramine (Option B) is incorrect. Diphenhydramine possesses anticholinergic and
antihistamine effects but would have no effect on the progression of dementia.
Levodopa (Option D) is incorrect. Levodopa is effective in Parkinsons disease and
degenerative disorders with Parkinsonian features.
Promethazine (Option E) is incorrect. Promethazine may be effective in reducing nausea but
has no impact on the progression of dementia.
46820

Next Question
https://mypastest.pastest.com/Secure/TestMe/Browser/429893#Top

1/2

8/11/2016

MyPastest

Previous Question

Tag Question

Feedback

End Review

Difficulty: Average
Peer Responses

Session Progress
Responses Correct:

Responses Incorrect:

207

Responses Total:

207

Responses - % Correct:

0%

Blog (https://www.pastest.com/blog) About Pastest (https://www.pastest.com/about-us)


Contact Us (https://www.pastest.com/contact-us) Help (https://www.pastest.com/help)
Pastest 2016

https://mypastest.pastest.com/Secure/TestMe/Browser/429893#Top

2/2

8/11/2016

MyPastest

Back to Filters (/Secure/TestMe/Filter/429893/QA)

Question 161 of 207

A 90-year-old man with chronic leukaemia presents with gout, which his general practitioner
treats with allopurinol.
What is the primary pharmacological action of allopurinol?
A

By competing for the uric acid transporter in the kidney

By enhancing its solubility

By inhibiting a step in urate breakdown

By inhibiting purine synthesis and urate production

By inhibiting the inflammatory response associated with leukaemia

Explanation

The answer is By inhibiting purine synthesis and urate production


Leukaemia is associated with gout because of the increased turnover of blood cells,
which leads to increased uric acid production. Other conditions associated with gout
include: myeloproliferative disorders, such as polycythaemia vera, glucose-6-phospate
dehydrogenase (G6PD) deficiency, chronic renal disease, drug therapy (thiazide
diuretics), lead toxicity, primary hyperparathyroidism, hypothyroidism, increased
production of lactic acid from alcohol, exercise, or starvation. Allopurinol blocks the
enzyme xanthine oxidase, which is responsible for the conversion of xanthine into urate,
which is the final step in the pathway of purine breakdown.

By competing for the uric acid transporter in the kidney (Option A) is incorrect. Probenecid
enhances excretion of uric acid.
By enhancing its solubility (Option B) is incorrect. Sodium bicarbonate creates an alkaline
environment and solubilises urate so that its renal clearance is enhanced.
By inhibiting a step in urate breakdown (Option C) is incorrect. There is limited urate
breakdown to allantoin by oxidative metabolism; other mammals that express urate oxidase
extensively metabolise urate to allantoin.
https://mypastest.pastest.com/Secure/TestMe/Browser/429893#Top

1/2

8/11/2016

MyPastest

By inhibiting the inflammatory response associated with leukaemia (Option E) is incorrect.


Treatment of classic acute gout involves the use of non-steroidal anti-inflammatory drugs
(NSAIDs) during the initial period.
46925

Next Question

Previous Question

Tag Question

Feedback

End Review

Difficulty: Average
Peer Responses

Session Progress
Responses Correct:

Responses Incorrect:

207

Responses Total:

207

Responses - % Correct:

0%

Blog (https://www.pastest.com/blog) About Pastest (https://www.pastest.com/about-us)


Contact Us (https://www.pastest.com/contact-us) Help (https://www.pastest.com/help)
Pastest 2016

https://mypastest.pastest.com/Secure/TestMe/Browser/429893#Top

2/2

8/11/2016

MyPastest

Back to Filters (/Secure/TestMe/Filter/429893/QA)

Question 162 of 207

A 16-year-old boy has been receiving antiepileptic medications for several years. He attends
the outpatient clinic for review and reports suffering from nausea and diplopia. On
examination, there are marked visual-field defects noted affecting central and peripheral
visual fields.
Which of the following drugs is most likely to be responsible?
A

Ethosuximide

Primidone

Sodium valproate

Tiagabine

Vigabatrin

Explanation

The answer is Vigabatrin


Vigabatrin is the drug of choice for infantile spasms in childhood. It is, however,
associated with a number of other adverse effects and as such is not widely used apart
from this indication. Adverse effects include aggression, alopecia, retinal atrophy and
reduced peripheral vision. Treatment should be stopped immediately.

Ethosuximide (Option A) is incorrect. Ethosuximide only rarely causes visual disturbance.


Primidone (Option B) is incorrect. Primidone is a prodrug of phenobarbital, which is capable
of causing diplopia, ataxia and slurred speech. It is not associated with visual-field defects.
Sodium valproate (Option C) is incorrect. Sodium valproate may cause liver toxicity,
hyperammonaemia, pancreatitis and weight gain.
Tiagabine (Option D) is incorrect. Tiagabine may be useful in partial seizures or secondary
generalised tonicclonic seizures, but is not prescribed for infantile spasms; it may also
impair peripheral vision (visual disturbance is less well characterised than for vigabatrin).
46758

https://mypastest.pastest.com/Secure/TestMe/Browser/429893#Top

1/2

8/11/2016

MyPastest

Next Question

Previous Question

Tag Question

Feedback

End Review

Difficulty: Average
Peer Responses

Session Progress
Responses Correct:

Responses Incorrect:

207

Responses Total:

207

Responses - % Correct:

0%

Blog (https://www.pastest.com/blog) About Pastest (https://www.pastest.com/about-us)


Contact Us (https://www.pastest.com/contact-us) Help (https://www.pastest.com/help)
Pastest 2016

https://mypastest.pastest.com/Secure/TestMe/Browser/429893#Top

2/2

8/11/2016

MyPastest

Back to Filters (/Secure/TestMe/Filter/429893/QA)

Question 163 of 207

You are trialling a new drug for type-2 diabetes for a major drug company. One of your jobs is
to determine whether it can be given as a once-per-day agent. To do this you need to have an
idea of the half-life and plasma concentration at various time points. The peak initial plasma
concentration is 2 mg/dl, achieved 20 min after oral administration. The half-life is measured
at 4 h.
What is the plasma concentration likely to be 12 h after peak concentration is reached?
A

0.1 mg/dl

1.8 mg/dl

0.25 mg/dl

0.66 mg/dl

0.5 mg/dl

Explanation
Pharmacokinetics
The half-life is the time taken for the concentration of a drug to reduce by 50%
We are given the half-life here, which is 4 h, and are required to calculate the plasma
concentration 12 h after administration of a quantity of drug that gives a peak initial
plasma concentration of 2 mg/dl
12 h is equal to three half-lives
Therefore, the plasma concentration at 12 h will be 2/(2 2 2) = 0.25
The tmax (time taken to reach peak concentration) has nothing at all to do with the
calculation
22423

Next Question

https://mypastest.pastest.com/Secure/TestMe/Browser/429893#Top

1/2

8/11/2016

MyPastest

Previous Question

Tag Question

Feedback

End Review

Difficulty: Easy
Peer Responses

Session Progress
Responses Correct:

Responses Incorrect:

207

Responses Total:

207

Responses - % Correct:

0%

Blog (https://www.pastest.com/blog) About Pastest (https://www.pastest.com/about-us)


Contact Us (https://www.pastest.com/contact-us) Help (https://www.pastest.com/help)
Pastest 2016

https://mypastest.pastest.com/Secure/TestMe/Browser/429893#Top

2/2

8/11/2016

MyPastest

Back to Filters (/Secure/TestMe/Filter/429893/QA)

Question 164 of 207

An 18-year-old woman attends the Emergency Department with clinical features that make
you suspicious of recreational drug use. A bedside urine test gives a positive reading for
amphetamine.
Which of the following agents would be most strongly implicated?
A

Cannabis

Cocaine

Cyanide

Ecstasy

Morphine

Explanation

The answer is Ecstasy


Ecstasy (3, 4-methylenedioxy methamphetamine) is a semi-synthetic amphetamine
derivative that is used as an illicit recreational substance and may cross-react with
amphetamine screening tests. The main adverse effects are nausea, muscle pains,
confusion, ataxia, hyperkalaemia, metabolic acidosis, hyperthermia, cardiovascular
collapse and acute respiratory distress. There is no specific antidote, and treatment
involves supportive care.

Cannabis (Option A) is incorrect. Cannabis and its metabolites may be detectable in urine for
several days to weeks after drug exposure.
Cocaine (Option B) is incorrect. Cocaine metabolites may be detectable in urine within
several hours after exposure.
Cyanide (Option C) is incorrect. Cyanide is converted to thiocyanates, which may be
detectable in blood or urine.
Morphine (Option E) is incorrect. Morphine and its metabolites may be readily detectable in
urine within several hours up to around 1 day after exposure.
46962

https://mypastest.pastest.com/Secure/TestMe/Browser/429893#Top

1/2

8/11/2016

MyPastest

46962

Next Question

Previous Question

Tag Question

Feedback

End Review

Difficulty: Average
Peer Responses

Session Progress
Responses Correct:

Responses Incorrect:

207

Responses Total:

207

Responses - % Correct:

0%

Blog (https://www.pastest.com/blog) About Pastest (https://www.pastest.com/about-us)


Contact Us (https://www.pastest.com/contact-us) Help (https://www.pastest.com/help)
Pastest 2016

https://mypastest.pastest.com/Secure/TestMe/Browser/429893#Top

2/2

8/11/2016

MyPastest

Back to Filters (/Secure/TestMe/Filter/429893/QA)

Question 165 of 207

An obstetric FY2 stops you in the corridor to ask for some advice about a 44-year-old
woman that presented at 24-weeks gestation. An obstetric ultrasound has identified
significant intrauterine growth retardation. The patient had inadvertently been exposed to a
number of different drugs during early pregnancy.
Which of the following drugs would be most likely to cause intrauterine growth retardation?
A

Aspirin

Bendroflumethiazide

Bisoprolol

Ramipril

Simvastatin

Explanation

The answer is Bisoprolol


Many drugs are capable of causing teratogenic risks and complications during
pregnancy. Beta-blockers may cause hypoglycaemia, intrauterine growth retardation and
fetal bradycardia.

Aspirin (Option A) is incorrect. Aspirin is a recognised cause of kernicterus.


Bendroflumethiazide (Option B) is incorrect. Thiazide diuretics may cause neonatal
thrombocytopaenia.
Ramipril (Option D) is incorrect. Angiotensin-converting enzyme (ACE) inhibitors may cause
oligohydramnios and impaired renal function.
Simvastatin (Option E) is incorrect. Statins are not typically associated with intrauterine
growth retardation.
46485

Next Question
https://mypastest.pastest.com/Secure/TestMe/Browser/429893#Top

1/2

8/11/2016

MyPastest

Previous Question

Tag Question

Feedback

End Review

Difficulty: Average
Peer Responses

Session Progress
Responses Correct:

Responses Incorrect:

207

Responses Total:

207

Responses - % Correct:

0%

Blog (https://www.pastest.com/blog) About Pastest (https://www.pastest.com/about-us)


Contact Us (https://www.pastest.com/contact-us) Help (https://www.pastest.com/help)
Pastest 2016

https://mypastest.pastest.com/Secure/TestMe/Browser/429893#Top

2/2

8/11/2016

MyPastest

Back to Filters (/Secure/TestMe/Filter/429893/QA)

Question 166 of 207

You are asked to review a 46-year-old lady who has been referred by her GP with a creatinine
of 342 micromol/l. Renal function was normal before commencing chemotherapy recently.
Which one of the following chemotherapeutic agents is most likely to result in
nephrotoxicity?
A

Ciclosporin

Cyclophosphamide

Methotrexate

Sulfasalazine

Vincristine

Explanation

The answer is Ciclosporin


Ciclosporin is used as an immunosuppressive agent after renal transplantation. It has a
narrow therapeutic window and can cause direct nephrotoxicity; as renal function
worsens plasma concentrations increase and renal function progressively deteriorates.

Cyclophosphamide (Option B) is incorrect. Cyclophosphamide is an alkylating agent that


typically causes bone marrow suppression.
Methotrexate (Option C) is incorrect. Methotrexate is a dihydrofolate reductase inhibitor that
is used to treat acute lymphoblastic leukaemia, non-Hodgkins lymphoma, some solid
tumours and psoriasis. Folinic acid is concomitantly administered to reduce the risk of
mucositis or myelosuppression.
Sulfasalazine (Option D) is incorrect. Sulfasalazine is an amino salicylate that is used in the
treatment of ulcerative colitis, Crohns disease and rheumatoid arthritis. Adverse effects
include diarrhea, nausea, rash, StevensJohnson syndrome and hepatitis. Interstitial nephritis
may occur, but this is rare and so ciclosporin is the preferred answer.

https://mypastest.pastest.com/Secure/TestMe/Browser/429893#Top

1/2

8/11/2016

MyPastest

Vincristine (Option E) is incorrect. Vincristine is a vinca alkaloid that is used to treat acute
leukaemia and some solid tumours. Its characteristic adverse effect profile is neurotoxicity; it
does not cause prominent bone marrow suppression or renal impairment.
46961

Next Question

Previous Question

Tag Question

Feedback

End Review

Difficulty: Average
Peer Responses

Session Progress
Responses Correct:

Responses Incorrect:

207

Responses Total:

207

Responses - % Correct:

0%

Blog (https://www.pastest.com/blog) About Pastest (https://www.pastest.com/about-us)


Contact Us (https://www.pastest.com/contact-us) Help (https://www.pastest.com/help)
Pastest 2016

https://mypastest.pastest.com/Secure/TestMe/Browser/429893#Top

2/2

8/11/2016

MyPastest

Back to Filters (/Secure/TestMe/Filter/429893/QA)

Question 167 of 207

A 70-year-old man is found by his home help lying on the floor of his lounge. An empty
bottle which contained 5 mg diazepam tablets and half a bottle of whisky are on the table.
He has a past medical history of severe rheumatoid arthritis and you understand his pain and
disability had worsened significantly over the past few months. On examination in the
Emergency Room his temperature is 34C. His blood pressure is 105/65 mmHg, with a pulse
of 60 beats per minute.
Investigations:
Hb

12.1 g/dl

White cell count 12.1 109 /l


Platelets

230 109 /l

Na+

141 mmol/l

K+

6.0 mmol/l

Creatinine

240 mol/l

ALT

550 U/l

Urine

blood ++

Which one of the following is the most likely diagnosis?


A

Additional paracetamol overdose

Rhabdomyolysis

Urinary sepsis

Hepatitis

Myocardial infarction

Explanation
https://mypastest.pastest.com/Secure/TestMe/Browser/429893#Top

1/2

8/11/2016

MyPastest

Multidrug overdose
This man has taken an overdose of both diazepam and whisky and it is likely he
remained on the floor for a significant period after losing consciousness
This period on the floor has led to significant muscle necrosis, resulting in
myoglobinuria (urine dipstick positive to blood), acute renal impairment and raised
alanine aminotransferase
Whilst we are not given the result for creatine kinase, it will be raised in this case
Key in the early management of this patient is treatment of his hyperkalaemia, with
insulin and dextrose infusion the first-line option along with gentle rewarming
20937

Next Question

Previous Question

Tag Question

Feedback

End Review

Difficulty: Easy
Peer Responses

Session Progress
Responses Correct:

Responses Incorrect:

207

Responses Total:

207

Responses - % Correct:

0%

Blog (https://www.pastest.com/blog) About Pastest (https://www.pastest.com/about-us)


Contact Us (https://www.pastest.com/contact-us) Help (https://www.pastest.com/help)
Pastest 2016

https://mypastest.pastest.com/Secure/TestMe/Browser/429893#Top

2/2

8/11/2016

MyPastest

Back to Filters (/Secure/TestMe/Filter/429893/QA)

Question 168 of 207

You are asked by one of the obstetrics staff to see a 24-year-old woman who is 13 weeks
pregnant and has been found to have had high blood pressure recordings during the past
three visits. Today her blood pressure is 170/110 mmHg. Her antenatal records indicate healthy
blood pressure before pregnancy.
What would be the most appropriate antihypertensive drug to use in this patient?
A

Doxazosin

Enalapril

Hydrochlorothiazide

Losartan

Methyldopa

Explanation

The answer is Methyldopa


One of the earliest clinical trials on the treatment of hypertension during pregnancy
involved a comparison of methyldopa with no treatment, which found no adverse effects
on pregnancy outcomes or subsequent development in childhood.

Doxazosin (Option A) is incorrect. Doxazosin may be safe, there is no evidence of


teratogenicity, but has less robust outcome data when compared to methyldopa.
Enalapril (Option B) is incorrect. ACE inhibitors may cause oligohydramnios.
Hydrochlorothiazide (Option C) is incorrect. Thiazides may cause neonatal
thrombocytopaenia, although this is a comparatively rare outcome. Insufficient data are
available to consider thiazides safe for use.
Losartan (Option D) is incorrect. Angiotensin receptor blockers may cause oligohydramnios
and neonatal hyperkalaemia.
46802

https://mypastest.pastest.com/Secure/TestMe/Browser/429893#Top

1/2

8/11/2016

MyPastest

Next Question

Previous Question

Tag Question

Feedback

End Review

Difficulty: Average
Peer Responses

Session Progress
Responses Correct:

Responses Incorrect:

207

Responses Total:

207

Responses - % Correct:

0%

Blog (https://www.pastest.com/blog) About Pastest (https://www.pastest.com/about-us)


Contact Us (https://www.pastest.com/contact-us) Help (https://www.pastest.com/help)
Pastest 2016

https://mypastest.pastest.com/Secure/TestMe/Browser/429893#Top

2/2

8/11/2016

MyPastest

Back to Filters (/Secure/TestMe/Filter/429893/QA)

Question 169 of 207

A 67-year-old man has recently started taking dothiepin for the management of depression.
His GP has contacted you for advice regarding the optimal dose, and for advice on how to
monitor for development of anticholinergice adverse effects.
Which of the following clinical features would be most strongly indicative of anticholinergic
drug effects?
A

Bradycardia

Constricted pupils

Hot facial sensation

Hypotension

Urinary incontinence

Explanation

The answer is Hot facial sensation


Common features include hot, dry skin, hypertension, tachycardia, dilated pupils, urinary
retention, constipation and delirium. Drugs with anticholinergic properties include
tricyclic antidepressants, antipsychotics and antihistamines. In most cases, withdrawal of
the offending drug is all that is required, along with supportive care.

Bradycardia (Option A) is incorrect. Tachycardia is a common feature.


Constricted pupils (Option B) is incorrect. Mydriasis is a recognised feature.
Hypotension (Option D) is incorrect. Slight blood pressure elevation may occur.
Urinary incontinence (Option E) is incorrect. Acute urinary retention is common.
46489

Next Question

https://mypastest.pastest.com/Secure/TestMe/Browser/429893#Top

1/2

8/11/2016

MyPastest

Previous Question

Tag Question

Feedback

End Review

Difficulty: Average
Peer Responses

Session Progress
Responses Correct:

Responses Incorrect:

207

Responses Total:

207

Responses - % Correct:

0%

Blog (https://www.pastest.com/blog) About Pastest (https://www.pastest.com/about-us)


Contact Us (https://www.pastest.com/contact-us) Help (https://www.pastest.com/help)
Pastest 2016

https://mypastest.pastest.com/Secure/TestMe/Browser/429893#Top

2/2

8/11/2016

MyPastest

Back to Filters (/Secure/TestMe/Filter/429893/QA)

Question 170 of 207

A 29-year-old man is found collapsed and unresponsive in the entrance to the hospital with
an empty bottle of medicine. His partner thinks he was supplied with a new medication when
he attended the psychiatry outpatient clinic on the day before. On examination he is drowsy,
heart rate is 140 bpm, and an ECG shows wide complexes.
Which one of the following is the best initial treatment?
A

Adenosine

Digoxin

Lidocaine

Propranolol

Sodium bicarbonate

Explanation

The answer is Sodium bicarbonate


The history is suggestive of an overdose of tricyclic antidepressants leading to
ventricular tachycardia. Symptoms and signs of tricyclic overdose include dry mouth,
hypotension, hypothermia, urinary retention, hyperreflexia, up-going planters, coma,
seizures and cardiac arrhythmias. Metabolic acidosis is a common feature, which
enhances cardiotoxicity. Sodium bicarbonate diminishes the sodium-channel-blocking
effect of tricyclics, thereby lessening the risk of seizures and arrhythmia.

Adenosine (Option A) is incorrect. Adenosine would not be expected to have any significant
effect.
Digoxin (Option B) is incorrect. Digoxin may increase the risk of cardiotoxicity in this
situation, and may worsen hyperkalaemia.
Lidocaine (Option C) is incorrect. Lidocaine is a class I antiarrhythmic agent that may worsen
the tricyclic-induced sodium channel blockade.

https://mypastest.pastest.com/Secure/TestMe/Browser/429893#Top

1/2

8/11/2016

MyPastest

Propranolol (Option D) is incorrect. Propranolol and other -blockers may be helpful in


controlling heart rate but are less effective in preventing arrhythmia and seizure, hence
sodium bicarbonate is the preferred answer.
46963

Next Question

Previous Question

Tag Question

Feedback

End Review

Difficulty: Average
Peer Responses

Session Progress
Responses Correct:

Responses Incorrect:

207

Responses Total:

207

Responses - % Correct:

0%

Blog (https://www.pastest.com/blog) About Pastest (https://www.pastest.com/about-us)


Contact Us (https://www.pastest.com/contact-us) Help (https://www.pastest.com/help)
Pastest 2016

https://mypastest.pastest.com/Secure/TestMe/Browser/429893#Top

2/2

8/11/2016

MyPastest

Back to Filters (/Secure/TestMe/Filter/429893/QA)

Question 171 of 207

A 24-year-old woman who is 16 weeks pregnant presents to her GP complaining of frequency


and dysuria. She has been relatively well in her pregnancy so far and has no significant past
medical history. On examination she is pyrexial 37.6C and has some suprapubic tenderness.
Investigations:
Hb

12.1 g/dl

White cell count 10.4 109/l


Platelets

201 109/l

Na+

140 mmol/l

K+

4.0 mmol/l

Creatinine

90 mol/l

Urine dipstick

blood +, protein ++, leukocytes ++

Which one of the following antibiotics should be avoided if at all possible in this patient?
A

Amoxicillin

Co-amoxiclav

Ciprofloxacin

Trimethoprim

Cephalexin

Explanation
Antibiotic prescribing in pregnancy
Data from ciprofloxacin exposure to pregnant women do not indicate any specific risk
of malformations
https://mypastest.pastest.com/Secure/TestMe/Browser/429893#Top

1/2

8/11/2016

MyPastest

That being said, in animal studies on juvenile and prenatal animals, the potential for
damage to articular cartilage was indicated
For this reason, as a precaution, use of ciprofloxacin in pregnancy is not recommended
Similarly, because of secretion in breast milk, ciprofloxacin should not be used by
breast-feeding women
Most data exist for amoxicillin, trimethoprim and cephalexin use in this population;
therefore, according to sensitivities, one of these three agents would be the most
appropriate choice
Trimethoprim is a folate antagonist and is therefore avoided if possible in early
pregnancy, but at 16 weeks it is a reasonable option for therapy
20936

Next Question

Previous Question

Tag Question

Feedback

End Review

Difficulty: Average
Peer Responses

Session Progress
Responses Correct:

Responses Incorrect:

207

Responses Total:

207

Responses - % Correct:

0%

Blog (https://www.pastest.com/blog) About Pastest (https://www.pastest.com/about-us)


Contact Us (https://www.pastest.com/contact-us) Help (https://www.pastest.com/help)
Pastest 2016

https://mypastest.pastest.com/Secure/TestMe/Browser/429893#Top

2/2

8/11/2016

MyPastest

Back to Filters (/Secure/TestMe/Filter/429893/QA)

Question 172 of 207

A 65-year-old chronic alcoholic is receiving warfarin therapy for left ventricular aneurysm
following a previous acute myocardial infarction 6 months ago. He now presents with
deranged liver function tests and ultrasound scan appearances are suggestive of cirrhosis.
What is the most appropriate change that should be made to his anticoagulation?
A

Decrease or stop the warfarin according to INR

Increase the dose of warfarin

Stop all anticoagulant treatment

Stop warfarin and start aspirin

Stop warfarin and start subcutaneous heparin injections

Explanation

The answer is Decrease or stop the warfarin according to INR


There are increased risks of bleeding associated with cirrhosis, and the potential for INR
to increase owing to impaired vitamin K metabolism and clotting factor synthesis. Ideally,
warfarin therapy should be continued to afford protection against thromboembolism
related to the left ventricular aneurysm. However, close INR monitoring is required to
minimise bleeding risk. He may be anticoagulated by liver failure to the extent that he no
longer needs warfarin therapy.

Increase the dose of warfarin (Option B) is incorrect. This would be hazardous and unhelpful.
Stop all anticoagulant treatment (Option C) is incorrect. Although this would reduce the risks
of bleeding complications, it would leave the patient exposed to the risks of
thromboembolism associated with the left ventricular aneurysm.
Stop warfarin and start aspirin (Option D) is incorrect. Switching to an antiplatelet might
reduce risks of bleeding, but would not offer effective protection against thromboembolism.
Stop warfarin and start subcutaneous heparin injections (Option E) is incorrect. Subcutanous
heparin might be more risky because it would be more difficult to measure the extent of
https://mypastest.pastest.com/Secure/TestMe/Browser/429893#Top

1/2

8/11/2016

MyPastest

anticoagulation, whereas warfarin may be monitored using INR.


46774

Next Question

Previous Question

Tag Question

Feedback

End Review

Difficulty: Average
Peer Responses

Session Progress
Responses Correct:

Responses Incorrect:

207

Responses Total:

207

Responses - % Correct:

0%

Blog (https://www.pastest.com/blog) About Pastest (https://www.pastest.com/about-us)


Contact Us (https://www.pastest.com/contact-us) Help (https://www.pastest.com/help)
Pastest 2016

https://mypastest.pastest.com/Secure/TestMe/Browser/429893#Top

2/2

8/11/2016

MyPastest

Back to Filters (/Secure/TestMe/Filter/429893/QA)

Question 173 of 207

A 34-year-old patient who is 28 weeks pregnant with her first baby is referred to you as an
emergency with newly diagnosed symptomatic hyperthyroidism.
Which of the following statements best describes the management of maternal
thyrotoxicosis?
A

Block and replace combination carbimazole and thyroxine is preferred in pregnancy

Carbimazole causes neonatal goitre in 75% of babies of treated mothers

Carbimazole does not cross the placenta

Neonatal goitre is a recognised complication of propylthiouracil

Propylthiouracil does not cross the placenta

Explanation

The answer is Neonatal goitre is a recognised complication of propylthiouracil


Management of thyrotoxicosis in pregnancy is similar to that for other patients, although
surgery should normally be avoided if possible, and radioactive iodine is contraindicated
in view of the radiation hazard to the fetus. Carbimazole and propylthiouracil are capable
of crossing the placenta and may cause neonatal hypothyroidism and goitre. Goitre may
interfere with childbirth and normally resolves after delivery. Both carbimazole and
propylthiouracil are excreted in maternal breast milk, theoretically leading to neonatal
hypothyroidism.

Block and replace combination carbimazole and thyroxine is preferred in pregnancy (Option
A) is incorrect. Both carbimazole and propylthiouracil cross the placenta, although thyroxine
does not. Therefore, the block and replace regimen is not appropriate as this creates a
greater risk of neonatal hypothyroidism than using carbimazole alone.
Carbimazole causes neonatal goitre in 75% of babies of treated mothers (Option B) is
incorrect. Neonatal hypothyroidism occurs in approximately 10% of babies.
Carbimazole does not cross the placenta (Option C) is incorrect. Both carbimazole and
propylthiouracil may cross the placenta and cause neonatal hypothyroidism and goitre.
https://mypastest.pastest.com/Secure/TestMe/Browser/429893#Top

1/2

8/11/2016

MyPastest

Propylthiouracil does not cross the placenta (Option E) is incorrect. Both carbimazole and
propylthiouracil may cross the placenta and cause neonatal hypothyroidism and goitre.
46486

Next Question

Previous Question

Tag Question

Feedback

End Review

Difficulty: Average
Peer Responses

Session Progress
Responses Correct:

Responses Incorrect:

207

Responses Total:

207

Responses - % Correct:

0%

Blog (https://www.pastest.com/blog) About Pastest (https://www.pastest.com/about-us)


Contact Us (https://www.pastest.com/contact-us) Help (https://www.pastest.com/help)
Pastest 2016

https://mypastest.pastest.com/Secure/TestMe/Browser/429893#Top

2/2

8/11/2016

MyPastest

Back to Filters (/Secure/TestMe/Filter/429893/QA)

Question 174 of 207

A 71-year-old man is admitted with a left-sided hemiplegia. He has a history of type-2


diabetes, which is managed with metformin 500 mg bd. He is poorly compliant with other
medications and smokes 20 cigarettes per day. On examination he has a blood pressure of
167/87 mmHg some 3 h after admission. His left-sided hemiplegia has undergone slight
improvement. A CT scan is reported as showing normal appearances.
What is the optimal timing of antihypertensive therapy?
A

Immediate treatment

More than 1 month

Should be avoided due to risk of hypotension

Within 1 month

Within 2 weeks

Explanation

The answer is Within 2 weeks


There is significant controversy concerning the benefits of early antihypertensive
therapy. Treatment should normally be started prior to discharge from hospital or 2
weeks after the stroke (which ever is the soonest). Treatment should only be introduced
urgently if there is accelerated hypertension or hypertensive emergency. Long-term
target blood pressure is 140/90 mmHg or less, and preferably to 130/80 mmHg.

Immediate treatment (Option A) is incorrect. The potential hazard of immediate therapy is


worsened cerebral ischaemia, and should only be contemplated with accelerated
hypertension or very high systolic blood pressure values.
More than 1 month (Option B) is incorrect. Treatment should be started within 2 weeks or at
discharge from hospital, whichever occurs soonest.
Should be avoided due to risk of hypotension (Option C) is incorrect. Hypertension is the
single largest risk factor for stroke, and optimal blood pressure control will reduce recurrent
events.
https://mypastest.pastest.com/Secure/TestMe/Browser/429893#Top

1/2

8/11/2016

MyPastest

Within 1 month (Option D) is incorrect. Treatment should be started within 2 weeks or at


discharge from hospital, whichever occurs soonest.
46985

Next Question

Previous Question

Tag Question

Feedback

End Review

Difficulty: Average
Peer Responses

Session Progress
Responses Correct:

Responses Incorrect:

207

Responses Total:

207

Responses - % Correct:

0%

Blog (https://www.pastest.com/blog) About Pastest (https://www.pastest.com/about-us)


Contact Us (https://www.pastest.com/contact-us) Help (https://www.pastest.com/help)
Pastest 2016

https://mypastest.pastest.com/Secure/TestMe/Browser/429893#Top

2/2

8/11/2016

MyPastest

Back to Filters (/Secure/TestMe/Filter/429893/QA)

Question 175 of 207

You see a 39-year-old man in the endocrinology clinic. He has a history of hyperthyroidism
and is currently receiving treatment with carbimazole.
What mechanism best explains the pharmacological actions of carbimazole?
A

Conversion to methimazole, which blocks the TSH receptor

Destruction of functioning thyroid cells

Inhibition of 5'-deiodinase

Inhibition of the iodination of tyrosine

Inhibition of thyroglobulin proteolysis

Explanation

The answer is Inhibition of the iodination of tyrosine


Carbimazole is converted to its active metabolite, methimazole, which inhibits iodination
of tyrosine, thereby preventing coupling of thyroxine precursors. It also has an
immunosuppressive action, leading to a reduction in serum thyroid-stimulating hormone
(TSH) receptor antibody (TRAb) concentrations.

Conversion to methimazole, which blocks the TSH receptor (Option A) is incorrect. It is


converted to methimazole, but its actions are unrelated to the TSH receptor.
Destruction of functioning thyroid cells (Option B) is incorrect. Carbimazole does not destroy
functioning thyroid cells, but disrupts the intracellular iodination of tyrosine and coupling,
therefore preventing formation of thyroxine and tri-iodothyronine.
Inhibition of 5'-deiodinase (Option C) is incorrect. Propylthiouracil not carbimazole inhibits
5'-deiodinase and prevents the release of free tri-iodothyronine and thyroxine from the
thyroid cell.
Inhibition of thyroglobulin proteolysis (Option E) is incorrect. Iodine but not carbimazole
inhibits thyroglobulin proteolysis.
46622

https://mypastest.pastest.com/Secure/TestMe/Browser/429893#Top

1/2

8/11/2016

MyPastest

Next Question

Previous Question

Tag Question

Feedback

End Review

Difficulty: Average
Peer Responses

Session Progress
Responses Correct:

Responses Incorrect:

207

Responses Total:

207

Responses - % Correct:

0%

Blog (https://www.pastest.com/blog) About Pastest (https://www.pastest.com/about-us)


Contact Us (https://www.pastest.com/contact-us) Help (https://www.pastest.com/help)
Pastest 2016

https://mypastest.pastest.com/Secure/TestMe/Browser/429893#Top

2/2

8/11/2016

MyPastest

Back to Filters (/Secure/TestMe/Filter/429893/QA)

Question 176 of 207

In a diabetes specialist clinic you are about to review a 59-year-old patient with type-2
diabetes mellitus who is not responding to dietary advice and weight reduction. You have
decided to prescribe an oral hypoglycaemic agent.
Which of the following statements most accurately expresses the mechanism of action of
hypoglycaemic agents used in the management of diabetes mellitus?
A

Acarbose inhibits -glucosidase enzymes within the gastrointestinal tract

Metformin may cause hypoglycaemia

Metformin stimulates pancreatic insulin secretion

Pioglitazone causes hepatotoxicity in 1520% of patients

Sulfonylureas reduce peripheral insulin sensitivity

Explanation

The answer is Acarbose inhibits -glucosidase enzymes within the gastrointestinal tract
Acarbose inhibits intestinal -glucosidase, which therefore delays the digestion and
absorption of starch and sucrose, and minimises the peak glucose concentrations arising
after a meal. Its main unwanted effects are flatus and gastrointestinal disturbance.
Biguanides (e.g. metformin) primarily decrease hepatic glucose secretion and do not
cause hypoglycaemia, even in overdose; the main unwanted effect is lactic acidosis, and
they are contraindicated in renal failure. Sulfonylureas (e.g. gliclazide, glimepiride)
increase insulin secretion in response to a given glucose load, and may cause weight gain
and hypoglycaemia. Pioglitazone reduces peripheral insulin resistance (increases insulin
sensitivity), and adverse effects include weight gain, hypoglycaemia, fluid retention and
congestive heart failure.

Metformin may cause hypoglycaemia (Option B) is incorrect. Metformin and biguanides do


not cause hypoglycaemia. Other medications may cause hypoglycaemia, particularly
sulfonylureas.

https://mypastest.pastest.com/Secure/TestMe/Browser/429893#Top

1/2

8/11/2016

MyPastest

Metformin stimulates pancreatic insulin secretion (Option C) is incorrect. Metformin does not
increase insulin secretion; biguanides decrease hepatic glucose secretion.
Pioglitazone causes hepatotoxicity in 1520% of patients (Option D) is incorrect. Pioglitazone
and rosiglitazone may cause fluid retention and congestive heart failure. Idiosyncratic liver
failure was caused by troglitazone and it was withdrawn.
Sulfonylureas reduce peripheral insulin sensitivity (Option E) is incorrect. The main
mechanism of sulfonylureas is enhanced pancreatic insulin secretion.
46382

Next Question

Previous Question

Tag Question

Feedback

End Review

Difficulty: Average
Peer Responses

Session Progress
Responses Correct:

Responses Incorrect:

207

Responses Total:

207

Responses - % Correct:

0%

Blog (https://www.pastest.com/blog) About Pastest (https://www.pastest.com/about-us)


Contact Us (https://www.pastest.com/contact-us) Help (https://www.pastest.com/help)
Pastest 2016

https://mypastest.pastest.com/Secure/TestMe/Browser/429893#Top

2/2

8/11/2016

MyPastest

Back to Filters (/Secure/TestMe/Filter/429893/QA)

Question 177 of 207

A 70-year-old man comes to the clinic complaining of blue vision. He has chronic atrial
fibrillation and hypertension but has been passed fit to take sildenafil by his doctor. On
examination he looks well, his pulse is 74 beats per minute, he has atrial fibrillation and his
blood pressure is 142/78 mmHg.
Investigations:
Hb

13.1 g/dl

White cell count 4.9 109/l


Platelets

182 109/l

Na+

142 mmol/l

K+

4.5 mmol/l

Creatinine

105 mol/l

Which one of the following is the most likely cause of his blue vision amongst the
medications he has been taking?
A

Temazepam

Sildenafil

Digoxin

Bisoprolol

Amlodipine

Explanation
Drug side-effects
Digoxin is associated with yellow vision in overdose
Bisoprolol, like all -blockers, is associated with increased dreams/possible night terrors
Sildenafil
https://mypastest.pastest.com/Secure/TestMe/Browser/429893#Top

1/2

8/11/2016

MyPastest

is a phosphodiesterase type-5 (PDE-5) inhibitor


also has some activity against PDE-6, which is involved in the functioning of
retinal photoreceptors
At high sildenafil doses, the PDE-6 inhibitory effect becomes clinically significant and
patients complain of blue vision
In total over half of men taking 200 mg or more of sildenafil experience some kind of
visual side-effects
21325

Next Question

Previous Question

Tag Question

Feedback

End Review

Difficulty: Average
Peer Responses

Session Progress
Responses Correct:

Responses Incorrect:

207

Responses Total:

207

Responses - % Correct:

0%

Blog (https://www.pastest.com/blog) About Pastest (https://www.pastest.com/about-us)


Contact Us (https://www.pastest.com/contact-us) Help (https://www.pastest.com/help)
Pastest 2016

https://mypastest.pastest.com/Secure/TestMe/Browser/429893#Top

2/2

8/11/2016

MyPastest

Back to Filters (/Secure/TestMe/Filter/429893/QA)

Question 178 of 207

A 22-year-old woman who is 5+ weeks pregnant presents with dysuria to the Emergency
Department. She has felt feverish for the past 24 h and has been to pass urine two to three
times per hour. On examination she is pyrexial at 37.8C and has suprapubic tenderness. She
has suffered a rash on penicillin in the past.
Investigations:
Hb

11.9 g/dl

White cell count 11.2 109/l


Platelets

191 109/l

Na+

139 mmol/l

K+

4.9 mmol/l

Creatinine

95 mol/l

Which one of the following is the most appropriate antibiotic choice for her?
A

Clarithromycin

Ciprofloxacin

Doxycycline

Cephalexin

Trimethoprim

Explanation
Managing urinary tract infection in pregnancy
Appropriate options for managing urinary tract infection in pregnancy include both
cephalexin and trimethoprim
https://mypastest.pastest.com/Secure/TestMe/Browser/429893#Top

1/2

8/11/2016

MyPastest

However, given her stage in pregnancy, the theoretical action of trimethoprim in


interfering with folate metabolism drives cephalexin as the most appropriate option
Both tetracyclines and quinolones are expressly contraindicated for use in pregnancy
Clarithromycin is unlikely to be effective for UTI
There is up to 20% crossover with sensitivity to cephalosporins in patients with
penicillin allergy, so she should be observed for signs of allergy
22464

Next Question

Previous Question

Tag Question

Feedback

End Review

Difficulty: Average
Peer Responses

Session Progress
Responses Correct:

Responses Incorrect:

207

Responses Total:

207

Responses - % Correct:

0%

Blog (https://www.pastest.com/blog) About Pastest (https://www.pastest.com/about-us)


Contact Us (https://www.pastest.com/contact-us) Help (https://www.pastest.com/help)
Pastest 2016

https://mypastest.pastest.com/Secure/TestMe/Browser/429893#Top

2/2

8/11/2016

MyPastest

Back to Filters (/Secure/TestMe/Filter/429893/QA)

Question 179 of 207

A 37-year-old woman is referred to the Acute Medical Unit for investigation of breathlessness.
She is 3 weeks post-partum and is solely breast feeding her infant.
Which of the following drugs, in normal therapeutic doses, should be avoided from the point
of view of breast-feeding?
A

Aspirin

Erythromycin

Heparin

Thyroxine

Warfarin

Explanation

The answer is Aspirin


Aspirin, when ingested by the mother, can cause neonatal bleeding problems, and there
is a risk of Reyes syndrome in the infant even after exposure to low quantities.

Erythromycin (Option B) is incorrect. Erythromycin, like many antibiotics, is water soluble,


and only small amounts cross into breast milk; it is unlikely to cause adverse effects in the
baby.
Heparin (Option C) is incorrect. Heparin crosses into breast milk in amounts too small to
produce significant anticoagulation in the neonate.
Thyroxine (Option D) is incorrect. Thyroxine crosses into breast milk in amounts too small to
appreciably affect thyroid function or to affect tests for neonatal hypothyroidism.
Warfarin (Option E) is incorrect. Warfarin crosses into breast milk in amounts too small to
produce significant anticoagulation in the neonate.
46910

Next Question
https://mypastest.pastest.com/Secure/TestMe/Browser/429893#Top

1/2

8/11/2016

MyPastest

Previous Question

Tag Question

Feedback

End Review

Difficulty: Average
Peer Responses

Session Progress
Responses Correct:

Responses Incorrect:

207

Responses Total:

207

Responses - % Correct:

0%

Blog (https://www.pastest.com/blog) About Pastest (https://www.pastest.com/about-us)


Contact Us (https://www.pastest.com/contact-us) Help (https://www.pastest.com/help)
Pastest 2016

https://mypastest.pastest.com/Secure/TestMe/Browser/429893#Top

2/2

8/11/2016

MyPastest

Back to Filters (/Secure/TestMe/Filter/429893/QA)

Question 180 of 207

A 54-year-old man has been diagnosed with type-2 diabetes and started on metformin in
addition to appropriate dietary and lifestyle advice. He asks you about possible adverse
effects of metformin treatment.
Which of the following adverse effects is most likely to be attributable to metformin
treatment?
A

Anaphylaxis

Cardiotoxicity

Lactic acidosis

Pulmonary fibrosis

Rhabdomyolysis

Explanation

The answer is Lactic acidosis


Metformin is a biguanide that is known to cause lactic acidosis, which may be a severe or
even fatal complication, particularly in the setting of intentional metformin overdose.
Metformin acts mainly by inhibiting hepatic gluconeogenesis, and inhibits lactate
dehydrogenase in the gut and liver. Haemodialysis may clear lactate and allow correction
of acidosis, although mortality remains high (over 30%), partly because of the organ
failures that commonly coexist.

Anaphylaxis (Option A) is incorrect. This is not a recognised feature of metformin therapy.


Cardiotoxicity (Option B) is incorrect. Fluid retention and cardiac failure may be seen in
patients receiving glitazone therapy.
Pulmonary fibrosis (Option D) is incorrect. Pulmonary fibrosis is not a recognised feature of
metformin therapy.
Rhabdomyolysis (Option E) is incorrect. Rhabdomyolysis is more likely to occur in
conjunction with statin or fibrate therapy, or especially with the combination of the two.
46803

https://mypastest.pastest.com/Secure/TestMe/Browser/429893#Top

1/2

8/11/2016

MyPastest

46803

Next Question

Previous Question

Tag Question

Feedback

End Review

Difficulty: Average
Peer Responses

Session Progress
Responses Correct:

Responses Incorrect:

207

Responses Total:

207

Responses - % Correct:

0%

Blog (https://www.pastest.com/blog) About Pastest (https://www.pastest.com/about-us)


Contact Us (https://www.pastest.com/contact-us) Help (https://www.pastest.com/help)
Pastest 2016

https://mypastest.pastest.com/Secure/TestMe/Browser/429893#Top

2/2

8/11/2016

MyPastest

Back to Filters (/Secure/TestMe/Filter/429893/QA)

Question 181 of 207

A 72-year-old woman with multiple medical problems is admitted to the Emergency


Department with progressive confusion over the past 6 weeks. She has a history of
depression, diabetes and hypertension. On examination her BP is 135/72 mmHg, pulse is
70/min and regular. Her BMI is 25. Venous blood gas sampling reveals a sodium of 118 mmol/l.
Which of the following agents is most likely to be the cause of her low sodium?
A

Atorvastatin

Bendroflumethiazide

Fluoxetine

Glicalzide

Ramipril

Explanation
The answer is Bendroflumethiazide
The drugs most commonly implicated in SIADH are:

Thiazide diuretics
Vincristine
Vinblastine
Cyclophosphamide

Others include:

Chlorpromamide
Carbamazepine
Phenothiazines
Tricyclic antidepressants
Clofibrate
Oxytocin
Vasopressin
Morphine
Barbiturates
Nicotine
36464

https://mypastest.pastest.com/Secure/TestMe/Browser/429893#Top

1/2

8/11/2016

MyPastest

36464

Next Question

Previous Question

Tag Question

Feedback

End Review

Difficulty: Average
Peer Responses

Session Progress
Responses Correct:

Responses Incorrect:

207

Responses Total:

207

Responses - % Correct:

0%

Blog (https://www.pastest.com/blog) About Pastest (https://www.pastest.com/about-us)


Contact Us (https://www.pastest.com/contact-us) Help (https://www.pastest.com/help)
Pastest 2016

https://mypastest.pastest.com/Secure/TestMe/Browser/429893#Top

2/2

8/11/2016

MyPastest

Back to Filters (/Secure/TestMe/Filter/429893/QA)

Question 182 of 207

A 52-year-old alcoholic was diagnosed with respiratory tuberculosis, and commenced on


antituberculous treatment. After receiving treatment for 3 months, he has complained of
tingling and numbness affecting both feet.
Which of the following drugs is most likely to be responsible for his symptoms?
A

Ethambutol

Isoniazid

Pyrazinamide

Rifampicin

Streptomycin

Explanation

The answer is Isoniazid


Peripheral neuropathy is rare with the standard 300 mg adult dose of isoniazid; however,
it is more likely to occur with (1) higher doses, (2) slow acetylator status, or (3) the
presence of predisposing conditions including malnutrition, alcoholism, diabetes, AIDS
and uraemia.

Ethambutol (Option A) is incorrect. The most common serious adverse event with
ethambutol is retrobulbar neuritis, causing loss of visual acuity and redgreen colour
blindness; regular ophthalmology review is required.
Pyrazinamide (Option C) is incorrect. Pyrazinamide causes hepatotoxicity (15% of cases),
drug fever and hyperuricaemia.
Rifampicin (Option D) is incorrect. Rifampicin is associated with thrombocytopaenia, orange
red staining of body fluids and hepatitis.
Streptomycin (Option E) is incorrect. Streptomycin has been used to treat tuberculosis and is
usually administered by intramuscular injection.
46776

https://mypastest.pastest.com/Secure/TestMe/Browser/429893#Top

1/2

8/11/2016

MyPastest

Next Question

Previous Question

Tag Question

Feedback

End Review

Difficulty: Average
Peer Responses

Session Progress
Responses Correct:

Responses Incorrect:

207

Responses Total:

207

Responses - % Correct:

0%

Blog (https://www.pastest.com/blog) About Pastest (https://www.pastest.com/about-us)


Contact Us (https://www.pastest.com/contact-us) Help (https://www.pastest.com/help)
Pastest 2016

https://mypastest.pastest.com/Secure/TestMe/Browser/429893#Top

2/2

8/11/2016

MyPastest

Back to Filters (/Secure/TestMe/Filter/429893/QA)

Question 183 of 207

A 65-year-old man has been receiving bendroflumethiazide for more than 15 years for
treatment of hypertension.
Which of the following mechanisms best explains the renal effect of thiazide diuretics?
A

Ascending limb of Henles loop

Collecting duct

Descending limb of Henles loop

Distal segment of the distal convoluted tubule

Proximal segment of the distal convoluted tubule

Explanation

The answer is Proximal segment of the distal convoluted tubule


Most diuretics act by inhibiting sodium and water reabsorption in the renal tubules.
Thiazide diuretics inhibit Na+/Cl cotransport in the proximal segment of the distal
convoluted tubule.

Ascending limb of Henles loop (Option A) is incorrect. The loop diuretics furosemide and
bumetanide inhibit Na+/K+/Cl co-transport in the ascending limb of Henles loop.
Collecting duct (Option B) is incorrect. Aldosterone causes sodium absorption and
potassium secretion from the collecting ducts; aldosterone antagonists including eplerenone
and spironolactone inhibit salt and water absorption within the collecting ducts.
Descending limb of Henles loop (Option C) is incorrect. This is not a recognised target for
diuretics.
Distal segment of the distal convoluted tubule (Option D) is incorrect. The potassium-sparing
diuretic amiloride inhibits sodium channels in the distal segment of the distal convoluted
tubule.
46940

https://mypastest.pastest.com/Secure/TestMe/Browser/429893#Top

1/2

8/11/2016

MyPastest

Next Question

Previous Question

Tag Question

Feedback

End Review

Difficulty: Average
Peer Responses

Session Progress
Responses Correct:

Responses Incorrect:

207

Responses Total:

207

Responses - % Correct:

0%

Blog (https://www.pastest.com/blog) About Pastest (https://www.pastest.com/about-us)


Contact Us (https://www.pastest.com/contact-us) Help (https://www.pastest.com/help)
Pastest 2016

https://mypastest.pastest.com/Secure/TestMe/Browser/429893#Top

2/2

8/11/2016

MyPastest

Back to Filters (/Secure/TestMe/Filter/429893/QA)

Question 184 of 207

A 28-year-old woman is taking a course of ciprofloxacin for pyelonephritis.


Which of the following best describes the mode of action of ciprofloxacin?
A

Inhibition of bacterial cell wall synthesis

Inhibition of bacterial ribosomal protein synthesis

Inhibition of DNA gyrase

Inhibition of folate synthesis

Production of -lactamase

Explanation

The answer is Inhibition of DNA gyrase


Quinolones such as ciprofloxacin primarily act by inhibiting bacterial DNA gyrase.

Inhibition of bacterial cell wall synthesis (Option A) is incorrect. Bacterial cell wall synthesis is
inhibited by penicillins, cephalosporins and vancomycin.
Inhibition of bacterial ribosomal protein synthesis (Option B) is incorrect. Aminoglycosides,
erythromycin and tetracyclines act via inhibition of bacterial ribosomal synthesis.
Inhibition of folate synthesis (Option D) is incorrect. Sulfonamides and trimethoprim are
inhibitors of folate synthesis.
Production of -lactamase (Option E) is incorrect. -lactamase production is one common
mechanism of antibiotic resistance, which is often conferred via transfer of plasmids between
bacteria.
46846

Next Question

https://mypastest.pastest.com/Secure/TestMe/Browser/429893#Top

1/2

8/11/2016

MyPastest

Previous Question

Tag Question

Feedback

End Review

Difficulty: Average
Peer Responses

Session Progress
Responses Correct:

Responses Incorrect:

207

Responses Total:

207

Responses - % Correct:

0%

Blog (https://www.pastest.com/blog) About Pastest (https://www.pastest.com/about-us)


Contact Us (https://www.pastest.com/contact-us) Help (https://www.pastest.com/help)
Pastest 2016

https://mypastest.pastest.com/Secure/TestMe/Browser/429893#Top

2/2

8/11/2016

MyPastest

Back to Filters (/Secure/TestMe/Filter/429893/QA)

Question 185 of 207

Four patients present to the Emergency Department after exposure to an unusual vapour at a
local supermarket. Reports from the ambulance crew and first responders indicate they are
dealing with a suspected chemical release, and you have been alerted that the patients might
have been exposed to sarin.
Which of the following clinical features would most strongly support a diagnosis of nerve
agent exposure?
A

Hypersalivation

Hypotension

Intense cough

Mydriasis

Tachycardia

Explanation

The answer is Hypersalivation


Organophosphates and nerve agents are rapidly absorbed through the gastrointestinal
and respiratory tract and skin. The principal action is inhibition of acetylcholinesterase
such that there is accumulation of acetylcholine at muscarinic and nicotinic receptors.
Muscarinic effects include miosis, hypersalivation, sweating, diarrhoea, excessive
bronchial secretions and bradycardia. Nicotinic effects include muscle fasciculation and
tremor. Central nervous system effects include anxiety, memory impairment, headache
and coma. Neuropathy may develop over several weeks, initially a flaccid paralysis, and
later hypertonia, hyperreflexia and a spastic paralysis. Treatments include supplemental
oxygen, atropine and pralidoxime, an activator of cholinesterase.

Hypotension (Option B) is incorrect. This is not a significant feature.


Intense cough (Option C) is incorrect. Cough is not a typical feature, unlike exposure to
respiratory irritants.
Mydriasis (Option D) is incorrect. Miosis is a characteristic clinical finding.
https://mypastest.pastest.com/Secure/TestMe/Browser/429893#Top

1/2

8/11/2016

MyPastest

Tachycardia (Option E) is incorrect. Bradycardia may occur due to muscarinic cardiac action.
45797

Next Question

Previous Question

Tag Question

Feedback

End Review

Difficulty: Average
Peer Responses

Session Progress
Responses Correct:

Responses Incorrect:

207

Responses Total:

207

Responses - % Correct:

0%

Blog (https://www.pastest.com/blog) About Pastest (https://www.pastest.com/about-us)


Contact Us (https://www.pastest.com/contact-us) Help (https://www.pastest.com/help)
Pastest 2016

https://mypastest.pastest.com/Secure/TestMe/Browser/429893#Top

2/2

8/11/2016

MyPastest

Back to Filters (/Secure/TestMe/Filter/429893/QA)

Question 186 of 207

A 38-year-old female patient has a history of brittle asthma and has received a number of
courses of corticosteroid treatments over the years for flare ups of her asthma. She has now
been diagnosed with vertebral fractures secondary to osteoporosis.
Which of the following treatments would be the most appropriate treatment for minimising
the risk of further fractures?
A

Alendronate

Calcitonin

Cyproterone acetate

Oestradiol and norethisterone acetate

Parathyroid hormone

Explanation

The answer is Alendronate


Glucocorticoid-induced osteoporosis (GIO) leads to fractures in 3050% of patients.
Bisphosphonates are currently the recommended first-line therapy for the prevention
and treatment of GIO. They have been shown to increase bone mineral density at the
spine and hip and to decrease the incidence of vertebral fractures (especially in
postmenopausal women). A broad range of therapies have been tested with less certain
benefits in terms of fracture risk, including selective oestrogen receptor modulators, oral
contraceptive pill, strontium, calcium, vitamin D, vitamin D analogues, sex hormones and
anabolic agents.

Calcitonin (Option B) is incorrect. Calcitonin increases lumbar spine bone mineral density but
has no proven fracture efficacy.
Cyproterone acetate (Option C) is incorrect. Antiandrogen therapy is unlikely to be of
benefit. Testosterone appears to increase bone mineral density, although the ability to
prevent fractures is uncertain.

https://mypastest.pastest.com/Secure/TestMe/Browser/429893#Top

1/2

8/11/2016

MyPastest

Oestradiol and norethisterone acetate (Option D) is incorrect. Female hormone replacement


therapy (HRT) has been found to increase lumbar spine bone mineral density in hypogonadal
patients on glucocorticoid therapy, but effects on the hip are inconsistent, and there are no
data concerning fracture prevention in GIO.
Parathyroid hormone (Option E) is incorrect. Parathyroid hormone treatment has recently
been shown to increase bone mineral density at the spine and hip, decreases the incidence of
new vertebral fractures, and may be an alternative to bisphosphonates where these are not
tolerated.
46786

Next Question

Previous Question

Tag Question

Feedback

End Review

Difficulty: Average
Peer Responses

Session Progress
Responses Correct:

Responses Incorrect:

207

Responses Total:

207

Responses - % Correct:

0%

Blog (https://www.pastest.com/blog) About Pastest (https://www.pastest.com/about-us)


Contact Us (https://www.pastest.com/contact-us) Help (https://www.pastest.com/help)
Pastest 2016

https://mypastest.pastest.com/Secure/TestMe/Browser/429893#Top

2/2

8/11/2016

MyPastest

Back to Filters (/Secure/TestMe/Filter/429893/QA)

Question 187 of 207

A 32-year-old woman with long-term epilepsy was concerned about weight gain due to
taking valproate. The only other medication she is taking is the oral contraceptive pill.
Which one of the following is the best alternative to valproate?
A

Carbamazepine

Lamotrigine

Phenobarbital

Phenytoin

Topiramate

Explanation

The answer is Lamotrigine


Lamotrigine is less likely to interfere with the oral contraceptive pill than other
antiepileptic agents. The overall risk of birth defects in women who take one
anticonvulsant is around 7%, compared with a background rate of 3% in the population
as a whole.

Carbamazepine (Option A) is incorrect. Carbamazepine is a powerful hepatic enzyme inducer


and is associated with decreased effectiveness of the oral contraceptive pill owing to
acceleration of the metabolism of oestrogens and progestogens.
Phenobarbital (Option C) is incorrect. Phenobarbital is a powerful hepatic enzyme inducer
and is associated with decreased effectiveness of the oral contraceptive pill owing to
acceleration of the metabolism of oestrogens and progestogens.
Phenytoin (Option D) is incorrect. Phenytoin is a powerful hepatic enzyme inducer and is
associated with decreased effectiveness of the oral contraceptive pill owing to acceleration
of the metabolism of oestrogens and progestogens.
Topiramate (Option E) is incorrect. Topiramate is a hepatic enzyme inducer and is associated
with decreased effectiveness of the oral contraceptive pill owing to acceleration of the
https://mypastest.pastest.com/Secure/TestMe/Browser/429893#Top

1/2

8/11/2016

MyPastest

metabolism of oestrogens and progestogens.


46930

Next Question

Previous Question

Tag Question

Feedback

End Review

Difficulty: Average
Peer Responses

Session Progress
Responses Correct:

Responses Incorrect:

207

Responses Total:

207

Responses - % Correct:

0%

Blog (https://www.pastest.com/blog) About Pastest (https://www.pastest.com/about-us)


Contact Us (https://www.pastest.com/contact-us) Help (https://www.pastest.com/help)
Pastest 2016

https://mypastest.pastest.com/Secure/TestMe/Browser/429893#Top

2/2

8/11/2016

MyPastest

Back to Filters (/Secure/TestMe/Filter/429893/QA)

Question 188 of 207

What is the most common cause of paranoid psychosis with visual hallucination?
A

Alcohol withdrawal

Amphetamine withdrawal

Benzodiazepine withdrawal

Opiate withdrawal

Selective serotonin re-uptake inhibitor withdrawal

Explanation

The answer is Alcohol withdrawal


Delirium tremens occurs in less than 5% of individuals withdrawing from alcohol and
happens around 24 days after cessation of alcohol intake. Untreated it carries a high
mortality of up to 15%. Features include coarse tremor, agitation, confusion, delusion and
visual hallucinations. Fever, sweating and tachycardia may also occur; rarely there is
associated ketoacidosis. Co-existent hypoglycaemia and WernickeKorsakoff psychosis
are considerations. Sedation with chlordiazepoxide is often necessary to prevent
seizures. Vitamin B supplements given intravenously (Pabrinex) should be considered.

Amphetamine withdrawal (Option B) is incorrect. Amphetamines are comparatively short


lived so that withdrawal features are less common.
Benzodiazepine withdrawal (Option C) is incorrect. Benzodiazepine withdrawal may provoke
agitation and seizures, particularly after abrupt cessation in patients that have been treated
with prolonged therapy at high doses.
Opiate withdrawal (Option D) is incorrect. Opioid withdrawal features may appear similar to
alcohol withdrawal, although physical symptoms usually predominate, including sweating,
tachycardia, cravings and diarrhoea.
Selective serotonin re-uptake inhibitor (SSRI) withdrawal (Option E) is incorrect. SSRI
withdrawal is associated with dysphoria and suicidal ideation, so guidelines advocate gradual
https://mypastest.pastest.com/Secure/TestMe/Browser/429893#Top

1/2

8/11/2016

MyPastest

dose reduction over several weeks.


46931

Next Question

Previous Question

Tag Question

Feedback

End Review

Difficulty: Average
Peer Responses

Session Progress
Responses Correct:

Responses Incorrect:

207

Responses Total:

207

Responses - % Correct:

0%

Blog (https://www.pastest.com/blog) About Pastest (https://www.pastest.com/about-us)


Contact Us (https://www.pastest.com/contact-us) Help (https://www.pastest.com/help)
Pastest 2016

https://mypastest.pastest.com/Secure/TestMe/Browser/429893#Top

2/2

8/11/2016

MyPastest

Back to Filters (/Secure/TestMe/Filter/429893/QA)

Question 189 of 207

You are called to the endoscopy suite by a 2222 emergency call. An 82-year-old woman is
being prepared for an upper gastrointestinal endoscopy to investigate a recent coffee-ground
vomit and melaena. Soon after receiving a dose of intravenous midazolam, she suffers a
cardiorespiratory arrest.
Which is the most appropriate choice of emergency treatment?
A

Dantrolene

Diazepam

Flumazenil

Naloxone

Neostigmine

Explanation

The answer is Flumazenil


Flumazenil, a benzodiazepine antagonist, is used to reverse the central sedative effects
of benzodiazepines after anaesthetic and similar procedures. Flumazenil has a shorter
half-life than that of diazepam and midazolam and there is a risk that patients may
require a repeated dose.

Dantrolene (Option A) is incorrect. Dantrolene is used in the treatment of malignant


hyperthermia such as occurs in the setting of serotonergic syndrome.
Diazepam (Option B) is incorrect. Diazepam would cause addive benzodiazepine effects and
worsen respiratory depression.
Naloxone (Option D) is incorrect. Naloxone is used for the treatment of opiate overdose.
Neostigmine (Option E) is incorrect. Neostigmine is used in the treatment of myasthenia
gravis.
46822

https://mypastest.pastest.com/Secure/TestMe/Browser/429893#Top

1/2

8/11/2016

MyPastest

Next Question

Previous Question

Tag Question

Feedback

End Review

Difficulty: Average
Peer Responses

Session Progress
Responses Correct:

Responses Incorrect:

207

Responses Total:

207

Responses - % Correct:

0%

Blog (https://www.pastest.com/blog) About Pastest (https://www.pastest.com/about-us)


Contact Us (https://www.pastest.com/contact-us) Help (https://www.pastest.com/help)
Pastest 2016

https://mypastest.pastest.com/Secure/TestMe/Browser/429893#Top

2/2

8/11/2016

MyPastest

Back to Filters (/Secure/TestMe/Filter/429893/QA)

Question 190 of 207

A 40-year-old woman is brought unconscious to the Emergency Department. She apparently


has swallowed a large number of pills belonging to her friend who is being treated for a
psychiatric disorder. On examination, she is apyrexial, her pulse is 130 bpm and her blood
pressure is 90/60 mmHg. Her pupils are dilated. ECG shows sinus tachycardia and occasional
ventricular ectopics.
Which of the following drugs is she most likely to have taken?
A

Chlordiazepoxide

Chlorpromazine

Fluoxetine

Imipramine

Lithium

Explanation

The answer is Imipramine


Overdose with tricyclic antidepressants can produce anticholinergic effects (dilated
pupils, tachycardia) and sodium channel blockade (QRS prolongation, arrhythmia and
seizures). Tricyclic overdose may also be associated with loss of consciousness, grossly
abnormal reflexes and divergent eye movements.

Chlordiazepoxide (Option A) is incorrect. Chlordiazepoxide overdose would cause central


sedation, respiratory depression and coma; it would not be expected to alter pupil size or
cause tachycardia.
Chlorpromazine (Option B) is incorrect. Phenothiazine (chlorpromazine) poisoning causes
dystonia (torticollis, opisthotonos).
Fluoxetine (Option C) is incorrect. Fluoxetine is a selective serotonin-reuptake inhibitor
(SSRI) and overdose can cause blurring of vision, fits, headache, tremors and, rarely,
serotonin syndrome (agitation, dysarthria, tremor, myoclonus and restlessness).
https://mypastest.pastest.com/Secure/TestMe/Browser/429893#Top

1/2

8/11/2016

MyPastest

Lithium (Option E) is incorrect. Lithium causes blurring of vision, diarrhoea, vomiting,


hypokalaemia, drowsiness, ataxia, coarse tremors, dysarthria, seizures, psychosis and coma.
Dilated pupils are not a recognised feature of lithium toxicity.
46764

Next Question

Previous Question

Tag Question

Feedback

End Review

Difficulty: Average
Peer Responses

Session Progress
Responses Correct:

Responses Incorrect:

207

Responses Total:

207

Responses - % Correct:

0%

Blog (https://www.pastest.com/blog) About Pastest (https://www.pastest.com/about-us)


Contact Us (https://www.pastest.com/contact-us) Help (https://www.pastest.com/help)
Pastest 2016

https://mypastest.pastest.com/Secure/TestMe/Browser/429893#Top

2/2

8/11/2016

MyPastest

Back to Filters (/Secure/TestMe/Filter/429893/QA)

Question 191 of 207

You admit a 65-year-old man, via the Emergency Department, who has suffered an acute
myocardial infarction. On examination of his medication history you note that he is taking a
newly licensed antihypertensive agent and you suspect an adverse drug reaction.
Which of the following is most accurate concerning the Yellow Card recording system
currently employed in the UK?
A

Approximately 50% of serious adverse drug reactions are identified through the
system

Follow-up of serious adverse events is voluntary for pharmaceutical companies

Only 10% of serious adverse drug reactions are identified by Yellow Cards

Reporting of adverse drug reactions is compulsory for health professionals

The scheme is administered by the ABPI

Explanation

The answer is Only 10% of serious adverse drug reactions are identified by Yellow Cards
The Yellow Card scheme is administered by the Medicines Healthcare Products
Regulatory Agency, a governmental organisation. It was set up over 40 years ago as a
means of identifying important adverse effects at an earlier stage than would be allowed
without systematic national data collection. It requires voluntary reporting; however, only
around 10% of adverse drug reactions (ADRs) are thought to be reported. When a new
product is launched there may be significant over-reporting compared to other agents in
the same established drug class.

Approximately 50% of serious adverse drug reactions are identified through the system
(Option A) is incorrect. Reporting rates are estimated to be around 10% overall.
Follow-up of serious adverse events is voluntary for pharmaceutical companies (Option B) is
incorrect. Pharmaceutical manufacturers are bound by very strict guidelines around rapid
reporting of ADRs and are obliged to follow up reported ADRs and notify the regulatory
authorities.
https://mypastest.pastest.com/Secure/TestMe/Browser/429893#Top

1/2

8/11/2016

MyPastest

Reporting of adverse drug reactions is compulsory for health professionals (Option D) is


incorrect. The Yellow Card reporting system is voluntary.
The scheme is administered by the ABPI (Option E) is incorrect. The Association for the
British Pharmaceutical Industry (ABPI) is a voluntary professional body that sets standards
and regulates performance in the different aspects of drug development, marketing and
safety reporting. The Yellow Card scheme is administered by the Medicines and Healthcare
products Regulatory Authority (MHRA).
46876

Next Question

Previous Question

Tag Question

Feedback

End Review

Difficulty: Average
Peer Responses

Session Progress
Responses Correct:

Responses Incorrect:

207

Responses Total:

207

Responses - % Correct:

0%

Blog (https://www.pastest.com/blog) About Pastest (https://www.pastest.com/about-us)


Contact Us (https://www.pastest.com/contact-us) Help (https://www.pastest.com/help)
Pastest 2016

https://mypastest.pastest.com/Secure/TestMe/Browser/429893#Top

2/2

8/11/2016

MyPastest

Back to Filters (/Secure/TestMe/Filter/429893/QA)

Question 192 of 207

A 62-year-old man is brought in by a friend who found him collapsed. In his pockets you find
a card from the Psychiatry Outpatients Department, along with a full box of haloperidol
tablets. On examination his temperature is 38.9C, and he is alert but not responsive to
commands. He has widespread muscular rigidity.
Which one of the following is the most likely diagnosis?
A

Bacterial meningitis

Catatonic schizophrenia

Cerebral malaria

Haloperidol overdose

Neuroleptic malignant syndrome

Explanation

The answer is Neuroleptic malignant syndrome


Neuroleptic malignant syndrome occurs in around 12% of patients taking phenothiazines
or butyrophenones (eg. haloperidol). It is less commonly recognised as a complication of
treatment with tricyclic antidepressants, selective serotonin re-uptake inhibitors (SSRIs)
and metoclopramide. There is inhibition of central dopamine receptors in the
hypothalamus and abnormal thermoregulation. Clinical features include hyperthermia,
rigidity, altered mental status, variable blood pressure and autonomic dysfunction. If
muscle rigidity is severe, there may also be metabolic acidosis and myoglobinuria. Onset
is most common within the first 30 days of starting treatment, but can occur at any time;
it typically lasts 23 weeks after stopping the causal drug, and mortality is 520%.
Treatment is symptomatic and supportive with active cooling and rehydration.

Bacterial meningitis (Option A) is incorrect. Bacterial meningitis may present with purpuric
rash, with symptoms of meningism (neck stiffness, headache, photophobia); generalised
muscle rigidity is not usually a presenting feature.

https://mypastest.pastest.com/Secure/TestMe/Browser/429893#Top

1/2

8/11/2016

MyPastest

Catatonic schizophrenia (Option B) is incorrect. Catatonic schizophrenia involves a


withdrawn state and would not typically be associated with high temperature or rigidity.
Cerebral malaria (Option C) is incorrect. Cerebral malaria caused by Plasmodium falciparum
is characterised by fever and reduced consciousness, and may also cause shock, metabolic
acidosis, hypoglycaemia, renal failure and adult respiratory distress syndrome.
Haloperidol overdose (Option D) is incorrect. Haloperidol may cause significant muscle
rigidity and extra-pyramidal features, but would be less likely to cause a high temperature
and collapse.
46964

Next Question

Previous Question

Tag Question

Feedback

End Review

Difficulty: Average
Peer Responses

Session Progress
Responses Correct:

Responses Incorrect:

207

Responses Total:

207

Responses - % Correct:

0%

Blog (https://www.pastest.com/blog) About Pastest (https://www.pastest.com/about-us)


Contact Us (https://www.pastest.com/contact-us) Help (https://www.pastest.com/help)
Pastest 2016

https://mypastest.pastest.com/Secure/TestMe/Browser/429893#Top

2/2

8/11/2016

MyPastest

Back to Filters (/Secure/TestMe/Filter/429893/QA)

Question 193 of 207

You review a 58-year-old man with tuberculosis and commence him on triple antituberculous
therapy. He has a past history of alcoholism. You are aware of the problem of peripheral
neuropathy with isoniazid therapy.
Which of the following is most characteristic of isoniazid-induced peripheral neuropathy?
A

It occurs because of a defect in butyrylcholinesterase

It only occurs in alcoholics

Thiamine prophylaxis is usually used to prevent its occurrence

Those with an N-acetyltransferase type-1 gene defect are predisposed to neuropathy

Those with an N-acetyltransferase type-2 gene defect are predisposed to


neuropathy

Explanation

The answer is Those with an N-acetyltransferase type-2 gene defect are predisposed to
neuropathy
Thankfully, isoniazid-associated peripheral neuropathy occurs in less than 1% of those
who take the drug, although the incidence is increased at higher drug doses. It is related
to a defect in the gene coding for N-acetyltransferase type 2, resulting in abnormal
isoniazid metabolism and slow acetylator status. Isoniazid is also associated with raised
transaminases in 1020% of those patients for whom it is prescribed, and rarely hepatic
necrosis and jaundice (liver toxicity is more likely to occur in fast acetylator status.
Rifampicin and pyrazinamide are two other antituberculous agents that may also be
associated with abnormal liver enzymes.

It occurs because of a defect in butyrylcholinesterase (Option A) is incorrect.


Butyrylcholinesterase defects are associated with suxamethonium-induced apnoea.
Butyrylcholinesterase is not relevant to isoniazid metabolism.
It only occurs in alcoholics (Option B) is incorrect. It is more common in alcoholics, but not
unique to this patient group.
https://mypastest.pastest.com/Secure/TestMe/Browser/429893#Top

1/2

8/11/2016

MyPastest

Thiamine prophylaxis is usually used to prevent its occurrence (Option C) is incorrect.


Pyridoxine (vitamin B6) is given as prophylaxis against peripheral neuropathy occurring, not
thiamine (vitamin B1).
Those with an N-acetyltransferase type-1 gene defect are predisposed to neuropathy (Option
D) is incorrect. NAT-1 mutations may be associated with increased risk of colorectal
adenomas.
46877

Next Question

Previous Question

Tag Question

Feedback

End Review

Difficulty: Average
Peer Responses

Session Progress
Responses Correct:

Responses Incorrect:

207

Responses Total:

207

Responses - % Correct:

0%

Blog (https://www.pastest.com/blog) About Pastest (https://www.pastest.com/about-us)


Contact Us (https://www.pastest.com/contact-us) Help (https://www.pastest.com/help)
Pastest 2016

https://mypastest.pastest.com/Secure/TestMe/Browser/429893#Top

2/2

8/11/2016

MyPastest

Back to Filters (/Secure/TestMe/Filter/429893/QA)

Question 194 of 207

A man had an argument with his girlfriend and drank anti-freeze in an impulsive attempt to
commit suicide. On admission his consciousness level is decreased, he is retching and
complaining of abdominal pain. Biochemistry tests show pH 7.1, HCO3 2 mmol/l, large base
deficit, calcium 1.98 mmol/l.
What would be the next immediate management after replacing calcium?
A

Haemodialysis

Start ethanol infusion

Start oral ethanol

Start peripheral sodium bicarbonate 8.4% infusion

Start sodium bicarbonate 8.4% infusion via central line

Explanation

The answer is Start sodium bicarbonate 8.4% infusion via central line
As little as 30 ml of ethylene glycol may be fatal in adults This patient, who has taken
ethylene glycol in an attempt to commit suicide, has very low HCO3 levels, severe
acidosis and a base deficit. In this situation the next immediate treatment is with sodium
bicarbonate; the 8.4% concentration should be given via central line over 3040 min. It is
also important to commence treatment with fomepizole to inhibit alcohol
dehydrogenase promptly.
Fomepizole or alcohol may be given as inhibitors of alcohol dehydrogenase.
Fomepizole has the advantage of not decreasing the level of consciousness.
It is likely that haemodialysis will be required; in patients requiring haemodialysis
either the alcohol infusion rate or the dose interval between doses of fomepizole
requires alteration.

https://mypastest.pastest.com/Secure/TestMe/Browser/429893#Top

1/2

8/11/2016

MyPastest

Haemodialysis (Option A) is incorrect. Conventional indications for dialysis include pH < 7.25,
acute renal failure or ethylene glycol level > 500 mg/l. In the first instance sodium
bicarbonate administration should be used to correct pH.
Start ethanol infusion (Option B) is incorrect. Intravenous ethanol infusion may be
considered, as a competitive inhibitor of alcohol dehydrogenase, but this is not an immediate
priority until pH is corrected.
Start oral ethanol (Option C) is incorrect. Oral ethanol is a lower priority than correction of
pH to prevent seizures.
Start peripheral sodium bicarbonate 8.4% infusion (Option D) is incorrect. Peripheral sodium
bicarbonate is venotoxic and should be given via a central line where possible.
46928

Next Question

Previous Question

Tag Question

Feedback

End Review

Difficulty: Average
Peer Responses

Session Progress
Responses Correct:

Responses Incorrect:

207

Responses Total:

207

Responses - % Correct:

0%

Blog (https://www.pastest.com/blog) About Pastest (https://www.pastest.com/about-us)


Contact Us (https://www.pastest.com/contact-us) Help (https://www.pastest.com/help)
Pastest 2016

https://mypastest.pastest.com/Secure/TestMe/Browser/429893#Top

2/2

8/11/2016

MyPastest

Back to Filters (/Secure/TestMe/Filter/429893/QA)

Question 195 of 207

A 59-year-old woman presents to hospital with features of altered behaviour with


aggressiveness and increased libido. Her husband says that prior to this she had been
markedly withdrawn and had low mood and suicidal thoughts for several weeks. A liaison
psychiatry review has indicated a suspected diagnosis of bipolar affective disorder and is
arranging transfer to a psychiatry unit. Her partner is asking for advice regarding long-term
therapies that might be considered.
Which of the following drugs would offer most effective long-term improvement in patients
with bipolar affective disorder?
A

Carbamazepine

Diazepam

Fluoxetine

Lithium

Phenytoin

Explanation

The answer is Lithium


Sodium valproate has been demonstrated to have antimanic effects and is now being
widely used for this indication. Lithium therapy is effective in treatment and prevention
of mania and depression, but is generally regarded as second-line therapy in patients
that fail to tolerate or responde to sodium valproate. Lithium is potentially a very toxic
drug and therapeutic monitoring is to minimise the risks of toxicity (polyuria, polydipsia,
diarrhoea, vomiting, tremor, confusion, ataxia, dizziness). Long-term use is associated
with development of nephrogenic diabetes insipidus, hypothyroidism and renal failure,
and it is not normally used for more than 5 years.

Carbamazepine (Option A) is incorrect. Carbamazepine may be considered for patients that


have failed to respond to sodium valproate or lithium.
Diazepam (Option B) is incorrect. Diazepam would be useful only in the manic phase.
https://mypastest.pastest.com/Secure/TestMe/Browser/429893#Top

1/2

8/11/2016

MyPastest

Fluoxetine (Option C) is incorrect. Fluoxetine may be effective in treating symptoms of the


depression phase but less effective overall than sodium valproate or lithium for bipolar
disorder.
Phenytoin (Option E) is incorrect. Phenytoin may have some role as a mood stabiliser but
efficacy is less well established than for other agents.
46783

Next Question

Previous Question

Tag Question

Feedback

End Review

Difficulty: Average
Peer Responses

Session Progress
Responses Correct:

Responses Incorrect:

207

Responses Total:

207

Responses - % Correct:

0%

Blog (https://www.pastest.com/blog) About Pastest (https://www.pastest.com/about-us)


Contact Us (https://www.pastest.com/contact-us) Help (https://www.pastest.com/help)
Pastest 2016

https://mypastest.pastest.com/Secure/TestMe/Browser/429893#Top

2/2

8/11/2016

MyPastest

Back to Filters (/Secure/TestMe/Filter/429893/QA)

Question 196 of 207

A 54-year-old man has suffered a myocardial infarction managed in a Coronary Care Unit. He
is thought to be in a stable condition and is moved to a general ward area. He is in mild left
ventricular failure (LVF), and echocardiography estimates his ejection fraction at around 34%.
You commence eplerenone therapy at a dose of 25 mg. His BP is 135/75 mmHg, pulse is
80/min and regular.
Which of the following adverse effects would be most important to observe for?
A

Constipation

Hyperkalaemia

Hypertension

Hypocalcaemia

Hyponatraemia

Explanation

The answer is Hyperkalaemia


Eplerenone is an aldosterone antagonist that improves survival when added to existing
therapy after myocardial infarction in stable patients with an ejection fraction of less than
40% and clinical evidence of heart failure; this benefit is in addition to existing treatment
with -blockers, ACE inhibitors and antiplatelet medications, and eplerenone may reduce
all-cause mortality by up to 15%.
Common adverse effects are hyperkalaemia, dizziness, hypotension, diarrhoea, nausea
and prerenal renal dysfunction. Eplerenone is metabolised via the CYP3A4 system, so
that inducers and inhibitors of the 3A4 enzyme subtype may precipitate drug
interactions.

Constipation (Option A) is incorrect. Diarrhoea is an uncommon but recognised adverse


effect.
Hypertension (Option C) is incorrect. Eplerenone may cause hypotension including postural
hypotension.
https://mypastest.pastest.com/Secure/TestMe/Browser/429893#Top

1/2

8/11/2016

MyPastest

Hypocalcaemia (Option D) is incorrect. Hypocalcaemia is a recognised complication of loop


diuretics, including furosemide and bumetanide.
Hyponatraemia (Option E) is incorrect. Eplerenone and spironolactone have only a modest
effect on sodium excretion, and are less likely to cause hyponatraemia than loop or thiazide
type diuretics.
46891

Next Question

Previous Question

Tag Question

Feedback

End Review

Difficulty: Average
Peer Responses

Session Progress
Responses Correct:

Responses Incorrect:

207

Responses Total:

207

Responses - % Correct:

0%

Blog (https://www.pastest.com/blog) About Pastest (https://www.pastest.com/about-us)


Contact Us (https://www.pastest.com/contact-us) Help (https://www.pastest.com/help)
Pastest 2016

https://mypastest.pastest.com/Secure/TestMe/Browser/429893#Top

2/2

8/11/2016

MyPastest

Back to Filters (/Secure/TestMe/Filter/429893/QA)

Question 197 of 207

A patient undergoing cancer chemotherapy complains of increased urinary frequency and


suprapubic pain. Investigations reveal he has microcytic, hypochromic anaemia, leucopenia,
thrombocytopaenia and haematuria.
Which of the following medications is the most likely cause of his symptoms?
A

Cisplatin

Chlorambucil

Cyclophosphamide

Melphalan

Methotrexate

Explanation

The answer is Cyclophosphamide


Cyclophosphamide is inactive unless metabolised by the liver to 4-hydroxyl
cyclophosphamide, which is metabolised into alkylating species as well as to
chloroacetaldehyde and acrolein. Acrolein causes chemical cystitis and therefore
hydration must be maintained during cyclophosphamide therapy.

Cisplatin (Option A) is incorrect. Cisplatin is associated with nephrotoxicity, hypokalaemia,


haemolysis, ototoxicity and peripheral neuropathy.
Chlorambucil (Option B) is incorrect. Chlorambucil causes myelosuppression, hepatotoxicity
and azoospermia.
Melphalan (Option D) is incorrect. Melphalan suppresses the bone marrow and has other
effects, including inappropriate secretion of ADH, amenorrhoea, sterility and pulmonary
infiltrates.
Methotrexate (Option E) is incorrect. Methotrexate blocks folic acid synthesis and may cause
macrocytic anaemia, stomatitis, alopecia, hepatotoxicity and acute pneumonitis.
46746

https://mypastest.pastest.com/Secure/TestMe/Browser/429893#Top

1/2

8/11/2016

MyPastest

Next Question

Previous Question

Tag Question

Feedback

End Review

Difficulty: Average
Peer Responses

Session Progress
Responses Correct:

Responses Incorrect:

207

Responses Total:

207

Responses - % Correct:

0%

Blog (https://www.pastest.com/blog) About Pastest (https://www.pastest.com/about-us)


Contact Us (https://www.pastest.com/contact-us) Help (https://www.pastest.com/help)
Pastest 2016

https://mypastest.pastest.com/Secure/TestMe/Browser/429893#Top

2/2

8/11/2016

MyPastest

Back to Filters (/Secure/TestMe/Filter/429893/QA)

Question 198 of 207

A 40-year-old woman has been established on warfarin therapy following a deep venous
thrombosis, with INR between 2.0 and 3.0. However, at her last clinic visit the INR was noted
to be 4.9, without any change in warfarin dose. On questioning, she admits to having recently
self-medicated with some of her partners medications.
Which of the following drugs is most likely to account for the change in INR?
A

Amoxicillin

Aspirin

Cimetidine

Diazepam

Indometacin

Explanation

The answer is Cimetidine


Cimetidine inhibits breakdown of warfarin by the hepatic cytochrome P450 system, so
that INR and bleeding risk increase.

Amoxicillin (Option A) is incorrect. Amoxicillin is neither a p450 enzyme inducer, nor


inhibitor, and as such is less likely to affect warfarin metabolism. It may interfere with gut
flora and interfere with warfarin bsorption so that there may be a small fall in INR.
Aspirin (Option B) is incorrect. Aspirin may increase bleeding by inhibiting platelet function,
but this will not affect the INR.
Diazepam (Option D) is incorrect. Diazepam does not significantly interact with warfarin.
Indometacin (Option E) is incorrect. Indometacin may increase free warfarin and INR owing
to a protein-binding interaction; however, the effect on INR is less obvious than the addition
of cimetidine, making cimetidine the preferred answer.
46896

https://mypastest.pastest.com/Secure/TestMe/Browser/429893#Top

1/2

8/11/2016

MyPastest

Next Question

Previous Question

Tag Question

Feedback

End Review

Difficulty: Average
Peer Responses

Session Progress
Responses Correct:

Responses Incorrect:

207

Responses Total:

207

Responses - % Correct:

0%

Blog (https://www.pastest.com/blog) About Pastest (https://www.pastest.com/about-us)


Contact Us (https://www.pastest.com/contact-us) Help (https://www.pastest.com/help)
Pastest 2016

https://mypastest.pastest.com/Secure/TestMe/Browser/429893#Top

2/2

8/11/2016

MyPastest

Back to Filters (/Secure/TestMe/Filter/429893/QA)

Question 199 of 207

A 63-year-old lady is found to be jaundiced. Biochemical results are as follows: bilirubin 62


mol/l (122), -glutamyl transferase 400 IU (435), alanine aminotransferase 42 IU (535)
and alkaline phosphatase 386 IU (45105). She is taking many different drugs and you
consider the possibility of drug-induced jaundice.
Which of the following is most likely to be responsible for the biochemical findings?
A

Amiodarone

Chlorpromazine

Isoniazid

Methotrexate

Methyldopa

Explanation

The answer is Chlorpromazine


The biochemical picture is one of cholestasis or obstructive jaundice, rather than
hepatitis. A number of drugs can cause cholestasis, including chlorpromazine,
chlorpropamide, erythromycin, nitrofurantoin, co-amoxiclav, nifedipine and statins
(statins and nifedipine may also cause hepatitis).

Amiodarone (Option A) is incorrect. Amiodarone may cause hepatitis, but not cholestasis.
Isoniazid (Option C) is incorrect. Isoniazide may cause hepatitis (via its key metabolite), and
this is more likely to occur in slow acetylators.
Methotrexate (Option D) is incorrect. Methotrexate may provoke a fibrotic reaction within the
liver.
Methyldopa (Option E) is incorrect. Methyldopa may cause an autoimmune hepatitis, but
does not cause cholestasis.
46908

https://mypastest.pastest.com/Secure/TestMe/Browser/429893#Top

1/2

8/11/2016

MyPastest

Next Question

Previous Question

Tag Question

Feedback

End Review

Difficulty: Average
Peer Responses

Session Progress
Responses Correct:

Responses Incorrect:

207

Responses Total:

207

Responses - % Correct:

0%

Blog (https://www.pastest.com/blog) About Pastest (https://www.pastest.com/about-us)


Contact Us (https://www.pastest.com/contact-us) Help (https://www.pastest.com/help)
Pastest 2016

https://mypastest.pastest.com/Secure/TestMe/Browser/429893#Top

2/2

8/11/2016

MyPastest

Back to Filters (/Secure/TestMe/Filter/429893/QA)

Question 200 of 207

You are responsible for initiating anticoagulation therapy for a 56-year-old woman with new
onset deep vein thrombosis.
Which one of the following is most strongly associated with warfarin therapy?
A

An initial loading dose is given because it has a short half-life (3 h)

Chronic use gives rise to osteoporosis in 10% of patients

May induce autoimmune thrombocytopaenia

Reduces protein C levels in the blood

Should be avoided in lactating women

Explanation

The answer is Reduces protein C levels in the blood


Warfarin competitively inhibits carboxylation of vitamin K-dependent factors: factor II,
factor VII, factor IX, factor X and protein C. The half-life of warfarin is approximately 44 h.
It is highly protein bound, and the quantities expressed in breast milk are too low to be
of any clinical significance.

An initial loading dose is given because it has a short half-life (3 h) (Option A) is incorrect.
The initial loading dose affords a more rapid onset of action, unrelated to the drug half-life.
Chronic use gives rise to osteoporosis in 10% of patients (Option B) is incorrect.
Osteoporosis is an important adverse effect of long-term heparin therapy, as might be used
in pregnancy, cancer patients, or patients who are intolerant of oral anticoagulants.
May induce autoimmune thrombocytopaenia (Option C) is incorrect. Autoimmune
thrombocytopaenia is a well characterised adverse effect of heparin treatment.
Should be avoided in lactating women (Option E) is incorrect. Warfarin does not penetrate
breast milk in sufficiently large quantities to pose a significant risk to the infant.
46625

https://mypastest.pastest.com/Secure/TestMe/Browser/429893#Top

1/2

8/11/2016

MyPastest

Next Question

Previous Question

Tag Question

Feedback

End Review

Difficulty: Average
Peer Responses

Session Progress
Responses Correct:

Responses Incorrect:

207

Responses Total:

207

Responses - % Correct:

0%

Blog (https://www.pastest.com/blog) About Pastest (https://www.pastest.com/about-us)


Contact Us (https://www.pastest.com/contact-us) Help (https://www.pastest.com/help)
Pastest 2016

https://mypastest.pastest.com/Secure/TestMe/Browser/429893#Top

2/2

8/11/2016

MyPastest

Back to Filters (/Secure/TestMe/Filter/429893/QA)

Question 201 of 207

A 21-year-old student suffers from hay fever and acne. She has been receiving long-term
erythromycin therapy for acne, and has attended clinic for advice about her hay fever, which
has been much worse recently. She is about to take her final examinations and is worried that
certain antihistamines may cause drowsiness.
Which antihistamine would be most suitable in these circumstances?
A

Chlormethiazole

Chlorphenamine maleate

Desloratadine

Loratadine

Promethiazine

Explanation

The answer is Desloratadine


Desloratadine is a long-acting selective H1-receptor antagonist that has little or no
clinically significant interaction with erythromycin. It has comparatively poor penetration
into the central nervous system and is not sedating. Cetirizine, desloratadine and
fexofenadine may be prescribed for allergic rhinitis (hay fever) and all three are equally
effective, but cetirizine and fexofenadine effects may be increased by erythromycin and
other macrolides.

Chlormethiazole (Option A) is incorrect. Chlormethiazole is particularly sedating and should


be avoided where this may be undesirable.
Chlorphenamine maleate (Option B) is incorrect. Chlorpheniramine maleate is a sedating
antihistamine.
Loratadine (Option D) is incorrect. Loratadine effects may be increased by concomitant
erythromycin.

https://mypastest.pastest.com/Secure/TestMe/Browser/429893#Top

1/2

8/11/2016

MyPastest

Promethiazine (Option E) is incorrect. Promethiazine is particularly sedating and should be


avoided where this may be undesirable.
46761

Next Question

Previous Question

Tag Question

Feedback

End Review

Difficulty: Average
Peer Responses

Session Progress
Responses Correct:

Responses Incorrect:

207

Responses Total:

207

Responses - % Correct:

0%

Blog (https://www.pastest.com/blog) About Pastest (https://www.pastest.com/about-us)


Contact Us (https://www.pastest.com/contact-us) Help (https://www.pastest.com/help)
Pastest 2016

https://mypastest.pastest.com/Secure/TestMe/Browser/429893#Top

2/2

8/11/2016

MyPastest

Back to Filters (/Secure/TestMe/Filter/429893/QA)

Question 202 of 207

A 65-year-old woman with type 2 diabetes of 11 years duration presents with poorly
controlled blood glucose levels. She was overweight and initially started on metformin
therapy. Her diabetes was well controlled until the last 12 months. Despite strict adherence to
diet, exercise and maximum daily doses of metformin, satisfactory blood glucose control has
proved difficult to achieve and the last Hb A1c was at 13%. You consider adding pioglitazone.
Which of the following best describes the pharmacological properties of pioglitazone?
A

A benzoic acid derivative

A sulfonylurea

An -glucosidase inhibitor

An insulin secretagogue, which stimulates -cell insulin secretion

An insulin sensitiser, which decreases peripheral insulin resistance

Explanation

The answer is An insulin sensitiser, which decreases peripheral insulin resistance


Pioglitazone exerts its effects by activating the peroxisome proliferation-activated
receptor-gamma (PPAR), which is a nuclear receptor that influences the differentiation
of fibroblasts into adipocytes and lowers free fatty acid levels. Its major therapeutic
action is to decrease peripheral insulin resistance, although at higher doses it may also
decrease hepatic glucose production. Pioglitazone appears to act as an insulin sensitiser,
and its full hypoglycaemic potency may take up to 12 weeks. Adverse events of
pioglitazone include fluid retention, weight gain and decreased bone mineral density.

A benzoic acid derivative (Option A) is incorrect. Benzoic acid derivatives have no


therapeutic action relevant to treatment of diabetes.
A sulfonylurea (Option B) is incorrect. Sulfonlyureas (eg gliclazide) stimulate insulin
secretion, and may provoke hypoglycaemia.
An -glucosidase inhibitor (Option C) is incorrect. Acarbose is an -glucosidase inhibitor that
delays glucose absorption from the gut. Acarbose acts locally in the small intestine by
https://mypastest.pastest.com/Secure/TestMe/Browser/429893#Top

1/2

8/11/2016

MyPastest

inhibiting -glucosidase enzymes, an action that slows digestion of ingested carbohydrates,


delays glucose absorption and reduces the increase in postprandial blood glucose.
An insulin secretagogue, which stimulates -cell insulin secretion (Option D), is incorrect.
Glitazones modulate the response to insulin, but do not directly stimulate or inhibit insulin
secretion.
46633

Next Question

Previous Question

Tag Question

Feedback

End Review

Difficulty: Average
Peer Responses

Session Progress
Responses Correct:

Responses Incorrect:

207

Responses Total:

207

Responses - % Correct:

0%

Blog (https://www.pastest.com/blog) About Pastest (https://www.pastest.com/about-us)


Contact Us (https://www.pastest.com/contact-us) Help (https://www.pastest.com/help)
Pastest 2016

https://mypastest.pastest.com/Secure/TestMe/Browser/429893#Top

2/2

8/11/2016

MyPastest

Back to Filters (/Secure/TestMe/Filter/429893/QA)

Question 203 of 207

A 54-year-old man attends the cardiology clinic for review. He has a history of myocardial
infarction 5 years ago and his current angina symptoms are stable. However, he mentions that
he has been suffering impotence recently, and is keen to start sildenafil.
Which one of the following medications is most likely to interact with sildenafil and should be
avoided?
A

Bisoprolol

Doxazosin

Furosemide

Isosorbide mononitrate

Nifedipine

Explanation

The answer is Isosorbide mononitrate


Interactions with phosphodiesterase type-5 inhibitors Glyceryl trinitrate, nicorandil and
isosorbide all act as nitric oxide donors, which can lead to profound hypotension when
they are combined with any of the phosphodiesterase type-5 (PDE5) inhibitors. Common
side-effects associated with sildenafil include headache, flushing, indigestion, nasal
congestion, dizziness and visual colour distortion.

Bisoprolol (Option A) is incorrect. Patients that have stable symptoms on a -blocker can
normally take sildenafil without difficulty.
Doxazosin (Option B) is incorrect. Alpha-blockers may precipitate hypotension when used in
conjunction with medications such as sildenafil and should be avoided if possible. To
minimise the hypotensive effect of -blockade, sildenafil is introduced at the lowest dose
first. The risks of nitrates are higher, hence isosorbide nitrate is the preferred answer.
Furosemide (Option C) is incorrect. Furosemide is unlikely to cause problems.

https://mypastest.pastest.com/Secure/TestMe/Browser/429893#Top

1/2

8/11/2016

MyPastest

Nifedipine (Option E) is incorrect. Patients that have stable symptoms on nifedipine can
normally take sildenafil without difficulty.
46991

Next Question

Previous Question

Tag Question

Feedback

End Review

Difficulty: Average
Peer Responses

Session Progress
Responses Correct:

Responses Incorrect:

207

Responses Total:

207

Responses - % Correct:

0%

Blog (https://www.pastest.com/blog) About Pastest (https://www.pastest.com/about-us)


Contact Us (https://www.pastest.com/contact-us) Help (https://www.pastest.com/help)
Pastest 2016

https://mypastest.pastest.com/Secure/TestMe/Browser/429893#Top

2/2

8/11/2016

MyPastest

Back to Filters (/Secure/TestMe/Filter/429893/QA)

Question 204 of 207

You are asked to see a 17-year-old woman in the Emergency Department. She has been
brought to the unit by her boyfriend, who is concerned that she may have taken an overdose
after being depressed about her mother, who is dying of carcinoma of the breast.
Which one of the following features would most strongly indicate that she has taken some of
her mothers morphine sulfate pills?
A

Dilated pupils

Hypertension

Increased libido

Sweating

Urinary incontinence

Explanation

The answer is Sweating


Psychological and physical dependence on morphine may occur. Common adverse
effects include nausea, vomiting, constipation, drowsiness, small pupils and confusion.
Less common effects include sweating, rash, flushing, oedema, headache and raised
intracranial pressure. Large doses typically cause respiratory depression, circulatory
failure and coma.

Dilated pupils (Option A) is incorrect. Pupils are small and may be very constricted pinpoint.
Hypertension (Option B) is incorrect. Flushing and hypotension may occur.
Increased libido (Option C) is incorrect. Libido is typically reduced.
Urinary incontinence (Option E) is incorrect. Constipation and urinary retention may occur.
46980

Next Question

https://mypastest.pastest.com/Secure/TestMe/Browser/429893#Top

1/2

8/11/2016

MyPastest

Previous Question

Tag Question

Feedback

End Review

Difficulty: Average
Peer Responses

Session Progress
Responses Correct:

Responses Incorrect:

207

Responses Total:

207

Responses - % Correct:

0%

Blog (https://www.pastest.com/blog) About Pastest (https://www.pastest.com/about-us)


Contact Us (https://www.pastest.com/contact-us) Help (https://www.pastest.com/help)
Pastest 2016

https://mypastest.pastest.com/Secure/TestMe/Browser/429893#Top

2/2

8/11/2016

MyPastest

Back to Filters (/Secure/TestMe/Filter/429893/QA)

Question 205 of 207

You are asked to review a pregnant patient with HIV disease.


Which of the following measures would be expected to be most effective in reducing the risk
of HIV infection in the newborn child?
A

Elective caesarean section

Oral zidovudine at the time of conception

Postnatal administration of zidovudine to the baby

Vaginal delivery with shortening of the second stage of labour

Zidovudine administration to the mother at the time of breast-feeding

Explanation

The answer is Postnatal administration of zidovudine to the baby


Oral zidovudine from 28 weeks gestation onwards can reduce the risk of transmission. A
range of studies suggest that zidovudine is effective in reducing the risk of HIV
transmission from mother to neonate by 8.3% to 18%. The optimal regimen is to
commence zidovudine as an intravenous infusion at the onset of labour and continue
treatment in the neonate up to 6 weeks of age. There is insufficient evidence that
intravenous zidovudine during labour is of any benefit, unless followed by treatment of
the child in the postnatal period.

Elective caesarean section (Option A) is incorrect. Elective caesarean section may reduce the
risk compared with vaginal delivery, but the effectiveness of this strategy is much less
certain, hence option C is the preferred answer.
Oral zidovudine at the time of conception (Option B) is incorrect. There is no evidence that
oral administration of zidovudine at the time of conception reduces the risk of HIV
transmission.
Vaginal delivery with shortening of the second stage of labour (Option D) is incorrect.
Shortening the second stage by operative delivery may increase the risk of transmission.
https://mypastest.pastest.com/Secure/TestMe/Browser/429893#Top

1/2

8/11/2016

MyPastest

Zidovudine administration to the mother at the time of breast-feeding (Option E) is


incorrect. Breast-feeding is generally avoided as this increases the risk of HIV transmission.
46751

Next Question

Previous Question

Tag Question

Feedback

End Review

Difficulty: Average
Peer Responses

Session Progress
Responses Correct:

Responses Incorrect:

207

Responses Total:

207

Responses - % Correct:

0%

Blog (https://www.pastest.com/blog) About Pastest (https://www.pastest.com/about-us)


Contact Us (https://www.pastest.com/contact-us) Help (https://www.pastest.com/help)
Pastest 2016

https://mypastest.pastest.com/Secure/TestMe/Browser/429893#Top

2/2

8/11/2016

MyPastest

Back to Filters (/Secure/TestMe/Filter/429893/QA)

Question 206 of 207

On admission examination, a 60-year-old man is noted to have reduced facial expression,


rigidity and bradykinesia. He has been taking a long-term medication and a diagnosis of
possible drug-induced parkinsonism is suspected.
Which of the following treatments is most likely to have caused these clinical features?
A

Domperidone

Droperidol

Imipramine

Orphenadrine

Selegiline

Explanation

The answer is Droperidol


Butyrophenones (eg haloperidol, droperidol) and phenothiazines are the most common
cause of drug-induced parkinsonism.

Domperidone (Option A) is incorrect. Domperidone (unlike metoclopramide) does not cross


the bloodbrain barrier and acts on peripheral dopaminergic receptors. It is much less likely
to cause extrapyramidal effects than metoclopramide.
Imipramine (Option C) is incorrect. Imipramine is a tricyclic antidepressant with some
anticholinergic effects within the central nervous system, but it is not expected to cause
parkinsonism.
Orphenadrine (Option D) is incorrect. Orphenadrine is an anticholinergic treatment that may
be effective for some symptoms of drug-induced parkinsonism.
Selegiline (Option E) is incorrect. Selegiline is a type B monoamine oxidase inhibitor (MAOI)
used to treat Parkinsons disease.
46899

https://mypastest.pastest.com/Secure/TestMe/Browser/429893#Top

1/2

8/11/2016

MyPastest

Next Question

Previous Question

Tag Question

Feedback

End Review

Difficulty: Average
Peer Responses

Session Progress
Responses Correct:

Responses Incorrect:

207

Responses Total:

207

Responses - % Correct:

0%

Blog (https://www.pastest.com/blog) About Pastest (https://www.pastest.com/about-us)


Contact Us (https://www.pastest.com/contact-us) Help (https://www.pastest.com/help)
Pastest 2016

https://mypastest.pastest.com/Secure/TestMe/Browser/429893#Top

2/2

8/11/2016

MyPastest

Back to Filters (/Secure/TestMe/Filter/429893/QA)

Question 207 of 207

A 20-year-old patient presents after taking an overdose involving her grandmothers aspirin
tablets. She has been complaing of nausea and vomiting, and has developed tinnitus,
sweating and dizziness.
Which one of the following is the best initial step in her management?
A

Acetylcysteine

Haemodialysis

Intravenous H2-receptor blocker

Intubation

Oral activated charcoal

Explanation

The answer is Oral activated charcoal


Oral activated charcoal should be considered if the patient presents within 1 h of
ingestion of the salicylate overdose to minimise drug absorption, and may be effective
even if administered beyond 1 h by increasing aspirin clearance.

Acetylcysteine (Option A) is incorrect. Acetylcysteine is effective for paracetamol toxicity.


Haemodialysis (Option B) is incorrect. Haemodialysis may be necessary for severe aspirin
poisoning, but would not be appropriate initial management. Haemodialysis is the treatment
of choice for severely poisoned patients, particularly those with features of central nervous
system toxicity and metabolic acidosis that fails to correct after intravenous sodium
bicarbonate.
Intravenous H2-receptor blocker (Option C) is incorrect. An intravenous H2-receptor blocker
may be helpful in preventing gastric irritation, but would do little to reduce systemic toxicity.
Intubation (Option D) is incorrect. Intubation is not indicated at this stage. Pulmonary
oedema occasionally complicates salicylate toxicity.
46816

https://mypastest.pastest.com/Secure/TestMe/Browser/429893#Top

1/2

8/11/2016

MyPastest

End Session

Previous Question

Tag Question

Feedback

End Review

Difficulty: Average
Peer Responses

Session Progress
Responses Correct:

Responses Incorrect:

207

Responses Total:

207

Responses - % Correct:

0%

Blog (https://www.pastest.com/blog) About Pastest (https://www.pastest.com/about-us)


Contact Us (https://www.pastest.com/contact-us) Help (https://www.pastest.com/help)
Pastest 2016

https://mypastest.pastest.com/Secure/TestMe/Browser/429893#Top

2/2

Вам также может понравиться